You are on page 1of 98

Board of Directors Cases 3C

I. POWER/AUTHORITY OF THE BOD AS CORPORATE AGENTS


T A B L E O F C O N T E N T S (Sec. 24)
20. RIOSA v. TABACO LA SUERTE CORP., GR 203786 ........................................................ 17
A. NATURE OF OFFICE 21. LA BUGA’AL v. RAMOS ......................................................................................................... 18
B. REQUIREMENTS 22. SHIPSIDE INCORPORATED vs. CA .................................................................................... 20
1. Qualifications/Qualifying Shares (Sec. 24) 23. CEBU MACTAN VS. MASAHIRA.......................................................................................... 21
1. VILLAFUERTE v. MORENO ...................................................................................................... 4 24. ABS-CBN v. CA....................................................................................................................... 22
2. BAGUIO v. CA ............................................................................................................................. 4 25. YAO KA SIN v. CA .................................................................................................................. 22
3. DETECTIVE & PROTECTIVE BUREAU, INC. vs. CLORIBEL and ALBERTO..................... 5 26. APT v. CA................................................................................................................................. 24
4. GRACE CHRISTIAN HIGH SCHOOL v. CA, GRACE VILLAGE ASSOCIATION, Inc., BELTRAN and GO 5 27. BA SAVINGS BANK vs. SIA and JOHN DOE ..................................................................... 24
5. LEE and LACDAO v. CA............................................................................................................. 6 28. MONTELIBANO v. BACOLOD MURCIA.............................................................................. 25
2. Requirements: Disqualifications 29. POWERS v. MARSHALL ....................................................................................................... 25
6. BRIAS v. HORD........................................................................................................................... 7 30. PREMIUM MARBLE v. CA and INTERNATIONAL CORPORATE BANK ........................ 26
C. ELECTION 31. RAMIREZ v. THE ORIENTALIST CORPORATION ............................................................ 27
1. Quorum J. DELEGATION OF AUTHORITY TO CORPORATE OFFICERS
2. Voting 1. Corporate Officers/Meaning of "Office" vis-à-vis "Employment"
7. AURBACH v. SANITARY WARES ............................................................................................ 7 32. REAL v. SANGU PHIL ............................................................................................................ 28
8. BATAAN SHIPYARD v. PCGG .................................................................................................. 8 33. MATLING v. COROS ............................................................................................................. 28
3. Election Contests 34. MANILA METAL CONTAINER CORP. v PNB, GR 166862 ............................................... 29
9. RICAFORT v. DICDICAN ........................................................................................................... 8 35. ONGKINGCO v. NLRC........................................................................................................... 30
D. REPORT ON ELECTION (Sec. 26) 36. LAO vs. CA .............................................................................................................................. 31
10. PREMIUM MARBLE v. CA and INTERNATIONAL CORPORATE BANK ........................ 10 37. DE TAVERA v. PHIL. TUBERCULOSIS SOCIETY ............................................................. 31
E. TERM OF OFFICE/HOLDOVER 2. Corporate Officers (Sec. 25); Qualifications and Disqualifications; Authority and Liabilities
11. SEC v. BAGUIO COUNTRY CLUB ....................................................................................... 10 38. MATLING INDUSTRIAL and COMMERCIAL CORPORATION, et al. v. COROS ........... 33
12. SEÑERES v. COMELEC ........................................................................................................ 12 39. OKOL v. SLIMMERS WORLD and MOY.............................................................................. 33
F. HOW REMOVED (Sec. 28) 40. GOMEZ v. PNOC DMC .......................................................................................................... 34
13. BERNAS, et. al. v. CINCO, et. al............................................................................................. 12 Section 11, Rule 14, 1997 Rules of Civil Procedure
14. LAMBERT v. FOX .................................................................................................................. 14 41. E.B. VILLAROSA & PARTNER., Ltd V. HON. BENITO...................................................... 35
G. HOW VACANCY FILLED (Sec. 29) 42. CAGAYAN VALLEY DRUG CORPORATION v. CIR.......................................................... 36
15. VALLE VERDE COUNTRY CLUB v. AFRICA...................................................................... 15 43. BOI v. SR METALS, Inc. ......................................................................................................... 36
H. HOW COMPENSATED (Sec. 30) 44. SSPC v. BARDAJE ................................................................................................................. 37
16. SINGSON et al. v. COA........................................................................................................... 15 45. PABON & CAMONAYAN v. NLRC ....................................................................................... 37
17. WESTERN INSTITUTE v. SALAS ......................................................................................... 16 46. VLASON ENTERPRISES CORP. v CA and DURAPROOF SERVICES.......................... 37
18. CENTRAL COOP EXCHANGE v. TIBE................................................................................ 17 47. PRIME WHITE CEMENT CORPORATION v. IAC .............................................................. 38
19. LINGAYEN GULF ELECTRIC POWER COMPANY, Inc. V. BALTAZAR.......................... 17 48. LOUIS VUITTON S.A. vs. JUDGE FRANCISCO DIAZ VILLANUEVA .............................. 39

1
Board of Directors Cases 3C
3. Executive Committee (Sec. 35) 74. QUEENSLAND-TOKYO COMMODITIES v. GEORGE...................................................... 63
4. "Doctrine of Apparent Authority" 75. GERALDO vs. BILL SENDER ............................................................................................... 64
49. AYALA LAND v. ASB .............................................................................................................. 39 76. WENSHA SPA CENTER and/or XU ZHI JIE v. YUNG........................................................ 65
50. BANATE v. PHILIPPINE COUNTRYSIDE ............................................................................ 41 77. CEBU MACTAN v. MASAHIRO ............................................................................................ 65
51. SARGASSO v. PPA ................................................................................................................ 41 78. ARMANDO DAVID v. NAFLU ................................................................................................ 66
52. ASSOCIATED BANK (now UOB) v. SPOUSES PRONSTROLLER.................................. 42 79. HILARIO P. SORIANO and ROSALINDA ILAGAN vs. PEOPLE, BSP and PDIC ............ 66
53. ACUÑA v. BATAC ................................................................................................................... 44 80. CEBU COUNTRY CLUB v. ELIZAGAQUE .......................................................................... 67
54. BOARD OF LIQUIDATORS v. KALAW................................................................................. 45 81. CALTEX vs. NLRC and STO. TOMAS.................................................................................. 68
55. TRINIDAD J. FRANCISCO vs. GOVERNMENT SERVICE INSURANCE SYSTEM ....... 45 82. ATRIUM MANAGEMENT v. CA ............................................................................................ 69
56. RURAL BANK OF MILAOR (Camarines Sur) v. FRANCISCA OCFEMIA, et.al. ............... 47 83. ARB CONSTRUCTION CO. INC. and MARK MOLINA v. CA............................................ 69
K. THREE-FOLD FIDUCIARY DUTIES (Sec. 31) 84. LIM v. CA.................................................................................................................................. 70
1. Duty of Obedience 85. FRANCISCO v. MEJIA ........................................................................................................... 70
57. ARNEL TY, MARIE TY, JASON ONG, WILLY DY, AND ALVIN TY v. NBI ....................... 48 86. DBP v. CA ................................................................................................................................ 71
2. Duty of Diligence: Business Judgment Rule 87. ATRIUM MANAGEMENT CORPORATION v. COURT OF APPEALS............................. 72
58. STEINBERG v. VELASCO ..................................................................................................... 48 88. AMERICAN HOSPITAL SUPPLIES/PHILIPPINES et al. vs. CA ........................................ 73
59. BALINGHASAY v. CASTILLO................................................................................................ 49 89. COMPLEX ELECTRONICS v. NLRC ................................................................................... 73
60. PHILIPPINE STOCK EXCHANGE Inc. vs COURT OF APPEALS..................................... 49 90. ERNESTINA CRISOLOGO-JOSE v. CA .............................................................................. 75
61. ONG YONG, et al. v. DAVID S. TIU, et al. ............................................................................ 50 91. FCY CONSTRUCTION and FRANCIS YU v. CA and LEY CONSTRUCTION ................ 75
3. Duty of Loyalty 92. LLAMADO v. CA ..................................................................................................................... 76
62. IENT and SCHULZE v. TULLETT PREBON (Philippines), Inc., .......................................... 51 93. MAM REALTY DEV’T CORPORATION and CENTENO v. NLRC and BALBASTRO..... 77
3.1 Self-Dealing Director/Officer (Sec. 32) 94. NAGUIAT, et al. v. NLRC........................................................................................................ 77
63. REPUBLIC v. COJUANGCO.................................................................................................. 52 95. PROGRESS HOMES and ALMEDA v. NLRC ..................................................................... 78
64. MEAD v. E. C. MCCULLOUGH, et al..................................................................................... 54 96. REAHS CORP v. NLRC ......................................................................................................... 79
65. PRIME WHITE CEMENT v. IAC and TE ............................................................................... 55 97. SANTOS vs. NLRC ................................................................................................................. 79
3.2 Interlocking Directors (Sec. 33) 98. SIA v PEOPLE......................................................................................................................... 80
66. PATLING v. SAN JOSE PETROLEUM ................................................................................. 56 99. TRAMAT MERCANTILE v. COURT OF APPEALS............................................................. 80
67. DBP v. CA ................................................................................................................................ 58 5. Watered Stocks (Sec. 65)
3.3 Doctrine of Corporate Opportunity (Sec. 34) 100. LIRAG TEXTILE MILLS and BASILIO LIRAG vs. SSS, HON. DE CASTRO .................. 81
68. GOKONGWEI, JR. v. SEC ..................................................................................................... 58 101. NAVA v. PEERS MARKETING ........................................................................................... 81
69. STRONG v. REPIDE............................................................................................................... 59 6. Derivative Suit
4. Personal Liability of Directors and other Corporate Officers/Liability of Corporation for Acts of Officers 102. JUANITO ANG (in behalf of SUNRISE) v. SPS. ROBERTO and RACHEL ANG ........... 82
70. SPI TECHNOLOGIES v. MAPUA .......................................................................................... 60 103. VILLAMOR v. UMALE .......................................................................................................... 83
71. HEIRS OF UY v. INTERNATIONAL EXCHANGE BANK.................................................... 61 104. CHING v. SUBIC ................................................................................................................... 84
72. EVER ELECTRICAL MANUFACTURING CORP. v SMEE-NAMAWU ............................. 62 105. LEGASPI TOWERS 300 v. MUER, et. al. ........................................................................... 85
73. HARPOON MARINE SERVICES v. FRANCSICO............................................................... 63 106. LISAM ENTERPRISES v. BDO ........................................................................................... 86

2
Board of Directors Cases 3C
107. STRADEC v. RADSTOCK & PNCC.................................................................................... 87
108. YU v. YUKAYGUAN.............................................................................................................. 90
109. GOCHAN et. al. v. YOUNG et. al ......................................................................................... 91
110. WESTERN INSTITUTE v. SALAS....................................................................................... 92
111. FIRST INTERNATIONAL BANK v. CA................................................................................ 92
112. COMMART PHILS v. SEC ................................................................................................... 93
113. CHASE v. BUENCAMINO .................................................................................................... 94
114. SMC v. KAHN ........................................................................................................................ 95
115. EVERETT v. ASIA BANKING............................................................................................... 95
116. RICARDO L. GAMBOA, et al. v. Hon. OSCAR R. VICTORIANO, et al. ........................... 96
117. REYES v. TAN....................................................................................................................... 96
118. PASCUAL v. OROZCO ........................................................................................................ 97

3
Delegation of Authority to Corporate Officers | Doctrine of Apparent Authority
Villafuerte and the other officers issued a Notice of National Elections to be National Congress was held, therefore he was unqualified to hold the
1. VILLAFUERTE v. MORENO held on 4 June 2008, the agenda of which included the election of officers, position of Chairman.
G.R. No. 186566 organization of standing committees, accreditation of new applicants for
October 2, 2009 membership, financial report, report on program for Nationwide A motion for reconsideration was filed but was denied.
FACTS: Development of Basketball and other matters.
ISSUE:
On 28 August 2006, at the sideline of the 18th FIBA World Congress held June 4 2008, respondents and the BAP-SBP members sympathetic to
at Tokyo, Japan, a Joint Communique ("Tokyo Communique") was their faction attended the National Congress, wherein the regular trustees Whether or not Villafuerte is not qualified to be the chairman.
entered into by the feuding Basketball Association of the Philippines and the executive officers of SBP were elected.
("BAP") and the newly formed Pilipinas Basketbol ("PB"), through their then HELD:
incumbent Presidents, Jose D. Lina, Jr. and Bernardo Gabriel L. Atienza, June 12 2008, the members of the Board of Trustees were then elected
respectively, and as witnessed not only by their other representatives but for the term of 2008 to 2012 and until their successors shall have been duly Yes. The Tokyo Communique’s directive to the three-man panel is for it to
also by the representative of the Philippine Olympic Committee ("POC") elected and qualified. review, verify, and validate the list of members as submitted by PB and
and the FIBA Secretary General Patrick Baumann. The main objectives of BAP to the FIBA Central Board Special Commission created to hear the
the Tokyo Communique are (1) to unify said rival basketball associations June 27, 2008, Petitioners filed before the Regional Trial Court of Manila a Philippine Case based on an agreed set of criteria for membership as
and (2) to facilitate the lifting of the suspension imposed by the Federation petition for declaration of nullity of the election of respondents as members formulated by said three-man panel. In other words, there is a given
Internationale de Basketball ("FIBA"), which prevented the country from of the Board of Trustees and Officers of BAP-SBP. process for validation of membership rather than the automatic grant of
participating in any international basketball competitions. voting or active membership status being insisted upon by petitioners.
Petitioners alleged that the June 12, 2008 election was a sham, illegal, and
The merger of the two associations will create a three man panel with the void. They also claimed to be the rightful and legally elected trustees and The membership validation resulted in the conferment of active
task to (1) writing and finalizing the organization’s constitution and by-laws; officers of the BAP-SBP and thus prayed that the corporate reins of BAP- membership status upon 19 BAP-SBP members, 17 of which participated
(2) reviewing, verifying and validating the list of members as submitted by SBP be turned over to them. in the June 12, 2008 meeting. Petitioners even constituted the majority of
BAP and PB to the FIBA Central Board Special Commission based on the Committee that undertook the task; they actively participated in the
agreed set of criteria for membership as formulated by the panel; and (3) Respondents argued that petitioners have no cause of action; that formulation of the validation rules based on the by-laws providing for this.
convening the National Congress of the united organization and to oversee Villafuerte never assumed the position of Chairman of the BAP-SBP
the election of officers. because he failed to qualify for the same; that before Villafuerte could Respondents, who were elected by 17 of the 19 active and voting
legally assume the Chairmanship of BAP-SBP, he must first be elected a members of the BAP-SBP during the meeting held on June 12, 2008, are
Jose De Lina, Bernardo Atienza and Manual Pangilinan composed of the member of the Board of Trustees. the legitimate officers of the organization, their election in accordance with
three man panel. the applicable rules on the said exercise.
September 3, 2008, the trial court ruled in favor of petitioners stating that
On 17 September 2006, in keeping with the merger and unification efforts the June 12 election was null and void. Villafuerte’s nomination must of necessity be understood as being subject
as embodied in the Tokyo Communique, the Samahang Basketbol ng to or in accordance with the qualifications set forth in the By-Laws of the
Pilipinas, Inc. ("SBP") was established and its constitutive documents The respondents appealed the case under a Petition for review under Rule BAP-SBP. Since the said by-laws require the Chairman of the Board of
consisting of the Articles of Incorporation were signed by the five (5) 43 of the Rules of Court. Trustees to be a trustee himself, petitioner Villafuerte was not qualified
incorporators, which include petitioner Pangilinan. On the same day, the since he had neither been elected nor appointed as one of the trustees of
incorporators likewise passed and signed its by-laws. November 18, 2008, the Court of Appeals rendered decision reversing and BAP-SBP. In other words, petitioner Villafuerte never validly assumed the
setting aside the Decision of the trial court and dismissing the petition for position of Chairman because he failed in the first place to qualify therefor.
February 4, 2007, the panel met in Bangkok, Thailand where they declaration of nullity of elections. As quoted the Bangkok Agreement
executed a MOA containing the terms and conditions as well as the should not be exploited as to clothe petitioners with the authority to 2. BAGUIO v. CA
renaming of "Samahang Basketbol ng Pilipinas, Inc." to "BAP-Samahang convene the National Congress and conduct themselves as trustees and CONSTANCIO BAGUIO vs CA, LAS PALMAS MANPOWER CORP.,
Basketbol ng Pilipinas, Inc." ("BAP-SBP"). officers of the BAP-SBP because the attendees of said June 4, 2008 SPS. DONALDO & CONSUELO PALMA
National Congress did not constitute a quorum; that only six of the GR No. 93417
February 5 2007, the first trustees of the SBP attended the unity congress attendees were active and voting members, while the rest were September 14, 1983
where the officers were elected. Villafuerte was elected as chairman while associates, or non-voting members, or even non-members.
Pangilinan was elected as President. Quiason, J.
The Court of Appeals found Villafuerte not qualified to hold the position of
However, the association was divided into two factions. Said dispute Chairman of BAP-SBP. It held that the organization’s By-laws require that FACTS:
evolved from the resolve of petitioner Pangilinan not to recognize the the Chairman of the Board of Trustees must first be a trustee. Since
election of respondent Villafuerte as Chairman of BAP-SBP on account of Villafuerte was not yet named as a trustee of the BAP-SBP when the In April 1982, Constancio Baguio was offered by the spouses
the alleged failure of the latter to qualify for the said position. Palmas a director position as vice-president of Las Palmas Manpower
4
Delegation of Authority to Corporate Officers | Doctrine of Apparent Authority
Corporations. Along with the offer are shareholdings of 600 shares at par 3. DETECTIVE & PROTECTIVE BUREAU, INC. vs. THE
value of 100 pesos per share, for a total of 60,000 pesos. After paying for HONORABLE GAUDENCIO CLORIBEL and FAUSTINO S. ALBERTO ISSUE:
the amount, the spouses assured that a resolution has been issued and a G.R. No. L-23428 November 29, 1968
secretary’s certification has been drawn and that it will be delivered to Should the writ of preliminary injunction against respondent be granted?
Baguio on December 1982. Baguio asked the spouses to issue a receipt FACTS:
establishing his proof of payment of the shares, however the spouses RULING:
assured him that the resolution and certification is sufficient proof of receipt The complaint, in Civil Case No. 56949 of the Court of First Instance of
of such payment. Manila, dated May 4, 1964, filed by Detective and Protective Bureau, Inc., Petitioner in support of its stand alleged that public interest demanded that
therein plaintiff (petitioner herein) against Fausto S. Alberto, therein the writ enjoining respondent Fausto Alberto from exercising the functions
December 1982 arrived and the Board Resolution and defendant (respondent herein), for accounting with preliminary injunction of managing director be maintained. Petitioner contended that respondent
Secretary’s Certificate was not yet delivered, Baguio inquired into the and receivership, alleged that Alberto had arrogated to himself the power of the Board of Directors of the
corporate books and he found out that the said Resolution and Certificate corporation because he refused to vacate the office and surrender the
were not recorded therein. This prompted Baguio to file a collection suit Detective & Protective Bureau, Inc. was a corporation duly organized and same to Jose de la Rosa who had been elected managing director by the
against the corporation and the spouses for non-delivery of the shares and existing under the laws of the Philippines. Faustino S. Alberto was Board to succeed him. This assertion, however, was disputed by
the return of the 60,000 pesos he paid. In their defense, the spouses and managing director of plaintiff corporation from 1952 until January 14, 1964 respondent Alberto who stated that Jose de la Rosa could not be elected
Las Palamas alleged that there is no ground for them to give the amount managing director because he did not own any stock in the corporation.
to Baguio as he was not able to present a receipt establishing the proof of In June, 1963, Alberto illegally seized and took control of all the assets as
payment. As a counter to the defense, Baguio presented the Board well as the books, records, vouchers and receipts of the corporation from There is in the record no showing that Jose de la Rosa owned a share of
Resolution and Secretary’s Certificate establishing that he is a holder of the accountant-cashier, concealed them illegally and refused to allow any stock in the corporation. If he did not own any share of stock, certainly he
600 shares in the corporation and contended that such holdings will member of the corporation to see and examine the same could not be a director pursuant to the mandatory provision of Section 30
constitute proof of payment. The RTC favored Baguio, the CA reversed of the Corporation Law, which in part provides:
the RTC and favored the Palamas, and Baguio appealed to the SC. On January 14, 1964, the stockholders, in a meeting, removed Alberto as Sec. 30. Every director must own in his own right at least one
managing director and elected Jose de la Rosa in his stead. However, share of the capital stock of the stock corporation of which he is
Issue: Alberto not only had refused to vacate his office and to deliver the assets a director, which stock shall stand in his name on the books of
Is the fact of being a director/VP in a corporation, and therefore and books to Jose de la Rosa, but also continued to perform unauthorized the corporations....
a holder of shares as such, tantamount as proof having paid for acts for and in behalf of plaintiff corporation.
the said shares as held? If he could not be a director, he could also not be a managing director of
Alberto had been required to submit a financial statement and to render an the corporation, pursuant to Article V, Section 3 of the By-Laws of the
Ruling: accounting of his administration from 1952 but defendant has failed to do Corporation which provides that:
No, the fact of being a director/VP and a shareholder does so. Alberto, contrary to a resolution adopted by the Board of Directors on The manager shall be elected by the Board of Directors from
not signify proof of payment. November 24, 1963, had been illegally disposing of corporate funds; that among its members.... (Record, p. 48)
defendant, unless immediately restrained ex-parte, would continue
The Supreme Court ruled that what Baguio presented purporting discharging the functions of managing director; and that it was necessary If the managing director-elect was not qualified to become managing
to be the Board Resolution and the Secretary’s Certificate merely to appoint a receiver to take charge of the assets and receive the income director, respondent Fausto Alberto could not be compelled to vacate his
established the fact that he is a shareholder of 600 shares and that he is a of the corporation. office and cede the same to the managing director-elect because the by-
director and vice-president of Las Palmas Manpower Corporation. It further laws of the corporation provides in Article IV, Section 1 that "Directors shall
ruled that inclusion as a director/VP does not itself constitute proof of Plaintiff prayed that a preliminary injunction ex-parte be issued restraining serve until the election and qualification of their duly qualified successor."
payment for the shares so-held. Thus, though Baguio presents a cause of defendant from exercising the functions of managing director and from
action, he failed to substantiate the same through sufficient evidence. disbursing and disposing of its funds; that Jose M. Barredo be appointed 4. GRACE CHRISTIAN HIGH SCHOOL v. THE COURT OF
receiver; that, after judgment, the injunction be made permanent and APPEALS, GRACE VILLAGE ASSOCIATION, INC., ALEJANDRO G.
Directorship in a corporation does not require that the shares so- defendant be ordered to render an accounting. BELTRAN, and ERNESTO L. GO
held must be paid in order to be qualified; what is only required is that at G.R. No. 108905. October 23, 1997
least one (1) share is held in the corporation as he is appointed as director. On June 18, 1964, respondent Judge granted the writ of preliminary
The CA is thereby affirmed. injunction prayed for, conditioned upon plaintiff's filing a bond of P5,000.00. Facts:
N.B. If you are the counsel for Baguio, how would you prove payment Plaintiff filed the bond, but while the same was pending approval defendant
of the shares? (PLEASE research, the decision is dubious, if not, unfair Fausto S. Alberto filed, on July 1, 1964, a motion to admit a counter-bond Grace Christian High School (Grace Christian) is an educational institution
for Baguio. Is it sound corporate practice, to allow an unpaid shareholder for the purpose of lifting the order granting the writ of preliminary injunction. offering preparatory up to secondary courses at Grace Christian Village
become a director/VP of a corporation?) Inspite of the opposition filed by plaintiff, respondent Judge issued, on
Quezon City while Grace Christian Village Association, Inc. (The
August 5, 1964, an order admitting the counterbond and setting aside the
writ of preliminary injunction.
5
Delegation of Authority to Corporate Officers | Doctrine of Apparent Authority
Association) is an organization composed of lot/building owners, lessees, ruled that the 1975 draft of by-laws is invalid since it does not comply with members. There may be corporations in which there are unelected
and residents of Grace Christian Village. the requirements for valid amendment under the law. Hence, this petition members in the board but it is clear that in the examples cited by petitioner
with the Supreme Court. the unelected members sit as ex officio members, i.e., by virtue of and for
The 1968 by-laws of the Village provides that the members will have an as long as they hold a particular office. But in the case of Grace Christian,
annual meeting and election of the board of directors composed of 11 Arguments/Issues: there is no reason at all for its representative to be given a seat in the
members that will serve for one year by plurality vote and by secret boar,nor does Grace Christian claim a right to such seat by virtue of an
balloting to be held every first Sunday of January at 2:00 pm. Grace Christian: office held. In fact it was not given such seat in the beginning. It was only in
1975 that a proposed amendment to the by-laws sought to give it one.
Sometime in December 20, 1975, a committee of the board of directors a. The 1975 draft of by-laws is valid and binding because there is
prepared a draft of the by-laws (1975 draft of by-laws) which provides really no provision of law prohibiting unelected members of b. It is probable that, in allowing Grace Christan’s representative to sit on
that the number of directors will be increased to fifteen (15), fourteen (14) boards of directors of corporations, citing as example is the the board, the members of The Association were not aware that this
thereof will be elected while the representative of Grace Christian will be Archbishop of Manila, that according to the by-laws of Pius XII was contrary to law. It should be noted that they did not actually
given a permanent position as one of the directors of The Association. Catholic Center, Inc. and Cardinal Santos Memorial Hospital, implement the provision in question except perhaps insofar as it
However, such draft was never presented to the general membership for Inc., whoever sits as the Archbishop of Manila is considered as increased the number of directors from 11 to 15, but certainly not the
approval. member of the Board of trustees without the benefit of an allowance of Grace Christian’s representative as an unelected
election. member of the board of directors. It is more accurate to say that the
Nevertheless, from 1975 to 1990, the representative of Grace Christian members merely tolerated Grace Christian’s representative and
was given a permanent seat in the board of directors of The Association. tolerance cannot be considered ratification.
In February 1990, The Association sent a letter to principal James Tan of b. It already acquired a vested right to a permanent seat in the
Grace Christian informing him their permanent seat in the board of Board of Directors of Grace Village Association since the right of
c. Since the provision in question is contrary to law, the fact that for
directors of the association would be reconsidered and further discussed the petitioner to an automatic membership in the board of the
fifteen years it has not been questioned or challenged but, on the
that all 15 members of the board should be elected by members of Association was granted by the members of the Association
contrary, appears to have been implemented by the members of The
association and if they let a person or entity seat as a permanent director, themselves and this grant has been implemented by members
Association cannot forestall a later challenge to its validity. Neither can
that would deprive the right of voters to vote for fifteen (15) members of the of the board themselves all through the years.
it attain validity through acceptance because, if it is contrary to law, it
Board, and it is undemocratic for a person or entity to hold office in c. The Practice of tolerating the automatic inclusion of petitioner as
is beyond the power of the members of the association to waive its
perpetuity. Hence, notices were sent to the members of the association a permanent member of the Board of Directors of the
invalidity. For that matter the members of the association may have
informing them that the 1968 by-laws will be followed for the election of the Association without the benefit of election is allowed under the
formally adopted the provision in question, but their action would be
board. law.
of no avail because no provision of the by-laws can be adopted if it is
Ruling:
contrary to law.
Grace Christian filed for a petition for mandamus to the Home Insurance
and Guaranty Corporation (HIGC) to compel the association to recognize a. The 1975 draft of by-laws is not valid and binding, since the same is
5. RAMON LEE AND ANTONIO LACDAO V. CA
the right of its representative to a permanent seat in the board basing its contrary to Section 23 of B.P. 68, or the corporation code of the G.R. No. 93695 February 4, 1992
claim under the 1975 draft of by-laws of The Association. The Association Philippines, to wit:
in its answer cited the opinion of the SEC which said that the practice of The Board of Directors or Trustees. - Unless otherwise
allowing unelected members in the board was contrary to the existing by- provided in this Code, the corporate powers of all corporations
Facts:
laws of the association and to 92 of the Corporation Code and that the formed under this Code shall be exercised, all business
basis of the petition of Grace Christian was merely proposed by-laws of the conducted and all property of such corporations controlled and On November 15, 1985, a complaint for a sum of money was
association which have not been voted for by the members and was not held by the board of directors or trustees to be elected from filed by the International Corporate Bank, Inc. against the private
yet approved by SEC. The hearing officer of HIGC dismissed the petition among the holders of stocks, or where there is no stock, from respondents who, in turn, filed a third party complaint against ALFA and
for mandamus and adopted the argument of The Association. Upon among the members of the corporation, who shall hold office for the petitioners
appeal, the appeals board of HIGC affirmed the ruling of the hearing officer one (1) year and until their successors are elected and qualified.
The trial court issued an order requiring the issuance of
of HIGC and further ruled that the long standing practice of giving Grace
In sum, the law provides that the board of directors of an alias summons upon ALFA through the DBP as a consequence of the
Christian a permanent seat has no legal basis. Grace Christian appealed petitioner's letter informing the court that the summons for ALFA was
with the Court of Appeals (CA), but the CA affirmed the ruling of HIGC and corporations must be elected from among the stockholders or

6
Delegation of Authority to Corporate Officers | Doctrine of Apparent Authority
erroneously served upon them considering that the management of ALFA It is not disputed that a resignation per verba is just as effective
had been transferred to the DBP. In order to be eligible as a director, what is material is the legal and binding as a resignation per scripta.
title to, not beneficial ownership of, the stock as appearing on the
The DBP claimed that it was not authorized to receive summons books of the corporation. The facts of this case show that the But it appears that the minutes of the proceedings of the meeting
on behalf of ALFA since the DBP had not taken over the company which petitioners, by virtue of the voting trust agreement executed in on September 26, kept by the Secretary, were destroyed.
has a separate and distinct corporate personality and existence. 1981 disposed of all their shares through assignment and
delivery in favor of the DBP, as trustee. Consequently, the The minutes of the transactions, prepared by its secretary or
The petitioners filed a motion for reconsideration submitting that petitioners ceased to own at least one share standing in their some person named or appointed for the purpose of keeping a record of
Rule 14, section 13 of the Revised Rules of Court is not applicable since names on the books of ALFA as required under Section 23 of the proceedings, are generally accepted, once approved by the Board, as
they were no longer officers of ALFA and that the private respondents the new Corporation Code. They also ceased to have anything prima facie evidence of what actually took place during that meeting. In this
should have availed of another mode of service under Rule 14, Section 16 to do with the management of the enterprise. The petitioners case, the minutes were not prepared by the secretary. His minutes had
of the said Rules. The petitioners maintain that with the execution of the ceased to be directors. Hence, the transfer of the petitioners' been destroyed. The minutes presented in evidence was prepared by the
voting trust agreement between them and the other stockholders of ALFA, shares to the DBP created vacancies in their respective board itself in the absence of Brias, and after the alleged resignation took
as one party, and the DBP, as the other party, the former assigned and positions as directors of ALFA. place.
transferred all their shares in ALFA to DBP, as trustee. They argue that by
b. No. Section 13 of Rule 14 of the Revised Rules of Court provides Brias maintains that he did not resign. It will be noted that no
virtue to of the voting trust agreement the petitioners can no longer be words are here attributed to him which indicate that he then and there
that if the defendant is a corporation organized under the laws of
considered directors of ALFA. absolutely and unequivocably resigned. The most that can be said is that
the Philippines or a partnership duly registered, service may be
made on the president, manager, secretary, cashier, agent or he “ceased to attend its meetings.” No words are attributed to him, even by
The private respondents argued that the voting trust agreement
any of its directors. The petitioners in this case do not fall under said minutes which show conclusively that he did then and there resign.
dated March 11, 1981 did not divest the petitioners of their positions as
president and executive vice-president of ALFA so that service of any of the enumerated officers. The service of summons upon
The Court finds that there is a marked and irreconcilable conflict
summons upon ALFA through the petitioners as corporate officers was ALFA, through the petitioners, therefore, is not valid.
in the statements of the different members of the board. Some say that
proper. Brias said that “he was resigning,” others say that he was “going to resign,”
6. BRIAS v. HORD while the minutes say that “he could not continue to belong to a
The trial court upheld the validity of the service of summons on G.R. No. L-8387; February 5, 1913 corporation” etc., and that “therefore, he withdrew from it and ceased to
ALFA through the petitioners, thus, denying the latter's motion for (Sorry di ko alam ano kinalaman nito sa DQs. Read at your own risk.) attend its meetings.” The Court finds that the members are not clear in their
reconsideration and requiring ALFA to file its answer through the understanding of just exactly what Brias said with reference to the alleged
petitioners as its corporate officers. FACTS: Enrique Brias was a duly elected member of the Board of resignation.
Directors of BPI, and also a member of the committee of credits. John Hord
Issues: is the acting president of said corporation. That members of boards become momentarily disgusted with
the method of procedure of their associates and withdraw, is not an
1. Is the execution of the voting trust agreement by a stockholders On August 15, 1912, Brias, acting as a member of the Board and infrequent occurrence. For the associates to take advantage of this
whereby all his shares to the corporation have been transferred the committee of credits, sought authority from Hord to examine and common weakness of men and distort the momentary action into a
to the trustee deprives the stockholders of their position as inspect the books of account of BPI, which is in Hord’s possession and meaning not really intended or justified by actual words would be to do
directors of the corporation? control. Hord refused and denied Brias repeatedly. great injustice to their fellows.
2. Is there a proper service of summons?
Ruling: Later on, Hord and the other Board Members declared Brias’
position as vacant by reason of Brias’ alleged resignation they said to have 7. AURBACH v. SANITARY WARES
a. Yes. Under section 59 of the Corporation Code, a voting trust happened during a board meeting where heated exchanges occurred G.R. No. 75875, December 15, 1989
agreement is an agreement in writing whereby one or more dated September 26, 1912.
stockholders of a corporation consent to transfer his or their Facts: Saniwares, a domestic corporation was incorporated for the primary
shares to a trustee in order to vest in the latter voting or other Brias instituted a complaint against Hord and the Board of BPI purpose of manufacturing and marketing sanitary wares. One of the
rights pertaining to said shares for a period not exceeding five praying that he be allowed tot eh use, exercise and enjoyment of his office incorporators, Mr. Baldwin Young went abroad to look for foreign partners,
years upon the fulfillment of statutory conditions and such other as a member of the Board and the committee of credits. European or American who could help in its expansion plans. ASI, a
terms and conditions specified in the agreement. The five year- foreign corporation domiciled in Delaware, United States entered into an
ISSUE: Did Brias resign as a member of the Board of Directors of BPI? Agreement with Saniwares and some Filipino investors whereby ASI and
period may be extended in cases where the voting trust is
executed pursuant to a loan agreement whereby the period is the Filipino investors agreed to participate in the ownership of an enterprise
HELD: No. which would engage primarily in the business of manufacturing in the
made contingent upon full payment of the loan.
Philippines and selling here and abroad vitreous china and sanitary wares.
7
Delegation of Authority to Corporate Officers | Doctrine of Apparent Authority
The parties agreed that the business operations in the Philippines shall be Agreement as well as the testimonial evidence shows that the parties 2. Give certain shareholders or groups of shareholders power to select a
carried on by an incorporated enterprise and that the name of the agreed to establish a joint venture and not a corporation. The history of the specified number of directors; 3. Give to the shareholders control over the
corporation shall initially be "Sanitary Wares Manufacturing Corporation." organization of Saniwares and the unusual arrangements which govern its selection and retention of employees; and
policy making body are all consistent with a joint venture and not with an 4. Set up a procedure for the settlement of disputes by arbitration
The Agreement has the following provisions relevant to the issues in these ordinary corporation. Under the Agreement there are two groups of
cases on the nomination and election of the directors of the corporation: stockholders who established a corporation with provisions for a special
contractual relationship between the parties, i. e., ASI and the other 8. BATAAN SHIPYARD v. PRESIDENTIAL COMMISSION ON GOOD
"3. Articles of Incorporation stockholders. GOVERNMENT
May 27, G.R. No. 75885
(a) The Articles of Incorporation of the Corporation shall be substantially in Section 5 (a) of the agreement uses the word "designated" and not
the form annexed hereto as Exhibit A and, insofar as permitted under "nominated" or "elected" in the selection of the nine directors on a six to FACTS:
Philippine law, shall specifically provide for three ratio. Each group is assured of a fixed number of directors in the
board. This case arose from a sequestration order issued by the PCGG
(1) Cumulative voting for directors: under authority given by the president. Such sequestration order was sent
Moreover, ASI in its communications referred to the enterprise as joint and received by petitioner. Pursuant to this sequestration orders, take over
xxxxxxxxx venture. Philippine Corporation Code itself recognizes the right of orders were also issued to protect public interest and to prevent the
stockholders to enter into agreements regarding the exercise of their voting disposal or dissipation of business enterprises and properties taken over
"5. Management rights. Saniwares is technically not a close corporation because it has more by the government of the Marcos Administration or by entities or persons
than 20 stockholders, the undeniable fact is that it is a close-held close to former President Marcos, until the transactions leading to such
(a) The management of the Corporation shall be vested in a Board corporation. Surely, appellants cannot honestly claim that Saniwares is a acquisition by the latter can be disposed of by the appropriate authorities.
of Directors, which shall consist of nine individuals. As long as public- issue or a widely held corporation. Express arrangements between However, among other facts, the petitioner questions the exercise of
American-Standard shall own at least 30% of the outstanding corporate joint ventures should be construed with less emphasis on the PCGGs right of ownership and management when it terminated several
stock of the Corporation, three of the nine directors shall be ordinary rules of law usually applied to corporate entities and with more contracts without the consent of both parties, to enter contracts, and to
designated by American-Standard, and the other six shall be consideration given to the nature of the agreement between the joint operate its quarry business, and especially its right to vote during
designated by the other stockholders of the Corporation. venturers. stockholders‘meetings.

ISSUE: Whether or not PCGG may vote in stockholders ‘meetings.


At the request of ASI, the agreement contained provisions designed to In addition, paragraph 2 of Sec. 100 of the Corporation Code does not
protect it as a minority group, including the grant of veto powers over a necessarily imply that agreements regarding the exercise of voting rights RULING:
number of corporate acts and the right to designate certain officers, such are allowed only in close corporations. Having entered into a well-defined
as a member of the Executive Committee whose vote was required for contractual relationship, it is imperative that the parties should honor and YES.
important corporate transactions. The annual stockholders' meeting was adhere to their respective rights and obligations thereunder. Clearly
held. The stockholders then proceeded to the election of the members of established minority position of ASI and the contractual allocation of board PCGG may properly exercise the prerogative to vote
the board of directors. The ASI group nominated three persons while the seats cannot be disregarded and the rights of the stockholders to sequestered stock of corporations, granted to it by the President of the
Philippine investors nominated six. The chairman, Baldwin Young ruled the cumulative voting should also be protected. Philippines through a Memorandum dated June 26, 1986. That
last two nominations out of order on the basis of section 5 (a) of the Memorandum authorizes the PCGG, pending the outcome of proceedings
Agreement, the consistent practice of the parties during the past annual Section 5 (a) of the Agreement relates to the manner of nominating the to determine the ownership of sequestered shares of stock, to vote such
stockholders' meetings to nominate only nine persons as nominees for the members of the board of directors while Section 3 (a) (1) relates to the shares of stock as it may have sequestered in corporations at all
nine-member board of directors, and the legal advice of Saniwares' legal manner of voting for these nominees. stockholders' meetings called for the election of directors, declaration of
counsel. dividends, amendment of the Articles of Incorporation, etc. Moreover, in
Consideration of the contractual intent of the parties is the consideration as the case at bar, there was adequate justification to vote the incumbent
Issue/s: regards the possible domination by the foreign investors of the enterprise directors out of office and elect others in their stead because the evidence
1) Was the business of the parties a joint venture or a corporation? in violation of the nationalization requirements enshrined in the Constitution showed prima facie that the former were just tools of President Marcos and
2) May the ASI Group may vote their additional 10% equity during and circumvention of the Anti-Dummy Act. It is to be noted that the same were no longer owners of any stock in the firm, if they ever were at all.
elections of Saniwares' board of directors? law also limits the election of aliens as members of the board of directors
in proportion to their allowance participation of said entity. 9. CORAZON H. RICAFORT v. ISAIAS P. DICDICAN
Ruling: The business is a joint venture. The rule is that whether the parties GR Nos. 202647-50, Mar 09, 2016 (REYES, J.:)
to a particular contract have thereby established among themselves a joint Thus, such stipulations are valid:
venture or some other relation depends upon their actual intention which 1. Require greater than majority vote for shareholder and director action;
is determined in accordance with the rules governing the interpretation and DOCTRINE:
construction of contracts. our examination of important provisions of the
8
Delegation of Authority to Corporate Officers | Doctrine of Apparent Authority
“An election contest refers to any controversy or dispute Gaitman, however, contended that:
involving title or claim to any elective office in a stock or non-stock a. the complaint must be barred by prescription( it was filed more Indeed, the Ricaforts are: (a) asserting their "right to choose the
corporation, the validation of proxies, the manner and validity of than two months after the ASM complained of,) because the persons who will direct, manage and operate the corporation is significant
elections, and the qualifications of candidates, including the proclamation complaint involved an election contest - since in effect it sought because it is the primary way in which a stockholder can have a voice in
of winners, to the office of director, trustee or other officer directly elected the management of corporate affairs,"; and (b) by praying for the voiding of
to nullify the election of the Board of Directors of NADECOR for
by the stockholders in a close corporation or by members of a non-stock the August 15, 2011 ASM, and for "other just and equitable reliefs," – but
corporation where the articles of incorporation or by-laws so provide.” FY2011-2012, under Section 3, Rule 6 of the Interim Rules of such act clearly shows that the Ricaforts were really seeking the holding of
Procedure Governing Intra-Corporate Controversies (Interim a new election for members of the Board of Directors of NADECOR for
FACTS: Rules) - it should have been filed within 15 days from the date of FY2011-2012.
The NADECOR is a domestic company which was first the election.
registered with the Securities and Exchange Commission (SEC) on b. That as shown in the affidavit of the NADECOR messenger, As the CA noted, by seeking to nullify the August 15, 2011 ASM
September 6, 1956 is the holder of a Mining Production Sharing Mario S. San Juan who mailed the notices on August 11, 2011 of NADECOR, "including all proceedings taken thereat, all the
Agreement (MPSA). with the Department of Environment and Natural consequences thereof, and all acts carried out pursuant thereto" - the
or four days prior to the scheduled ASM.
Resources (DENR), which covers the King-king Gold and Copper Project petitioners were clearly challenging the validity of the election of the new
(King-king Project), a 1,656-hectare gold and copper mining concession in c. that the plaintiffs although physically absent were in fact Board of Directors. As the NADECOR's Amended By-Laws itself expressly
Barangay King-king, Municipality of Pantukan, Province of Compostela represented by their proxy, JG Ricafort, by virtue of irrevocable provides, the purpose of the ASM is "for the election of Directors and
Valley in Mindanao. The King-king Project is the second largest copper proxies which they executed; that JG Ricafort attended and for the transaction of general business of its office." To nullify the
and gold mine in the country with proven copper deposits of 5.4 billion signed the attendance sheet as the plaintiffs' proxy and August 15, 2011 ASM would have had no practical effect except to void
pounds and gold deposits of 10.3 million troy ounces. participated in the ASM for himself as well as in the plaintiffs' the election of the Board of Directors.
behalf; that the true and beneficial owner of the shares of stock
Pursuant to Section 1, Article I of NADECOR's Amended By- In the case of Yujuico, the Court expressly ruled that where one
issued in the plaintiffs' names is JG Ricafort, not the plaintiffs, as
Laws, its regular annual stockholders' meeting (ASM) was held on August of the reliefs sought in the complaint is to nullify the election of the Board of
15, 2011 to elect its Board of Directors for Fiscal Year (FY) 2011-2012. shown in the Nominee Agreements which they executed; that Directors at the ASM, the complaint involves an election contest. Both
Gatmaitan, NADECOR Corporate Secretary, attested to the presence of a aided by the irrevocable proxies and Nominee Agreements, JG cases put in issue the validity of the ASM and, expressly in Yujuico, the
quorum representing 94.81% of NADECOR's outstanding shares of stock, Ricafort won election to the NADECOR Board. election of the members of the Board of Directors. The ostensible
and the election of new set of its Board of Directors, namely, Calalang, difference is that in the present case, the Ricaforts invoked lack of notice of
Jose G. Ricafort (JG Ricafort), Jose P. De Jesus (De Jesus), Romulo, Ricaforts countered that - not being an election contest, because: the August 15, 2011 ASM, while in Yujuico the ground invoked was
Ayala, Victor P. Lazatin (Lazatin), Ethelwoldo E. Fernandez (Fernandez), a. they are not claiming any elective office in NADECOR improper venue.
Nitorreda and Engle. b. Neither are they questioning the manner and validity of the
Additional:
elections, and qualifications of the candidates for directorship.
On October 20, 2011, Corazon H. Ricafort, wife of JG Ricafort, On alleged deprivation to participate -
along with their children (Jose,Marie, Maria Theresa) – stockholders in c. Their prayer is clear that they seek to have the August 15, 2011 1) they were in fact represented by JG Ricafort under an
record, filed a complaint to declare null and void "the 15 August 2011 [ASM] [ASM] declared null and void due to fatal defects committed irrevocable proxy which they executed on April 26, 2010.
of NADECOR, including all proceedings taken thereat, all the prior to said meeting. The nullification of the proceedings, Gatmaitan, NADECOR Corporate Secretary, categorically
consequences thereof, and all acts carried out pursuant thereto," as well including the elections is not only incidental or the logical declared under oath that JG Ricafort held a valid irrevocable
as the election of the members of its Board of Directors. consequence of a declaration of nullity of the [ASM].
The Ricaforts alleged that: proxy from the Ricaforts to attend and vote their shares at all
ISSUE: meetings of the stockholders, and that JG Ricafort signed the
a. they had no knowledge or prior notice of, and were thus unable Does the case involve an election contest, which shall be subject
to attend, participate in, and vote at, the said [ASM]" - since they attendance sheet for and in behalf of the plaintiffs as shown by
to the 15 day period prescription?
received the notice of the ASM only on August 16, 2011, or one RULING: his signatures in the rows in the said attendance sheet for the
day late, in violation of the three-day notice provided in names of the plaintiffs who had appointed him as his proxy.
NADECOR's By-Laws. YES. Content of the Irrevocable Proxy expressly authorized JG
b. that due to lack of notice, they failed to attend the said ASM and Under SEC. 2 of Rule 6 of the Interim Rules, “An election contest Ricafortto: “xxx… to attend and represent the undersigned at
refers to any controversy or dispute involving title or claim to any elective [any and all meetings of the shareholders of the Company], and
to exercise their right as stocldiolders to participate in the
office in a stock or non-stock corporation, the validation of proxies, the
management and control of NADECOR. for and on behalf of the undersigned, to vote upon any and all
manner and validity of elections, and the qualifications of candidates,
c. the notice announced a time and venue different from those set including the proclamation of winners, to the office of director, trustee or matters to be taken up at said meeting, according to the number
forth in the By-Laws. other officer directly elected by the stockholders in a close corporation or of share(s) of stock of the Company of which the undersigned
by members of a non-stock corporation where the articles of incorporation are the lawful record and beneficial owners,and which they
or by-laws so provide. ” would be entitled to vote if personally present…. xxx” Thus, not
9
Delegation of Authority to Corporate Officers | Doctrine of Apparent Authority
only did the SPA acknowledge JG Ricafort's proxy authority from Premium Marble Resources, Inc., assisted by Atty. Arnulfo Dumadag as is still pending, is a matter that is also addressed, considering the premises,
the petitioners, it even expanded his authority to include naming counsel, filed an action for damages against International Corporate Bank. to the sound judgment of the Securities & Exchange Commission.
another person as proxy of the petitioners.
The complaint alleges that Ayala Investment and Development 11. SEC v. BAGUIO COUNTRY CLUB
2) Also, Corazon Ricafort misled the trial court into thinking that they
Corporation issued three checks in the aggregate amount of P31,663.88
had an inherent right to vote as an incident of their ownership of
payable to the plaintiff and drawn against Citibank. That former officers of
corporate stock, although they always knew that JG Ricafort was G.R. No. 165146, August 12, 2015
the plaintiff corporation headed by Saturnino G. Belen, Jr., without any
the real and beneficial owner(under the Nominee Agreements) authority whatsoever from the plaintiff deposited the above-mentioned
and that he himself attended the stockholders' meeting and checks to the current account of his conduit corporation, Intervest SECURITIES AND EXCHANGE COMMISSION AND VERNETTE G.
voted as their "proxy" the shares in their names. Merchant Finance which the latter maintained with the defendant bank UMALI, Petitioners, v. BAGUIO COUNTRY CLUB
3) It defies reason, too, that he could not have informed his wife and CORPORATION, Respondent.
children, who live in the same house with him, of the scheduled Although the checks were clearly payable to the plaintiff corporation and
crossed on their face and for payees account only, defendant bank [G.R. N0. 165209]
ASM.
Lack/late notice accepted the checks to be deposited to the current account of Intervest
and thereafter presented the same for collection from the drawee bank RAMON K. ILUSORIO AND ERLINDA K.
1) As shown in the Affidavit dated October 13, 2011 of San Juan,
which subsequently cleared the same thus allowing Intervest to make use ILUSORIO, Petitioners, v. BAGUIO COUNTRY CLUB
NADECOR's messenger, he mailed the notices for the August CORPORATION, Respondent.
of the funds to the prejudice of the plaintiff.
15, 2011 ASM to the petitioners' address at the Ortigas Post
Office on August 11, 2011, four days prior to the ASM. Meantime, the same corporation, i.e., Premium, but this time represented On December 17, 1998, the Securities and Exchange
2) under Article I, Section 3 of NADECOR's Amended By-Laws, by Siguion Reyna, Montecillio and Ongsiako Law Office as counsel, filed a Commission (SEC) approved the amended by-laws submitted by the
what is required is the mailing out of notices by registered mail at motion to dismiss on the ground that the filing of the case was without Baguio Country Club Corporation (BCCC). Article 5, Section 2 thereof
least three days before the ASM – “xxx… or by mailing it … at authority from its duly constituted board of directors as shown by the reads:
least three days before such meeting … Failure to give excerpt of the minutes of the Premiums board of directors meeting.
notice of annual meeting, or any irregularity in such notice, Election and Term. The Board of Directors
ISSUE:
shall not affect the validity of such annual meeting or of any shall be elected at the regular meetings or
proceedings at such meeting ..xxx” stockholders and shall hold office for two (2) years and
Whether or not Premium through Atty. Dumadag was authorized to file the
3) The shorter notice of three days instead of two weeks for until their successors are elected and qualified
present case
stockholders' regular or special meeting is clearly allowed under
Section 50 of the Corporation Code “…That written notice of RULING:
On September 27, 2002, Atty. Manuel R. Singson, acting for and
regular meetings shall be sent to all stockholders or members of No. Petitioner, through the first set of officers, viz., Mario Zavalla, Oscar in behalf of Ramon K. Ilusorio and Erlinda Ilusorio (the Ilusorios) requested
record at least two (2) weeks prior to the meeting, unless a Gan, Lionel Pengson, Jose Ma. Silva, Aderito Yujuico and Rodolfo Millare, the SEC, via a letter-complaint, to compel BCCC to hold the annual
different period is required by the by-laws…. Special presented the Minutes of the meeting of its Board of Directors held on April election of the board of directors for 2002 in view of the nullity of the above-
meetings of stockholders or members shall be held at any time quoted provision in the amended by-laws.4 He informed the SEC that
1, 1982, as proof that the filing of the case against private respondent was
sometime in 2001, a stockholder of BCCC requested for the opinion of the
deemed necessary or as provided in the by-laws: Provided, authorized by the Board. However, While the Minutes of the Meeting of the SEC on the validity of the amendment, particularly the two (2) year term of
however, That at least one (1) week written notice shall be sent Board on April 1, 1982 states that the newly elected officers for the year the board of directors; and that in response, the SEC opined that the
to all stockholders or members, unless otherwise provided in 1982 were Oscar Gan, Mario Zavalla, Aderito Yujuico and Rodolfo Millare, amendment increasing the term of office to two (2) years is contrary to law,
the by-laws. Notice of any meeting may be waived, petitioner failed to show proof that this election was reported to the SEC. In particularly Section 23 of the Corporation Code which limits the term of
expressly or impliedly, by any stockholder or member….” fact, the last entry in their General Information Sheet with the SEC, as of office to only one (1) year.
1986 appears to be the set of officers elected in March 1981.
On November 13, 2002, the SEC, through the Corporation
We agree with the finding of public respondent Court of Appeals, that in the Registration and Monitoring Department, issued an Order6 ruling that
absence of any board resolution from its board of directors the authority to Article 5, Section 2 of the amended by-laws of BCCC violates Section 23
10. PREMIUM MARBLE RESOURCES, INC., vs. THE COURT OF of the Corporation Code on the term of office of members of the board of
APPEALS and INTERNATIONAL CORPORATE BANK act for and in behalf of the corporation, the present action must necessarily
directors and should be amended to conform to the rules. The SEC also
G.R. No. 96551 November 4, 1996 fail. The power of the corporation to sue and be sued in any court is lodged ordered BCCC to conduct the annual election of members of the board.
with the board of directors that exercises its corporate powers. Thus, the
FACTS: issue of authority and the invalidity of plaintiff-appellants subscription which On March 18, 2003, Ramon Ilusorio, as stockholder of BCCC,
formalized Atty. Singson 's letter-request through a petition with the
10
Delegation of Authority to Corporate Officers | Doctrine of Apparent Authority
SEC.9 He alleged among others, that the BCCC refused to conduct a The OSG argues that the matter at hand is not an intra-corporate its regulatory powers motu propio, without the complaint or initiative of
stockholders' meeting for the election of board members, and that the dispute. The authority to accept, reject, or order the modification or anyone, although it may exercise its regulatory powers upon the complaint
individuals claiming to be officers of the BCCC used their positions to amendment of BCCC's by-laws and direct the performance of an act or initiative of private parties.
manipulate stockholders' meeting to their advantage and harass those relative thereto is administrative in nature and does not partake of a quasi-
In its Comment, BCCC further argues that inasmuch as the SEC
who have opposed them. The petition prayed for the SEC to call and judicial function.
is powerless to nullify BCCC's by-laws, any act in connection thereof, such
conduct, under its control and supervision, a stockholder's meeting in the
as the calling a meeting for the purpose of an election is also necessarily
BCCC for the election of the members of the board of directors.
void.48 Finally, BCCC states that there was no need for the CA to discuss
In its Comment, BCCC claims that it was subjected to grave and
In its August 15, 2003 Order,11 the SEC observed that the only the other collateral issues raised by the Ilusorios, since in any case, all
oppressive acts by the SEC when it issued a series of patently void orders.
issue that must be resolved is whether or not the SEC can call a proceedings before the SEC are null and void.
These orders were not issued in the exercise of SEC's regulatory powers,
stockholders' meeting for the purpose of conducting an election of the
but rather in the nature of quasi-judicial powers, which the SEC no longer Meanwhile, the Ilusorios tiled their Urgent Manifestation and
BCCC board of directors.12 It ruled that under the Corporation Code, it has
possesses in view of the transfer of said quasi-judicial power to the RTCs Motion dated October 28, 2004, stating that the Corporate Secretary of
the power to call such a meeting and to order the conduct of an election of
as provided in RA No. 8799. BCCC also maintains that there is an intra- BCCC issued a Notice of Annual Meeting of Stockholders, said meeting to
new board members in the BCCC.13 Thus it ordered, among others, the
corporate dispute because the unverified letter and the petition in the SEC be held on November 11, 2004. According to the Ilusorios, the scheduled
calling and conduct of a stockholders meeting for the election of the
alleged that Erlinda and Ramon Ilusorio are stockholders. stockholders' meeting would prejudice the instant petition. On November
members of the board under the control and supervision of the SEC.
10, 2004, the Court issued a resolution directing the parties to maintain
According to the BCCC, the SEC's authority to order the conduct
On September 26, 2003, BCCC filed a petition15 for certiorari the status quo.
of an election of directors is limited to situations when there is no person
and prohibition with the Court of Appeals (CA). It added that the matter is
authorized to call a meeting or if no meeting is being called in contravention
within the exclusive jurisdiction of the trial court, being an intra-corporate Nonetheless, BCCC and its counsel were made aware of the status
of the by-laws. In this case, however, the SEC is aware and is always
dispute. quo order only in the afternoon of November 11, 2004; way after it
notified by BCCC of its regular and annual stockholders' meeting
conducted the stockholders' meeting in the morning of the same date.
In its Decision16 dated March 26, 2004, the CA granted the conducted by authorized officers of the BCCC. In addition, there is a need
BCCC sought reconsideration of the status quo order52 but its motion was
petition, set aside the SEC's Orders and dismissed the letter-complaint of for a valid petition for the holding of a stockholders' meeting filed by a valid
denied by the Court on December 15, 2004.
Ramon Ilusorio. According to the CA, the matter between the parties is an stockholder before the SEC may compel the same.
intra corporate dispute, being between a stockholder and the corporation
In their Reply, the Ilusorios maintain that the SEC's act of calling
itself, as well as other stockholders, particularly those occupying positions On January 10, 2005, we ordered the consolidation of the two (2) cases.
for an election is not exercise of its quasi-judicial power, but rather its
in the board of directors. Further, the SEC's jurisdiction over all cases
regulatory power against a corporation to ensure compliance with the
enumerated under Section 5 of Presidential Decree No. 902-A, including
Corporation Code. Moreover, they clarify that contrary to BCCC's On July 19, 2005, BCCC filed a Motion for Leave to Admit Manifestation
intra-corporate controversies has been transferred to the appropriate
insistence that there is an intra-corporate dispute, there is in fact no dispute with Manifestation, stating that in a meeting held on June 29, 2005, the
Regional Trial Courts by virtue of Republic Act (RA) No. 8799 (The
at all, since they are not asserting any right against the respondent, nor board of directors of BCCC approved the amendment to its bylaws,
Securities Regulation Code). Thus, the dispute pertains to the regular
seeking any positive relief for their personal benefit. For all intents and modifying the term of its directors from two (2) years to one ( 1) year.
courts.
purposes, the controversy is limited to the non-compliance of BCCC's by- According to the BCCC, the amendment was made "to reciprocate the
On September 1, 2004, the CA denied the SEC's motion for laws to the Corporation Code. humble gesture" of the SEC who admitted that the approval of the two-year
reconsideration for lack of merit.21 Hence, these petitions. term of the BCCC's board of directors was an honest and inadvertent
Defending its actions, the SEC maintained that it merely
mistake. BCCC prayed that in view of the amendment of BCCC's by-laws
implemented the statutory term of office provided in Section 23 of the
to reflect a term of one year for its board of directors, the primary legal
Corporation Code. The law being clear and categorical, there is no room
G.R. No. 165146 contention of the petitioners should now be deemed moot and academic.
for interpretation nor construction; there is only room for application. The
We denied the manifestation due to BCCC's failure to attach its annexes.
SEC clarifies that calling for a meeting and ordering the conduct of
In G.R. No. 165146, the petitioner SEC, through the Office of the Solicitor elections is necessary in view of the expired term of the members of the
On September 21, 2005, BCCC filed another Motion for Leave to Admit
General (OSG), BCCC board of directors; hence there is no one authorized to call a
Manifestation with Manifestation, stating that on August 8, 2005 the SEC
meeting except the SEC
According to the OSG, the one (1) year term rule for members issued a certificate approving BCCC's amended by-laws (modifying the
of the board of directors is mandatory, and cannot be shortened or term of office of its directors from two [2] years to one [1] year). It added
extended by agreement of the parties or by those interested in the position, that the SEC also approved the amendments to BCCC's articles of
G.R. No. 165209
thus BCCC's amended by-laws granting its board of directors a two (2) incorporation59 extending its corporate life and converting BCCC from a
year term is void, notwithstanding the SEC's prior approval.23 Pursuant to In G.R. No. 165209, The Ilusorios maintain that the SEC is stock to a non-stock corporation. BCCC reiterated that the SEC's approval
Section 5 of the Securities Regulation Code, the SEC has the authority to empowered under RA No. 8799 (The Securities Regulation Code) to call of its amended by-laws has caused the petition to be moot and academic.
compel BCCC to amend its by-laws to conform with Section 23 of the for a meeting for the conduct of an election, even if there are authorized
Corporation Code, and to impose sanction on the recalcitrant BCCC persons to call such a meeting.37 In any case, pursuant to the Corporation
officers and board members. Code and the Securities Regulation Code, the SEC Can act and exercise
11
Delegation of Authority to Corporate Officers | Doctrine of Apparent Authority
Banking on the same amended by-laws and articles of the signature of Robles as BUHAY president. On March 29, 2007, Robles prohibit a hold-over situation. As such, since no successor was ever
incorporation, the SEC tiled a Manifestation and Motion praying that the signed and filed a Certificate of Nomination of BUHAYs nominees for the elected or qualified, Robles remained the President of BUHAY in a hold-
petition be considered terminated on the ground of mootness 2007 elections. over capacity.

For their part, petitioners Ramon and Erlinda Ilusorio maintain that the Earlier, however, or on March 27, 2007, petitioner Hans Christian Señeres, Authorities are almost unanimous that one who continues with the
amendment of the by-laws did not render the petition moot since the holding himself up as acting president and secretary-general of BUHAY, discharge of the functions of an office after the expiration of his or her legal
validity of the amendment is not the only subject matter of the assailed SEC also filed a Certificate of Nomination with the COMELEC, and including termno successor having, in the meantime, been appointed or chosenis
Order.62 They claim that they also raised other issues63 in their himself in the nominees. Señeres filed with the COMELEC a Petition to commonly regarded as a de facto officer, even where no provision is made
memorandum before the CA. Further, even assuming, without conceding Deny Due Course to Certificates of Nomination. He alleged that he was by law for his holding over and there is nothing to indicate the contrary. By
that the petition covers only the validity of the amendment extending the the acting president and secretary-general of BUHAY, having assumed fiction of law, the acts of such de facto officer are considered valid and
that position since August 17, 2004 when Robles vacated the position. effective. So it must be for the acts of Robles while serving as a hold-over
term of directors to two (2) years, the amendment restoring the term to one
Pushing the point, Señeres would claim that the nominations made by Buhay President.
(1) year did not render the petition moot because the fundamental issue Robles were, for lack of authority, null and void owing to the expiration of
decided by the CA is the jurisdiction of the SEC in issuing the assailed SEC the latter’s term as party president. However, the National Council of
Order. BUHAY adopted a resolution expelling Señeres as party member for his
act of submitting a Certificate of Nomination for the party.
ISSUE: Whether or not the Commission can call a stockholders' meeting
for the purpose of conducting an election of the BCCC board of directors. 3. BERNAS, et. al. v. CINCO, et. al.
Robles filed an Urgent Motion to Declare Null and Void the Certificate of G.R. Nos. 163356 & 163357
Nomination and Certificates of Acceptance filed by Hans Christian M. CINCO, et. al. v. BERNAS, et.al.
RULING:
Señeres. G.R. Nos. 163368 & 163368
The petitions have been rendered moot by the 2005 amendment July 10, 2015
On July 2007, the COMELEC issued two resolutions proclaiming BUHAY
of the by-laws. The validity of the two (2) year term provision and the calling
as a winning party-list organization for the May 2007 elections entitled to FACTS:
of meeting for the election of members of the board of directors to replace
three (3) House seats. This was followed by the en banc COMELEC of a
those holding a two (2) year term should no longer be in issue. Resolution recognizing and declaring Robles as the president of BUHAY Makati Sports Club (MSC) is a domestic corporation duly organized for the
and, as such, was the one duly authorized to sign documents. Explaining social, cultural, recreational and athletic activities among its members.
A moot and academic case is one that ceases to present a
its action, COMELEC stated that since no party election was held to Bernas, Cheng, Africa, Maramara, Frondoso, Macrohon and Lim (Bernas
justiciable controversy by virtue of supervening events, so that a
replace Robles as party president, then he was holding the position in a Group) were the members of the BOD of MSC whose terms were to expire
declaration thereon would be of no practical use or value. hold-over capacity. either on 1998 and 1999.
As can be gleaned from the SEC's Order, the calling of the
Aggrieved, petitioner filed the instant petition. Because of rumored anomalies in handling the corporate funds by the
meeting for the conduct of an election was made to rectify the inadvertent
approval of the two (2) year term for the members of the board. With the Bernas Group, stockholders composing of at least 100 shares sought the
ISSUE: assistance of the MSC Oversight Committee (MSCOC), who are
return of the one (1) year term, there is no more actual controversy that
composed of the past presidents of MSC. The request was granted and
warrants the exercise of our judicial power. An actual case or controversy Is Robles the duly authorized representative of BUHAY? the MSCOC called for Special Stockholders Meeting and sent notices to
exists when there is a conflict of legal rights or an assertion of opposite legal
stockholders thereto.
claims, which can be resolved on the basis of existing law and RULING:
jurisprudence. For failure of the Bernas Group to secure an injunction before the SEC,
Yes. Cinco, Librea and Pardo (Cinco Group) were elected new members of the
12. Dr. Hans Christian M. SEÑERES v. COMELEC BOD during the questioned 17 December 1997 Special Stockholders
G.R. No. 178678 As a general rule, officers and directors of a corporation hold over after the Meeting. Thus, the Bernas Group were removed from office.
April 16, 2009 expiration of their terms until such time as their successors are elected or
appointed. The holdover doctrine accords validity to what would Bernas Group filed an action for nullification of the 1997 Special Meeting
FACTS: otherwise be deemed as dubious corporate acts and gives continuity to a before the SEC on the ground that it was improperly called (SEC Case
corporate enterprise in its relation to outsiders. This is the analogical 5840).
In 1999, private respondent Robles was elected president and chairperson situation obtaining in the present case. The voting members of BUHAY
of BUHAY, a party-list group. The constitution of BUHAY provides for a duly elected Robles as party President in October 1999. And although his Arguments:
three-year term for all its party officers, without re-election. BUHAY regular term as such President expired in October 2002, no election was
participated in the 2001 and 2004 elections, with Robles as its held to replace him and the other original set of officers. Further, the Bernas Group: They cite Section 28 of the Corporation Code (CC) arguing
president. As in the past two elections, the manifestation to participate bore constitution and by-laws of BUHAY do not expressly or impliedly that the authority to call a meeting lies with the Corporate Secretary and
12
Delegation of Authority to Corporate Officers | Doctrine of Apparent Authority
not with the MSCOC which functions merely as an oversight committee non-stock corporation, by a vote of at least two-thirds
and is not vested with the power to call corporate meetings. Hence, their (2/3) of the members entitled to vote: Provided, That Illegal acts of a corporation which are contrary to law, morals or public
removal and the election of the new ones should be nullified. such removal shall take place either at a regular order, or public policy are, like similar transaction between individuals, are
meeting of the corporation or at a special meeting void. They cannot serve as basis of a court action nor acquire validity by
Cinco Group: They insisted that the meeting is sanctioned by the CC and called for the purpose, and in either case, after performance, ratification or estoppel. While, those which are merely ultra
the MSC By-laws. They cite Section 25 of the same arguing that the previous notice to stockholders or members of the vires, or those which are not illegal or void ab initio, are merely voidable
Corporate Secretary is merely authorized to issue notices of meetings and corporation of the intention to propose such removal and may become binding and enforceable when ratified by the
nowhere does it state that such authority solely belongs to him. Moreover, at the meeting. A special meeting of the stockholders stockholders.
it is useless to request a call thru the Secretary because he refused to call or members of a corporation for the purpose of
a special stockholders meeting. removal of directors or trustees, or any of them, must The 17 December 1997 Meeting belongs to the former, that it is void ab
be called by the secretary on order of the president or initio and cannot be validated/ratified.
Meanwhile, the Cinco Group also found that Bernas was guilty of on the written demand of the stockholders
irregularities, hence, the new BOD expelled him from the club and also sold representing or holding at least a majority of the Hence, the 1997 Meeting cannot have any legal effect. The removal and
his shares at public auction. outstanding capital stock, or, if it be a non-stock election is void, and since the Cinco Group has no legal right to sit in the
corporation, on the written demand of a majority of the BOD, the act of expelling Bernas as member and the selling of his shares,
Prior to the resolution of the SEC Case 5840, an Annual Stockholder’s members entitled to vote. Should the secretary fail or are likewise invalid.
Meeting was held on 20 April 1998. It was attended by 1,017 stockholders refuse to call the special meeting upon such demand
representing 2/3 of the outstanding shares. They ratified, among others, or fail or refuse to give the notice, or if there is no 3. The Cinco Group cannot invoke the application of de
the calling and holding of the 1997 Special Stockholders Meeting, including secretary, the call for the meeting may be addressed facto officership doctrine. The proper recourse is to file
the removal of Bernas Group and the election of the Cinco Group. directly to the stockholders or members by any a petition to the SEC.
stockholder or member of the corporation signing the
Due to the filing of several petitions for and against the removal of the demand. Notice of the time and place of such meeting, The doctrine cannot justify the action taken after the invalid election since
Bernas Group, the SEC En Banc supervised the holding of the 1999 as well as of the intention to propose such removal, the operation of the principle is limited to third person who were originally
Annual Stockholders Meeting. Once again, they ratified and confirmed the must be given by publication or by written notice not part of the corporation but became such by reason of voting of
holding of the 1997 Special Stockholders Meeting. prescribed in this Code. Removal may be with or government-sequestered shares.
without cause: Provided, That removal without cause
It was also the same in the 2000 Annual Stockholders Meeting. may not be used to deprive minority stockholders or The case would have been different if the petitioning stockholders went
members of the right of representation to which they directly to the SEC and sought its assistance to call a meeting citing the
ISSUES: may be entitled under Section 24 of this Code.” refusal of the Secretary to call a meeting.

1. Is the 1997 Special Stockholders Meeting initiated by the Further, under MSC by-laws, only the President and the BOD are Section 50 of the Corporation Code provides:
MSCOC valid? authorized to call a special meeting. If they refuse, fail or neglect to call a
2. If not valid, did the subsequent ratifications made during the meeting, then stockholders who composed of at least 100 shares, upon “Regular and special meetings of stockholders or
Annual Stockholders Meeting make it valid? written request, may file a petition to call a special meeting. members. - Regular meetings of stockholders or
3. Can the Cinco Group invoke the de facto officership doctrine? If members shall be held annually on a date fixed in the
not, what is the proper recourse to remove the BOD? In this case, the 1997 Special Stockholders Meeting was called neither by by-laws, or if not so fixed, on any date in April of every
4. Are the 1998 and 1999 Annual Stockholders Meeting valid? the President nor the BOD but by the MSCOC. It is nowhere in the law or year as determined by the board of directors or
5. Can the Bernas Group invoke the hold-over principle? MSC By-Laws that the MSCOC is authorized to call for a meeting, rather it trustees: Provided, That written notice of regular
is solely vested on the President and the BOD. meetings shall be sent to all stockholders or members
RULING: of record at least two (2) weeks prior to the meeting,
Moreover, relative to the powers of the BOD, it is nowhere can it be unless a different period is required by the by-laws.
1. No, the 1997 Special Stockholders Meeting was not gathered that the Oversight Committee is authorized to step in wherever
valid. there is breach of fiduciary duty and call a special meeting for the purpose Special meetings of stockholders or members shall be
of removing the existing offices and electing their replacements even if held at any time deemed necessary or as provided in
Section 28 of the Corporation Code states that: such call was made upon the request of shareholders. the by-laws: Provided, however, That at least one (1)
week written notice shall be sent to all stockholders or
“Any director or trustee of a corporation may be 2. No, the subsequent ratifications made by the members, unless otherwise provided in the by-laws.
removed from office by a vote of the stockholders stockholders did not cure the defect.
holding or representing at least two-thirds (2/3) of the Notice of any meeting may be waived, expressly or
outstanding capital stock, or if the corporation be a The defect was in the authority of the persons who called for it. impliedly, by any stockholder or member.
13
Delegation of Authority to Corporate Officers | Doctrine of Apparent Authority

Whenever, for any cause, there is no person 1. The 1997 Special Meeting was prematurely or invalidly called by
authorized to call a meeting, the Secretaries and the Cinco Group. Thus, it had no legal effect and did not effectively remove Defendant’s defense: the stipulation in the contract suspending the
Exchange Commission, upon petition of a stockholder the Bernas Group as directors of the Makati Sports Club. power to sell the stock referred to therein is an illegal stipulation, is in
or member on a showing of good cause therefor, may 2. The expulsion of Bernas as a member and the sale of his shares restraint of trade and, therefore, offends public policy.
issue an order to the petitioning stockholder or in a public auction is void because the Cinco Group was not authorized to
member directing him to call a meeting of the perform corporate acts as their election as new BOD is void.
corporation by giving proper notice required by this 3. The ratification of the removal of Bernas Group, the expulsion of
Code or by the by-laws. The petitioning stockholder or Bernas and the sale of his shares in the regular 1998, 1999 and 2000 ISSUE:
member shall preside thereat until at least a majority Annual Meetings, is void because they were not the proper party to cause
Was the contract violated by Defendant Fox or not?
of the stockholders or members present have been the ratification.
chosen one of their number as presiding officer. (24, 4. However, all actions of the Cinco Group and stockholders during
26)” the regular 1998, 1999 and 2000 Annual Meetings, including the election
of the Cinco Group as BOD after the expiration of the term of office of RULING:
Given the broad administrative and regulatory powers of the SEC, the Bernas Group, are valid.
Cinco Group cannot claim that it was left without recourse after the The contract was violated by the Defendant Fox.
Secretary refused to heed the demands. Even if it is true that the Secretary
refused, the remedy of the Stockholders would have been to file a petition 14. LAMBERT v. FOX
to the SEC.
Interpretation and construction should by the instruments last resorted to
G.R. No. L-7991 January 29, 1914 by a court in determining what the parties agreed to. Where the language
4. Yes, the subsequent Annual Meetings are valid.
used by the parties is plain, then construction and interpretation are
First, the 1998 Annual Meeting was valid because it was sanctioned by FACTS: unnecessary and, if used, result in making a contract for the parties.
Section 8 of the by-laws:
Early in 1911 the firm known as John R. Edgar & Co., engaged in the retail
“SEC. 8. Annual Meetings. The annual meeting of book and stationery business, found itself in such condition financially that
its creditors, including the plaintiff and the defendant, together with many Construction and interpretation come only after it has been demonstrated
stockholder shall be held at the Clubhouse on the third
Monday of April of every year x x x” others, agreed to take over the business, incorporate it and accept stock that application is impossible or inadequate without them. They are the
therein in payment of their respective credits. This was done, the plaintiff very last functions which a court should exercise. The majority of the law
Second, the 1999 Annual Meeting was also valid because in addition to and the defendant becoming the two largest stockholders in the new need no interpretation or construction. They require only application, and if
the fact that it was sanctioned by Section of the MSC by-laws, such corporation called John R. Edgar & Co., Incorporated. A few days after the there were more application and less construction, there would be more
meeting was supervised by the SEC in the exercise of its regulatory and incorporation was completed plaintiff and defendant entered into na stability in the law, and more people would know what the law is."
administrative powers to implement the CC. agreement of mutually and reciprocally agree not to sell, transfer, or
otherwise dispose of any part of their present holdings of stock in said John What we said in that case is equally applicable to contracts between
Hence, it gave rise to the presumption that the officers who won the elected R. Edgar & Co. Inc., till after one year. persons. In the case at bar the parties expressly stipulated that the contract
during the Annual Meeting can be rightfully considered as de jure officers.
should last one year. No reason is shown for saying that it shall last only
As de jure officials, they can lawfully perform acts that are within the scope
Defendant Fox sold his stock to E. C. McCullough. This sale was made by nine months. Whatever the object was in specifying the year, it was their
of the business of the corporation except ratification of actions that are void
ab initio. the defendant against the protest of the plaintiff and with the warning that agreement that the contract should last a year and it was their judgment
he would be held liable under the contract. and conviction that their purposes would not be subversed in any less time.
5. No, the Bernas Group cannot use the hold-over What reason can give for refusing to follow the plain words of the men who
principle. made the contract? We see none.
The trial court decided the case in favor of the defendant upon the ground
that the intention of the parties as it appeared from the contract in question
Considering that a new set of officers were duly elected during the 1998 The suspension of the power to sell has a beneficial purpose, results in the
was to the effect that the agreement should be good and continue only until
and 1999 Annual Meetings, the Bernas Group cannot be permitted to use protection of the corporation as well as of the individual parties to the
the holdover principle as a shield to perpetuate office, unless they were the corporation reached a sound financial basis, and that that event having
occurred some time before the expiration of the year mentioned in the contract, and is reasonable as to the length of time of the suspension.
reelected. They had no right to hold-over because they fail to perform the
duty incumbent to them. contract, the purpose for which the contract was made and had been
fulfilled and the defendant accordingly discharged of his obligation
To summarize everything: thereunder. The complaint was dismissed upon the merits.
14
Delegation of Authority to Corporate Officers | Doctrine of Apparent Authority
from January 1996 to December 1998, petitioners received their
Section 23 of the Corporation Code states that term of the corresponding RATA in the total amount of P1,565,000.00.
members of the board of directors shall be only for one year; their term
expires one year after election to the office. Thus, when an incumbent The then PICCI Corporate Auditor addressed to petitioner
15. VALLE VERDE COUNTRY CLUB v. VICTORIA AFRICA member of the board of directors continues to serve in a holdover capacity, Villanueva disallowing in audit the payment of petitioners’ Representation
GR No. 151969, September 4, 2009 it implies that the office has a fixed term, which has expired, and the and Transportation Allowance (RATA) in the total amount of
incumbent is holding the succeeding term. P1,565,000.00, and directing them to settle immediately the said
Facts: disallowances, due to the following reasons: (a) As to petitioner Villanueva,
On February 27, 1996, during the Annual Stockholders' Meeting The word "term" is defined as the time during which the officer there was double payment of RATA to her as member of the PICCI Board
of petitioner Valle Verde Country Club, Inc., the following were elected as may claim to hold the office as of right, and fixes the interval after which the and as OIC of PICCI, which was in violation of Section 8, Article IX-B of the
members of the VVCC Board of Directors: Dinglasan, Makalintal, several incumbents shall succeed one another. Term is distinguished from 1987 Constitution and, moreover, Compensation Policy Guideline No. 6
Francisco Ortigas III, Victor Salta, Amado M. Santiago, Jr., Fortunato Dee, tenure in that an officer's "tenure" represents the term during which the provides that an official already granted commutable RATA and
Augusto Sunico, and Ray Gamboa. incumbent actually holds office. designated by competent authority to perform duties in concurrent capacity
The term of office is not affected by the holdover. The term is as OIC of another position whether or not in the same agency and entitled
In the years 1997, 1998, 1999, 2000, and 2001, however, the fixed by statute and it does not change simply because the office may have to similar benefits, shall not be granted said similar benefits, except where
requisite quorum for the holding of the stockholders' meeting could not be become vacant, nor because the incumbent holds over in office beyond said similar allowances are higher in rates than those of his regular position,
obtained. Consequently, the above-named directors continued to serve in the end of the term due to the fact that a successor has not been elected in which case he may be allowed to collect the difference thereof; and (b)
the VVCC Board in a hold-over capacity. and has failed to qualify. As to her co-petitioners, there was double payment of RATA to them as
members of the PICCI Board and as officers of BSP, which was in violation
On September 1, 1998, Dinglasan resigned. In a meeting, the After the lapse of one year from his election as member of the of the Constitution and PICCI By-laws.
remaining directors, still constituting a quorum of VVCC's nine-member VVCC Board in 1996,
board, elected Roxas to fill in the vacancy created by the resignation of Makalintals term as well as those of other directors of VVCC board is The Director of the Corporate Audit Office I of COA affirmed the
Dinglasan. A year later, Makalintal also resigned. He was replaced by deemed to have already expired. That they continued to serve in the VVCC disallowance of the RATA received by petitioners in their capacity as
Ramirez, who was elected by the remaining members of the VVCC Board. Board in a holdover capacity cannot be considered as extending his term. Directors of the PICCI Board. He stated that except for per diems, the
This holdover period, however, is not to be considered as part of his term, PICCI By-Laws prohibits the payment of salary to directors in the form of
Respondent Africa a member of VVCC, questioned the election which, as declared, had already expired. compensation or reimbursement of expenses, based upon the principle
of Roxas and Ramirez as members of the VVCC Board with the SEC and expression unius est exclusio alterius (the express mention of one thing in
the Regional Trial Court. In his nullification complaint before the RTC, Africa When a vacancy is created by the expiration of a term, logically, a law means the exclusion of others not expressly mentioned). Neither can
alleged that the election of Roxas was contrary to Section 29. Sec. 29. there is no more unexpired term to speak of. Hence, Section 29 declares the payment of RATA be legally founded on Section 30 of the Corporation
Vacancies in the office of director or trustee. - Any vacancy occurring in the that it shall be the corporation's stockholders who shall possess the Code where it bears stressing that the directors of a corporation shall not
board of directors or trustees other than by removal by the stockholders or authority to fill in a vacancy caused by the expiration of a member's term. receive any compensation for being members of the board of directors,
members or by expiration of term, may be filled by the vote of at least a except for reasonable per diems. The power to fix the compensation which
majority of the remaining directors or trustees, if still constituting a quorum; the directors shall receive, if any, is left to the corporation, to be determined
otherwise, said vacancies must be filled by the stockholders in a regular or in its by-laws or by the vote of stockholders. The PICCI By-Laws allows
special meeting called for that purpose. A director or trustee so elected to 6. GABRIEL C. SINGSON et al. v. COMMISSION ON AUDIT only the payment of per diem to the directors. Thus, the BSP board
fill a vacancy shall be elected only for the unexpired term of his predecessor G.R. No. 159355 resolution granting RATA of P1,500.00 to petitioners violated the PICCI By-
in office. xxx. August 9, 2010 Laws. Although MB Resolution No. 15, as amended, would have the effect
of amending the PICCI By-laws, and may render the grant of RATA valid,
Africa claimed that a year after Makalintal's election as member Facts: such amendment, however, had no effect because it failed to comply with
of the VVCC Board in 1996, as well as those of the other members of the The Philippine International Convention Center, Inc. (PICCI) is a the procedural requirements set forth under Section 48 of the Corporation
VVCC Board should be considered to have already expired. Thus, government corporation whose sole stockholder is the Bangko Sentral ng Code.
according to Africa, the resulting vacancy should have been filled by the Pilipinas (BSP). Petitioner Araceli E. Villanueva was then a member of the COA confirmed the Notice of Disallowance and ruled that
stockholders in a regular or special meeting called for that purpose, and PICCI Board of Directors and Officer-in-Charge (OIC) of PICCI together petitioners’ receipt of the P1, 500.00 RATA from the BSP for every meeting
not by the remaining members of the VVCC Board, as was done in this with his co-petitioners who were then members of the PICCI BOD and they attended as members of the PICCI Board of Directors was not valid.
case. SEC and RTC ruled in favor of Africa. officials of the BSP. By virtue of the PICCI By-Laws, petitioners were Petitioners contend that since PICCI was incorporated with SEC and has
authorized to receive P1,000.00 per diem each for every meeting attended. no original charter, it should be governed by Section 30 of the Corporation
Issue: Can the remaining directors in fill in a vacancy caused by the Pursuant to its Monetary Board (MB) Resolution No. 15, as amended, the Code. According to petitioners, their receipt of RATA as directors of PICCI
resignation of a hold-over director? BSP MB granted additional monthly RATA, in the amount of P1,500.00, to was sanctioned by PICCI’s sole stockholder, BSP, per MB Resolution No.
each of the petitioners, as members of the BOD of PICCI. Consequently, 15. Respondent counters that said provision does not apply to petitioners
Ruling: No. as the PICCI By-laws provides that the compensation of the members of
15
Delegation of Authority to Corporate Officers | Doctrine of Apparent Authority
the PICCI Board of Directors shall be given only through per diems and not limited only to per diem of P1,000.00 for every meeting attended, by virtue 1. Are the respondents entitled to compensation?
RATA. Moreover, respondent maintains that there is double payment of of the PICCI By-Laws. In the same vein, it also clarifies that there has been
RATA which violated National Compensation Circular (NCC) No. 67, since no double compensation despite the fact that, apart from the RATA they Yes. There is no argument that directors or trustees, as the case may be,
petitioners membership in the PICCI Board is a mere adjunct of their have been receiving from the BSP, petitioners have been granted the are not entitled to salary or other compensation when they perform nothing
positions as BSP officials; petitioners were already drawing RATA from RATA of P1,500.00 for every board meeting they attended, in their capacity more than the usual and ordinary duties of their office. This proscription,
their mother agencies and, hence, their receipt of RATA from PICCI was as members of the BOD of PICCI, pursuant to MB Resolution No. 15 of however, against granting compensation to directors/trustees of a
without legal basis and constituted double compensation of RATA; that the BSP. In this regard, the Court took into consideration the good faith of corporation is not a sweeping rule. The unambiguous implication in the law
double compensation refers to two sets of compensations for two different petitioners. As petitioners believed in good faith that they are entitled to the is that members of the board may receive compensation, in addition to
offices held concurrently by one officer; and that while there is no general RATA of P1,500.00 for every board meeting they attended, in their capacity reasonable per diems, when they render services to the corporation in a
prohibition against holding two offices which are not incompatible, when an as members of the BOD of PICCI, pursuant to MB Resolution No. 15, the capacity other than as directors/trustees. In the case at bench, Resolution
officer accepts a second office, he can draw the salary attached to such Court sees no need for them to refund their RATA respectively, in the total No. 48, s. 1986 granted monthly compensation to private respondents not
second office only when he is specifically authorized by law which does not amount of P1,565,000.00, covering the period from 1996-1998. in their capacity as members of the board, but rather as officers of the
exist in the present case. corporation, more particularly as Chairman, Vice-Chairman, Treasurer and
Secretary of Western Institute of Technology
Issue: 17. WESTERN INSTITUTE V SALAS
Was there double compensation of RATA to the petitioners? G.R. No. 113032. August 21, 1997 The Articles of Incorporation of the Panay Educational Institution, Inc., now
the Western Institute of Technology, Inc. provides that the officers of the
Held: FACTS: corporation shall receive such compensation as the Board of Directors
NONE. may provide. The grant of compensation or salary to the accused in their
Private respondents: Ricardo, Salvador, Soledad, Antonio (lahat
capacity as officers of the corporation is authorized by both the Articles of
In all cases, commutable and reimbursable RATA shall be paid Salas) are the majority stockholders and members of the board of trustees
Incorporation and the By-Laws of the Corporation. To state otherwise is to
from the amount appropriated for the purpose and other personal services of Western Institute, an educational institution. Petitioners the minority
depart from the clear terms of the said articles and by-laws
savings of the agency or project from where the officials and employees stockholders of WIT. A meeting was held on June 1, 1986; in said meeting,
covered under this Circular draw their salaries. No one shall be allowed to the Board of Trustees passed Resolution No. 48, s. 1986, granting monthly
compensation to the private respondents as corporate officers retroactive 2. Was there falsification?
collect RATA from more than one source. National Compensation Circular
(NCC) No. 67 seeks to prevent the dual collection of RATA by a national June 1, 1985.
official from the budgets of more than one national agency. It is No. The prosecution omitted to submit the complete minutes of the regular
In 1991, petitioners filed a complaint against private respondents for meeting of the Board of Trustees on March 30, 1986; it only presented the
emphasized that the other source referred to in the prohibition is another
falsification of a public document and for estafa. The charge for falsification 5th page. Had the complete minutes consisting of five (5) pages, been
national agency. Stated otherwise, when a national official is on detail with
of public document was anchored on the private respondents submission submitted, it can readily be seen and understood that Resolution No. 48,
another national agency, he should get his RATA only from his parent
of WITs income statement for the fiscal year 1985-1986 with the SEC Series of 1986 giving compensation to corporate officers, was indeed
national agency and not from the other national agency he is detailed to.
reflecting therein the disbursement of corporate funds for the taken up and passed on March 30, 1986. This evidence by implication
compensation of private respondents based on Resolution No. 4, series of presented cannot prevail over the Minutes and cannot ripen into proof
Moreover, Section 6 of The New Central Bank Act defines that
1986, making it appear that the same was passed by the board on March beyond reasonable doubt.
the powers and functions of the BSP shall be exercised by the BSP
Monetary Board. MB Resolution No. 15, as amended, are valid 30, 1986, when in truth, the same was actually passed on June 1, 1986, a
corporate acts of petitioners that became the bases for granting date not covered by the corporations fiscal year 1985-1986 (beginning May 3. Did respondents commit estafa?
them additional monthly RATA of P1,500.00, as members of the 1, 1985 and ending April 30, 1986).The charge for estafa was anchored
Board of Directors of PICCI. The RATA is distinct from salary (as a on the argument that respondents disbursed the corporations funds to No. The grant of compensation or salary to the respondents is authorized
form of compensation). Unlike salary which is paid for services themselves which is without legal basis. Petitioners maintain that this grant by both the Articles of Incorporation and the By-Laws of the Corporation.
rendered, the RATA is a form of allowance intended to defray of compensation to private respondents is proscribed under Section 30 of To state otherwise is to depart from the clear terms of the said articles and
expenses deemed unavoidable in the discharge of office. Hence, the the Corporation Code. Thus, private respondents are obliged to return by-laws.
these amounts to the corporation with interest.
RATA is paid only to certain officials who, by the nature of their offices, incur
representation and transportation expenses. Indeed, aside from the The trial court acquitted the respondents without imposing any civil 4. Is the case a derivative suit?
RATA that they have been receiving from the BSP, the grant of liability against the accused therein. Petitioners want to make private
P1,500.00 RATA to each of the petitioners for every board meeting respondents civilly liable despite their acquittal, hence this case. No. It is merely an appeal on the civil aspect of Criminal Cases
they attended, in their capacity as members of the Board of Directors filed. Among the basic requirements for a derivative suit to prosper is that
of PICCI, in addition to their P1,000.00 per diem, does not run afoul the minority shareholder who is suing for and on behalf of the corporation
the constitutional proscription against double compensation. must allege his complaint before the proper forum that he is suing on a
The Court upholds the findings of respondent that petitioners’ RULING:
derivative cause of action on behalf of the corporation and all other
right to compensation as members of the PICCI Board of Directors is shareholders similarly situated who wish to join. This is necessary to vest
16
Delegation of Authority to Corporate Officers | Doctrine of Apparent Authority
jurisdiction upon the tribunal in line with the rule that it is the allegations in Lingayen Gulf Electric Power Company is a domestic No.
the complaint that vests jurisdiction upon the court or quasi-judicial body corporation with an authorized capital stock of P300,000 divided into 3,000 As regards the compensation of President claimed by defendant
concerned over the subject matter and nature of the action. This was not shares with a par value of P100 per share. Irineo Baltazar appears to have and appellant, it is clear that he is not entitled to the same. The by-laws of
complied with by the petitioners either in their complaint. subscribed for 600 shares on account of which he had paid upon the the company are silent as to the salary of the President. And, while
organization of the corporation the sum of P15,000. After incorporation, the resolutions of the incorporators and stockholders provide salaries for the
defendant made further payments on account of his subscription, leaving general manager, secretary-treasurer and other employees, there was no
a balance of P18,500 unpaid for, which amount, the plaintiff now claims in provision for the salary of the President. On the other hand, other
18. CENTRAL COOP EXCHANGE v. CONCORDIO TIBE & CA this action. resolutions provide for per diems to be paid to the President and the
Jun 30, G.R. No. L-27972 On July 23, 1946, a majority of the stockholders of the directors of each meeting attended, P10 for the President and P8 for each
corporation, among them the herein defendant, held a meeting and director, which were later increased to P25 and P15, respectively. This
FACTS: adopted stockholders' resolution No. 17. By said resolution, it was leads to the conclusions that the President and the board of directors were
agreed upon by the stockholders present to call the balance of all unpaid expected to serve without salary, and that the per diems paid to them were
The petitioner is a national federation of farmers' cooperative subscribed capital stock as of July 23, 1946, the first 50 per cent payable sufficient compensation for their services. Furthermore, for defendant's
marketing associations, or FACOMAS, scattered throughout the country. within 60 days beginnning August 1, 1946, and the remaining 50 per cent several years of service as President and up to the filing of the action
Under its by- laws "The compensation, if any, and the per diems for payable within 60 days beginning October 1, 1946. The resolution also against him, he never filed a claim for salary. He thought of claiming it only
attendance at meetings of the members of the Board of Directors shall be provided, that all unpaid subscription after the due dates of both calls when this suit was brought against him
determined by the members at any annual meeting in special meeting of would be subject to 12 per cent interest per annum. Lastly, the resolution
the Exchange called for the purpose." In the annual stockholders‘ meeting provided, that after the expiration of 60 days' grace which would be on
it was resolved that the members of the Board of Directors attending the December 1, 1946, for the first call, and on February 1, 1947, for the 20. RIOSA v TABACO LA SUERTE CORP., GR 203786
second call, all subscribed stocks remaining unpaid would revert to the OCTOBER 23, 2013
CCE board meetings be entitled to actual transportation expenses plus the
corporation.
per diems of P30.00 and actual expenses while waiting. In this regard, Tibe
Lingayen Gulf wrote a letter to the defendant reminding him that J. MENDOZA
collected the said amounts however the petitioner refused to give on the the first 50 per cent of his unpaid subscription. The Baltazar requested to
ground that the resolutions are invalid. The trial court ruled in favor of Tibe. "kindly advise the company thru the undersigned your decision regarding FACTS:
this matter.” Baltazar answered asking the corporation that he be allowed
ISSUE: Whether or not the resolutions are valid.
to pay his unpaid subscriptiona at a later date. However, he made no In his complaint, Aquiles alleged that he was the owner and in
RULING: paymenrs hence his unpaid subscription would be reverted to the actual possession of a 52-square meter commercial lot situated in
corporation. Baltazar subsequently wrote another letter to the members of Barangay Quinale, Tabaco City, Albay; that he acquired the said property
NO. The Court ruled that resolutions are contrary to the By-Laws the Board of Directors of the plaintiff corporation, offering to withdraw through a deed of cession and quitclaim executed by his parents, Pablo
of the federation and, therefore, are not within the power of the board of completely from the corporation by selling out to the corporation all his Riosa, Sr. and Sabiniana Biron; that he declared the property in his name
directors to enact. The ByLaws, in the aforequoted Section 8, explicitly shares of stock in the total amount of P23,000. Apparently this offer of the and had been religiously paying the realty tax on the said property; that
reserved unto the stockholders the power to determine the compensation defendant was left unacted upon by the plaintiff. thereafter, his daughter, Annie Lyn Riosa Zampelis, renovated the
The Board of Directors of the Lingayen Gulf held a meeting, and commercial building on the lot and introduced improvements costing no
of members of the board of directors, and the stockholders did restrict such
in the course of the said meeting they adopted Resolution No. 17. This less than ₱300,000.00; that subsequently, on three (3) occasions, he
compensation to "actual transportation expenses plus the per diems of
resolution in effect set aside the stockholders resolution approved on June obtained loans from Sia Ko Pio in the total amount of ₱50,000.00; that as
P30.00 and actual expenses while waiting." Even without the express 23, 1946, on the ground that said stockholders' resolution was null and
reservation of said power, the directors are not entitled to compensation a security for the payment of loans, Sia Ko Pio requested from him a
void, and because the plaintiff corporation was not in a financial position to photocopy of the deed of cession and quitclaim; that Sia Ko Pio presented
under the Corporation Code. The directors, in assigning themselves absorb the unpaid balance of the subscribed capital stock. to him a document purportedly a receipt for the ₱50,000.00 loan with an
additional duties, such as the visitation of FACOMAS, acted within their The legal counsel of the Lingayen Gulf wrote a letter to the undertaking to pay the total amount of ₱52,000.00 including the ₱2,000.00
power, but, by voting for themselves compensation for such additional Baltazar, demanding the payment of the unpaid balance of his subscription attorney’s fees; that without reading the document, he affixed his signature
duties, they acted in excess of their authority, as expressed in the ByLaws. amounting to P18,500. Baltazar ignored the said demand. Hence an action thereon; and that in September 2001, to his surprise, he received a letter
was instituted. By way of counterclaim, the Baltazar also claims from from La Suerte informing him that the subject lot was already registered in
Lingayen Gulf a reasonable compensation at the rate of P700 per month its name.
as president of the company, for the period from March 1, 1946 to
19. LINGAYEN GULF ELECTRIC POWER COMPANY, INC. V. December 31, 1948.
BALTAZAR Aquiles claimed that by means of fraud, misrepresentation and deceit
G.R. No. L-6244; June 30, 1953 ISSUE: employed by Sia Ko Pio, he was made to sign the document which he
Is Baltazar entitled to salary as president during his term? thought was a receipt and undertaking to pay the loan, only to find out later
FACTS: that it was a document of sale. Aquiles averred that he did not appear
RULING: before the notary public to acknowledge the sale, and that the notary
17
Delegation of Authority to Corporate Officers | Doctrine of Apparent Authority
public, a municipal judge, was not authorized to notarize a deed of It is the board of directors or trustees which exercises almost all the Aquiles’ property. It is, thus, clear that the loan obtained by Aquiles from
conveyance. He further claimed that he could not have sold the corporate powers in a corporation. Thus, the Corporation Code provides: Sia Ko Pio was a personal loan from the latter, not a transaction between
commercial building on the lot as he had no transmissible right over it, as it Aquiles and La Suerte. There was no evidence to show that Sia Ko Pio
was not included in the deed of cession and quitclaim. He, thus, prayed for SEC. 23. The board of directors or trustees. — Unless otherwise provided was clothed with authority to use his personal fund for the benefit of La
the nullification of the deed of sale and certificate of title in the name of La in this Code, the corporate powers of all corporations formed under this Suerte. Evidently, La Suerte was never in the picture.
Suerte and the reconveyance of the subject property to him. Code shall be exercised, all business conducted and all property of such
corporations controlled and held by the board of directors or trustees to be
In its Answer, La Suerte averred that it was the actual and lawful owner of elected from among the holders of stock, or where there is no stock, from
the commercial property, after purchasing it from Aquiles on December 7, among the members of the corporation, who shall hold office for one (1) 21. LA BUGA’AL TRIBAL ASSOCIATION INC. v. RAMOS
1990; that it allowed Aquiles to remain in possession of the property to year and until their successors are elected and qualified. x x x GR No. 127882 December 1, 2004
avoid the ire of his father from whom he had acquired property inter vivos,
subject to his obligation to vacate the premises anytime upon demand; that SEC. 36. Corporate powers and capacity. — Every corporation FACTS:
on February 13, 1991, the Register of Deeds of Albay issued Transfer incorporated under this Code has the power and capacity:
Certificate of Title (TCT) No. T-80054 covering the subject property in its The Petition for Prohibition and Mandamus before the Court challenges
name; that Aquiles necessarily undertook the cost of repairs and did not the constitutionality of (1) Republic Act No. [RA] 7942 (The Philippine
pay rent for using the premises; that Aquiles transacted with it, through Sia xxxx
Mining Act of 1995); (2) its Implementing Rules and Regulations (DENR
Ko Pio, now deceased, who was then its Chief Executive Officer; that his Administrative Order No. [DAO] 96-40); and (3) the FTAA dated March 30,
opinion that only the land was sold was absurd because the sale of the 7. To purchase, receive, take or grant, hold, convey, sell, lease, pledge, 1995,6 executed by the government with Western Mining Corporation
principal included its accessories, not to mention that he did not make any mortgage and otherwise deal with such real and personal property, (Philippines), Inc. (WMCP). On January 27, 2004, the Court en banc
reservation at the time the deed was executed; that it repeatedly asked including securities and bonds of other corporations, as the transaction of promulgated its Decision granting the Petition and declaring the
Aquiles to vacate the premises but to no avail; that, instead, he tried to a lawful business of the corporation may reasonably and necessarily unconstitutionality of certain provisions of RA 7942, DAO 96-40, as well as
renovate the building in 2001 which prompted it to lodge a complaint with require, subject to the limitations prescribed by the law and the Constitution. of the entire FTAA executed between the government and WMCP, mainly
the Office of the Mayor on the ground that the renovation work was without on the finding that FTAAs are service contracts prohibited by the 1987
a building permit; and that Aquiles’ complaint was barred by prescription, xxxx Constitution. The Decision struck down the subject FTAA for being similar
laches, estoppel and indefeasibility of La Suerte’s title. to service contracts, which, though permitted under the 1973 Constitution,
were subsequently denounced for being antithetical to the principle of
Under these provisions, the power to purchase real property is vested in sovereignty over our natural resources, because they allowed foreign
ISSUE/S:
the board of directors or trustees. While a corporation may appoint agents control over the exploitation of our natural resources, to the prejudice of the
to negotiate for the purchase of real property needed by the corporation, Filipino nation. According to the Decision, the 1987 Constitution (Section 2
WHETHER THE HONORABLE COURT OF APPEALS the final say will have to be with the board, whose approval will finalize the of Article XII) effectively banned such service contracts. Subsequently,
COMMITTED SERIOUS ERROR OF LAW IN HOLDING THAT THE transaction. A corporation can only exercise its powers and transact its respondents filed separate Motions for Reconsideration.
PERSONAL LOAN OF PETITIONER OBTAINED AND GRANTED BY business through its board of directors and through its officers and agents
SIA KO PIO IS A CONSIDERATION OF SALE OF THE PROPERTY IN when authorized by a board resolution or its by-laws. As held in AF Realty ISSUE:
FAVOR OF THE RESPONDENT CORPORATION LA SUERTE & Development, Inc. v. Dieselman Freight Services, Co.:
CORPORATION.
Did the Philippine Mining Act and the WMCP FTAA divest the State of
Section 23 of the Corporation Code expressly provides that the corporate control and ownership over the natural resources?
RULING: powers of all corporations shall be exercised by the board of directors. Just
as a natural person may authorize another to do certain acts in his behalf, RULING:
And for said reasons, the CA should not have favorably so may the board of directors of a corporation validly delegate some of its
considered the validity of the deed of absolute sale absent any written functions to individual officers or agents appointed by it. Thus, contracts or No.
authority from La Suerte’s board of directors for Sia Ko Pio to negotiate and acts of a corporation must be made either by the board of directors or by a
purchase Aquiles property on its behalf and to use its money to pay the corporate agent duly authorized by the board. Absent such valid All mineral resources are owned by the State. Their exploration,
purchase price. The Court notes that when Sia Ko Pio’s son, Juan was delegation/authorization, the rule is that the declarations of an individual development and utilization (EDU) must always be subject to the full
presented as an officer of La Suerte, he admitted that he could not find in director relating to the affairs of the corporation, but not in the course of, or control and supervision of the State. More specifically, given the
the records of the corporation any board resolution authorizing his father to connected with, the performance of authorized duties of such director, are inadequacy of Filipino capital and technology in large-scale EDU activities,
purchase disputed property.20 In Spouses Firme v. Bukal Enterprises and held not binding on the corporation.22 [Emphases supplied] the State may secure the help of foreign companies in all relevant matters
Development Corporation,21 it was written: -- especially financial and technical assistance -- provided that, at all times,
In the case at bench, Sia Ko Pio, although an officer of La Suerte, had no the State maintains its right of full control.
authority from its Board of Directors to enter into a contract of sale of
18
Delegation of Authority to Corporate Officers | Doctrine of Apparent Authority
Full control is not anathematic to day-to-day management by the (Clause 3.2-a); requires approval by the State for incorporation of lands into prevent the mining operations from grinding to a complete halt as a result
contractor, provided that the State retains the power to direct overall the contract area (Clause 4.3-c); requires Bureau of Forest Development of possible delays of more than 60 days in the governments processing
strategy; and to set aside, reverse or modify plans and actions of the approval for inclusion of forest reserves as part of the FTAA contract area and approval of submitted work programs and budgets. Clause 8.3 seeks
contractor. The idea of full control is similar to that which is exercised by the (Clause 4.5); obligates the contractor to periodically relinquish parts of the to provide a temporary, stop-gap solution in case a disagreement between
board of directors of a private corporation: the performance of managerial, contract area not needed for exploration and development (Clause 4.6); the State and the contractor (over the proposed work program or budget
operational, financial, marketing and other functions may be delegated to requires submission of a declaration of mining feasibility for approval by the submitted by the contractor) should result in a deadlock or impasse, to
subordinate officers or given to contractual entities, but the board retains State (Clause 4.6-b); obligates the contractor to report to the State the avoid unreasonably long delays in the performance of the works.
full residual control of the business. results of its exploration activities (Clause 4.9); requires the contractor to
obtain State approval for its work programs for the succeeding two year The State, despite Clause 8.3, still has control over the contract area, and
Who or what organ of government actually exercises this power of control periods, containing the proposed work activities and expenditures budget it may, as sovereign authority, prohibit work thereon until the dispute is
on behalf of the State? The Constitution is crystal clear: the President. related to exploration (Clause 5.1); requires the contractor to obtain State resolved, or it may terminate the FTAA, citing substantial breach thereof.
Indeed, the Chief Executive is the official constitutionally mandated to enter approval for its proposed expenditures for exploration activities (Clause Hence, the State clearly retains full and effective control.
into agreements with foreign owned corporations. On the other hand, 5.2); requires the contractor to submit an annual report on geological, Clause 8.5, which allows the contractor to make changes to approved work
Congress may review the action of the President once it is notified of every geophysical, geochemical and other information relating to its explorations programs and budgets without the prior approval of the DENR secretary,
contract entered into in accordance with this [constitutional] provision within within the FTAA area (Clause 5.3-a); requires the contractor to submit subject to certain limitations with respect to the variance/s, merely provides
thirty days from its execution. In contrast to this express mandate of the within six months after expiration of exploration period a final report on all the contractor a certain amount of flexibility to meet unexpected situations,
President and Congress in the EDU of natural resources, Article XII of the its findings in the contract area (Clause 5.3-b); requires the contractor after while still guaranteeing that the approved work programs and budgets are
Constitution is silent on the role of the judiciary. However, should the conducting feasibility studies to submit a declaration of mining feasibility, not abandoned altogether. And if the secretary disagrees with the actions
President and/or Congress gravely abuse their discretion in this regard, the along with a description of the area to be developed and mined, a taken by the contractor in this instance, he may also resort to
courts may -- in a proper case -- exercise their residual duty under Article description of the proposed mining operations and the technology to be cancellation/termination of the FTAA as the ultimate sanction.
VIII. Clearly then, the judiciary should not inordinately interfere in the employed, and the proposed work program for the development phase, for
exercise of this presidential power of control over the EDU of our natural approval by the DENR secretary (Clause 5.4); obligates the contractor to Clause 4.6 of the WMCP FTAA gives the contractor discretion to select
resources. complete the development of the mine, including construction of the parts of the contract area to be relinquished. The State is not in a position
production facilities, within the period stated in the approved work program to substitute its judgment for that of the contractor, who knows exactly
On the basis of this control standard, this Court upholds the constitutionality (Clause 6.1); requires the contractor to submit for approval a work program which portions of the contract area do not contain minerals in commercial
of the Philippine Mining Law, its Implementing Rules and Regulations -- covering each period of three fiscal years (Clause 6.2); requires the quantities and should be relinquished. Also, since the annual occupation
insofar as they relate to financial and technical agreements -- as well as the contractor to submit reports to the secretary on the production, ore fees paid to government are based on the total hectarage of the contract
subject Financial and Technical Assistance Agreement (FTAA). reserves, work accomplished and work in progress, profile of its work force area, net of the areas relinquished, the contractors self-interest will assure
and management staff, and other technical information (Clause 6.3); proper and efficient relinquishment.
RA 7942 provides for the State’s control and supervision over mining subjects any expansions, modifications, improvements and replacements
operations. The setup under RA 7942 and DAO 96-40 hardly relegates the of mining facilities to the approval of the secretary (Clause 6.4); subjects to Clause 10.2(e) of the WMCP FTAA does not mean that the contractor can
State to the role of a passive regulator dependent on submitted plans and State control the amount of funds that the contractor may borrow within the compel government to use its power of eminent domain. It contemplates a
reports. On the contrary, the government agencies concerned are Philippines (Clause 7.2); subjects to State supervisory power any situation in which the contractor is a foreign-owned corporation, hence, not
empowered to approve or disapprove -- hence, to influence, direct and technical, financial and marketing issues (Clause 10.1-a); obligates the qualified to own land. The contractor identifies the surface areas needed
change -- the various work programs and the corresponding minimum contractor to ensure 60 percent Filipino equity in the contractor within ten for it to construct the infrastructure for mining operations, and the State then
expenditure commitments for each of the exploration, development and years of recovering specified expenditures unless not so required by acquires the surface rights on behalf of the former. The provision does not
utilization phases of the mining enterprise. subsequent legislation (Clause 10.1); gives the State the right to terminate call for the exercise of the power of eminent domain (or determination of
In other words, the FTAA contractor is not free to do whatever it pleases the FTAA for unremedied substantial breach thereof by the contractor just compensation); it seeks to avoid a violation of the anti-dummy law.
and get away with it; on the contrary, it will have to follow the government (Clause 13.2); requires State approval for any assignment of the FTAA by
line if it wants to stay in the enterprise. Ineluctably then, RA 7942 and DAO the contractor to an entity other than an affiliate (Clause 14.1). Clause 10.2(l) of the WMCP FTAA giving the contractor the right to
96-40 vest in the government more than a sufficient degree of control and mortgage and encumber the mineral products extracted may have been a
supervision over the conduct of mining operations. In short, the aforementioned provisions of the WMCP FTAA, far from result of conditions imposed by creditor-banks to secure the loan
constituting a surrender of control and a grant of beneficial ownership of obligations of WMCP. Banks lend also upon the security of encumbrances
WMCP FTAA Likewise Gives the mineral resources to the contractor in question, vest the State with control on goods produced, which can be easily sold and converted into cash and
State Full Control and Supervision and supervision over practically all aspects of the operations of the FTAA applied to the repayment of loans. Thus, Clause 10.2(l) is not something
contractor, including the charging of pre-operating and operating out of the ordinary. Neither is it objectionable, because even though the
The WMCP FTAA obligates the contractor to account for the value of expenses, and the disposition of mineral products. contractor is allowed to mortgage or encumber the mineral end-products
production and sale of minerals (Clause 1.4); requires that the contractors themselves, the contractor is not thereby relieved of its obligation to pay the
work program, activities and budgets be approved by the State (Clause There is likewise no relinquishment of control on account of specific government its basic and additional shares in the net mining revenue. The
2.1); gives the DENR secretary power to extend the exploration period provisions of the WMCP FTAA. Clause 8.2 provides a mechanism to
19
Delegation of Authority to Corporate Officers | Doctrine of Apparent Authority
contractors ability to mortgage the minerals does not negate the States cumulative net mining revenue. Whichever option or computation is used, control. To avoid compromising the States full control and supervision over
right to receive its share of net mining revenues. the additional government share has nothing to do with taxes, duties, fees the exploitation of mineral resources, there must be no attempt to impose
or charges. The portion of revenues remaining after the deduction of the a minimum 60 percent rule. It is sufficient that the State has the power and
Clause 10.2(k) which gives the contractor authority to change its equity basic and additional government shares is what goes to the contractor. means, should it so decide, to get a 60 percent share (or greater); and it is
structure at any time, means that WMCP, which was then 100 percent not necessary that the State does so in every case.
foreign owned, could permit Filipino equity ownership. Moreover, what is The basic government share and the additional government share do not
important is that the contractor, regardless of its ownership, is always in a yet take into account the indirect taxes and other financial contributions of
position to render the services required under the FTAA, under the mining projects, which are real and actual benefits enjoyed by the Filipino
direction and control of the government. people; if these are taken into account, total government share increases 22. SHIPSIDE INCORPORATED vs. THE HON. COURT OF
to 60 percent or higher (as much as 77 percent, and 89 percent in one APPEALS, HON. REGIONAL TRIAL COURT, BRANCH 26 & the
Clauses 10.4(e) and (i) bind government to allow amendments to the instance) of the net present value of total benefits from the project. REPUBLIC OF THE PHILIPPINES
FTAA if required by banks and other financial institutions as part of the G.R. No. 143377, February 20, 2001
conditions of new lendings. There is nothing objectionable here, since The third or last paragraph of Section 81 of RA 7942 is slammed for
Clause 10.4(e) also provides that such financing arrangements should in deferring the payment of the government share in FTAAs until after the FACTS
no event reduce the contractors obligations or the governments rights contractor shall have recovered its pre-operating expenses, exploration
under the FTAA. Clause 10.4(i) provides that government shall favourably and development expenditures. Allegedly, the collection of the States Originally four lots were owned Rafael Galvez. He subsequently sold lot 1
consider any request for amendments of this agreement necessary for the share is rendered uncertain, as there is no time limit in RA 7942 for this and 4 in favor of Filipina Mamaril, Cleopatra Llana, Regina Bustos, and
contractor to successfully obtain financing. There is no renunciation of grace period or recovery period. But although RA 7942 did not limit the Erlinda Balatbat in a deed of sale. Mamaril later sold lot 1 to Lepanto
control, as the proviso does not say that government shall automatically grace period, the concerned agencies (DENR and MGB) in formulating the Consolidated Mining Company. Later, unknown to Lepanto, the RTC
grant any such request. Also, it is up to the contractor to prove the need for 1995 and 1996 Implementing Rules and Regulations provided that the declared the OCT registered in the name of Galvez as null and void and
the requested changes. The government always has the final say on period of recovery, reckoned from the date of commercial operation, shall ordered the cancellation thereof. On October 28, 1963, Lepanto
whether to approve or disapprove such requests. be for a period not exceeding five years, or until the date of actual recovery, Consolidated Mining Company sold to herein petitioner Lots No. 1 and 4.
whichever comes earlier. Meanwhile the decision of the CA became final and executory and a writ
In fine, the FTAA provisions do not reduce or abdicate State control.
was issued, however said writ remained unsatisfied for 24 years.
In fine, the challenged provisions of RA 7942 cannot be said to surrender
No Surrender of financial benefits from an FTAA to the foreign contractors. Office of the Solicitor General filed a complaint for revival of judgment and
Financial Benefits
cancellation of titles before the Regional Trial Court of the First Judicial
Moreover, there is no concrete basis for the view that, in FTAAs with a
Region against the successors of Galvez and herein petitioner and its
The second paragraph of Section 81 of RA 7942 has been denounced for foreign contractor, the State must receive at least 60 percent of the after-
allegedly limiting the States share in FTAAs with foreign contractors to just tax income from the exploitation of its mineral resources, and that such motion for reconsideration was likewise turned down. The CA affirmed the
taxes, fees and duties, and depriving the State of a share in the after-tax share is the equivalent of the constitutional requirement that at least 60 same, hence this petition.
income of the enterprise. However, the inclusion of the phrase among percent of the capital, and hence 60 percent of the income, of mining
other things in the second paragraph of Section 81 clearly and companies should remain in Filipino hands. Even if the State is entitled to
unmistakably reveals the legislative intent to have the State collect more a 60 percent share from other mineral agreements (CPA, JVA and MPSA), ISSUE
than just the usual taxes, duties and fees. that would not create a parallel or analogous situation for FTAAs. We are
dealing with an essentially different equation. Here we have the old apples Whether or not the filing of the petition was authorized by the BOD of
Thus, DAO 99-56, the Guidelines Establishing the Fiscal Regime of and oranges syndrome. petitioner.
Financial or Technical Assistance Agreements, spells out the financial
benefits government will receive from an FTAA, as consisting of not only a The Charter did not intend to fix an iron-clad rule of 60 percent share,
basic government share, comprised of all direct taxes, fees and royalties, applicable to all situations, regardless of circumstances. There is no
as well as other payments made by the contractor during the term of the indication of such an intention on the part of the framers. Moreover, the RULING
FTAA, but also an additional government share, being a share in the terms and conditions of petroleum FTAAs cannot serve as standards for
earnings or cash flows of the mining enterprise, so as to achieve a fifty-fifty mineral mining FTAAs, because the technical and operational YES.
sharing of net benefits from mining between the government and the requirements, cost structures and investment needs of off-shore petroleum
contractor. exploration and drilling companies do not have the remotest resemblance The Court of Appeals dismissed the petition for certiorari on the ground that
to those of on-shore mining companies. Lorenzo Balbin, the resident manager for petitioner, who was the signatory
The additional government share is computed using one of three (3) in the verification and certification on non-forum shopping, failed to show
options or schemes detailed in DAO 99-56, viz., (1) the fifty-fifty sharing of To take the position that governments share must be not less than 60 proof that he was authorized by petitioner's board of directors to file such a
cumulative present value of cash flows; (2) the excess profit-related percent of after-tax income of FTAA contractors is nothing short of this petition.
additional government share; and (3) the additional sharing based on the Court dictating upon the government. The State resultantly ends up losing
20
Delegation of Authority to Corporate Officers | Doctrine of Apparent Authority
It was clear from the record that when the general manager filed the to sign, endorse and deliver all checks and promissory notes on behalf of or acquiescence in the general course of business. This Court has held,
petition, there was no proof attached as to the authorization by the Board. the corporation. thus:
However, when the petitioner filed its motion for reconsideration a
resolution or secretary‘s certification stating that that on October 11, 1999, CMMCI, on the other hand, denied borrowing the amount from
Tsukahara, and claimed that both loans were personal loans of A corporate officer or agent may represent and bind
or ten days prior to the filing of the petition, Balbin had been authorized by the corporation in transactions with third persons to the
petitioner's board of directors to file said petition. The Court accepted this Sugimoto. The company also contended that if the loans were those of
extent that [the] authority to do so has been conferred
certification, although belatedly presented, as a valid authorization. The CMMCI, the same should have been supported by resolutions issued by upon him, and this includes powers which have been
Court was reiterated that belated submission of a verification is allowed the CMMCIs Board of Directors. intentionally conferred, and also such powers as, in
same being not a mandatory and jurisdictional requirement, and as to the the usual course of the particular business, are
The RTC, ruled in favor of the plaintiff, and ordered the
non-forum shopping the same was considered to be valid because the incidental to, or may be implied from, the powers
defendants to pay jointly and severally the plaintiffs. The CA affirmed the intentionally conferred, powers added by custom and
case of the petitioner must be litigated based on its merit and must not be
decision of the RTC. usage, as usually pertaining to the particular officer or
dismissed based on technical and procedural infirmities, which were
actually cured. agent, and such apparent powers as the corporation
has caused persons dealing with the officer or agent
Issue: to believe that it has conferred.

23. CEBU MACTAN VS. MASAHIRA Is CMMCI liable for the loan contracted by its President without In this case, the corporate by-laws of CMMCI provide:
GR No. 159624 July 17, 2009 a resolution by the CMMI Board of Directors.

ARTICLE III
Facts: Held:
Officers
In February 1994, petitioner Cebu Mactan Members Center, A corporation, being a juridical entity, may act through its board
of directors, which exercises almost all corporate powers, lays down all xxx
Inc. (CMMCI), through Mitsumasa Sugimoto (Sugimoto), the President
and Chairman of the Board of Directors of CMMCI, obtained a loan corporate business policies and is responsible for the efficiency of
amounting to P6,500,000 from respondent Masahiro Tsukahara. As management. The general rule is that, in the absence of authority from the 2. President. The President shall be elected by the
payment for the loan, CMMCI issued seven postdated checks of CMMCI board of directors, no person, not even its officers, can validly bind a Board of Directors from their own number. He shall
corporation. Section 23 of the Corporation Code of the Philippines have the following powers and duties:
payable to Tsukahara.
provides:
On 13 April 1994, CMMCI, through Sugimoto, obtained another
SEC. 23. The Board of Directors or xxx
loan amounting to P10,000,000 from Tsukahara. Sugimoto executed and
Trustees. Unless otherwise provided in this Code, c. Borrow money for the company by any legal
signed a promissory note in his capacity as CMMCI President and
the corporate powers of all corporations formed under means whatsoever, including the arrangement of
Chairman, as well as in his personal capacity.
this Code shall be exercised, all business conducted letters of credit and overdrafts with any and all banking
Upon maturity, the seven checks were presented for payment and all property of such corporations controlled and institutions;
held by the board of directors or trustees x x x.
by Tsukahara, but the same were dishonored by PNB, the drawee bank.
d. Execute on behalf of the company all contracts and
After several failed attempts to collect the loan amount In Peoples Aircargo and Warehousing Co., Inc. v. Court of agreements which the said company may enter into;
totaling P16,500,000, Tsukahara filed the instant case for collection of Appeals, we held that under Section 23, the power and the responsibility
sum of money against CMMCI and Sugimoto. e. Sign, indorse, and deliver all checks, drafts, bill of
to decide whether the corporation should enter into a contract that will bind
exchange, promissory notes and orders of payment of
the corporation are lodged in the board of directors, subject to the articles
Tsukahara alleged that the amount of P16,500,000 was used of incorporation, by-laws, or relevant provisions of law. However, just as a sum of money in the name and on behalf of the
by CMMCI for the improvement of its beach resort, which included the natural person may authorize another to do certain acts for and on his corporation;
construction of a wave fence, the purchase of airconditioners and behalf, the board of directors may validly delegate some of its functions and
curtains, and the provision of salaries of resort employees. He also powers to officers, committees or agents. The authority of such individuals
asserted that Sugimoto, as the President of CMMCI, has the power to to bind the corporation is generally derived from law, corporate by-laws or It is clear from the foregoing that the president of CMMCI is given
borrow money for said corporation by any legal means whatsoever and authorization from the board, either expressly or impliedly by habit, custom
the power to borrow money, execute contracts, and sign and indorse

21
Delegation of Authority to Corporate Officers | Doctrine of Apparent Authority
ABS-CBN and VIVA executed a Film Exhibition Agreement for specific purposes. Delegation to officers makes the latter agents of the
checks and promissory notes, in the name and on behalf of CMMCI. With whereby Viva gave ABS-CBN an exclusive right to exhibit some Viva films. corporation; accordingly, the general rules of agency as to the binding
In accordance with the said agreement stating that- ABS-CBN shall have effects of their acts would apply. For such officers to be deemed fully
such powers expressly conferred under the corporate by-laws, the CMMCI
the right of first refusal to the next twenty-four Viva films for TV telecast clothed by the corporation to exercise a power of the Board, the latter must
president, in exercising such powers, need not secure a resolution from under such terms as may be agreed upon by the parties hereto, provided, specially authorize them to do so. That Del Rosario did not have the
however, that such right shall be exercised by ABS-CBN from the actual authority to accept ABS-CBNs counter-offer was best evidenced by his
the companys board of directors. We quote with approval the ruling of the offer in writing. submission of the draft contract to VIVAs Board of Directors for the latter’s
In a letter, Mrs. Concio signified her refusal of 8 films. approval. In any event, there was between Del Rosario and Lopez III no
appellate court, viz: Subsequently, Del Rosario and ABS-CBNs general manager, Eugenio meeting of minds. The following findings of the trial court are instructive: A
x x x The court a quo correctly ruled that a board Lopez III, met at the Tamarind Grill Restaurant in Quezon City to discuss number of considerations militate against ABS-CBNs claim that a contract
resolution in this case is a superfluity given the express the package proposal of VIVA. What transpired in that lunch meeting is the was perfected at that lunch meeting on April 02, 1992 at the Tamarind Grill.
provision of the corporate by-laws. subject of conflicting versions. Mr. Lopez testified that he and Mr. Del There is no proof that a contract was perfected in the said
Rosario allegedly agreed that ABS-CBN was granted exclusive film rights meeting. Lopez’ testimony about the contract being written in a napkin is
to fourteen films for a total consideration of P36 million; that he allegedly not corroborated because the napkin was never produced in court. Further,
To insist that a board resolution is still required in order put this agreement as to the price and number of films in a napkin and there is no meeting of the minds because Del Rosario’s offer was of 104
to bind the corporation with respect to the obligations signed it and gave it to Mr. Del Rosario. On the other hand. Del Rosario films for P60 million was not accepted. And that the alleged counter-offer
contracted by its president is to defeat the purpose of denied having made any agreement with Lopez regarding the 14 Viva made by Lopez on the same day was not also accepted because there’s
the by-laws. By-laws of a corporation should be films; denied the existence of a napkin in which Lopez wrote something; no proof of such. The counter offer can only be deemed to have been
construed and given effect according to the general and insisted that what he and Lopez discussed at the lunch meeting was made days after the April 2 meeting when Santos-Concio sent a letter to
rules governing the construction of contracts. They, as Vivas film package offer of 104 films (52 originals and 52 re-runs) for a total Del Rosario containing the counter-offer. Regardless, there was no
the self-imposed private laws of a corporation, have, price of P60 million. Mr. Lopez promising to make a counter proposal which showing that Del Rosario accepted. But even if he did accept, such
when valid, substantially the same force and effect as came in the form of a proposal contract Annex C of the complaint. acceptance will not bloom into a perfected contract because Del Rosario
laws of the corporation, as have the provisions of its Viva made an agreement with Republic Broadcasting has no authority to do so.
charter insofar as the corporation and the persons Corporation (referred to as RBS – or GMA 7) which gave exclusive rights
within it are concerned. They are in effect written into to RBS to air 104 Viva films including the 14 films initially requested by As a rule, corporate powers, such as the power; to enter into
the charter and in this sense, they become part of the ABS-CBN. ABS-CBN now filed a complaint for specific performance contracts; are exercised by the Board of Directors. But this power may be
fundamental law of the corporation. And the against Viva as it alleged that there is already a perfected contract between delegated to a corporate committee, a corporate officer or corporate
corporation and its directors (or trustees) and officers Viva and ABS-CBN in the meeting. Lopez testified that Del Rosario agreed manager. Such a delegation must be clear and specific. In the case at bar,
are bound by and must comply with them. to the counterproposal and he (Lopez) even put the agreement in a napkin there was no such delegation to Del Rosario. The fact that he has to
which was signed and given to Del Rosario. ABS-CBN also filed an present the counteroffer to the Board of Directors of Viva is proof that the
injunction against RBS to enjoin the latter from airing the films. contract must be accepted first by the Viva’s Board. Hence, even if Del
The corporation is now estopped from denying the
authority of its president to bind the former into Rosario accepted the counter-offer, it did not result to a contract because
contractual relations. x x x it will not bind Viva sans authorization.
ISSUE:
Was the contract made between Del Rosario and Lopez valid? 25. YAO KA SIN v. CA
Thus, given the presidents express powers under the CMMCIs by-laws,
15 June 1992
RULING:
Sugimoto, as the president of CMMCI, was more than equipped to enter
No. Facts:
into loan transactions on CMMCIs behalf. Accordingly, the loans obtained In the case at bar, ABS-CBN made no unqualified acceptance
of VIVAs offer hence, they underwent period of bargaining. ABS-CBN then The root of this controversy is the undated letter-offer of Constancio B.
by Sugimoto from Tsukahara on behalf of CMMCI are valid and binding formalized its counter-proposals or counter-offer in a draft contract. VIVA Maglana, President and Chairman of the Board of private respondent
through its Board of Directors, rejected such counter-offer. Even if it be Prime White Cement Corporation, hereinafter referred to as PWCC, to Yao
against the latter, and CMMCI may be held liable to pay such loans. conceded arguendo that Del Rosario had accepted the counter-offer, the Ka Sin Trading, hereinafter referred to as YKS, which describes itself as "a
acceptance did not bind VIVA, as there was no proof whatsoever that Del business concern of single proprietorship," and is represented by its
Rosario had the specific authority to do so. manager, Mr. Henry Yao.
24. ABS-CBN v. CA Under the Corporation Code, unless otherwise provided by said
G.R. No. 1286904; January 21,1999 Code, corporate powers, such as the power to enter into contracts, are In its meeting in Cebu City on 30 June 1973, or twenty-three (23) days after
exercised by the Board of Directors. However, the Board may delegate the signing of Exhibit "A", the Board of Directors of PWCC disapproved the
FACTS: such powers to either an executive committee or officials or contracted same; the rejection is evidenced by the following Minutes (Exhibit "10"):
managers. The delegation, except for the executive committee, must be
22
Delegation of Authority to Corporate Officers | Doctrine of Apparent Authority
"the 10,000 bags of white cement sold to Yao Ka Sin Trading is sold not cement, not under Exhibit "A", but under a separate contract prepared by 1. The resolution contained in defendant's letter to plaintiff dated July 5,
because of the alleged letter-contract adhered to by them, but must be the Board; (f) the rejection by the Board of Exhibit "A" was made known to 1973, on the 10,000 bags of white cement delivered to plaintiff was not by
understood as a new and separate contract, and has in no way to do with YKS through various letters sent to it, copies of which were attached to the reason of the letter contract, Exhibit "A", which was totally disapproved by
the letter-offer which they (sic) distinct consideration, as the letter-contract Answer as Annexes 1, 2 and 3; (g) YKS knew, per Delivery Order and defendant corporation's board of directors, clearly stating that 'If within ten
which they now hang on (sic) as consummated is by this resolution, totally Official Receipt issued by PWCC, that only 10,000 bags were sold to it, (10) days from date hereof, we will not hear from you but you will withdraw
disapproved and is unacceptable to the corporation." without any terms or conditions, at P24.30 per bag FOB Asturias, Cebu; cement at P24.30 per bag from our plant, then we will deposit your check
of P243,000.00 dated June 7, 1973 issued by the Producers Bank of the
On 5 July 1973, PWCC wrote a letter (Exhibit "1") to YKS informing it of the (h) YKS is solely to blame for the failure to take complete delivery of 10,000
Philippines, per instruction of the Board.' (Annex "1" to defendant's
disapproval of Exhibit "A". Pursuant, however, to its decision with respect bags for it did not send its boat or truck to PWCC's plant; and (i) YKS has,
Answer).
to the 10,000 bags of cement, it issued the corresponding Delivery Order therefore, no cause of action.
(Exhibit "4") and Official Receipt No. 0394 (Exhibit "5") for the payment of 2. Letter of defendant to plaintiff dated August 4, 1973 that defendant 'only
the same in the amount of P243,000.00. This is the same amount received PWCC presented evidence to prove that Exhibit "A" is not binding upon it committed to you and which you accordingly paid 10,000 bags of white
and acknowledged by Maglana in Exhibit "A". because Mr. Maglana was not authorized to make the offer and sign the cement of which 4,150 bags were already delivered to you as of August 1,
contract in behalf of the corporation. Per its By-Laws (Exhibit "8"), only the 1973' (Annex "2" of defendant's Answer).
YKS accepted without protest both the Delivery and Official Receipts. Board of Directors has the power "x x x (7) To enter into (sic) agreement or
contract of any kind with any person in the name and for and in behalf of 3. Letter dated August 21, 1973 to plaintiff reiterating defendant's letter of
On 4 August 1973, PWCC wrote a letter (Exhibit "2") to YKS in answer to August 4, 1973 (Annex "3" to defendant's Answer).
the latter's 4 August 1973 letter stating that it is "withdrawing or taking the corporation through its President, subject only to the declared objects
delivery of not less than 10,000 bags of white cement on August 6-7, 1973 and purpose of the corporation and the existing provisions of law." Per 4. Letter to stores dated August 21, 1973,
at Asturias, Cebu, thru M/V Taurus." In said reply, PWCC reminded YKS standard practice of the corporation, contracts should first pass through the
of its (PWCC's) 5 July 1973 letter (Exhibit "1") and told the latter that PWCC marketing and intelligence unit before they are finalized. Because of its 5. Receipt from plaintiff (sic) P243,000.00 in payment of 10,000 bags of
"only committed to you and which you correspondingly paid 10,000 bags interest in the PWCC, the NIDC, through its comptroller, goes over white cement at 24.30 per bag (Annex "5" to defendant's Answer).
of white cement of which 4,150 bags were already delivered to you as of contracts involving funds of and white cement produced by the PWCC.
Plaintiff is deemed to have admitted, not only the due execution and
August 1, 1973."Unfortunately, no copy of the said 4 August 1973 letter of Finally, among the duties of its legal counsel is to review proposed
genuiness (sic) of said documents, (Rule 8, Sec. 8, Rules of Court) but also
YKS was presented in evidence. contracts before they are submitted to the Board. While the president may the allegations therein (Rule 9, Sec. 1, Rules of Court). All of the foregoing
be tasked with the preparation of a contract, it must first pass through the documents tend to prove that the letter-offer, Exhibit "A", was rejected by
On 10 September 1973, YKS, through Henry Yao, wrote a letter to PWCC
legal counsel and the comptroller of the corporation. defendant corporation's Board of Directors and plaintiff was duly notified
as a follow-up to the letter of 15 August 1973; YKS insisted on the delivery
of 45,000 bags of white cement. thereof and that the P243,000.00 check was considered by both parties as
The trial court interpreted the provision of the By-Laws -- granting its Board payment of the 10,000 bags of cement under a separate transaction.
As of 7 December 1973, PWCC had delivered only 9,775 bags of white of Directors the power to enter into an agreement or contract of any kind
cement. with any person through the President -- to mean that the latter may enter Third. Maglana's signing the letter-offer prepared for him in the Solidbank
into such contract or agreement at any time and that the same is not was made clearly upon the condition that it was subject to the approval of
On 9 February 1974, YKS wrote PWCC a letter (Exhibit "H") requesting, subject to the ratification of the board of directors but "subject only to the the board of directors of defendant corporation. We find consistency herein
for the last time, compliance by the latter with its obligation under Exhibit declared objects and purpose of the corporation and existing laws." Hence, because according to the Corporation Law, and the By-Laws of defendant
"A". Exhibit "A" was validly entered into by Maglana and thus binds the corporation, all corporate commitments and business are conducted by,
corporation. and contracts entered into through, the express authority of the Board of
On 27 February 1974, PWCC sent an answer (Exhibit "7") to the Directors (Sec. 28, Corp. Law, Exh "I" or "8").
aforementioned letter of 9 February 1974; PWCC reiterated the The Court of Appeals reversed the decision of the Trial Court
unenforceability of Exhibit "A". Fourth. What Henry Yao and Maglana agreed upon as embodied in Exhibit
First, the defendant corporation is supervised and principally financed by "A", insofar as defendant corporation is concerned, was an unauthorized
On 4 March 1974, YKS filed with the then Court of First Instance of Leyte the National Investment and Development Corporation (NIDC), a contract (Arts. 1317 and 1403 (1), Civil Code). And because Maglana was
a complaint for Specific Performance with Damages against PWCC. subsidiary investment of the Philippine National Bank (PNB), with cash not authorized by the Board of Directors of defendant corporation nor was
financial exposure of some P10,000,000.00. PNB is a government his actuation ratified by the Board, the agreement is unenforceable (Art.
PWCC denied under oath the material averments in the complaint and financial institution whose Board is chairmaned (sic) by the Minister of 1403 (1), Civil Code; Raquiza et al. vs. Lilles et al., 13 CA Rep. 343; Gana
alleged that: (a) YKS "has no legal personality to sue having no legal National Defense. This fact is very material to the issue of whether vs. Archbishop of Manila, 43 O.G. 3224).
personality even by fiction to represent itself;" (b) Mr. Maglana, its President defendant corporation's president can bind the corporation with his own
and Chairman, was lured into signing Exhibit "A"; (c) such signing was act. It clearly results from the foregoing that the judgment appealed from is
subject to the condition that Exhibit "A" be approved by the Board of untenable. Having no cause of action against defendant corporation,
Second, for failure to deny under oath the following actionable documents plaintiff is not entitled to any relief.
Directors of PWCC, as corporate commitments are made through it; (d)
in support of defendant's counterclaim:
the latter disapproved it, hence Exhibit "A" was never consummated and
Issue:
is not enforceable against PWCC; (e) it agreed to sell 10,000 bags of white
23
Delegation of Authority to Corporate Officers | Doctrine of Apparent Authority
Whether or not the letter-offer (Exhibit A) is binding upon the private conducting the same according to the order, directives or resolutions of legally binding and implementable. As in all other contracts, there must be
respondent. the Board of Directors or of the president." It goes without saying then that a meeting of minds of the parties; the PNB and DBP must have to validly
Mr. Maglana did not have a direct and active hand in the management of adopt and ratify such FRP before they can be bound by it. In this case,
Ruling: the business and operations of the corporation. Besides, no evidence PLAINTIFFS simply relied on a legal doctrine of promissory estoppel to
was adduced to show that Mr. Maglana had, in the past, entered into support its allegation.
No. contracts similar to that of Exhibit "A" either with the petitioner or with The doctrine of promissory estoppel can hardly find application here.
other parties. The representatives of PNB and DBP are separate and distinct from the
The letter-offer is not binding. Mr. Maglana, its President and Chairman,
said bans.
was not empowered to execute it.
As a rule, a corporation has the power to appoint agents to enter into
Since a corporation, such as the private respondent, can act only through a contract in its behalf, ,but the agent, should not exceed his authority.
its officers and agents, "all acts within the powers of said corporation may 26. APT v. CA In the case at bar, the plaintiffs failed to show that the representatives
be performed by agents of its selection; and, except so far as limitations or G.R. No. 121171. December 29, 1998 of PNB and DBP had the authority to enter into the FRP contract. And if
restrictions may be imposed by special charter, by-law, or statutory they had such authority, there was no showing that the banks had ratified
provisions, the same general principles of law which govern the relation of FACTS: the FRP.
agency for a natural person govern the officer or agent of a corporation, of Marinduque Mining and Industrial Corporation (MMIC) is a MMIC DAMAGES
whatever status or rank, in respect to his power to act for the corporation; domestic corporation engaged in mining with respondents Jesus S. MMICs credit reputation was no longer a desirable one. The
and agents when once appointed, or members acting in their stead, are Cabarrus, Sr. as President. company was already suffering from serious financial crisis which projects
subject to the same rules, liabilities and incapacities as are agents of In 1968, the government undertook to support the financing of an image not compatible with good and wholesome reputation. So it could
individuals and private persons.” Moreover, "x x x a corporate officer or MMIC and it then issued debenture bonds which enable the latter to take not be said that there was a besmirched reputation by the act of
agent may represent and bind the corporation in transactions with third out loans from the DBP and PNB. The loans were mortgaged by MMIC’s foreclosure.
persons to the extent that authority to do so has been conferred upon him, assets. The case being a derivative suit, MMIC should have been impleaded
and this includes powers which have been intentionally conferred, and also MMIC had trouble paying and this exposed the government, as a party. It was not joined as a party plaintiff or party defendant at any
such powers as, in the usual course of the particular business, are because of the debenture bonds, that reached P22 billion obligation. In stage of the proceedings. As it is, the award of damages to MMIC, which
incidental to, or may be implied from, the powers intentionally, conferred, order to mitigate MMIC’s loan liability, FRP was drafted in the presence of was not a party before the Arbitration Committee, is a complete nullity.
powers added by custom and usage, as usually pertaining to the particular MMIC’s representatives as well as representatives from DBP and PNB. in a derivative suit, the corporation is the real party in interest while
officer or agent, and such apparent powers as the corporation has caused The two banks however never formally approved the said FRP. Eventually, the stockholder filing suit for the corporations behalf is only nominal party.
persons dealing with the officer or agent to believe that it has conferred.” the staggering loans became overdue and PNB and DBP chose to
foreclose MMIC’s assets, FRP no longer feasible at that point. CABBARIUS DAMAGES
While there can be no question that Mr. Maglana was an officer -- the So the assets were foreclosed and were eventually assigned to
President and Chairman -- of private respondent corporation at the time he the Asset Privatization Trust (APT). It is a basic postulate that s corporation has a personality
signed Exhibit "A", the above provisions of said private respondent's By- Later, Jesus Cabarrus, Sr., a stockholder of MMIC initiated a separate and distinct from its stockholders. The properties
Laws do not in any way confer upon the President the authority to enter derivative suit against PNB and DBP with APT being impleaded as the foreclosed belonged to MMIC, not to its stockholders. Hence, if
into contracts for the corporation independently of the Board of Directors. successor in interest of the two banks. He alleged that the foreclosure was wrong was committed in the foreclosure, it was done against the
That power is exclusively lodged in the latter. Nevertheless, to expedite or invalid because the FRP was adopted by PNB and DBP because of the corporation.
facilitate the execution of the contract, only the President -- and not all the presence of its representatives when the said FRP was drafted. The suit
members of the Board, or so much thereof as are required for the act -- NOTE:
was filed in the RTC but while the case was pending, the parties agreed to
shall sign it for the corporation. This is the import of the submit the case for arbitration. A debenture is a type of debt instrument that is not secured by
words through the president in Exhibit "8-A" and the clear intent of the Arbitration Committee ruled in favor of MMIC and declared that physical assets or collateral. Debentures are backed only by the
power of the chairman "to execute and sign for and in behalf of the (a) FRP is valid; (b) in awarded damages to MMIC; and (c) in awarded general creditworthiness and reputation of the issuer.
corporation all contracts and agreements which the corporation may enter moral damages to Jesus S. Cabarrus, Sr.
into" in Exhibit "I-1". Both powers presuppose a prior act of the corporation
exercised through the Board of Directors. No greater power can be implied ISSUE: 27. BA SAVINGS BANK vs. ROGER T. SIA, TACIANA U. SIA and
from such express, but limited, delegated authority. Neither can it be Was the foreclosure made by the 2 banks valid? JOHN DOE
logically claimed that any power greater than that expressly conferred is
G.R. No. 131214
inherent in Mr. Maglana's position as president and chairman of the RULING: July 27, 2000
corporation.
PNB and DBP had the legitimate right to foreclose the mortgages of
We note that the private corporation has a general manager who, under MMIC whose obligations were past due. FACTS:
its By-Laws has, inter alia, the following powers: "(a) to have the active The mere fact that MMIC adopted the FRP does not mean that DBP-
and direct management of the business and operation of the corporation, PNB lost the option to foreclose. Neither does it mean that the FRP is
24
Delegation of Authority to Corporate Officers | Doctrine of Apparent Authority
August 6, 1997, the Court of Appeals issued a Resolution denying due non-forum shopping and other instruments necessary for such action and from Bacolod-Murcia the modification of the concession shares pursuant
course to a Petition for Certiorari filed by BA Savings Bank, on the ground proceeding. The Resolution was sufficient to vest such persons with the to the 1936 Board Resolution.
that the Certification on anti-forum shopping incorporated in the petition authority to bind the corporation and was specific enough as to the acts
was signed not by the duly authorized representative of the petitioner, as they were empowered to do. In its defense, the Resolution is void for being patently defective.
required under Supreme Court Circular No. 28-91, but by its counsel, in It adopted an amendment of the 1919 Milling Contract when the same was
contravention of said circular. Circular 28-91 was prescribed by the Supreme Court to prohibit and only signed after the issuance of the Board Resolution. The amended
penalize the evils of forum shopping. There is no circumvention of this concessions given to the planters under the resolution is in the form of a
A Motion for Reconsideration was subsequently filed by the petitioner, rationale if the certificate was signed by the corporations specifically donation since it was unilaterally given to them, which is thus, beyond the
attached to which was a BA Savings Bank Corporate Secretary’s authorized counsel, who had personal knowledge of the matters required powers of the corporation, and therefore, ultra vires. The trial court favored
Certificate, dated August 14, 1997. The Certificate showed that the in the Circular. the milling company, hence the appeal to the SC.
petitioners Board of Directors approved a Resolution on May 21, 1996,
authorizing the petitioners lawyers to represent it in any action or Issue:
proceeding before any court, tribunal or agency; and to sign, execute and Is the Board of Directors vested with the authority to adopt the
deliver the Certificate of Non-forum Shopping, among others. 28. MONTELIBANO v. BACOLOD MURCIA amended contract that was not perfected?
ALFREDO MONTELIBANO, ET AL. vs. BACOLOD-MURCIA
October 24, 1997, the Motion for Reconsideration was denied by the Court MILLING CO., INC. Ruling:
of Appeals on the ground that Supreme Court Revised Circular No. 28-91 GR No. L-15092 Yes, the Board of Directors has such authority.
requires that it is the petitioner, not the counsel, who must certify under oath May 18, 1962
to all of the facts and undertakings required therein. The Supreme Court ruled that the adoption of the amended
Quiason, J. concession agreement on August 20, 1936 operates as a supplement to
ISSUE: the said amendment; in fact, the Court held that the resolution amended
Facts: the 1936 agreement.
Whether or not a corporation is allowed to authorize its counsel to execute Montelibano, et al. and the limited co-partnership of Gonzaga
a certificate of non-forum shopping for its behalf. and Company, had been sugar planters and in the sugar industry, and has By the time the amendment was signed in September 10, 1936,
adhered to the concessions of Bacolod-Murcia Milling Co. the act of signing the same by both parties already signifies the meeting of
HELD: the minds of both parties as to the agreement. The Supreme Court
The sequence of relevant events as established is as follows: presumed that both parties understood that the Proposed Amended
Yes. Concession and the Resolution, purporting to have amended the former,
1919: the parties entered into a milling contract for a term of 30 years contain the controlling terms for both parties.
A corporation, such as the petitioner, has no powers except those beginning from the crop year 1920-1921. The shares of the
expressly conferred on it by the Corporation Code and those that are planters and mill will be divided into 45% for the mill and 55% for The Resolution does not pertain to a different transaction, in fact,
implied by or are incidental to its existence. In turn, a corporation exercises the planters the reference to the amended concessions was clear: the term of the
said powers through its board of directors and/or its duly authorized officers concessions was the same; the cause and consideration were likewise the
and agents. Physical acts, like the signing of documents, can be performed 1936: the parties drafted a proposed amendment of the 1919 Milling same; it reiterated the rights and obligations of the planters and it reiterated
only by natural persons duly authorized for the purpose by corporate contract, which provided among others, the extension of the term the 15-year extension of the 1919 Milling Contract.
bylaws or by a specific act of the board of directors. All acts within the to 45 years and modifying the shares into 40% for the mill and
powers of a corporation may be performed by agents of its selection; and, 60% for the planters Thus, the Supreme Court held in favor of Montelibano the appeal
except so far as limitations or restrictions which may be imposed by special is granted and hereby reversing the trial court decision.
charter, by-law, or statutory provisions, the same general principles of law August 20, 1936: The Board issued a Resolution providing that the Board
which govern the relation of agency for a natural person govern the officer shall adopt the terms and conditions of the 1936 Amendment.
or agent of a corporation, of whatever status or rank, in respect to his power Section 9.a also provides that it shall synchronize its concession 29. POWERS v. MARSHALL
to act for the corporation; and agents once appointed, or members acting shares with that of the shares established by the Negros sugar ANTHONY POWERS, BERTEL FASSNACHT, RICHARD I
in their stead, are subject to the same rules, liabilities and incapacities as centrals, namely (La Carlota, Binalbagan-Isabela, and San GUARDIAN, JOANN KELLY, LANDLESS, AMADO MACASAET,
are agents of individuals and private persons. Carlos) JAVIER MACICIORATUSHI NAKAI KAY NG, JAMES ROBERSON,
FREDERICK SEGGERMAN, ARTHUR YANG, EZRA TOEG, ISIDRO
The corporation’s board of directors issued a Resolution specifically September 10, 1936: the Amended Milling Contract was signed. CO, In behalf of themselves and 316 other Associate Members all
authorizing its lawyers to act as their agents in any action or proceeding other Associate Members similarly situated, and in behalf of and for
before the Supreme Court, the Court of Appeals, or any other tribunal or In 1953, the concessions of the Negros Sugar Centrals declared the benefit of the INTERNATIONAL SCHOOL, INC., vs. DONALD I.
agency[;] and to sign, execute and deliver in connection therewith the a standardized share of 62.5% in favor of the planters. This prompted MARSHALL, CHARLES ANGEVINE, CARLOS D. ARGUELLES,
necessary pleadings, motions, verification, affidavit of merit, certificate of herein respondents to demand, through an action for specific performance, BRYCE F. BASTIAN, GABRIEL DIMACHE, JOSE FLORENTO,
25
Delegation of Authority to Corporate Officers | Doctrine of Apparent Authority
JAMES T. HODGE, ROSEMARY IYAS, EUSEBIO R. LUZURIAGA, On July 16, 1975 the plaintiffs filed a complaint for injunction against the school's standard of instruction, which is ordained in fact by Presidential
THOMAS C. NIBLOCK Board of Trustees of the International school. Decree No. 732, the expansion cannot result in any radical or fundamental
School, Inc., and MAX SNYDER Superintendent, International change in the kind of activity being conducted by the school that might
School, Inc. The trial court issued an order temporarily restraining the defendants or require the consent of the members composing it.
their authorized representatives and agents: from executing and/or
FACTS: enforcing in any manner the development program that they had adopted Since the collection of the development fee had been approved by the
for the raising of funds to put up new building and provide for the Board of Trustees of the International School, Inc., it was a valid exercise
On July 16, 1975, the fourteen (14) plaintiffs, all associate members of the remodelling of existing ones belonging to the International School, Inc., or of corporate power by the Board, and said assessment was binding upon
International School, Inc., brought an action for injunction in the Court of from otherwise requiring as a pre-requisite to the re-enrollment for the all the members of the corporation. Their action to stop the collection of
First Instance of Rizal, against the ten (10) members of the Board of school year 1975-1976 of the children of the plaintiffs and other similarly said fee was correcty dismissed by the trial court for lack of a valid cause
Trustees of the school, praying that said Trustees be enjoined from situated the payment of the development fee or charge imposed under the of action against the school.
collecting a "development fee" of P2,625.00 per child-enrollee per school said development plan; and from requiring the payment of the matriculation
year for a period of twelve (12) years, beginning with the school year 1975- fee likewise imposed pursuant to said development as a pre-requisite for
1976, as a pre-requisite for re-enrollment in said school. the enrollment for the first time of the children of the associate members of 30. PREMIUM MARBLE V. CA AND INTERNATIONAL CORPORATE
the International School, Inc. BANK
On May 19, 1975, Donald I. Marshall, president of the Board of Trustees After the submission of the parties' memoranda the trial court issued an G.R. No. 96551, November 4, 1996
of the International School in Makati, Metro Manila, sent a letter addressed order on November 18, 1975, dismissing the complaint for lack of valid
to the parents of the students, giving notice that the Board of Trustees had cause of action, and dissolved the restraining order of July 17, 1975. Facts:
decided to embark on a program to construct new buildings and remodel
existing ones to accommodate the increasing enrollment in the school, and ISSUE: Premium Marble Resources, Inc. (Premium), assisted by Atty.
that it was necessary for the school to raise P35,000,000.00 for this
Arnulfo Dumadag (Atty. Dumadag) as counsel, filed an action for damages
purpose. The Board intended to raise the needed funds primarily through Was Board of Trustees of the International School authorized to adopt the
subscriptions to capital notes and prepayment certificates, and any development plan for which the disputed fee was being collected from the against International Corporate Bank (The Bank). The case started when
deficiency from these sources would be covered by collecting a so-called students? Ayala Investment and Development Corporation issued three (3) checks
"development fees" of P2,625 from each enrollee starting with the school payable to Premium and drawn against Citibank. Then, former officers of
year 1975-1976 and continuing up to the school year 1986-1987. RULING: Premium headed by Saturnino Belen, Jr. (Belen, Jr.) without any authority
from Premium deposited the said checks to the current account of his
On June 11, 1975, the school superintendent, Dr. Max Snyder, acting Section 2 of Article 3 of the By-Laws of the International School, Inc.
conduit corporation, Intervest Merchant Finance (Intervest). Although the
under instructions from the Board of Trustees, wrote a letter to the parents provides:
of returning students, enclosing an Application for Admission which Section 2. Powers and Duties. — The Board of Trustees, in check was a cross check payable only to Premium, The Bank accepted
specifically advised that the payment of the development fee was a pre- addition to the powers conferred by these By-Laws, shall have the checks and deposit the same with the account of Intervest, thus
requisite for re-enrollment. the right to such powers and do such acts as may be lawfully allowing the latter to use the funds to the prejudice of Premium. Premium
exercised or performed by the corporation, subject to applicable then demanded from The Bank to restitute the amount representing the
The plaintiffs, who are associate members of the International School, Inc., laws and to the provisions of the articles of incorporation and the value of the check but The Bank refused to do so. Hence, Premium filed
protested against the imposition of the development fee of P2,625.00 per By-Laws x x x (Article III,By-Laws.) the case.
student per year for twelve (12) years. In a petition dated June 18, 1975,
they requested the Board of Trustees "to suspend the implementation of Section 2 (b) of P.D. No. 732 granting certain rights to the International
In the meantime, the same corporation Premium, but this time
the requirement of payment of the development fee as a pre-requisite to School, Inc., expressly authorized the Board of Trustees "upon
final enrollment or re-enrollment for the school year 1975-1976. consultation with the Secretary of Education and Culture, ... to determine represented by Siguion Reyna, Montecillio and Ongsiako Law Office
the amount of fees and assessments which may be reasonably imposed (Siguion Reyna Law Firm) as counsel, filed a motion to dismiss on the
On July 7, 1975, Donald Marshall, signing for the Board of Trustees and upon its students, to maintain or conform to the school standard of ground that the filing of the case was without authority from its duly
as President of the School: education." Such consultation had been made with the Secretary of constituted board of directors. Premium thru Atty. Dumadag, in its
1. extended the deadline for the selection of the option by whch the Education and Culture who expressed his conformity with the opposition to the motion to dismiss contended that the persons who
Development Fee is to be paid from July 15 to July 22, 1975; reasonableness of the assessment of P2,625.00 per student for the whole signed the board resolution namely Belen, Jr., Alberto Nograles, &
2. allowed deferred payment thereof from August 1, 1975 to school year to carry out its development program.
Jose Reyes, are not directors of the corporation and were allegedly
October 13, 1975 (beginning of the second quarter) and allowing
quarterly payment thereof; The expansion of the school facilities, which is to be done by improving old former officers and stockholders of Premium who were dismissed for
3. granted "assistance" on a case to case basis. buildings and/or constructing new ones, is an ordinary business various irregularities and fraudulent acts; that Siguion Reyna Law office
transaction well within competence of the Board of Trustees to act upon, ... is the lawyer of Belen and Nograles and not of Premium and that the
Being directly related to the purpose of elevating and maintaining the Articles of Incorporation of Premium shows that Belen, Nograles and
26
Delegation of Authority to Corporate Officers | Doctrine of Apparent Authority
Reyes are not majority stockholders. Siguion Reyna Law Firm as Augusto I. Galace Treasurer Jose Ramirez, as representative of his father, placed in the
counsel of Premium asserted that it is the general information sheet filed hands of Ramon J. Fernandez an offer stating detail the terms upon which
with the Securities and Exchange Commission, among others, that is the Jose L.R. Reyes Secretary/Director the plaintiff would undertake to supply from Paris the aforesaid films.
Accordingly, Ramon J. Fernandez had an informal conference with all the
best evidence that would show who are the stockholders of a corporation
Pido E. Aguilar Director members of the company's board of directors except one, and with
and not the Articles of Incorporation since the latter does not keep track of approval of those with whom he had communicated, addressed a letter to
the many changes that take place after new stockholders subscribe to Jose Ramirez accepting the offer contained in the memorandum for the
Saturnino G. Belen, Jr. Chairman of the Board.
corporate shares of stocks. The Bank filed its manifestation adopting the exclusive agency of the Eclair films. A few days later, he addressed
motion to dismiss filed by premium thru Atty. Dumadag. another letter couched in the same terms, likewise accepting the office of
While the Minutes of the Meeting of the Board on April 1, 1982 states that
the exclusive agency for the Milano Films.
the newly elected officers for the year 1982 were Oscar Gan, Mario
The RTC granted the motion to dismiss and ruled that the
Zavalla, Aderito Yujuico and Rodolfo Millare, Premium failed to show proof
persons represented by Atty. Dumadag do not yet have the legal capacity These communications were signed in which it will be noted the
that this election was reported to the SEC. In fact, the last entry in their separate signature of R. J. Fernandez, as an individual, is placed
to sue for and in behalf of the Premium and for filing of the case for
General Information Sheet with the SEC, as of 1986 appears to be the set somewhat below and to the left of the signature of the Orientalist Company
damages. It found that Alberto Nograles, Fernando Hilario, Augusto
of officers elected in March 4, 1981. The claim, therefore, of Premium as as singed by R. J. Fernandez, in the capacity of treasurer.
Galace, Jose L.R. Reyes, Pido Aguilar and Saturnino Belen, Jr., are
represented by Atty. Dumadag, that Zaballa, et al., are the incumbent
presumably the officers of the Premium as indicated in its Articles of
officers of Premium has not been fully substantiated. In the absence of an However, when the films arrived, Orientalist was without fund to
Incorporation. On appeal, the Court of Appeals affirmed the trial court’s
authority from the board of directors, no person, not even the officers of the pay the cost and expenses incident to each shipment. In effect the
order.
corporation, can validly bind the corporation. company‘s president B. Hernandez paid said obligations and treated the
films by him as his own property; and they in fact never came into the actual
Issue: possession of the Orientalist Company as owner at all, though it is true
Hernandez rented the films to the Orientalist Company and they were
Was the filing of the case for damages against The Bank was authorized exhibited by it in the Oriental Theater under an arrangement which was
31. J.F. RAMIREZ V. THE ORIENTALIST CORPORATION
by a duly constituted Board of Directors of the petitioner corporation? made between him and the theater's manager. However, subsequent
G.R. No. 11897 September 24, 1918
deliveries were no longer paid by any of the concerned party.
Ruling:
Facts:
Thereupon this action was instituted by the plaintiff against the
No. The filing for damages against The Bank was not authorized. Orientalist Company, and Ramon J. Fernandez. As the films which
The Orientalist Company is a corporation, duly organized under accompanied the dishonored were liable to deteriorate, the court, upon
In the absence of any board resolution from its board of directors the laws of the Philippine Islands. It is engaged in the business of application of the plaintiff, and apparently without opposition on the part of
of the authority to act for and in behalf of the corporation, the action for maintaining and conducting a theatre in the city of Manila for the exhibition the defendants, appointed a receiver who took charge of the films and sold
damages must necessarily fail. The power of the corporation to sue and of cinematographic films. them. The amount of P6018.93 was realized from this sale and was
be sued in any court is lodged with the board of directors that exercises its applied to the satisfaction of the plaintiff's claim and was accordingly
The plaintiff J. F. Ramirez was, a resident of the city of Paris, delivered to him in part payment thereof.
corporate powers. Thus, the issue of authority and the invalidity of plaintiff-
France, and was engaged in the business of marketing films for
appellants subscription which is still pending, is a matter that is also
manufacturers, there engaged in the production or distribution of Issue:
addressed, considering the premises, to the sound judgment of the cinematographic material.
Securities & Exchange Commission
1. Is Orientalist Co. liable for the acts of Ramon Fernandez, its
Certain of the directors of the Orientalist Company became treasurer?
Following the general information sheet and the Certification issued by the
apprised of the fact that the plaintiff in Paris had control of the agencies for 2. Whether Jose Ramirez is considered a guarantor
SEC on August 19, 1986 that as of March 4, 1981, the officers and two different marks of films, namely, the "Eclair Films" and the Ruling:
members of the board of directors of the Premium Marble Resources, Inc. "Milano Films;" and negotiations were begun with said officials of the
were: Orientalist Company by Jose Ramirez, as agent of the plaintiff, for the
purpose of placing the exclusive agency of these films in the hands of the 1. Yes. Section 28 of the Corporation Law provides that the
Alberto C. Nograles President/Director Orientalist Company. The defendant Ramon J. Fernandez, one of the corporate power shall be exercised by the board of directors
directors of the Orientalist Company and also its treasure, was chiefly including the power to make contracts. However, Section 103 of
Fernando D. Hilario Vice President/Director active in this matter. the Code of Civil Procedures provides that if the foundation of
the suit is a written instrument and it is not denied upon oath, it
27
Delegation of Authority to Corporate Officers | Doctrine of Apparent Authority
shall be deemed to be admitted. It requires that the Answer The Labor Arbiter, in its decision, declared Real as having been claim that he is a corporate officer. Having said this, we find that there is no
setting up the defense of lack of authority of an officer of a illegally dismissed. The LA found no convincing proof of the causes for intra-corporate relationship between the parties insofar as Real’s complaint
corporation to bind it by a contract should be verified and the which Real was terminated and that there was complete absence of due for illegal dismissal is concerned and that same does not satisfy the
denial contemplated must be specific. process. relationship test.

In this case, the failure of the defendant corporation to make any Sangu appealed to the NLRC citing lack of jurisdiction claiming ISSUE: Does the case involve an intra-corporate controversy?
issue in its answer with regard to the authority of Ramon J. that Real is both a stockholder and a corporate officer of Sangu, hence, the
Fernandez to bind it, and particularly its failure to deny action is an intra-corporate controversy to which the LA has no jurisdiction. SC: No.
specifically under oath the genuineness and due execution of the
contracts sued upon, have the effect of eliminating the question While admitting that he is indeed a stockholder of Sangu, Real To determine whether a case involves an intra-corporate
of his authority from the case. nevertheless posited that his being a stockholder and his being a controversy, and is to be heard and decided by the branches of the RTC
managerial employee do not ipso facto confer upon him the status of a specifically designated by the Court to try and decide such cases, two
Moreover, if a corporation knowingly permits one of its officers, corporate officer. He argues that a corporate officer is one who holds an elements must concur: (a) the status or relationship of the parties, and (2)
or any other agent, to do acts within the scope of an apparent elective position as provided in the Articles or one who is appointed to such the nature of the question that is the subject of their controversy.
authority and thus holds him out to the public as possessing other positions as provided in the By-Laws. He says he was neither elected
power to do those acts, the corporation will, as against anyone nor appointed but rather he was hired to be the manager of Sangu by From the previous issue, we have determined that there is no
who has in good faith dealt with the corporation through such another board member. intra-corporate relationship between Sangu and Real as it has been settled
agent, be estopped from denying his authority. In this case, it will that Real was neither elected nor appointed to his position as manager, but
be observed that Ramon J. Fernandez was the particular officer ISSUE: Is Real a corporate officer? was rather hired, and thus, a mere employee.
and member of the board of directors who was most active in the
effort to secure the films for the corporation. The negotiations HELD: No. We now go to the nature of controversy test. As earlier stated,
were conducted by him with the knowledge and consent of other respondents terminated the services of petitioner for the following reasons:
members of the board; and the contract was made with their "‘Corporate officers’ in the context of Presidential Decree No. (1) continuous absences; (2) loss of trust and confidence; and, (3) to cut
prior approval. As appears from the papers in this record, 902-A are those officers of the corporation who are given that character by down operational expenses. From these, it is not difficult to see that the
Fernandez was the person to who keeping was confided the the Corporation Code or by the corporation’s by-laws. There are three reasons given by Sangu for dismissing Real have something to do with his
printed stationery bearing the official style of the corporation, as specific officers whom a corporation must have under Section 25 of the being a Manager of the corporation and nothing with his being a director or
well as rubber stencil with which the name of the corporation Corporation Code. These are the president, secretary and the treasurer. stockholder. Thus, when Real sought for reinstatement, he wanted to
could be signed to documents bearing its name. The number of officers is not limited to these three. A corporation may have recover his position as Manager, a position which we have, however,
such other officers as may be provided for by its by-laws like, but not limited earlier declared to be not a corporate position. He is not trying to recover a
2. Yes. As appears upon the face of the contracts, the signature of to, the vice-president, cashier, auditor or general manager. The number of seat in the board of directors or to any appointive or elective corporate
Fernandez, in his individual capacity, is not in line with the corporate officers is thus limited by law and by the corporation’s by-laws." position which has been declared vacant by the board. Certainly, what we
have here is a case of termination of employment which is a labor
signature of the Orientalist Company, but is set off to the left of
Sangu claims Real was appointed by virtue of Section 1, Article controversy and not an intra-corporate dispute. In sum, we hold that Real’s
the company's signature and somewhat who sign contracts in IV of the Sangu By-Laws which says “The Board, may from time to time, complaint likewise does not satisfy the nature of controversy test.
some capacity other than that of principal obligor to place their appoint such other officers as it may determine to be necessary or
signature alone would justify a court in holding that Fernandez proper.” With the elements of intra-corporate controversy being absent in
here took upon himself the responsibility of a guarantor rather this case, we thus hold that petitioner’s complaint for illegal dismissal
than that of a principal obligor. An examination of the records show that there is no copy of any against Sangu is not intra-corporate. Real’s termination thru a board
board resolution or any other document evidencing that the board resolution does not satisfy the requirement of due process, and thus, he
appointed Real as manager. While Sangu repeatedly claims that Real was was illegally dismissed.
32. RENATO REAL v. SANGU PHILS. appointed as Manager pursuant to the corporation’s By-Laws, the
G.R. No. 168757; January 19, 2011 inconsistencies in their allegations as to how Real was placed in said
position, coupled by the fact that they failed to produce any documentary
FACTS: Renato Real was the manager of Sangu Phils. Corp. In 2001, evidence to prove that he was appointed thereto by action or with approval 33. MATLING v. COROS
Real was terminated by Sangu from his position as manager thru a board of the board, only leads this Court to believe otherwise. 13 October 2010
resolution. Real complained that he was not notified of such meeting. He
just received a letter stating that he had been terminated for the following It has been consistently held that "[a]n ‘office’ is created by the Facts:
reasons: (1) continuous absences; (2) loss of trust and confidence; and (3) charter of the corporation and the officer is elected (or appointed) by the After his dismissal by Matling as its Vice President for Finance and
to cut down on operational expenses. directors or stockholders." Clearly here, Sangu failed to prove that Real Administration, the Ricardo R. Coros filed a complaint for illegal suspension
was appointed by the board of directors. Thus, we cannot subscribe to their and illegal dismissal against Matling and some of its corporate officers in
28
Delegation of Authority to Corporate Officers | Doctrine of Apparent Authority
the NLRC. Matling moved to dismiss the complaint, raising the ground, Matling's President to create non-corporate offices to be occupied by
among others, that the complaint pertained to the jurisdiction of the Section 25. Corporate officers, quorum.--Immediately after their election, ordinary employees of Matling. Such powers were incidental to the
Securities and Exchange Commission (SEC) due to the controversy being the directors of a corporation must formally organize by the election of a President's duties as the executive head of Matling to assist him in the daily
intra-corporate inasmuch as the respondent was a member of Matling's president, who shall be a director, a treasurer who may or may not be a operations of the business. Not every conflict between a corporation and
Board of Directors aside from being its Vice-President for Finance and director, a secretary who shall be a resident and citizen of the Philippines, its stockholders involves corporate matters that only the SEC can resolve
Administration prior to his termination. and such other officers as may be provided for in the by-laws. Any two (2) in the exercise of its adjudicatory or quasi-judicial powers. If, for example,
or more positions may be held concurrently by the same person, except a person leases an apartment owned by a corporation of which he is a
Ricardo insisted that his status as a member of Matling's Board of Directors that no one shall act as president and secretary or as president and stockholder, there should be no question that a complaint for his ejectment
was doubtful, considering that he had not been formally elected as such; treasurer at the same time. for non-payment of rentals would still come under the jurisdiction of the
that he did not own a single share of stock in Matling, considering that he regular courts and not of the SEC. By the same token, if one person injures
had been made to sign in blank an undated indorsement of the certificate The directors or trustees and officers to be elected shall perform the duties another in a vehicular accident, the complaint for damages filed by the
of stock he had been given in 1992; that Matling had taken back and enjoined on them by law and the by-laws of the corporation. Unless the victim will not come under the jurisdiction of the SEC simply because of the
retained the certificate of stock in its custody; and that even assuming that articles of incorporation or the by-laws provide for a greater majority, a happenstance that both parties are stockholders of the same corporation.
he had been a Director of Matling, he had been removed as the Vice majority of the number of directors or trustees as fixed in the articles of A contrary interpretation would dissipate the powers of the regular courts
President for Finance and Administration, not as a Director, a fact that the incorporation shall constitute a quorum for the transaction of corporate and distort the meaning and intent of PD No. 902-A.
notice of his termination. business, and every decision of at least a majority of the directors or
trustees present at a meeting at which there is a quorum shall be valid as The circumstances surrounding his appointment to office must be fully
LA: Ricardo was a corporate officer because he was occupying the a corporate act, except for the election of officers which shall require the considered to determine whether the dismissal constituted an intra-
position of Vice President for Finance and Administration and at the same vote of a majority of all the members of the board. corporate controversy or a labor termination dispute. Obviously enough,
time was a Member of the Board of Directors of Matling; and that, the respondent was not appointed as Vice President for Finance and
consequently, his removal was a corporate act of Matling and the Directors or trustees cannot attend or vote by proxy at board meetings. Administration because of his being a stockholder or Director of Matling.
controversy resulting from such removal was under the jurisdiction of the his promotion to the position of Vice President for Finance and
SEC, pursuant to Section 5, paragraph (c) of Presidential Decree No. 902. Conformably with Section 25, a position must be expressly mentioned in Administration in 1987 was by virtue of the length of quality service he had
the By-Laws in order to be considered as a corporate office. Thus, the rendered as an employee of Matling. His subsequent acquisition of the
NLRC: Ricardo’s complaint for illegal dismissal was properly cognizable by creation of an office pursuant to or under a By-Law enabling provision is status of Director/stockholder had no relation to his promotion.
the LA, not by the SEC, because he was not a corporate officer by virtue not enough to make a position a corporate office.
of his position in Matling, albeit high ranking and managerial, not being
among the positions listed in Matling's Constitution and By-Laws. An "office" is created by the charter of the corporation and the officer is
elected by the directors or stockholders. On the other hand, an employee 34. MANILA METAL CONTAINER CORP. v PNB, GR 166862
CA: Coros' alleged illegal dismissal therefrom is, therefore, within the occupies no office and generally is employed not by the action of the DECEMBER 20, 2006
jurisdiction of the labor arbiter. directors or stockholders but by the managing officer of the corporation
who also determines the compensation to be paid to such employee. J. CALLEJO SR.
Issue: Was Ricardo a corporate officer of Matling?
This interpretation is the correct application of Section 25 of the Corporation FACTS:
Held: No. Matling's By-Laws did not list his position as Vice President for Code, which plainly states that the corporate officers are the President,
Finance and Administration as one of the corporate offices; that Matling's Secretary, Treasurer and such other officers as may be provided for in the
Petitioner was the owner of a 8,015 square meter parcel of land
By-Law No. III listed only four corporate officers, namely: President, By-Laws. Accordingly, the corporate officers in the context of PD No. 902-
located in Mandaluyong (now a City), Metro Manila. The property was
Executive Vice President, Secretary, and Treasurer; that the corporate A are exclusively those who are given that character either by the
covered by Transfer Certificate of Title (TCT) No. 332098 of the Registry
offices contemplated in the phrase "and such other officers as may be Corporation Code or by the corporation's By-Laws. Moreover, the Board
of Deeds of Rizal. To secure a P900,000.00 loan it had obtained from
provided for in the by-laws" found in Section 25 of the Corporation Code of Directors of Matling could not validly delegate the power to create a
respondent Philippine National Bank (PNB), petitioner executed a real
should be clearly and expressly stated in the By-Laws; that the fact that corporate office to the President, in light of Section 25 of the Corporation
estate mortgage over the lot. Respondent PNB later granted petitioner a
Matling's By-Law No. III dealt with Directors & Officers while its By-Law No. Code requiring the Board of Directors itself to elect the corporate officers.
new credit accommodation of P1,000,000.00; and, on November 16,
V dealt with Officers proved that there was a differentiation between the Verily, the power to elect the corporate officers was a discretionary power
1973, petitioner executed an Amendment4 of Real Estate Mortgage over
officers mentioned in the two provisions, with those classified under By- that the law exclusively vested in the Board of Directors, and could not be
its property. On March 31, 1981, petitioner secured another loan of
Law No. V being ordinary or non-corporate officers; and that the officer, to delegated to subordinate officers or agents. The office of Vice President for
P653,000.00 from respondent PNB, payable in quarterly installments of
be considered as a corporate officer, must be elected by the Board of Finance and Administration created by Matling's President pursuant to By
P32,650.00, plus interests and other charges.5 On August 5, 1982,
Directors or the stockholders, for the President could only appoint an Law No. V was an ordinary, not a corporate, office.
respondent PNB filed a petition for extrajudicial foreclosure of the real
employee to a position pursuant to By-Law No. V.
estate mortgage and sought to have the property sold at public auction for
To emphasize, the power to create new offices and the power to appoint
P911,532.21, petitioner's outstanding obligation to respondent PNB as of
Section 25 of the Corporation Code provides: the officers to occupy them vested by By-Law No. V merely allowed
29
Delegation of Authority to Corporate Officers | Doctrine of Apparent Authority
June 30, 1982,6 plus interests and attorney's fees. After due notice and was why it had paid P725,000.00. Petitioner warned respondent PNB that property for P1,574,560.47. Any acceptance by the SAMD of petitioner's
publication, the property was sold at public auction on September 28, 1982 it would seek judicial recourse should PNB insist on the position.18 On June offer would not bind respondent. As this Court ruled in AF Realty
where respondent PNB was declared the winning bidder for 4, 1985, respondent PNB informed petitioner that the PNB Board of Development, Inc. vs. Diesehuan Freight Services, Inc.:60
P1,000,000.00. The Certificate of Sale7 issued in its favor was registered Directors had accepted petitioner's offer to purchase the property, but for
with the Office of the Register of Deeds of Rizal, and was annotated at the P1,931,389.53 in cash less the P725,000.00 already deposited with it.19 Section 23 of the Corporation Code expressly provides that the
dorsal portion of the title on February 17, 1983. Thus, the period to redeem On page two of the letter was a space above the typewritten name of corporate powers of all corporations shall be exercised by the
the property was to expire on February 17, 1984. petitioner's President, Pablo Gabriel, where he was to affix his signature. board of directors. Just as a natural person may authorize
However, Pablo Gabriel did not conform to the letter but merely indicated another to do certain acts in his behalf, so may the board of
Petitioner sent a letter dated August 25, 1983 to respondent therein that he had received it.20 Petitioner did not respond, so PNB directors of a corporation validly delegate some of its functions to
PNB, requesting that it be granted an extension of time to requested petitioner in a letter dated June 30, 1988 to submit an amended individual officers or agents appointed by it. Thus, contracts or
redeem/repurchase the property.8 In its reply dated August 30, 1983, offer to repurchase. acts of a corporation must be made either by the board of
respondent PNB informed petitioner that the request had been referred to directors or by a corporate agent duly authorized by the board.
its Pasay City Branch for appropriate action and recommendation.9 In a Petitioner rejected respondent's proposal in a letter dated July Absent such valid delegation/authorization, the rule is that the
letter10 dated February 10, 1984, petitioner reiterated its request for a one 14, 1988. It maintained that respondent PNB had agreed to sell the declarations of an individual director relating to the affairs of the
year extension from February 17, 1984 within which to redeem/repurchase property for P1,574,560.47, and that since its P725,000.00 downpayment corporation, but not in the course of, or connected with the
the property on installment basis. It reiterated its request to repurchase the had been accepted, respondent PNB was proscribed from increasing the performance of authorized duties of such director, are held not
property on installment.11 Meanwhile, some PNB Pasay City Branch purchase price of the property.21 Petitioner averred that it had a net balance binding on the corporation.
personnel informed petitioner that as a matter of policy, the bank does not payable in the amount of P643,452.34. Respondent PNB, however,
accept "partial redemption."12 Since petitioner failed to redeem the rejected petitioner's offer to pay the balance of P643,452.34 in a letter Thus, a corporation can only execute its powers and transact its business
property, the Register of Deeds cancelled TCT No. 32098 on June 1, 1984, dated August 1, 1989.22 On August 28, 1989, petitioner filed a complaint through its Board of Directors and through its officers and agents when
and issued a new title in favor of respondent PNB.13 Petitioner's offers had against respondent PNB for "Annulment of Mortgage and Mortgage authorized by a board resolution or its by-laws.
not yet been acted upon by respondent PNB. Meanwhile, the Special Foreclosure, Delivery of Title, or Specific Performance with Damages."
Assets Management Department (SAMD) had prepared a statement of While the case was pending, respondent PNB demanded, on September
account, and as of June 25, 1984 petitioner's obligation amounted to 20, 1989, that petitioner vacate the property within 15 days from notice,27
P1,574,560.47. This included the bid price of P1,056,924.50, interest, but petitioners refused to do so. On March 18, 1993, petitioner offered to
advances of insurance premiums, advances on realty taxes, registration repurchase the property for P3,500,000.00.28 The offer was however 35. BIENVENIDO ONGKINGCO, as President and GALERIA DE
expenses, miscellaneous expenses and publication cost.14 When apprised rejected by respondent PNB, in a letter dated April 13, 1993. According to MAGALLANES CONDOMINIUM ASSOCIATION, INC., v. NLRC
of the statement of account, petitioner remitted P725,000.00 to respondent it, the prevailing market value of the property was approximately G.R. No. 119877. March 31, 1997 (KAPUNAN, J.:)
PNB as "deposit to repurchase," and Official Receipt No. 978191 was P30,000,000.00, and as a matter of policy, it could not sell the property for
issued to it.15 less than its market value.29 On June 21, 1993, petitioner offered to DOCTRINE:
purchase the property for P4,250,000.00 in cash.30 The offer was again “A corporate officer's dismissal is always a corporate act and/or
rejected by respondent PNB on September 13, 1993. an intra-corporate controversy and that nature is not altered by the reason
In the meantime, the SAMD recommended to the management
or wisdom which the Board of Directors may have in taking such action.”
of respondent PNB that petitioner be allowed to repurchase the property
for P1,574,560.00. In a letter dated November 14, 1984, the PNB
FACTS:
management informed petitioner that it was rejecting the offer and the
Petitioner Galeria de Magallanes Condominium Association,
recommendation of the SAMD. It was suggested that petitioner purchase ISSUE/S: Inc. (Galeria) is a non-stock, non-profit corporation formed in accordance
the property for P2,660,000.00, its minimum market value. Respondent
with R.A. No. 4726, otherwise known as the Condominium Act, whose
PNB gave petitioner until December 15, 1984 to act on the proposal;
WHETHER OR NOT PETITIONER AND RESPONDENT PNB primary purpose is to hold title to the common areas of the Galeria de
otherwise, its P725,000.00 deposit would be returned and the property
HAD ENTERED INTO A PERFECTED CONTRACT FOR PETITIONER Magallanes Condominium Project and to manage and administer the
would be sold to other interested buyers.16 Petitioner, however, did not
TO REPURCHASE THE PROPERTY FROM RESPONDENT. same for the use and convenience of the residents and/or owners."
agree to respondent PNB's proposal. Instead, it wrote another letter dated
Petitioner Bienvenido Ongkingco was the president of Galeria at the time
December 12, 1984 requesting for a reconsideration. Respondent PNB
private respondent filed his complaint.
replied in a letter dated December 28, 1984, wherein it reiterated its
Galeria's Board of Directors appointed private respondent
proposal that petitioner purchase the property for P2,660,000.00. PNB
Federico B. Guilas as Administrator/Superintendent (monthly salary
again informed petitioner that it would return the deposit should petitioner RULING: of P10,000). As Administrator, private respondent was tasked with the
desire to withdraw its offer to purchase the property.17 On February 25,
maintenance of the "performance and elegance of the common areas of
1985, petitioner, through counsel, requested that PNB reconsider its letter
There is no evidence that the SAMD was authorized by the condominium and external appearance of the compound thereof for
dated December 28, 1984. Petitioner declared that it had already agreed
respondent's Board of Directors to accept petitioner's offer and sell the the convenience and comfort of the residents as well as to keep up the
to the SAMD's offer to purchase the property for P1,574,560.47, and that
30
Delegation of Authority to Corporate Officers | Doctrine of Apparent Authority
quality image, and hence the value of the investment for the owners In the present case, Guilas was indeed a corporate officer. He was Lao brought a complaint for accounting and damages against
thereof." appointed directly by the Board of Directors not by any managing officer of the corporation. During the pendency of the said civil case, Esteban co,
However, on 17 March 1992, through a resolution passed by the the corporation and his salary was, likewise, set by the same Board. representing the corporation as its new vice-president filed an estafa case
Board of Directors of Galeria, private respondent was not re-appointed as Having thus determined, his dismissal or non-appointment is clearly an against Lao. Without awaiting the termination of the criminal case, Lao
Administrator. As a result, Guilas instituted a complaint against petitioners intra-corporate matter and jurisdiction, therefore, properly belongs to the lodged a complaint for malicious prosecution. The court ruled in favor of
for illegal dismissal and non-payment of salaries with the NLRC. SEC and not the NLRC. Lao declaring that the estafa case was filed without probable cause and
Ongkingco filed a motion to dismiss alleging that it is the SEC, and not the
Guilas also attack the SEC's jurisdiction over the instant case on with malice and orders the corporation and Esteban Co to jointly and
labor arbiter, which has jurisdiction over the subject matter of the complaint.
grounds that Guilas was not elected by the Board of Directors but was severally pay the Laos.
LA granted the motion to dismiss. NLRC reversed the LA.
merely appointed. This particular argument baffles us. P.D. 902-A cannot
be any clearer. Sec. 5(c) of said law expressly covers both election and ISSUE:
ISSUE:
Is Guilas a corporate officer or a mere employee? appointment of corporate directors, trustees, officers and managers.
Whether or not Esteban Co is solidarily liable with the Corporation in
Does the case fall under the exclusive jurisdiction of the SEC? It is of no consequence, likewise, that the complaint of private payment of damages against Lao
respondent for illegal dismissal includes money claims, jurisdiction remains
RULING: with the SEC as ruled in the case of Cagayan de Oro Coliseum, Inc. v. RULING:
Office of the MOLE, a close scrutiny thereof shows that said claims are
Specifically delineated in P.D. 902-A are the cases over which actually part of the perquisites of his position in, and therefore interlinked No. A perusal of Lao’s affidavit-complaint reveals that at the time he filed
the SEC exercises exclusive jurisdiction: “c) Controversies in the election with his relations with the corporation. In Dy vs. NLRC, the Court said: the same petitioner Co was the vice-president of the Corporation. As a
or appointment of directors, trustees, officers, or managers of such "(t)he question of remuneration involving as it does, a person who is not a corporate officer, his power to bind the Corporation as its agent must be
corporations, partnerships or associations.” mere employee but a stockholder and officer, an integral part, it might be sought from statute, charter, by-laws, a delegation of authority to a
Guilas is an officer of petitioner corporation and not its mere employee said, of the corporation, is not a simple labor problem but a matter that corporate officer, or from the acts of the board of directors formally
cannot be questioned. The by-laws of the Galeria de Magallanes comes within the area of corporate affairs and, management, and is in fact expressed or implied from a habit or custom of doing business. In this case,
Condominium Association specifically includes the a corporate controversy in contemplation of the Corporation Code." no such sources of petitioner’s authority from which to deduce whether or
Superintendent/Administrator in its roster of corporate officers: “Section
1. Executive Officers The Executive officers of the corporation shall be a not he was acting beyond the scope of his responsibilities as corporate
President, a Vice President, a Treasurer, all of whom shall be elected by vice-president are mentioned, much less proven. It is thus logical to
the Board of Directors. They may be removed with or without cause at any conclude that the board of directors or by-laws of the corporation vested
36. ANDRES LAO vs. COURT OF APPEALS, THE ASSOCIATED
meeting by the concurrence of four directors. The Board of Directors may ANGLO-AMERICAN TOBACCO CORPORATION and ESTEBAN CO petitioner Co with certain executive duties one of which is a case for
appoint a Superintendent or Administrator and such other officers and G.R. No. 47013 February 17, 2000 the Corporation.
employees and delineate their powers and duties as the Board shall find
necessary to manage the affairs of the corporation.” FACTS: That petitioner Co was authorized to institute the estafa case is buttressed
by the fact that the Corporation failed to make an issue out of his
Under the recent case of Tabang v. NLRC, the president, vice- The Associated Anglo-American Tobacco Corporation entered into a authority to file the case. Upon well-established principles of pleading, lack
president, secretary and treasurer are commonly regarded as the principal “Contract of Sales Agent “ with Andres Lao. Under the contract, Lao agrees of authority of an officer of a corporation to bind it by contract executed by
or executive officers of a corporation, and modern corporation statutes to sell cigarettes manufactured and shipped by the corporation to him in its name, is a defense which should have been specially pleaded by
usually designate them as the officers of the corporation. However, other his business address in Tacloban City. Lao would in turn remit the sales. the corporation. The Corporation’s failure to interpose such a defense
offices are sometimes created by the charter or by-laws of a corporation,
could only mean that the filing of the affidavit-complaint by petitioner Co
or the board of directors may be empowered under the by-laws of a However, in February 1968, Lao failed to accomplish his monthly sales
corporation to create additional offices as may be necessary. It has been was with the consent and authority of the Corporation. In the same vein,
report. Since Lao appeared to encounter difficulties in complying with his
held that an "office" is created by the charter of the corporation and the petitioner Co may not be held personally liable for acts performed in
obligations under the contract of agency, the Corporation sent Ngo Kheng
officer is elected by the directors or stockholders. On the other hand, an pursuance of an authority and therefore, holding him solidarily liable with
to supervise Lao’s sales operations in Leyte and Samar. Ngo Kheng
"employee" usually occupies no office and generally is employed not by the Corporation for the damages awarded to respondent Lao does accord
discovered that, contrary to Lao’s allegation that he still had huge
action of the directors or stockholders but by the managing officer of the with law and jurisprudence.
collectibles from his customers, nothing was due the Corporation from
corporation who also determines the compensation to be paid to such
Lao’s clients. From then on, Lao no longer received shipments from the
employee.
Also, under the case of Lozon v. NLRC and Espino v. NLRC, the Corporation which transferred its vehicles to another compound controlled 37. DE TAVERA v. PHIL. TUBERCULOSIS SOCIETY
Court held that acorporate officer's dismissal is always a corporate act by Ngo Kheng. Shipments of cigarettes and the corresponding invoices GR. No. L-48928 February 25, 1982
and/or an intra-corporate controversy and that nature is not altered by the were also placed in the name of Ngo Kheng.
reason or wisdom which the Board of Directors may have in taking such FACTS:
action.
31
Delegation of Authority to Corporate Officers | Doctrine of Apparent Authority
On March 23, 1976, plaintiff-appellant Mita Pardo de Tavera filed temporary in nature, terminable at a moment's notice without need to show employment shall have been fixed in their contract of
with the Court of First Instance of Rizal a complaint against the Philippine that the termination was for cause; employment.
Tuberculosis Society, Inc. (hereinafter referred to as the Society), Miguel On October 13, 1976, plaintiff filed a Motion for Reconsideration
Canizares, Ralph Nubla, Bernardo Pardo, Enrique Garcia, Midpantao Adil, to which defendants filed an Opposition. On November 25, 1976, the xxx xxx xxx
Alberto Romulo, and the present Board of Directors of the Philippine court a quo denied the motion for Reconsideration.
Tuberculosis Society, Inc. Dissatisfied with the decision and the order denying the motion
for reconsideration. plaintiff filed a Notice of Appeal and an Urgent Motion Section 7.04. Removal of Officers and Employees. —
for Extension of Time to File Record on Appeal, which was granted in an All officers and employees shall be subject to
On April 12, 1976, plaintiff-appellant filed an amended complaint suspension or removal for a sufficient cause at any
impleading Francisco Ortigas, Jr. as party defendant. order dated December 15, 1976. However, on December 20, 1976, the
court a quo issued an amended order where it qualified the action as time by affirmative vote of a majority of an the
principally one for quo warranto and hence, dispensed with the filing of a members of the Board of Directors, except that
The complaint alleged that plaintiff is a doctor of Medicine by record on appeal as the original records of the case are required to be employees appointed by the President alone or by the
profession and a recognized specialist in the treatment of tuberculosis, elevated to the Court of Appeals. other officers alone at the pleasure of the officer
having been in the continuous practice of her profession since 1945; that On August 8, 1978, the Court of Appeals issued a resolution appointing him.
she is a member of the Board of Directors of the defendant Society, in certifying this case to this Court considering that the appeal raises no
representation of the Philippine Charity Sweepstakes Office; that she was factual issues and involves only issues of law, In the organizational meeting of the Society on April 25, 1973, the minutes
duly appointed on April 27, 1973 as Executive Secretary of the Society; The nature of the instant suit is one involving a violation of the of the meeting reveal that the Chairman mentioned the need of appointing
that on May 29, 1974, the past Board of Directors removed her summarily rights of the plaintiff under the By-Laws of the Society, the Civil Code and a permanent Executive Secretary and stated that the former Executive
from her position, the lawful cause of which she was not informed, through the Constitution, which allegedly renders the individuals responsible Secretary, Dr. Jose Y. Buktaw, tendered his application for optional
the simple expedient of declaring her position vacant; that immediately therefore, accountable for damages, as may be gleaned from the following retirement, and while on terminal leave, Dr. Mita Pardo de Tavera was
thereafter, defendant Alberto Romulo was appointed to the position by an allegations in the complaint as constituting the plaintiff's causes of action, appointed Acting Executive Secretary. In view thereof, Don Francisco
affirmative vote of seven directors, with two abstentions and one objection; Ortigas, Jr. moved, duly seconded, that Dr. Mita Pardo de Tavera be
and that defendants Pardo, Nubla, Garcia and Adil, not being members of ISSUE: Whether or not petitioner was illegally removed from her position appointed Executive Secretary of the Philippine Tuberculosis Society, Inc.
defendant Society when they were elevated to the position of members of as executive secretay in violation of Code of By-Laws of the Society. The motion was unanimously approved.
the Board of Directors, are not qualified to be elected as such and hence,
all their acts in said meeting of May 29, 1974 are null and void. RULING:
The defendants filed their answer on May 12, 1976, specifically Although the minutes of the organizational meeting show that the
Section 7.01. Officers of the Society. — The executed officers f Chairman mentioned the need of appointing a "permanent" Executive
denying that plaintiff was illegally removed from her position as Executive the Society shag be the President a Vice-President, a Treasurer who shall
Secretary and averring that under the Code of By-Laws of the Society, said Secretary, such statement alone cannot characterize the appointment of
be elected by the Board of Directors, Executive Secretary, and an Auditor, petitioner without a contract of employment definitely fixing her term
position is held at the pleasure of the Board of Directors and when the who shall be appointed by the Board of Directors, all of whom shall
pleasure is exercised, it only means that the incumbent has to vacate the because of the specific provision of Section 7.02 of the Code of By-Laws
exercise the functions. powers and prerogatives generally vested upon
same because her term has expired; skich officers, the functions hereinafter set out for their respective offices
On the same date, defendant Adil filed a Motion to Dismiss on and such other duties is from time to time, may be prescribed by the Board The absence of a fixed term in the letter addressed to petitioner informing
the ground that the complaint states no cause of action, or if it does, the of Directors. One person may hold more than one office except when the her of her appointment as Executive Secretary is very significant. This
same has prescribed. Inasmuch as plaintiff seeks reinstatement, he functions thereof are incompatible with each other. could have no other implication than that petitioner held an appointment at
argued that the complaint is an action for quo warranto and hence, the the pleasure of the appointing power.
same should be commenced within one year from May 29, 1974 when the
plaintiff was ousted from her position. It is petitioner's contention that she is subject, to removal
pursuant to Section 7.04 of the Code of By-laws which respondents An appointment held at the pleasure of the appointing power is in essence
correctly dispute citing Section 7.02 of the same Cede. The temporary in nature. It is co-extensive with the desire of the Board of
Plaintiff filed an Opposition to Motion to Dismiss on May 28, Directors. Hence, when the Board opts to replace the incumbent,
1976, stating that the complaint is a suit for damages filed under the aforementioned provisions state as follows:
technically there is no removal but only an expiration of term and in an
authority of Section 6, Article 11 of the present Constitution in relation to expiration of term, there is no need of prior notice, due hearing or sufficient
Articles 12 and 32(6) of the New Civil Code, Section 7.02. Tenure of Office. — All executive officers grounds before the incumbent can be separated from office. The protection
On September 3, 1976, the coturt a quo rendered a decision of the Society except the Executive Secretary and the afforded by Section 7.04 of the Code of By-Laws on Removal of Officers
holding that the present suit being one for quo warranto it should be filed Auditor shall be elected the Board of Directors, for a and Employees, therefore, cannot be claimed by petitioner.
within one year from plaintiff's outer from office; that nevertheless, plaintiff term of one rear ind shall hold office until their
was not illegally rendered or used from her position as Executive Secretary successors are elected and have qualified. The
in The Society since plaintiff as holding an appointment all the pleasure of Executive secretary, the Auditor and all other office ers Moreover, the act of the Board in declaring her position as vacant is not
the appointing power and hence her appointment in essence was and employees of the Society shall hold office at the only in accordance with the Code of By-Laws of the Society but also meets
pleasure of the Board of Directors, unless their term of the exacting standards of honesty and good faith. The meeting of May 29,

32
Delegation of Authority to Corporate Officers | Doctrine of Apparent Authority
1974, at which petitioner's position was declared vacant, was caged removal was a corporate act of Matling and the controversy resulting from
specifically to take up the unfinished business of the Reorganizational such removal was under the jurisdiction of the SEC.
Meeting of the Board of April 30, 1974. Hence, and act cannot be said to 39. OKOL v. SLIMMERS WORLD and MOY
impart a dishonest purpose or some moral obliquity and conscious doing ISSUE: G.R. No. 160146
to wrong but rather emanates from the desire of the Board to reorganize December 11, 2009
itself. Was Coros’ position of Vice President for Administration and Finance a
corporate office? FACTS:

RULING: Slimmers World International (Slimmers) employed Leslie Okol (Okol) as


38. MATLING INDUSTRIAL AND COMMERCIAL CORPORATION, a management trainee on 1992. She rose up the ranks to become Head
No. Office Manager and then Director and VP from 1996 until she was
ET AL. V. COROS
G.R. No. 157802 dismissed on September 1999.
Under Section 25 of the Corporation Code, a position must be expressly
October 13, 2010
mentioned in the By-Laws in order to be considered as a corporate office. Prior to Okol’s dismissal, Slimmers preventively suspended Okol, on July
The creation of an office pursuant to or under a By-Law enabling provision 1999, arising from the seizure of the Customs of seven Precor elliptical
FACTS:
is not enough to make a position a corporate office. The only officers of a machines and seven treadmills, belonging to or consigned to Slimmers,
corporation were those given that character either by the Corporation because it was undervalued. The shipment was named under Okol.
After his dismissal by Matling as its Vice President for Finance and
Code or by the By-Laws; the rest of the corporate officers could be
Administration, respondent Coros filed a complaint for illegal suspension
considered only as employees or subordinate officials. On 2 September 1999, she received a memo that her suspension is
and illegal dismissal against petitioners Matling and some of its corporate
officers in the NLRC. The petitioners moved to dismiss extended to 1 October pending investigation. She was asked to explain but
The corporate officers are the President, Secretary, Treasurer and such Slimmers found it unsatisfactory. Hence, a letter dated 22 September
the complaint, raising the ground, among others, that the complaint
other officers as may be provided for in the By-Laws. signed by its president Ronald Joseph Moy (Moy), Slimmers terminated
pertained to the jurisdiction of the Securities and Exchange Commission
Thus, pursuant to the above provision (Section 25 of the Corporation Okol.
(SEC) due to the controversy being intra-corporate inasmuch as the
Code), whoever are the corporate officers enumerated in the by-laws are
respondent was a member of Matlings Board of Directors aside from being
the exclusive Officers of the corporation and the Board has no power to Okol filed a complaint with the Arbitration branch of the NLRC against
its Vice-President for Finance and Administration prior to his termination.
create other Offices without amending first the corporate By- Slimmers and Moy for illegal suspension, illegal dismissal, unpaid
They contend that the position of Vice President for Finance and
laws. However, the Board may create appointive positions other than the commissions and attorney’s fees.
Administration was a corporate office, having been created by Matlings
positions of corporate Officers, but the persons occupying such positions
President pursuant to By-Law No. V.
are not considered as corporate officers within the meaning of Section 25 Respondents filed a Motion to Dismiss alleging that NLRC had no
of the Corporation Code and are not empowered to exercise the functions jurisdiction over the subject matter of the complaint.
The respondent counters that Matlings By-Laws did not list his position as
of the corporate Officers, except those functions lawfully delegated to them.
Vice President for Finance and Administration as one of the corporate
Their functions and duties are to be determined by the Board of The Labor Arbiter granted the motion since the issue involved a corporate
offices; that Matlings By-Law No. III listed only four corporate officers,
Directors/Trustees. officer, the dispute was an intra-corporate controversy falling outside the
namely: President, Executive Vice President, Secretary, and
Treasurer; that the corporate offices contemplated in the phrase and such jurisdiction of the arbitration branch.
Moreover, the Board of Directors of Matling could not validly delegate the
other officers as may be provided for in the by-laws found in Section 25 of
power to create a corporate office to the President, in light of Section 25 of Okol filed an appeal with the NLRC, the NLRC reversed and set aside the
the Corporation Code should be clearly and expressly stated in the By-
the Corporation Code requiring the Board of Directors itself to elect the order. The Respondents filed an MR, but was denied. It referred the case
Laws; that the fact that Matlings By-Law No. III dealt with Directors &
corporate officers. Verily, the power to elect the corporate officers was a to the CA, the CA reversed the order of the NLRC and affirmed the order
Officers while its By-Law No. V dealt with Officers proved that there was a
discretionary power that the law exclusively vested in the Board of of the Labor Arbiter.
differentiation between the officers mentioned in the two provisions, with
Directors, and could not be delegated to subordinate officers or
those classified under By-Law No. V being ordinary or non-
agents. The office of Vice President for Finance and Administration Arguments:
corporate officers; and that the officer, to be considered as a corporate
created by Matlings President pursuant to By Law No. V was an ordinary,
officer, must be elected by the Board of Directors or the stockholders, for
not a corporate, office. Okol: She claims that she is not a corporate officer because even as vice-
the President could only appoint an employee to a position pursuant to By-
Law No. V. president, the work that she performed conforms to that of a regular
To emphasize, the power to create new offices and the power to appoint employee, rather than a corporate officer. Mere title or designation will not
the officers to occupy them vested by By-Law No. V merely allowed by itself, determine the existence of an employer-employee relationship. It
The LA granted the petitioners motion to dismiss, ruling that the
Matlings President to create non-corporate offices to be occupied by is the “four-fold” test which must be applied. She claims that she was under
respondent was a corporate officer because he was occupying the position
of Vice President for Finance and Administration and at the same time was
ordinary employees of Matling. Such powers were incidental to the the power and control of Moy, Slimmer’s President, because (1) she
Presidents duties as the executive head of Matling to assist him in the daily receives salaries by pay slips, (2) Moy deducted Medicare and SSS, and
a Member of the Board of Directors of Matling; and that, consequently, his
operations of the business.
33
Delegation of Authority to Corporate Officers | Doctrine of Apparent Authority
(3) she was dismissed from employment not through a board resolution to his authority under the PDMC Approvals Manual. A new board of
but by virtue of a letter from Moy. 4. Vice-President Like the Chairman of the Board and directors for PDMC took over the company.
the President, the Vice-President shall be elected by
Slimmers and Moy: Okol is a corporate officer as supported by the General the Board of Directors from [its] own members. PDMC’S ALLEGATION: The new board of directors of respondent
Information Sheet (GIS) and Director’s Affidavit that she was an officer. PDMC removed petitioner Gomez as corporate secretary. The board
Absence of any board resolution approving her termination does not The Vice-President shall be vested with all the powers questioned her continued employment as administrator. In answer, she
constitute that she was not an officer. Moreover, Okol was a stockholder and authority and is required to perform all the duties presented the former presidents letter that extended her term. The Board’s
and director; which facts provide further that NLRC has no jurisdiction. of the President during the absence of the latter for any advice from the legal department expressed the view that Gomezs term
cause. extension was an ultra vires act of the former president. It reasoned that,
ISSUE: since her position was functionally that of a vice-president or general
The Vice-President will perform such duties as the manager, her term could be extended under the companys by-laws only
Is Okol a regular employee or a corporate officer of Slimmers? Board of Directors may impose upon him from time to with the approval of the board.
time.”
RULING: The Office of the Government Corporate Counsel (OGCC) held the view
Clearly, Okol was a director and officer of Slimmers. Hence, the that while respondent PDMCs board did not approve the creation of the
Okol is a corporate officer. case falls within the jurisdiction of the SEC. The determination of the rights position of administrator that Gomez held, such action should be deemed
of a director and corporate officer dismissed from his employment as well ratified since the board had been aware of it since 1994. But the OGCC
Section 25 of the Corporation Code (CC), enumerates corporate officers as the corresponding liability of a corporation, if any, is an intra-corporate ventured that the extension of her term beyond retirement age should have
as the president, secretary, treasurer and such other officers as may be dispute subject to the jurisdiction of the regular courts. been made with the boards approval.
provided for in the by-laws.
PETITIONER GOMEZ’S DEFENSE: Petitioner Gomez for her part
In Tabang v. NLRC, the Court held that an “office” is created by the charter 40. GOMEZ v. PNOC DMC conceded that as corporate secretary, she served only as a corporate
of the corporation and the officer is elected by the directors or stockholders. GLORIA V. GOMEZ v. PNOC DEVELOPMENT AND MANAGEMENT officer. But, when they named her administrator, she became a regular
On the other hand, an “employee” usually occupies no office and generally CORPORATION (PDMC) (formerly known as FILOIL managerial employee. The respondent PDMCs board did not have to
is employed not by action of the directors or stockholders but by the DEVELOPMENT AND MANAGEMENT CORPORATION [FDMC]), approve either her appointment as such or the extension of her term in
managing officer of the corporation. G.R. No. 174044, November 27, 2009 1998.
FACTS:
In the present case, the GIS, Minutes of meeting and Secretary’s LA and NLRC decisions: The Labor Arbiter granted the motion to dismiss
Certificate, and the Amended By-Laws of Slimmers show that Okol was a Petitioner Gloria V. Gomez used to work as Manager of the Legal by the respondent upon a finding that Gomez was a corporate officer and
corporate officer. It was indicated there that Okol was a member of the Department of Petron Corporation. With Petrons privatization, she availed that her case involved an intra-corporate dispute that fell under the
BOD, holding one subscribed share of the capital stock and an elected of the companys early retirement program and left that organization on jurisdiction of the Securities and Exchange Commission (SEC) pursuant to
corporate officer. April 30, 1994. On the following day, May 1, 1994, however, Filoil Refinery Presidential Decree (P.D.) 902-A. The National Labor Relations
Corporation (Filoil) appointed her its corporate secretary and legal Commission (NLRC) Third Division set aside the Labor Arbiters order and
Amended By-Laws: counsel, with the same managerial rank, compensation, and benefits that remanded the case to the arbitration branch for further proceedings.
she used to enjoy at Petron. But Filoil was later on also identified for Gomez was a regular employee, not a corporate officer;
Article II privatization. To facilitate its conversion, the Filoil board of directors created
The Board of Directors a five-member task force headed by petitioner Gomez who had been CA Ruling: The CA held that since Gomezs appointment as administrator
designated administrator. With the privatization temporarily shelved, Filoil required the approval of the board of directors, she was clearly a corporate
1. Qualifications and Election The general underwent reorganization and was renamed Filoil Development officer.
management of the corporation shall be vested in a Management Corporation (FDMC), which later became the respondent
board of five directors who shall be stockholders and PNOC Development Management Corporation (PDMC). When this ISSUE:
who shall be elected annually by the stockholders and happened, Gomez’s task force was abolished and its members, including Is Petitioner Gomez an ordinary employee or a corporate officer?
who shall serve until the election and qualification of Gomez, were given termination notices.
their successors. RULING:
In the meantime, petitioner Gomez continued to serve as corporate Ordinary company employees are generally employed not by action of the
xxx secretary of respondent PDMC. On September 23, 1996 its president re- directors and stockholders but by that of the managing officer of the
hired her as administrator and legal counsel of the company. In corporation who also determines the compensation to be paid such
Article III accordance with company guidelines, it credited her the years she served employees. Corporate officers, on the other hand, are elected or
Officers with the Filoil task force. On May 24, 1998, the next president of PDMC appointed[22] by the directors or stockholders, and are those who are given
xxx extended her term as administrator beyond her retirement age, pursuant
34
Delegation of Authority to Corporate Officers | Doctrine of Apparent Authority
that character either by the Corporation Code or by the corporations by- What is more, respondent PDMC enrolled petitioner Gomez with the office Villa Gonzalo, CDO, and evidenced by the signature on the face of
laws.[23] Social Security System, the Medicare, and the Pag-Ibig Fund. It even the original copy of the summons."
issued certifications dated October 10, 2008 stating that Gomez was a
Here, it was the PDMC president who appointed petitioner Gomez permanent employee and that the company had remitted combined Villarosa prayed for the dismissal of the complaint on the ground
administrator, not its board of directors or the stockholders. The president contributions during her tenure. The company also made her a member of of improper service of summons and for lack of jurisdiction over the person
alone also determined her compensation package. Moreover, the the PDMCs savings and provident plan[31] and its retirement plan.[32] It of the defendant. Villarosa contends that the RTC did not acquire
administrator was not among the corporate officers mentioned in the grouped her with the managers covered by the companys group jurisdiction over its person since the summons wasimproperly served upon
PDMC by-laws. The corporate officers proper were the chairman, hospitalization insurance.[33] Likewise, she underwent regular employee its employee in its branch office at CDO who is not one of those persons
president, executive vice-president, vice-president, general manager, performance appraisals,[34] purchased stocks through the employee stock named in Sec. 11, Rule 14 upon whom service of summons may be made.
treasurer, and secretary. option plan,[35] and was entitled to vacation and emergency
leaves.[36] PDMC even withheld taxes on her salary and declared her as RTC Judge Benito denied the motion to dismiss stating that
Respondent PDMC claims, however, that since its board had under its an employee in the official Bureau of Internal Revenue forms.[37] These are there was substantial compliance with the rule on service of summons
by-laws the power to create additional corporate offices, it may be deemed all indicia of an employer-employee relationship which respondent PDMC since the summons and the complaint where in fact served to the
to have simply ratified its presidents creation of the corporate position of failed to refute. corporation.
administrator. But creating an additional corporate office was definitely not
respondent PDMCs intent based on its several actions concerning the Estoppel, an equitable principle rooted on natural justice, prevents a Issue: Can an agent of a corporation can receive summons in behalf of
position of administrator. person from rejecting his previous acts and representations to the their corporation?
prejudice of others who have relied on them.[38] This principle of law applies
Respondent PDMC never told Gomez that she was a corporate officer until to corporations as well. The PDMC in this case is estopped from claiming Ruling: NO.
the tail-end of her service after the board found legal justification for getting that despite all the appearances of regular employment that it weaved
rid of her by consulting its legal department and the OGCC which supplied around petitioner Gomezs position it must have technically hired her only The designation of persons or officers who are authorized to
an answer that the board obviously wanted. Indeed, the PDMC president as a corporate officer. The board and its officers made her stay on and accept summons for a domestic corporation or partnership is now limited
first hired her as administrator in May 1994 and then as administrator/legal work with the company for years under the belief that she held a regular and more clearly specified in Section 11, Rule 14 of the1997 Rules of Civil
counsel in September 1996 without a board approval. The president even managerial position. Procedure. The rule now states "general manager" instead of
extended her term in May 1998 also without such approval. The companys only"manager"; "corporate secretary" instead of "secretary"; and
mindset from the beginning, therefore, was that she was not a corporate That petitioner Gomez served concurrently as corporate secretary for a "treasurer" instead of "cashier." The phrase "agent, or any of its directors"
officer. time is immaterial. A corporation is not prohibited from hiring a corporate is conspicuously deleted in the new rule.
officer to perform services under circumstances which will make him an It should be noted that even prior to the effectivity of the 1997 Rules of Civil
Respondent PDMC claims that as administrator petitioner Gomez employee.[39] Indeed, it is possible for one to have a dual role of officer and Procedure, strict compliance with the rules has been enjoined.
performed functions that were similar to those of its vice-president or its employee.
general manager, corporate positions that were mentioned in the A strict compliance with the mode of service is necessary to
companys by-laws. It points out that Gomez was third in the line of confer jurisdiction of the court over a corporation. The officer upon whom
command, next only to the chairman and president, and had been 41. E.B. VILLAROSA & PARTNER., LTD V. HON. HERMINIO I. service is made must be one who is named in the statute; otherwise the
empowered to make major decisions and manage the affairs of the BENITO service is insufficient. The purpose is to render it reasonably certain that
company. 312 S 65 the corporation will receive prompt and proper notice in an action against it
or to insure that the summons be served on a representative so integrated
But the relationship of a person to a corporation, whether as officer or agent Facts: with the corporation that such person will know what to do with the legal
or employee, is not determined by the nature of the services he performs E.B. Villarosa& Partners is a limited partnership with principal papers served on him. In other words, "to bring home to the corporation
but by the incidents of his relationship with the corporation as they actually officeaddress at 102 Juan Luna St., Davao City and with branch offices at notice of the filing of the action." The liberal construction rule cannot be
exist. Here, respondent PDMC hired petitioner Gomez as an ordinary Parañaque and Cagayan de Oro City (CDO) . Villarosa and Imperial invoked and utilized as a substitute for the plain legal requirements as to
employee without board approval as was proper for a corporate Development (ID) executed an Agreement wherein Villarosa agreed to the manner in which summons should be served on a domestic
officer. When the company got her the first time, it agreed to have her retain develop certain parcels of land in CDO belonging to ID into a housing corporation.
the managerial rank that she held with Petron. Her appointment paper said subdivision. ID, filed a Complaint for Breach of Contract and Damages
that she would be entitled to all the rights, privileges, and benefits that against Villarosa before the RTC allegedly for failure of the latter to comply Accordingly, it is ruled that the service of summons upon the
regular PDMC employees enjoyed. This is in sharp contrast to what the with its contractual obligation. branch manager of petitioner at its branch office at Cagayan de Oro,
former PDMC presidents appointment paper stated: he was elected to the instead of upon the general manager at its principal office at Davao City is
position and his compensation depended on the will of the board of Summons, together with the complaint, were served upon improper. Consequently, the trial court did not acquire jurisdiction over the
directors. Villarosa, through its Branch Manager Wendell Sabulbero at the address person of the petitioner. Accordingly, the filing of a motion to dismiss,
at CDO but the Sheriff’s Return of Service stated that the summons was whether or not belatedly filed by the defendant, his authorized agent or
duly served "E.B. Villarosa& Partner thru its Branch Manager at their new attorney, precisely objecting to the jurisdiction of the court over the person
35
Delegation of Authority to Corporate Officers | Doctrine of Apparent Authority
of the defendant can by no means be deemed a submission to the for the 20% sales discounts it granted to qualified senior citizens. The law
jurisdiction of the court. There being no proper service of summons, the Issues: then applicable on this point is clear and without any qualification Sec. 4 (a)
trial court cannot take cognizance of a case for lack of jurisdiction over the 1) Can the petitioner corporation’s President sign the subject of RA 7432. It is thus clear that petitioner is entitled to a tax credit for the full
person of the defendant. Any proceeding undertaken by the trial court will verification and certification without the approval of its Board 20% sales discounts it extended to qualified senior citizens for taxable year
consequently be null and void. of Directors? 1995. Considering that the CTA has not disallowed the PhP 123,083 sales
discounts petitioner claimed before the BIR and CTA, the Court is
2) Is petitioner entitled to tax refund or tax credit? constrained to grant them as tax credit in favor of petitioner.
42. CAGAYAN VALLEY DRUG CORPORATION v. CIR
G.R. No. 151413
February 13, 2008 Held: 43. BOARD OF INVESTMENT v. SR METALS, INC.
YES. G.R. No. 219927; October 03, 2018
Facts: As a general rule, Sec. 23, in relation to Sec. 25 of the
Petitioner, Cagayan Valley, is a duly licensed retailer of medicine Corporation Code, clearly enunciates that all corporate powers are FACTS:
and other pharmaceutical products. It operates two drugstores, one in exercised, all business conducted, and all properties controlled by the Petitioner Board oflnvestments (BOI) is a government agency
Tuguegarao, Cagayan, and the other in Roxas, Isabela, under the name board of directors. A corporation has a separate and distinct personality created under Republic Act (RA) No. 5186.5 It is an attached agency of the
and style of “Mercury Drug”. It alleged that in 1995, it granted 20% sales from its directors and officers and can only exercise its corporate powers Department of Trade and Industry (DTI) and is the lead government
discounts to qualified senior citizens on purchases of medicine pursuant to through the board of directors. Thus, it is clear that an individual corporate agency responsible for the promotion of investments in the Philippines.6
Republic Act No. (RA) 7432 and its IRR. In compliance with Revenue officer cannot solely exercise any corporate power pertaining to the Respondent SR Metals, Inc., on the other hand, is a corporation engaged
Regulation No. (RR) 2-94, petitioner treated the 20% sales discounts as corporation without authority from the board of directors. Only individuals in the business of mining in Tubay, Agusan Del Norte.
deductions from the gross sales in order to arrive at the net sales, instead vested with authority by a valid board resolution may sign the certificate of
of treating them as tax credit as provided by Section 4 of RA 7432. In 1996, non-forum shopping on behalf of a corporation. An action can be dismissed The case arose from a suit initiated by Municipality of Tubay with
petitioner filed with the Bureau of Internal Revenue (BIR) a claim for tax if the certification was submitted unaccompanied by proof of the signatory’s the BOI for the revocation of respondent’s Certificate of Registration. In
refund/tax credit of the full amount of the 20% sales discount it granted to authority. However, the Court have recognized the authority of some acting on said action, the BOI resolved to withdraw the Income Tax Holiday
senior citizens for the year 1995, allegedly totaling to PhP 123,083. corporate officers to sign the verification and certification against forum (ITH) of respondent for failing to comply with the requirements set under
shopping. It has held that the following officials or employees of the the 2007 Investment Properties Plan (IPP). The case elevated to the CA
The BIR’s inaction on petitioners claim for refund/tax credit company can sign the verification and certification without need of a where the ITH entitlement of SR Metals was restored.
compelled petitioner to file a petition for review in 1998 before the CTA. board resolution: (1) the Chairperson of the Board of Directors, (2)
CTA dismissed the petition notwithstanding its entitlement to the tax credit the President of a corporation, (3) the General Manager or Acting The case was appealed by petitioner. As one of the arguments
due to its net loss in 1995. It also sustained that the 20% sales discounts General Manager, (4) Personnel Officer, and (5) an Employment of respondent, it raised the procedural issue that there is lack of authority
should be treated as tax credit and not as deductions from the gross sales Specialist in a labor case. of the Officer-in-Charge (OIC), BOI Managing Head, Ma. Corazon Halili-
as erroneously interpreted in RR 2-94. The CTA reiterated its consistent Dichosa (OIC Halili-Dichosa), to sign the verification and certification of
holdings that RR 2-94 is an invalid administrative interpretation of the law; In the case at bar, there was substantial compliance with the non-forum shopping as well as the failure of petitioner to attach material
it purports to implement as it contravenes and does not conform to the Revised Rules on Civil Procedure. First, the requisite board resolution has portions of the records of the case. Respondent argues that there was
standards RA 7432 prescribes. The CTA rejected the refund as it is clear been submitted albeit belatedly by petitioner. Second, the President of nothing in Memorandum Order No. 2015- 080, series of 2015, dated
that RA 7432 only grants the 20% sales discounts extended to qualified petitioner is in a position to verify the truthfulness and correctness of the October 9, 2015 to indicate that the OIC is authorized to sign the verification
senior citizens as tax credit and not as tax refund. If no tax has been paid allegations in the petition. Third, the President of petitioner has signed the and certification of non-forum shopping as it is not among the list of official
or if no amount is due and collectible from the taxpayer, then a tax credit is complaint before the CTA at the inception of this judicial claim for refund or documents mentioned in Department Order No. 14-39, series of 2014.
unavailing. Moreover, it held that before allowing recovery for claims for a tax credit.
refund or tax credit, it must first be established that there was an actual
collection and receipt by the government of the tax sought to be recovered.
In the instant case, the CTA found that petitioner did not pay any tax by YES. RULING:
virtue of its net loss position in 1995.
The 20% sales discounts petitioner granted to qualified senior 1. Is the OIC authorized to sign the verification and certification for non-
The CA dismissed the petition on procedural grounds. The CA citizens should be deducted from petitioner’s income tax due and not from forum shopping?
held that the person who signed the verification and certification of absence petitioners gross sales as erroneously provided in RR 2-94. However, the
of forum shopping, a certain Jacinto J. Concepcion, President of Cagayan CTA erred in denying the tax credit to petitioner on the ground that Yes. Although it appears that the verification and certification of non-forum
Valley, failed to adduce proof that he was duly authorized by the Board of petitioner had suffered net loss in 1995, and ruling that the tax credit is shopping was not among the list of official documents mentioned in
Directors to do so since the rule is that if there is absence of an authority unavailing. It is true that petitioner did not pay any tax in 1995 since it Department Order No. 14-39, series of 2014, the Court is still inclined to
from the BOD, no person, not even the officers of the corporation, can suffered a net loss for that taxable year. This fact, however, without more, uphold the authority of OIC Halili-Dichosa to sign the same. In
validly bind the corporation. does not preclude petitioner from availing of its statutory right to a tax credit Memorandum Order No. 2015-080, Supervising Director Halili-Dichosa
36
Delegation of Authority to Corporate Officers | Doctrine of Apparent Authority
was designated OIC of petitioner in the interest of service as the company rules. He thus filed a case for illegal dismissal. The LA ruled for enterprise" safeguards the corporation from possible fraud being
Undersecretary/Managing Head was on an official trip. Considering the the respondent, while the NLRC reversed said decision. The CA ruled committed adverse to its own corporate interest.
rationale of the said Memorandum, the Court finds that any doubt as to the favoring the LA. Although it may be true that the service of summons was made
authority of OIC Halili-Dichosa to file the instant case and to sign the on a person not authorized to receive the same in behalf of the petitioner,
verification and certification of non-forum shopping should be resolved in ISSUE: Whether or not Regan Sy, the president of SSPC, may be held nevertheless since it appears that the summons and complaint were in fact
favor of the government. Obviously, OIC Halili-Dichosa caused the filing of solidarily liable with the latter. received by the corporation through its said clerk, the Court finds that there
the instant Petition in the performance of her duties and in order to protect was substantial compliance with the rule on service of summons. Indeed,
the interests of the government. RULING: the purpose of said rule as above stated to assure service of summons on
the corporation had thereby been attained. The need for speedy justice
NO. It appears that respondent impleaded SSPC President must prevail over technicality.
Besides, in recent cases, certain officials and employees to were allowed
to sign the verification and certification of non-forum shopping on behalf of Regan Sy only because he is an officer/agent of the company. However,
the company without need of a board resolution. These are the the court ruled that he cannot be made solidarily liable because for the
chairperson of the board of directors, the president of a corporation, the termination of respondent‘s employment, since there is no showing that the
general manager or acting general manager, the personnel officer, the dismissal was attended with malice or bad faith. The rule still stand that the 46. VLASON ENTERPRISES CORP. v CA AND DURAPROOF
employment specialist in a labor case, and other officials and employees liabilities of a corporation should not be directly imputed to its officers and it SERVICES
who are "in a position to verify the truthfulness and correctness of the shall be borne entirely by the corporation itself. GR 121662-64 JULY 6, 1999
allegations in the petition." In this case, the Court considers OIC Halili-
Dichosa to be in a position to verify the truthfulness and correctness of the J. PANGANIBAN
allegations stated in the instant Petition.
45. PABON & CAMONAYAN v. NLRC FACTS:
G.R. No.120457; September 24, 1998
Poro Point Shipping Services, then acting as the local agent of
SUBSTANTIAL ISSUE (Just in case) FACTS: Omega Sea Transport Company of Honduras & Panama, a Panamanian
On May 24, 1994 and June 22, 1994, complaints for illegal company, (hereafter referred to as Omega), requested permission for its
dismissal and non-payment of benefits were filed by petitioners Salome vessel M/V Star Ace, which had engine trouble, to unload its cargo and to
Petitioner claims that the CA erred in setting aside the withdrawal of
Pabon and Vicente Camonayan against private respondent Senior store it at the Philippine Ports Authority (PPA) compound in San Fernando,
respondent's ITH incentives since respondent did not build a beneficiation
Marketing Corporation (SMC) and its Field Manager, R-Jay Roxas La Union while awaiting transshipment to Hongkong. The request was
plant. The Sc ruled that a commitment to build a beneficiation plant does
Summons and notices of hearings were sent to Roxas at private approved by the Bureau of Customs. 4 Despite the approval, the customs
not necessarily require the construction of an industrial building or
respondent's provincial office in 13 Valley Homes, Patul Road, Santiago, personnel boarded the vessel when it docked on January 7, 1989, on
structure, as a beneficiation plant could also be an assemblage of
Isabela which were received by its bookkeeper, Mina Villanueva. On suspicion that it was the hijacked M/V Silver Med owned by Med Line
equipment and machineries where the beneficiation process could be
September 15, 1994, the Labor Arbiter rendered a judgment by default Philippines Co., and that its cargo would be smuggled into the country. 5
done. In this case, respondent was able to prove that it has a beneficiation
after finding that private respondent tried to evade all the summons and The district customs collector seized said vessel and its cargo pursuant to
plant.
orders of hearing by refusing to claim all the registered mail addressed to Section 2301, Tariff and Customs Code. A notice of hearing of SFLU
it. Seizure Identification No. 3-89 was served on its consignee, Singkong
Trading Co. of Hongkong, and its shipper, Dusit International Co., Ltd. of
Thailand.While seizure proceedings were ongoing, La Union was hit by
ISSUE: three typhoons, and the vessel ran aground and was abandoned. On June
Was the bookkeeper authorized to receive summons? 8, 1989, its authorized representative, Frank Cadacio, entered into a
salvage agreement with private respondent to secure and repair the vessel
44. SUPREME STEEL PIPE CORPORATION v. ROGELIO BARDAJE RULING: at the agreed consideration of $1 million and "fifty percent (50%) [of] the
G.R. No. 170811, April 24, 2007 Yes. Bookkeeper can be considered as an agent of private cargo after all expenses, cost and taxes." 6Finding that no fraud was
respondent corporation within the purview of Section 13, Rule 14 of the old committed, the District Collector of Customs, Aurelio M. Quiray, lifted the
FACTS: Rules of Court. The rationale of all rules with respect to service of process warrant of seizure on July 16, 1989. 7 However, in a Second Indorsement
on a corporation is that such service must be made to an agent or a dated November 11, 1989, then Customs Commissioner Salvador M.
Petitioner Supreme Steel Pipe Corporation (SSPC) was
representative so integrated with the corporation sued as to make it a priori Mison declined to issue a clearance for Quiray's Decision; instead, he
primarily engaged in the business of manufacturing steel pipes. It supposable that he will realize his responsibilities and know what he should
employed respondent Rogelio Bardaje as a warehouseman on March 14, forfeited the vessel and its cargo in accordance with Section 2530 of the
do with any legal papers served on him. The bookkeeper's task is one Tariff and Customs Code. 8 Accordingly, acting District Collector of
1994. SSPC employees were required to wear a uniform (a yellow t-shirt under consideration. The job of a bookkeeper is so integrated with the
with a logo and the marking "Supreme") while at work. Due to an incident, Customs John S. Sy issued a Decision decreeing the forfeiture and the
corporation that his regular recording of the corporation's "business sale of the cargo in favor of the government.9To enforce its preferred
his employment was terminated on the ground of multiple infractions of accounts" and "essential facts about the transactions of a business or
37
Delegation of Authority to Corporate Officers | Doctrine of Apparent Authority
salvor's lien, herein Private Respondent Duraproof Services filed with the Answer. 25Declared in default in an Order issued by the trial court on discretion to realize the importance of the legal papers served and to relay
Regional Trial Court of Manila a Petition for Certiorari, Prohibition and January 23, 1991, were the following: Singkong Trading Co., the same to the president or other responsible officer of the corporation
Mandamus 10 assailing the actions of Commissioner Mison and District Commissioner Mison, M/V Star Ace and Omega. 26 Private respondent being sued. 80 The secretary of the president satisfies this criterion. This
Collector Sy. Also impleaded as respondents were PPA Representative filed, and the trial court granted, an ex parte Motion to present evidence rule requires, however, that the secretary should be an employee of the
Silverio Mangaoang and Med Line Philippines, Inc. against the defaulting respondents. 27 Only private respondent, Atty. corporation sought to be summoned. Only in this manner can there be an
Tamondong, Commissioner Mison, Omega and M/V Star Ace appeared assurance that the secretary will "bring home to the corporation [the] notice
On January 10, 1989, private respondent amended its Petition in the next pretrial hearing; thus, the trial court declared the other of the filing of the action" against it.
11 to include former District Collector Quiray; PPA Port Manager Adolfo Ll. respondents in default and allowed private respondent to present evidence
Amor Jr; Petitioner Vlason Enterprises as represented by its president, against them. 28 Cesar Urbino, general manager of private respondent, In the present case, Bebero was the secretary of Angliongto, who was
Vicente Angliongto; Singkong Trading Company as represented by Atty. testified and adduced evidence against the other respondents, including president of both VSI and petitioner, but she was an employee of VSI, not
Eddie Tamondong; Banco Du Brasil; Dusit International Co., Inc.; Thai- herein petitioner. As regards petitioner, he declared: "Vlason Enterprises of petitioner. The piercing of the corporate veil cannot be resorted to when
Nan Enterprises Ltd. and Thai-United Trading Co., Ltd. 12 In both Petitions, represented by Atty. Sy and Vicente Angliongto thru constant intimidation serving summons. 81 Doctrinally, a corporation is a legal entity distinct and
private respondent plainly failed to include any allegation pertaining to and harassment of utilizing the PPA Management of San Fernando, La separate from the members and stockholders who compose it. However,
petitioner, or any prayer for relief against it.1âwphi1.nêtSummonses for the Union . . . further delayed, and [private respondent] incurred heavy when the corporate fiction is used as a means of perpetrating a fraud,
amended Petition were served on Atty. Joseph Capuyan for Med Line overhead expenses due to direct and incidental expenses . . . causing evading an existing obligation, circumventing a statute, achieving or
Philippines: Angliongto (through his secretary, Betty Bebero), Atty. irreparable damages of about P3,000,000 worth of ship tackles, rigs, and perfecting a monopoly or, in generally perpetrating a crime, the veil will be
Tamondong and Commissioner Mison. 13 Upon motion of the private appurtenances including radar antennas and apparatuses, which were lifted to expose the individuals composing it. None of the foregoing
respondent, the trial court allowed summons by publication to be served taken surreptitiously by persons working for Vlason Enterprises or its exceptions has been shown to exist in the present case. Quite the contrary,
upon the alien defendants who were not residents and had no direct agents[.] 29On December 29, 1990, private respondent and Rada, the piercing of the corporate veil in this case will result in manifest injustice.
representatives in the country. 14On January 29, 1990, private respondent representing Omega, entered into a Memorandum of Agreement This we cannot allow. Hence, the corporate fiction remains.
moved to declare respondents in default, but the trial court denied the stipulating that Rada would write and notify Omega regarding the demand
motion in its February 23, 1990 Order, 15 because Mangaoang and Amor for salvage fees of private respondent; and that if Rada did not receive any
had jointly filed a Motion to Dismiss, while Mison and Med Line had moved instruction from his principal, he would assign the vessel in favor of the
separately for an extension to file a similar motion. 16 Later it rendered an salvor.
47. PRIME WHITE CEMENT CORPORATION v. IAC
Order dated July 2, 1990, giving due course to the motions to dismiss filed
G.R. No. 68555, March 19, 1993
by Mangaoang and Amor on the ground of litis pendentia, and by the ISSUE/S:
commissioner and district collector of customs on the ground of lack of
FACTS:
jurisdiction. 17 In another Order, the trial court dismissed the action against WHETHER/NOT SERVICE OF SUMMONS WAS
Med Line Philippines on the ground of litis pendentia. 18On two other PROPERLY SERVED Alejandro Te and Primewhite Cement Corporation (PCC) entered into a
occasions, private respondent again moved to declare the following in
dealership agreement whereby Te was obligated to act as the executive
default: petitioner, Quiray, Sy and Mison on March 26, 1990; 19 and Banco
RULING: dealer and/or distributor of PCC of he latter’s cement products in the entire
Du Brazil, Dusit International Co., Inc., Thai-Nan Enterprises Ltd. and Thai-
Mindanao area for a term of five years.
United Trading Co., Ltd. on August 24, 1990. 20 There is no record,
however, that the trial court acted upon the motions. On September 18, The sheriff's return shows that Angliongto who was president of
Immediately after, Te placed advertisements in newspapers regarding the
1990, petitioner filed another Motion for leave to amend the petition, 21 petitioner corporation, through his secretary Betty Bebero, was served
cement products. Te then entered into written agreements with several
alleging that its counsel failed to include the following "necessary and/or summons on January 18, 1990. 78 Petitioner claims that this service was
hardware stores dealing in buying and selling white cement. Te was
indispensable parties": Omega represented by Cadacio; and M/V Star Ace defective for two reasons: (1) Bebero was an employee of Vlasons
assured by his buyers that 20,000 bags of cement would be bought, hence
represented by Capt. Nahon Rada, relief captain. Aside from impleading Shipping, Inc., which was an entity separate and distinct from Petitioner
Te wrote a letter to PCC informing the latter of its duty to comply with the
these additional respondents, private respondent also alleged in the Vlason Enterprises Corporation (VEC); and (2) the return pertained to the
dealership agreement. However, despite demands, PCC refused to
Second (actually, third) Amended service of summons for the amended Petition, not for the "Second
comply with the dealership agreement. PCC however, sent a letter-reply
Petition 22 that the owners of the vessel intended to transfer and alienate Amended Petition with Supplemental Petition," the latter pleading having
stating that the BoD of PCC decided to impose several conditions on the
their rights and interests over the vessel and its cargo, to the detriment of superseded the former.
sale of white cement. Te was then forced to cancel the agreements with
the private respondent.
third parties.
A corporation may be served summons through its agents or officers who
The trial court granted leave to private respondent to amend its under the Rules are designated to accept service of process. A summons Notwithstanding the dealership agreement with Te, PCC entered into an
Petition, but only to exclude the customs commissioner and the district addressed to a corporation and served on the secretary of its president exclusive dealership agreement with Napolean Co., hence the complaint
collector. 23 Instead, private respondent filed the "Second Amended binds that corporation. 79 This is based on the rationale that service must by Te.
Petition with Supplemental Petition" against Singkong Trading Company; be made on a representative so integrated with the corporation sued, that
and Omega and M/V Star Ace, 24 to which Cadacio and Rada filed a Joint it is safe to assume that said representative had sufficient responsibility and
38
Delegation of Authority to Corporate Officers | Doctrine of Apparent Authority
After trial, the trial court adjudged the corporation liable to Alejandro Te in that in September, 1970, the price per bag was P14.50, and by the middle NO.
the amount of P3,302,400.00 as actual damages, P100,000.00 as moral of 1975, it was already P37.50 per bag. Despite this, no provision was
damages, and P10,000.00 as and for attorney's fees and costs. The made in the "dealership agreement" to allow for an increase in price The ground which was relied upon by the trial court in acquitting the
appellate court affirmed the said decision. Hence, this petition before the mutually acceptable to the parties. Instead, the price was pegged at P9.70 accused finds basis in the well-settled doctrine that a corporation has a
Court. per bag for the whole five years of the contract. Fairness on his part as a distinct personality from that of its stockholders/owners. A corporation is
director of the corporation from whom he was to buy the cement, would vested by law with a personality of its own, separate and distinct from that
ISSUE: Is there a valid dealership agreement between PCC and Te? require such a provision. In fact, this unfairness in the contract is also a of its stockholders and from that of its officers who manage and run its
basis which renders a contract entered into by the President, without affairs. This decision is assailed to be unjust mainly because it did not
RULING: None. authority from the Board of Directors, void or voidable, although it may have consider the Prosecution's Memorandum with Motion and Motion for Early
All corporate powers shall be exercised by the Board of Directors, except been in the ordinary course of business. We believe that the fixed price of Resolution filed by private prosecutor, herein complainant, on February 8,
as otherwise provided by law. The Board may expressly delegate specific P9.70 per bag for a period of five years was not fair and reasonable.
1991 and February 11, 1991, respectively.
powers to its President or any of its officers. In the absence of such express
delegation, a contract entered into by its President, on behalf of the As director, specially since he was the other party in interest, respondent According to complainant, had respondent judge taken the former motion
corporation, may still bind the corporation if the board should ratify the Te's bounden duty was to act in such manner as not to unduly prejudice into account, he would not have acquitted the accused, Jose V. Rosario.
same expressly or impliedly. These rules are basic, but are all general and the corporation. In the light of the circumstances of this case, it is to Us quite
Instead, he would have been held guilty for giving others an opportunity
thus quite flexible. They apply where the President or other officer, clear that he was guilty of disloyalty to the corporation; he was attempting
purportedly acting for the corporation, is dealing with a third person, i.e., a in effect, to enrich himself at the expense of the corporation. There is no engage in unfair competition as prescribed by Article 189 of the Revised
person outside the corporation. showing that the stockholders ratified the "dealership agreement" or that Penal Code.
they were fully aware of its provisions. The contract was therefore not valid
In the first place, it would not have made any difference because Jose v.
The situation is quite different where a director or officer is dealing with his and this Court cannot allow him to reap the fruits of his disloyalty.
own corporation. In the instant case respondent Te was not an ordinary Rosario was charged as owner/proprietor. COD is not a single
stockholder; he was a member of the Board of Directors and Auditor of the As a result of this action which has been proven to be without legal basis, proprietorship but one that is run and owned by a corporation, Rosario
corporation as well. He was what is often referred to as a "self-dealing" petitioner corporation's reputation and goodwill have been prejudiced. Bros., Inc., of which the accused is stockholder and Executive Vice-
director. However, there can be no award for moral damages under Article 2217 President. A stockholder generally does not have a hand in the
and succeeding articles on Section 1 of Chapter 3 of Title XVIII of the Civil management of the corporate affairs. On the other hand, the VicePresident
A director of a corporation holds a position of trust and as such, he owes a Code in favor of a corporation. had no inherent power to bind the corporation.
duty of loyalty to his corporation. In case his interests conflict with those of
the corporation, he cannot sacrifice the latter to his own advantage and As general rule, his duties must be specified in the by-laws. In the criminal
benefit. As corporate managers, directors are committed to seek the case, the information did not specify his duties as Executive Vice-
maximum amount of profits for the corporation. This trust relationship "is 48. LOUIS VUITTON S.A. vs. JUDGE FRANCISCO DIAZ President. The trial court had no basis for holding that as such, the accused
not a matter of statutory or technical law. It springs from the fact that VILLANUEVA entered into a contract with the concessionaire thereby giving the latter an
directors have the control and guidance of corporate affairs and property A.M. No. MTJ-92-643 November 27, 1992 opportunity to practice unfair competition. Whereas, Section 23 of the
and hence of the property interests of the stockholders Corporation Code is explicit that the directors, acting as a body, exercise
FACTS corporation powers and conduct the corporation's business.
Granting arguendo that the "dealership agreement" involved here would
be valid and enforceable if entered into with a person other than a director In Criminal Case No. XXXVI-62431, entitled "People of the Philippines vs.
or officer of the corporation, the fact that the other party to the contract was Jose V. Rosario", Louis Vuitton, S.A. accused the latter of unfair
a Director and Auditor of the petitioner corporation changes the whole competition as defined by paragraph 1 of Article 189, Revised Penal Code. 49. AYALA LAND V. ASB
situation. First of all, We believe that the contract was neither fair nor G.R. No. 210043 September 26, 2018
reasonable. The "dealership agreement" entered into in July, 1969, was to Complainant also assailed respondent judge's findings that there was no
sell and supply to respondent Te 20,000 bags of white cement per month, unfair competition because the elements of the crime were not met, and Facts:
for five years starting September, 1970, at the fixed price of P9.70 per bag. that he seized articles did not come close to the appearance of a genuine
Respondent Te is a businessman himself and must have known, or at Louis Vuitton product, the counterfeit items having been poorly, done. ALI and ASBRC are domestic corporations engaged in real estate
least must be presumed to know, that at that time, prices of commodities development. On the other hand, EMRASON is a domestic corporation
in general, and white cement in particular, were not stable and were ISSUE principally organized to manage a 372- hectare property located in
expected to rise. At the time of the contract, petitioner corporation had not Dasmariñas, Cavite (Dasmariñas Property ).
even commenced the manufacture of white cement, the reason why Whether or not respondent judge is guilty of knowingly rendering a
delivery was not to begin until 14 months later. He must have known that manifestly unjust judgment.
The parties' respective versions of the factual antecedents are, as follows:
within that period of six years, there would be a considerable rise in the
price of white cement. In fact, respondent Te's own Memorandum shows RULING
39
Delegation of Authority to Corporate Officers | Doctrine of Apparent Authority
Version of the Petitioner Board Resolution, which approved the Letter-Agreement and authorized Issue:
Ramos, Sr. and Antonio to sign the same, was in tum likewise approved
by EMRASON 's stockholders on the same date, May 17, 1994. 1.) Whether or not the Ramos children had the authority to
ALI claimed that, sometime in August 1992, EMRASON's enter into a Contract to Sell on behalf of EMRASON.
brokers sent a proposal for a joint venture agreement (JVA) between ALI After ASBRC learned about the Contract to Sell executed 2.) Whether or not Ramos Sr. had the authority to execute and
and EMRASON for the development of EMRASON's Dasmariñas between ALI and the Ramos children and the annotation of the Contract to enter into the Letter Agreement with ASBRC.
Property. ALI initially declined but eventually negotiated with Ramos, Jr., Sell on the transfer certificates of title (TCTs) covering the Dasmariñas Held:
Antonio B. Ramos (Antonio), and Januario to discuss the terms of the JVA. Property, ASBRC and EMRASON filed a Complaint for the nullification of
According to ALI, EMRASON made it appear that Ramos, Jr., Antonio, Contract to sell and the cancellation of the annotations on the TCTs over 1.) A perusal of the August 3, 1993 letter shows that
and Januario had full authority to act on EMRASON's behalf in relation to the Dasmariñas Property. EMRASON, through Ramos, Sr. authorized Ramos, Jr.
the JVA. ALI alleged that Emerita Ramos, Sr. (Ramos, Sr.), then and Antonio merely to "collaborate and continue
EMRASON's President and Chairman, wrote to ALI and therein The RTC declared that the Contract to Sell between ALI and negotiating and discussing with [ALI] terms and
acknowledged that Ramos, Jr. and Antonio were fully authorized to Ramos children void because of the latter’s lack of authority to sign the conditions that are mutually beneficial to the parties therein.
represent EMRASON in the JVA, as shown in Ramos, Sr.'s letter dated Contract to Sell on behalf of EMRASON. The RTC held that the Ramos Nothing more, nothing less. To construe the letter as a
August 3, 1993. children failed to adduce evidence to show that they have been validly virtual carte blanche for the Ramos children to enter into a
authorized by the BOD of EMRASON to enter into a Contract to Sell with Contract to Sell regarding the Dasmariñas Property would
ALI and the Ramos children subsequently entered into a ALI. be unduly stretching one's imagination. Acts done by the
Contract to Sell dated May 18, 1994, under which ALI agreed to purchase corporate officers beyond the scope of their authority
the Dasmariñas Property. In addition, the RTC declared valid the Letter-Agreement cannot bind the corporation unless it has ratified such acts
deeding the Dasmariñas Property to ASBRC. Following this Court's ruling expressly or is estopped from denying them. What is clear
ALI alleged that it came to know that a Letter-Agreement dated in People's Aircargo and Warehousing Company, Inc. v. Court of Appeals, from the letter is that EMRASON authorized the Ramos
May 21, 1994 (Letter-Agreement) and a Real Estate Mortgage respecting the RTC held that Ramos, Sr., as President of EMRASON, had the children only to negotiate the terms of a potential sale over
the Dasmariñas Property had been executed by Ramos, Sr. and Antonio authority to enter into the Letter-Agreement because "the president is the Dasmariñas Property, and not to sell the property in an
for and in behalf of EMRASON, on one hand, and ASBRC on the other. It presumed to have the authority to act within the domain of the general absolute way or act as signatories in the contract.
also alleged that the Ramos children wrote to Luke C. Roxas, ASBRC's objectives of [a company's] business and within the scope of [the
President, informing the latter of the Contract to Sell between ALI and president's] usual duties. Equally misplaced is ALI's reliance on our
EMRASON. pronouncement in People's Aircargo Warehousing v. Court
The RTC further explained that, assuming arguendo that the
of Appeals, where we said that the authority of the apparent
Version of the Respondents signing of the Letter-Agreement was outside the usual powers of Emerito
agents may be expressly or impliedly shown by habit,
Ramos, Sr., as president, EMRASON's ratification of the Letter-
custom or acquiescence in the general course of
For their part, respondents averred that ALI submitted to Agreement via a stockholders' meeting on March 6, 1995, cured the defect
business. For, indeed, ALI never mentioned or pointed to
EMRASON and Ramos, Sr. its proposal to purchase the Dasmariñas caused by Ramos, Sr.'s apparent lack of authority.
certain palpable acts by the Ramos children which were
Property which proposal was however rejected.
The CA dismissed the appeal and affirmed the RTC's findings. indicative of a habit, custom, or acquiescence in the general
On May 17, 1994, EMRASON, through Ramos, Sr., informed The CA reiterated the RTC's pronouncement that the Ramos children course of business that compel the conclusion that
ALI that it had decided to accept the proposal of ASBRC because the failed to prove their authority to enter into a Contract to Sell on behalf of EMRASON must be deemed to have been bound thereby
latter's terms were more beneficial and advantageous to EMRASON. As a EMRASON. Citing ALI's letters addressed to Ramos, Sr. and the latter's implacably and irretrievably. ALI's bare allegation that "the
result, ASBRC and EMRASON entered into a Letter-Agreement on May uncontroverted deposition "that he is the corporation's sole and exclusive Ramos children submitted corporate documents to ALI to
21, 1994. The following day, or on May 22, 1994, EMRASON executed a authorized representative in the sale of the Dasmariñas Property" vis-a-vis convince it that it was negotiating with the controlling
Real Estate Mortgage in compliance with its obligations under the said the Ramos children's limited authority to negotiate for the best terms of a shareholders of EMRASON" is gratuitous and self-serving,
Letter-Agreement. sale, the CA then declared that ALI knew or was aware of the Ramos hence, does not merit this Court's consideration. As an
children's lack of authority. In sustaining the validity of the Letter-Agreement established business entity engaged in real estate, ALI
between EMRASON and ASBRC, the appellate court effectively held that should know that a corporation acts through its Board of
Ramos, Sr. was invested with the presumed authority to enter into the said Directors and not through its controlling shareholders.
Prior to the execution of the Letter-Agreement, a special
Letter-Agreement.
stockholders' meeting was held on May 17, 1994 during which
EMRASON's stockholders "authorized, approved, confirmed and ratified 2.) Here, Ramos, Sr.'s authority to execute and enter into the
the Resolution of EMRASON's Board of Directors (Board Resolution). The Letter-Agreement with ASBRC was clearly proven. ALI's
40
Delegation of Authority to Corporate Officers | Doctrine of Apparent Authority
argument that respondents failed to establish that [Ramos], Section 23 of the Corporation Code expressly provides that the Being a mere branch manager alone is insufficient to support the
Sr. had been in the habit of executing contracts on behalf of corporate powers of all corporations shall be exercised by the board of conclusion that Mondigo has been clothed with apparent authority to
EMRASON is negated by the fact that correspondences directors. The power and the responsibility to decide whether the verbally alter terms of written contracts, especially when viewed against the
between ALI and EMRASON had always been addressed corporation should enter into a contract that will bind the corporation are telling circumstances of this case: the unequivocal provision in the
to Ramos, Sr. In fact, ALI must be deemed to have lodged in the board, subject to the articles of incorporation, bylaws, or mortgage contract; PCRBs vigorous denial that any agreement to release
acknowledged the authority of Ramos, Sr. to act on behalf relevant provisions of law. In the absence of authority from the board of the mortgage was ever entered into by it; and, the fact that the purported
directors, no person, not even its officers, can validly bind a corporation. agreement was not even reduced into writing considering its legal effects
of EMRASON when ALI relied on the August 3, 1993 letter
on the parties interests. To put it simply, the burden of proving the authority
of Ramos, Sr. In any case, this Court clarified in People's
However, just as a natural person may authorize another to do of Mondigo to alter or novate the mortgage contract has not been
Aircargo that it is not the quantity of similar acts which certain acts for and on his behalf, the board of directors may validly established.
establishes apparent authority, but the vesting of a delegate some of its functions and powers to its officers, committees or
corporate officer with the power to bind the corporation. agents. The authority of these individuals to bind the corporation is
Together with this Court's pronouncement that "a party generally derived from law, corporate bylaws or authorization from the 51. SARGASSO v. PPA
dealing with the president of a corporation is entitled to board, either expressly or impliedly by habit, custom or acquiescence in Sargasso Construction and Development Corporation, Pick and
assume that he has the authority to enter, on behalf of the the general course of business. Shovel, Inc. and Atlantic Erectors, Inc., a joint venture vs Philippine
corporation, into contracts that are within the scope of the Ports Authority
powers of said corporation and that do not violate any The doctrine of apparent authority, on the other hand, with 5 July 2010
statute or rule on public policy," the inevitable conclusion is special reference to banks, had long been recognized in this
that Ramos, Sr. was properly authorized to, and validly jurisdiction. The existence of apparent authority may be ascertained Facts:
executed with ASBRC, the said Letter-Agreement. through:
Plaintiff Sargasso was awarded the construction of Pier 2 and the rock
1) the general manner in which the corporation causeway (R.C. Pier 2) for the port of San Fernando, La Union, after a
holds out an officer or agent as having the power public bidding conducted by the defendant PPA. Implementation of the
0. BANATE v. PHILIPPINE COUNTRYSIDE to act, or in other words, the apparent authority to project commenced on August 14, 1990. The port construction was in
G.R. No. 163825; July 13, 2010 act in general, with which it clothes him; or pursuance of the development of the Northwest Luzon Growth
2) the acquiescence in his acts of a particular Quadrangle. Adjacent to Pier 2 is an area of P4,280 square meters
FACTS: nature, with actual or constructive knowledge intended for the reclamation project as part of the overall port development
thereof, within or beyond the scope of his plan.
Spouses Maglasang obtained a loan from PCRB which was ordinary powers.
evidenced by a promissory note payable on January 1998. A mortgage Mr. Melecio J. Go, Executive Director of the consortium, plaintiff offered to
over the properties owned by Sps. Cortel was executed to secure the loan. undertake the reclamation between the Timber Pier and Pier 2 of
Two other loans were subsequently obtained from PCRB which were Accordingly, the authority to act for and to bind a corporation may the Port of San Fernando, La Union, as an extra work to its existing
covered by separate PNs and secured by mortgages on their other be presumed from acts of recognition in other instances when the power construction of R.C. Pier 2 and Rock Causeway for a price
properties. was exercised without any objection from its board or shareholders. of P36,294,857.03.
In 1997, before the loan became due, Sps. Maglasang and Sps.
Cortel asked PCRB’s permission to sell the mortgaged properties. Under the doctrine of apparent authority, acts and contracts of Defendant replied thru its Assistant General Manager Teofilo H. Landicho
According to them, the PCRB Branch Manager, Pancrasio Mondigo, the agent, as are within the apparent scope of the authority conferred on stating that “Your proposal to undertake the project at a total cost of
verbally agreed to their request but required the full payment of the 1st loan. him, although no actual authority to do such acts or to make such contracts THIRTY SIX MILLION TWO HUNDRED NINETY FOUR THOUSAND
Thereafter, the property was sold to Banate. Title over the properties was has been conferred, bind the principal. The principals liability, however, is EIGHT HUNDRED FIFTY SEVEN AND 03/100 PESOS
given to Banate, but with an annotation of mortgage lien in favor of PCRB. limited only to third persons who have been led reasonably to believe by (P36,294,857.03) is not acceptable to PPA. If you can reduce your offer to
PCRB refused to comply with the request to Release Mortgage hence this the conduct of the principal that such actual authority exists, although none THIRTY MILLION SEVEN HUNDRED NINETY FOUR THOUSAND
action for specific performance by the spouses. was given. TWO HUNDRED THIRTY AND 89/100 (P30,794,230.89) we may
The RTC ruled in favor of the spouses. CA reversed RTC, hence, this consider favorably award of the project in your favor, subject to the
petition. In the present case, no proof of the course of business, usages approval of higher authority.”
and practices of the bank about, or knowledge that the board had or is
ISSUE: Does the Branch Manager have apparent authority to change the presumed to have of, its responsible officers acts regarding bank branch Notice of Award signed by PPA General Manager Rogelio Dayan was sent
terms and conditions of the contract? affairs, was ever adduced to establish the branch managers to plaintiff for the phase I Reclamation Contract in the amount
apparent authority to verbally alter the terms of mortgage of P30,794,230.89 and instructing it to enter into and execute the contract
RULING: None. contracts. Neither was there any allegation, much less proof, that PCRB agreement with this Office and to furnish the documents representing
ratified Mondigos act or is estopped to make a contrary claim. performance security and credit line. Defendant likewise stated [and] made
41
Delegation of Authority to Corporate Officers | Doctrine of Apparent Authority
it a condition that fendering of Pier No. 2 Port of San Fernando, and basis considering that individual contractors of the joint venture have as the power to affect the legal relations of another person by transactions
the Port of Tabaco is completed before the approval of the contract for the undertaken separately bigger projects, and have been such individual with third persons arising from the others manifestations to such third
reclamation project. Installation of the rubber dock fenders in the said ports contractors for almost 16 years. person such that the liability of the principal for the acts and contracts of his
was accomplished in the year 1994. PPA Management further set a agent extends to those which are within the apparent scope of the authority
condition [that] the acceptance by the contractor that Defendant PPA thru the Office of the Government Corporate Counsel conferred on him, although no actual authority to do such acts or to make
mobilization/demobilization cost shall not be included in the contract and (OGCC) filed its Answer with Compulsory Counterclaim contending that such contracts has been conferred.
that escalation shall be reckoned upon approval of the Supplemental the alleged Notice of Award has already been properly revoked when the
Agreement. Supplemental Agreement which should have implemented the award was Apparent authority, or what is sometimes referred to as the
denied approval by defendants Board of Directors. As to plaintiffs pre- holding out theory, or doctrine of ostensible agency, imposes liability, not
At its meeting held on September 9, 1994, the Board decided not to disqualification from participating in the bidding for the extension of R.C. as the result of the reality of a contractual relationship, but rather because
approve the contract proposal, as reflected in the following excerpt of the Pier No. 2 Project the same is based on factual determination by the of the actions of a principal or an employer in somehow misleading the
minutes taken during said board meeting: defendant that plaintiff lacked IAC Registration and Classification and public into believing that the relationship or the authority exists. The
equipment for the said project as communicated in the August 9, existence of apparent authority may be ascertained through (1) the general
After due deliberation, 1996 letter. Defendant disclaimed any liability for whatever damages manner in which the corporation holds out an officer or agent as having the
the Board advised suffered by the plaintiff when it jumped the gun by committing its alleged power to act or, in other words, the apparent authority to act in general, with
Management to bid the project resources for the reclamation project despite the fact that no Notice to which it clothes him; or (2) the acquiescence in his acts of a particular
since there is no strong legal Proceed was issued to plaintiff by the defendant. nature, with actual or constructive knowledge thereof, whether within or
basis for Management to beyond the scope of his ordinary powers. It requires presentation of
award the supplemental The lower court rendered a decision in favor of the plaintiff. The evidence of similar act(s) executed either in its favor or in favor of other
contract through negotiation. trial court added that the tenor of the Notice of Award implied that parties.
The Board noted that the Pier 2 respondents general manager had been empowered by its Board of
Project was basically for the Directors to bind respondent by contract. It noted that whereas the letter- Easily discernible from the foregoing is that apparent authority is
construction of a pier while the reply contained the phrase approval of the higher authority, the determined only by the acts of the principal and not by the acts of the agent.
supplemental agreement refers to conspicuous absence of the same in the Notice of Award supported the The principal is, therefore, not responsible where the agents own conduct
reclamation. Thus there is no finding that the general manager had been vested with authority to enter and statements have created the apparent authority.
basis to compare the terms and into the contract for and in behalf of respondent. To the trial court, the
conditions of the reclamation disapproval by the PPA Board of the supplementary contract for the In this case, not a single act of respondent, acting through its
project with the original contract reclamation on a ground other than the general managers lack of authority Board of Directors, was cited as having clothed its general manager with
(Pier 2 Project) of Sargasso. was an explicit recognition that the latter was so authorized to enter into the apparent authority to execute the contract with it.
purported contract.
Plaintiff learned that the Board was not inclined to favor its Supplemental
Agreement, Mr. Go wrote General Manager Agustin requesting that the The CA also found the disapproval of the contract on a ground
same be presented again to the Board meeting for approval. However, other than the general managers lack of authority rather inconsequential 52. ASSOCIATED BANK (now UOB) v. SPOUSES PRONSTROLLER
no reply was received by plaintiff from the defendant. because Executive Order 380 expressly authorized the governing boards Gr no. 148444, July 14, 2008
of government-owned or controlled corporations to enter into negotiated
On June 30, 1997, plaintiff filed a complaint for specific infrastructure contracts involving not more than fifty million (P50 million). FACTS:
performance and damages before the Regional Trial Court of Manila The CA further noted that the Notice of Award was only one of those Spouses Vaca executed a REM in favor of the UOB over their
alleging that defendant PPAs unjustified refusal to comply with its documents that comprised the entire contract and, therefore, did not in itself parcel of residential land but failed to pay their obligation, the subject
undertaking, unnecessarily leading to the delay in the implementation of evidence the perfection of a contract. property was sold at public auction with the UOB as the highest bidder.
the award under the August 26, 1993 Notice of Award, has put on hold
plaintiffs men and resources earmarked for the project, aside from The spouses Vaca commenced an action for the nullification of
effectively tying its hands in undertaking other projects for fear that plaintiffs Issue: REM and the foreclosure sale. UOB, on the other hand, filed a petition for
incapacity to undertake work might be spread thinly and it might not be able Whether or not the doctrine of apparent authority is applicable in this case. the issuance of a writ of possession which the CA granted.
to function efficiently if the PPA project and other projects should require
simultaneous attention. Plaintiff averred that it sought reconsideration of Ruling: During the pendency of the cases, Respondents Rafael and
the August 9, 1996 letter of PPA informing it that it did not qualify to bid for No. Petitioners invocation of the doctrine of apparent authority is Monaliza Pronstroller offered to purchase the property for P7,500,000.00.
the proposed extension of RC Pier No. 2, Port of San Fernando, La Union misplaced. This doctrine, in the realm of government contracts, has been Said offer was made through Atty. Soluta, UOB’s Vice-President,
for not having IAC Registration and Classification and not complying with restated to mean that the government is NOT bound by unauthorized acts Corporate Secretary and a member of its Board of Directors. UOB
equipment requirement. In its letter dated September 19, 1996, plaintiff of its agents, even though within the apparent scope of their accepted it and respondents paid petitioner P750,000.00 as down
pointed out that the disqualification was clearly unjust and totally without authority. Under the law on agency, however, apparent authority is defined payment.
42
Delegation of Authority to Corporate Officers | Doctrine of Apparent Authority
For failure of the parties to reach an agreement, respondents, Naturally, the third person has little or no information as to
March 18, 1993 UOB, through Atty. Soluta, and respondents, through their counsel, informed petitioner that they would be enforcing what occurs in corporate meetings since it is entirely an internal matter.
executed a Letter-Agreement that the down payment and remaining their agreement dated July 14, 1993.2 Petitioner countered that it was the public has the right to rely on the trustworthiness of bank officers
balance would be deposited under escrow. The deposit shall be made not aware of the existence of the July 14 agreement and that Atty. and their acts Hence, petitioner may not impute negligence on the part of
within ninety (90) days from date hereof. Soluta was not authorized to sign for and on behalf of the bank. It, the respondents in failing to find out the scope of Atty. Solutas authority.
likewise, reiterated the rescission of their previous agreement
Prior to the expiration of the 90-day period, in view of the because of the breach committed by respondents. respondents already requested a modification of the earlier
pendency of the case between the spouses Vaca and UOB involving the agreement such that the full payment should be made upon receipt of this
subject property, respondents requested that the balance of the purchase On July 14, 1994, in the Vaca case, this Court upheld Courts decision confirming petitioners right to the subject property. The
price be made payable only upon service on them of a final decision petitioners right to possess the subject property. matter was brought to the petitioners attention and was in fact discussed
affirming UOB right to possess the subject property. Atty. Soluta referred by the members of the Board. Instead of acting on said request, the board
respondents proposal to petitioners Asset Recovery and Remedial On July 28, 1994, respondents commenced the instant suit by deferred action on the request. It was only after one year and after the
Management Committee (ARRMC) but the latter deferred action thereon. filing a Complaint for Specific Performance before the RTC. banks reorganization that the board rejected respondents request. We
Respondents prayed that petitioner be ordered to sell the subject property cannot therefore blame the respondents in relying on the July 14, 1993
On July 14, 1993, a month after they made the request and after to them in accordance with their letter-agreement of July 14, 1993. Letter-Agreement.
the payment deadline had lapsed respondents and Atty. Soluta, acting for For its part, petitioner contended that their contract had already
the petitioner, executed another Letter-Agreement allowing the former to been rescinded because of respondents failure to deposit in escrow the if a corporation knowingly permits its officer, or any other agent,
pay the balance of the purchase price upon receipt of a final order from this balance of the purchase price within the stipulated period to perform acts within the scope of an apparent authority, holding him out
Court and/or the delivery of the property to them free from occupants. to the public as possessing power to do those acts, the corporation will, as
During the pendency of the case, petitioner sold the subject against any person who has dealt in good faith with the corporation through
Towards the end of 1993, petitioner reorganized its property to the spouses Vaca, who eventually registered the sale; and As such agent, be estopped from denying such authority.
management. Atty. Dayday became petitioners Assistant Vice-President new owners, the spouses Vaca started demolishing the house on the
and Head of the Documentation Section, while Atty. Soluta was relieved of subject property which, however, was not completed by virtue of the writ of Was the second-letter agreement validly rescinded by
his responsibilities. Atty. Dayday discovered that respondents failed to preliminary injunction issued by the court.
deposit the balance of the purchase price of the subject property and found Petitioner UOB?
that respondents requested for an extension of time within which to pay ISSUE:
which late disapproved by ARRMC, hence, the contract was rescinded Is the petitioner bound by the July 14, 1993 Letter-Agreement
due to respondents breach of contract. No. This is so because there was in the first place, no
signed by Atty. Soluta under the doctrine of apparent authority?
breach of contract, as the date of full payment had already been
Atty. Dayday informed respondents that their request for RULING:
extension was disapproved by ARRMC and, in view of their breach of the modified by the later agreement. Neither can the July 14, 1993
YES. A corporation may authorize another to do certain acts for
contract, petitioner was rescinding the same and forfeiting their deposit. and on his behalf. The authority of a corporate officer or agent in dealing agreement be considered abandoned by respondents act of
Petitioner added that if respondents were still interested in buying the with third persons may be actual or apparent. Apparent authority may be
subject property, they had to submit their new proposal. Respondents making a new offer, which was unfortunately rejected by petitioner.
ascertained through 1) the apparent authority to act in general, with which
reiterated the Letter-Agreement of July 14, 1993 to show that they were it clothes him; or 2) the acquiescence in his acts of a particular nature, with such offer was made only to demonstrate their capacity to
granted an extension. However, Atty. Dayday claimed that the letter was a actual or constructive knowledge thereof, within or beyond the scope of his
mistake and that Atty. Soluta was not authorized to give such extension purchase the subject property.
ordinary powers. Accordingly, the authority to act for and to bind a
corporation may be presumed from acts of recognition in other instances,
respondents proposed to pay the balance of the purchase price as follows: wherein the power was exercised without any objection from its board or LITIS PENDENTIA
P3,000,000.00 and the balance after six (6) months.1 However, the shareholders
proposal was disapproved petitioner advised respondents that the former
would accept the latters proposal only if they would pay interest at the rate It is clear that petitioner had previously allowed Atty. Soluta to Admittedly, during the pendency of the case, respondents timely
of 24.5% per annum on the unpaid balance. Petitioner also allowed enter into the first agreement without a board resolution expressly
respondents a refund of their deposit of P750,000.00 if they would not registered a notice of lis pendens to warn the whole world that the property
authorizing him; thus, it had clothed him with apparent authority to
agree to petitioners new proposal. modify the second letter-agreement. was the subject of a pending litigation.

43
Delegation of Authority to Corporate Officers | Doctrine of Apparent Authority
This registration, therefore, gives the court clear authority to May 6, 1962, all the Board of Directors, President Verano, Vice President DEFENDANTS CONTENTION:
Narciso, unanimously authorized Verano, by a formal resolution, "to
cancel the title of the spouses Vaca, since the sale of the subject property
execute any agreement with any person or entity, on behalf of the In support of the motion defendants alleged that the contract for services
was made after the notice of lis pendens. Settled is the rule that the notice corporation, for the purpose of securing additional funds for the corporation, was never perfected because it was not approved or ratified but was
as well as to secure the services of such person or entity, in the collection instead disapproved by the Board of Directors of defendant Batac
is not considered a collateral attack on the title,3 for the indefeasibility of the
of all payments due to the corporation from the PVTA for any tobacco sold Procoma, Inc., and that on the basis of plaintiff's pleadings the contract is
title shall not be used to defraud another especially if the latter performs and delivered to said administration”. void and unenforceable. Defendants further denied the fact that plaintiff
had performed his part of the contract, alleging that he had not in any
acts to protect his rights such as the timely registration of a notice of lis
Plaintiff was made to understand by all of said defendants that the original manner intervened in the delivery and payment of tobacco pertaining to the
pendens. understanding between him and defendant Leon Q. Verano was defendant corporation.
acceptable to the corporation, except that the remuneration for the plaintiff's
services would be P0.30 per kilo of tobacco. The trial court ruled in favor of defendants ruling that the agreement was
not perfected and was therefore null and void and that the complaint states
53. ACUÑA v. BATAC May 10, 1962, the formal "Agreement" was executed between plaintiff and no cause of action.
defendant Leon Q. Verano, as Manager of the defendant corporation, duly
EMILIANO ACUÑA vs. authorized by its Board of Directors for such purpose, and signed by Plaintiff thus raises the appeal.
BATAC PRODUCERS COOPERATIVE MARKETING ASSOCIATION, defendants Justino Galano and Dr. Emmanuel Bumanglag as
INC., JUSTINO GALANO, TEODORO NARCISO, PABLO BACTIN, instrumental witnesses and acknowledged by Atty. Fernando Alcantara,
(DR.) EMMANUEL BUMANGLAG, VENANCIO DIRIC, MARCOS the Secretary and Legal Counsel of the defendant corporation.
ESQUIVEL, EVARISTO CAOILI, FIDEL BATTULAYAN, DAMIAN ISSUE:
ROSSINI, RAYMUNDO BATALLONES, PLACIDO QUIAOIT, and May 10, 1962, plaintiff gave and turned over to the defendant corporation,
LEON Q. VERANO defendants-appellees. thru its treasurer, Dominador T. Cocson the sum of P20,000.00, in the Whether or not the “agreement” is valid and binding.
presence of defendants Verano, Galano, Dr. Bumanglag and Atty. o
Alcantara, for which said treasurer issued to plaintiff its corresponding
G.R. No. L-20333
Official Receipt No. 130852.
HELD:
June 30, 1967 From then on, plaintiff diligently and religiously kept his part of the
agreement even incurring personal expenses in order to comply with such YES.
agreement.
CIVIL PROCEDURE:
June 6, 1962, After the defendant corporation was able to replenish its
FACTS: funds with collections from the tobacco delivered (as serviced by plaintiff) A MOD based on the ground that the complaint does not state a cause of
and with a collection of P381,495.00, the Board of Directors disapproved action, the averments in the complaint are deemed hypothetically admitted
May 5, 1962 Emiliano Acuña and defendant Leon Q. Verano, as Manager and the inquiry is limited to whether or not they make out a case on which
the Agreement.
of the defendant Batac Procoma, Inc., that the former would seek and relief can be granted. The lower court should not have gone beyond, and
obtain the sum of not less, than P20,000.00 to be advanced to the Thus the filing of the complaint by Acuna against Batac Procoma and its it should have limited itself, to the facts alleged in the complaint in
defendant Batac Procoma, Inc., to be utilized by it as additional funds for officers. considering and resolving said motion to dismiss.
its Virginia tobacco buying operations during the current redrying season.
Acuna would be constituted as the corporation's representative in Manila August 14, 1962, the trial court ordered the issuance of preliminary CORPO:
to assist in handling and facilitating its continuous shipments of tobacco attachment against properties of defendant corporation.
and their delivery to the redrying plants and in speeding up the prompt A perusal of the complaint reveals that it contains sufficient allegations
payment and collection of all amounts due to the corporation for such August 22, 1962, the defendants filed a motion to dismiss the complaint on indicating such approval or at least subsequent ratification. On the first
shipments. For his services, Acuna will be paid renumeration of P0.50 per the ground that it stated no cause of action and to discharge the preliminary point we note the following averments: that on May 9th the plaintiff met with
kilo of tobacco. attachment on the ground that it was improperly or irregularly issued. each and all of the individual defendants (who constituted the entire Board
of Directors) and discussed with them extensively the tentative agreement
This agreement was received by the Board of Directors (names are stated August 25, 1962, plaintiff filed a written opposition to the motion to dismiss and he was made to understand that it was acceptable to them, except as
as defendants in title) of Batac Procoma Inc. and to discharge the preliminary attachment. to plaintiff's remuneration; that it was finally agreed between plaintiff and all
said Directors that his remuneration would be P0.30 per kilo (of tobacco);

44
Delegation of Authority to Corporate Officers | Doctrine of Apparent Authority
and that after the agreement was formally executed he was assured by ventures took a turn for the worse when The Philippines was hit by four (4) company without formal authorization from the board. Thus, the Court
said Directors that there would be no need of formal approval by the Board. typhoons some time in December 1947. ruled that there has been established a practice that a corporate officer
It should be noted in this connection that although the contract required entrusted with the general management and control of its business has
such approval it did not specify just in what manner the same should be On January 30, 1948, Maximo Kalaw and the board approved implied authority to make any contract or do any other act which is
given. the various contracts despite knowing that the same are no longer deemed necessary or appropriate to the ordinary business of the corporation
profitable in light of the disasters the struck in the preceeding month leading without prior authority from the board.
1. plaintiff delivered to the defendant corporation the sum of to massive damage in the copra agricultural industry. As expected, the said
P20,000.00 as called for in the contract contracts were only partially fulfilled and NACOCO was threatened with It should be emphasized further that the need for ratification of
2. that he rendered the services he was required to do suits for the said contracts. Ultimately, the Government abolished the said contracts is no longer necessary in light of the established practice
3. that he furnished said defendant 3,000 sacks at a cost of NACOCO and subjected the liquidation of its assets through the Board of as earlier discussed. If anything, ratification of the Kalaw contracts has
Liquidators duly appointed by the Government. been reduced to a mere formality. Furthermore, this practice was not
P6,000.00 and advanced to it the further sum of P5,000.00
questioned in the long years of NACOCO’s existence and it was only
4. and that he did all of these things with the full knowledge, In a suit for damages, NACOCO sought to recover 1.34 Million questioned when the contracts entered into through that practice were not
acquiescence and consent of each and all of the individual pesos from board chairman Maximo Kalaw, now represented by his heirs, profitable.
defendants who constitute the Board of Directors of the and directors Bocar, Garcia (represented by his Estate) and Moll for having
defendant corporation. allowed Kalaw to enter into contracts that are doomed to fail without board Bad faith cannot also be imputed upon Kalaw when he entered
There is abundant authority in support of the proposition that ratification approval. The defendants contend that Kalaw legitimately entered into into the contracts. There is no proof evidencing that he intended to defraud
may be express or implied, and that implied ratification may take diverse those contracts in accordance with an established custom within the firm the government and its contractors. Kalaw and the directors were in good
forms, such as by silence or acquiescence; by acts showing approval or that a general manager’s transactions are no longer subject to board faith. Finally, this is a case of damnum absque injuria (damage without
adoption of the contract; or by acceptance and retention of benefits flowing approval. The trial court ruled in favor of Kalaw, et al. and thus this appeal injury) as the losses incurred by NACOCO were attributable to the
there from. by the Board of Liquidators to the SC. typhoons that gravely affected the copra industry; were it not for the
typhoons, NACOCO could have complied with its contractual obligations.
Since the undertaking of upgrading tobacco is deemed illegal before the Issue:
trial court that is why the case was dismissed, it is only incidental to the Are the contracts entered into by Kalaw during the subsistence
issue. The affidavit of plaintiff shows that he is not aware of the of NACOCO’s existence valid, binding, and therefore, 55. TRINIDAD J. FRANCISCO vs. GOVERNMENT SERVICE
terminologies of the tobacco industry and thus only relied in good faith of enforceable? INSURANCE SYSTEM
the assurance of the defendant corporation. This is merely incidental and -----------------------------
must be tried and heard with facts and evidences presented. Ruling: G.R. No. L-18155 March 30, 1963
Yes, the contracts entered into Kalaw are valid and binding. TRINIDAD J. FRANCISCO vs. GOVERNMENT SERVICE
Thus the order of dismissal is set aside and the case is remanded to the INSURANCE SYSTEM
court a qou for further proceeding. The Supreme Court noted that the argument set forth by the March 30, 1963
Board of Liquidators is banking on the provisions of NACOCO by-laws,
stating that the general manager may execute on behalf of the Corporation FACTS:
all contracts necessary for its business with prior board approval. But the
54. BOARD OF LIQUIDATORS v. KALAW Court nonetheless skirted this argument in light of the actual factual milieu On 10 October 1956, the plaintiff, Trinidad J. Francisco, in consideration of
BOARD OF LIQUIDATORS, REPRESENTING THE GOVERNMENT of the case. a loan in the amount of P400,000.00, out of which the sum of P336,100.00
vs. HEIRS OF MAXIMO M. KALAW, ET AL. was released to her, mortgaged in favor of the defendant, GSIS a parcel
GR No. L-18805 It was found that in the years of corporate existence of of land known as Vic-Mari Compound, located at Baesa, Quezon City,
August 14, 1967 NACOCO, its general manager has always entered into contracts in its payable within ten (10) years in monthly installments of P3,902.41, and
behalf without board approval. Because of the tedious and volatile nature with interest of 7% per annum compounded monthly.
SANCHEZ, J. of the copra industry there has been established a practice of letting a
general manager make the forward transactions without the need of board On 6 January 1959, the GSIS extrajudicially foreclosed the mortgage on
Facts: approval to expedite the procedures. It was proven on record that the ground that up to that date the plaintiff-mortgagor was in arrears on her
This case involves the National Coconut Corporation NACOCO left the forward sales of copra to the discretion of Kalaw. It was monthly installments in the amount of P52,000.00. Payments made by the
(NACOCO) and its ill-fated business ventures. Having been established even found that the predecessors of Kalaw as the general manager also plaintiff at the time of foreclosure amounted to P130,000.00. The System
since May 7, 1940 by virtue of its charter, CA 518, it is granted powers to entered into transactions without prior board approval. itself was the buyer of the property in the foreclosure sale.
deal in the market with coconut, copra and dessicated coconuts and their
by-products. From July to October 1947, NACOCO entered into various In light of the circumstances, despite the provisions of the by- On 20 February 1959, the plaintiff's father, Atty. Vicente J. Francisco, sent
contracts through Kalaw involving the trade and industry of copra. Its laws, this general practice and custom has become a matter of policy that a letter to the general manager of the defendant corporation, Mr. Rodolfo
has been established in the course of business thereby binding the P. Andal:
45
Delegation of Authority to Corporate Officers | Doctrine of Apparent Authority
(a) Atty. Francisco will pay pay said amount of P30,000 to the GSIS The offer of compromise made by plaintiff in the letter had been validly
if it would agree that after such payment the foreclosure of his Then the System sent three (3) letters asking the plaintiff for a proposal for accepted, and was binding on the defendant. The terms of the offer were
daughter's mortgage would be set aside the payment of her indebtedness, since according to the GSIS the one- clear, and over the signature of defendant's general manager, Rodolfo
(b) As regards the balance, the following arrangements were year period for redemption had expired. Andal, plaintiff was informed telegraphically that her proposal had been
proposed: accepted. There was nothing in the telegram that hinted at any anomaly,
i. for the GSIS to take over the administration of the In reply, Atty. Francisco sent a letter, dated 11 March 1960, protesting or gave ground to suspect its veracity, and the plaintiff, therefore, can not
mortgaged property against the System's request for proposal of payment and inviting its be blamed for relying upon it.
ii. to collect the monthly installments, amounting to about attention to the concluded contract generated by his offer of 20 February
P5,000, due on the unpaid purchase price of more 1959, and its acceptance by telegram of the same date, the compliance of There is no denying that the telegram was within Andal's apparent
than 31 lots and houses therein and the monthly the terms of the offer already commenced by the plaintiff, and the authority, but the defense is that he did not sign it, but that it was sent by
installments collected shall be applied to the payment misapplication by the System of the remittances she had made, and the Board Secretary in his name and without his knowledge. Assuming this
of Miss Francisco's arrearage until the same is fully requesting the proper corrections. to be true, how was appellee to know it? Corporate transactions would
covered. It is requested, however, that from the speedily come to a standstill were every person dealing with a corporation
amount of the monthly installments collected, the sum By letter, dated 31 May 1960, the defendant countered the preceding held duty-bound to disbelieve every act of its responsible officers, no matter
of P350.00 be deducted for necessary expenses, protest that that the telegram should be disregarded in view of its failure to how regular they should appear on their face.
such as to pay the security guard, the street-caretaker, express the contents of the board resolution due to the error of its minor
the Meralco Bill for the street lights and sundry items. employees in couching the correct wording of the telegram. A copy of the It has also been decided that —
(c) The proposed administration by the GSIS of the mortgaged excerpts of the resolution of the Board of Directors (No. 380, February 20, A very large part of the business of the country is carried on by
property will continue even after Miss Francisco's account shall 1959) was attached to the letter, showing the approval of Francisco's offer corporations. It certainly is not the practice of persons dealing
have been kept up to date. However, once the arrears shall have — with officers or agents who assume to act for such entities to
been paid, whatever amount of the monthly installments ... subject to the condition that Mr. Vicente J. Francisco shall pay insist on being shown the resolution of the board of directors
collected in excess of the amortization due on the loan will be all expenses incurred by the GSIS in the foreclosure of the authorizing the particular officer or agent to transact the particular
turned over to Miss Francisco. mortgage. business which he assumes to conduct. A person who knows
that the officer or agent of the corporation habitually transacts
On the same date, 20 February 1959, Atty. Francisco received the Inasmuch as, according to the defendant, the remittances previously made certain kinds of business for such corporation under
following telegram: by Atty. Francisco were allegedly not sufficient to pay off her daughter's circumstances which necessarily show knowledge on the part of
VICENTE FRANCISCO arrears, including attorney's fees incurred by the defendant in foreclosing those charged with the conduct of the corporate business
SAMANILLO BLDG. ESCOLTA. the mortgage, and the one-year period for redemption has expired, said assumes, as he has the right to assume, that such agent or
GSIS BOARD APPROVED YOUR REQUEST RE defendant, on 5 July 1960, consolidated the title to the compound in its officer is acting within the scope of his authority. (Curtis Land &
REDEMPTION OF FORECLOSED PROPERTY OF name, and gave notice thereof to the plaintiff on 26 July 1960 and to each Loan Co. vs. Interior Land Co., 137 Wis. 341, 118 N.W. 853, 129
YOUR DAUGHTER occupant of the compound. Am. St. Rep. 1068; as cited in 2 Fletcher's Encyclopedia, Priv.
ANDAL" Corp. 263, perm. Ed.)
Hence, the plaintiff instituted the present suit, for specific performance and
On 28 February 1959, Atty. Francisco remitted to the System, through damages. The defendant answered, pleading that the binding acceptance Indeed, it is well-settled that —
Andal, a check for P30,000.00, with an accompanying letter, which quoted of Francisco's offer was the resolution of the Board, and that Andal's If a private corporation intentionally or negligently clothes its
verbatim the telegram sent by Andal telegram, being erroneous, should be disregarded. officers or agents with apparent power to perform acts for it, the
corporation will be estopped to deny that such apparent authority
The defendant received the amount of P30,000.00, and issued therefor its After trial, the court below found that the offer of Atty. Francisco, dated 20 is real, as to innocent third persons dealing in good faith with
official receipt No. 1209874, dated 4 March 1959. It did not, however, take February 1959, made on behalf of his daughter, had been unqualifiedly such officers or agents. (2 Fletcher's Encyclopedia, Priv. Corp.
over the administration of the compound. In the meantime, the plaintiff accepted, and was binding, and rendered judgment as noted at the start of 255, Perm. Ed.)
received the monthly payments of some of the occupants thereat; then on this opinion.
4 March 1960, she remitted, through her father, the amount of P44,121.29, Hence, even if it were the board secretary who sent the telegram, the
representing the total monthly installments that she received from the ISSUE: corporation could not evade the binding effect produced by the telegram.
occupants for the period from March to December 1959 and January to
February 1960, minus expenses and real estate taxes. The defendant also Did the telegram generate a contract that is valid and binding upon the The defendant-appellant does not disown the telegram, and even asserts
received this amount, and issued the corresponding official receipt. parties? that it came from its offices, as may be gleaned from the letter, dated 31
Remittances, all accompanied by letters, corresponding to the months of May 1960, to Atty. Francisco, and signed "R. P. Andal, general manager
March, April, May, and June, 1960 and totalling P24,604.81 were also sent RULING: by Leovigildo Monasterial, legal counsel", wherein these phrases occur:
by the plaintiff to the defendant from time to time, all of which were received "the telegram sent ... by this office" and "the telegram we sent your"
and duly receipted for. (emphasis supplied), but it alleges mistake in couching the correct wording.
46
Delegation of Authority to Corporate Officers | Doctrine of Apparent Authority
This alleged mistake cannot be taken seriously, because while the Bank. However, Juanita and Felicisimo were not able to redeem the Yes. In this case, for the property to be transferred in their names, the
telegram is dated 20 February 1959, the defendant informed Atty. mortgaged properties hence the properties were foreclosed and was Register of Deeds required the submission of a board resolution from the
Francisco of the alleged mistake only on 31 May 1960, and all the while it transferred in the name of the Bank. Out of the 7 parcels of land, 5 of them bank confirming both the Deed of Sale and the authority of the bank
accepted the various other remittances, starting on 28 February 1959, sent
were sold by the Bank to the parents of Marife, who is Francisca and manager, Fe S. Tena, to enter into such transaction. The issue of the case
by the plaintiff to it in compliance with her performance of her part of the
new contract. Renato, as evidenced by a Deed of Sale executed by the Bank in favor of started when the Bank refused to issue the boad resolution.
Francisca and Renato. However, these 5 parcels of land cannot be
The inequity of permitting the System to deny its acceptance become more transferred in the name of the parents of Marife O. Nino as there is a need The General rule is that when a bank, by its acts and failure to act, has
patent when account is taken of the fact that in remitting the payment of to have a document of sale registered with the Register of Deeds. When clearly clothed its manager with apparent authority to sell an acquired asset
P30,000 advanced by her father, plaintiff's letter to Mr. Andal quoted Marife O. Nino went to the Register of Deeds with the needed document, in the normal course of business, it is legally obliged to confirm the
verbatim the telegram of acceptance. This was in itself notice to the the Register of Deeds said again that the document of sale cannot be transaction by issuing a board resolution to enable the buyers to register
corporation of the terms of the allegedly unauthorized telegram. the property in their names. It has a duty to perform necessary and lawful
registered without a board resolution executed by the Rural Bank. Marife
then went to the Rural Bank and requested for a board resolution so that acts to enable the other parties to enjoy all benefits of the contract which it
Yet, notwithstanding this notice, the defendant System pocketed the
amount, and kept silent about the telegram not being in accordance with the property can be transferred to the name of Francisca and Renato. had authorized.
the true facts, as it now alleges. This silence, taken together with the
unconditional acceptance of three other subsequent remittances from The Rural Bank then said to Marife that they cannot issue a board In this case, the Bank acknowledged, by its own acts or failure to act, the
plaintiff, constitutes in itself a binding ratification of the original agreement resolution because the new manager had no record of the sale. She was authority of Fe S. Tena to enter into binding contracts. After the execution
(Civil Code, Art. 1393). requested to furnish the bank a copy of the deed of sale, receipt for of the Deed of Sale, The Ocfemia childen occupied the properties in
ART. 1393. Ratification may be effected expressly or tacitly. It is payment, and an authority from her parents and siblings which she dispute and paid the real estate taxes due thereon. If the bank
understood that there is a tacit ratification if, with knowledge of management believed that it had title to the property, it should have taken
complied. After two weeks, no actions were taken by the Rural Bank.
the reason which renders the contract voidable and such reason
Marife asked the services of a lawyer and the lawyer wrote a letter to the some measures to prevent the infringement or invasion of its title thereto
having ceased, the person who has a right to invoke it should
execute an act which necessarily implies an intention to waive Rural Bank asking why is there no action taken by them. The bank replied and possession thereof.
his right. to theletter sent and again requested that a machine copy of the receipt of
payment be furnished to the bank which in fact was already complied with Tena had previously transacted business on behalf of the Bank, and the
Nowhere else do the circumstances call more insistently for the application by Marife. After several days, Marife reiterated her request for a board latter had acknowledged her authority. A bank is liable to innocent third
of the equitable maxim that between two innocent parties, the one who resolution but still no actions were taken by the Rural Bank for the reason persons where representation is made in the course of its normal business
made it possible for the wrong to be done should be the one to bear the by an agent like Manager Tena, even though such agent is abusing her
resulting loss. that they have no records of the sale. This prompted Marife and his lawyer
to file a petition for Mandamus with the Regional Trial Court (RTC) to authority. Clearly, persons dealing with her could not be blamed for
compel the Rural Bank to execute the Board Resolution. By the time the believing that she was authorized to transact business for and on behalf of
petition was filed, Francisca was already sick with a deteriorating condition the bank. In this light, the bank is estopped from questioning the authority
56. RURAL BANK OF MILAOR (Camarines Sur) v. FRANCISCA
OCFEMIA, et.al. and the Ocfemia children were not able to hospitalize their sick mother of the bank manager to enter into the contract of sale.
G.R. No. 137686, February 8, 2000 because of lack of funds. So they needed the property to be able to
mortgage the same and have funds for hospitalization.
Facts:
The RTC granted the petition. The CA affirmed the decision of the RTC.
The Respondents in this case, Marife O. Nino(Marife), Rowena Barrogo,
Felicisimo Ocfemia, Renato Ocfemia Jr. and Winston Ocfemia (The Issue:
Ocfemia children), are the children of Francisca Ocfemia and the late
Renato Ocfemia (Francisca and Renato) while the petitioner in this case is May the board of directors of a rural banking corporation be compelled to
the Rural Bank of Milaor (Rural Bank), where the subject properties were confirm a deed of absolute sale of real property owned by the corporation
mortgaged and the one who foreclosed such property. Juanita Arellano which deed of sale was executed by the bank manager without prior
Ocfemia(Juanita) and Felicisimo Ocfemia (Felicisimo) are the authority of the board of directors of the rural banking corporation?
grandparents of Marife O. Nino and of his co-respondents, were the
owners of the 7 parcels of land which is the subject property of this Ruling:
case.The Juanita and Felicisimo mortgaged 7 parcels of land to the Rural
47
Derivative Suit
57. ARNEL TY, MARIE TY, JASON ONG, WILLY DY, AND ALVIN TY Are the petitioners liable? the corporation had accounts payable amounting to P13,807.50, most of
v. NBI which were unpaid at the time petition for the dissolution of the corporation
G.R. No. 182147 December 15, 2010 Ruling: was financial condition, in contemplation of an insolvency and dissolution.

Facts: No, except as to petitioner Arnel U. Ty who is indisputably the As a second cause of action, Steinberg alleges that on July 24,
President of Omni. 1922, the officers and directors of the corporation approved a resolution for
Petitioners are stockholders of Omni Gas Corporation (Omni). the payment of P3,000 as dividends to its stockholders, which was
Omni is in the business of trading and refilling of Liquefied Petroleum Gas It may be noted that Sec. 4 above enumerates the persons who wrongfully done and in bad faith, and to the injury and fraud of its creditors.
(LPG) cylinders. The case all started when Joaquin Guevara Adarlo & may be held liable for violations of the law, viz: (1) the president, (2) general That at the time the petition for the dissolution of the corporation was
Caoile Law Offices (JGAC Law Offices) sent a letter to the NBI requesting, manager, (3) managing partner, (4) such other officer charged with the presented it had accounts payable in the sum of P9,241.19, "and
on behalf of their clients Shellane Dealers Association, Inc., Petron Gasul management of the business affairs of the corporation or juridical entity, or practically worthless accounts receivable."
Dealers Association, Inc., and Totalgaz Dealers Association, Inc., for the (5) the employee responsible for such violation.
surveillance, investigation, and apprehension of persons or establishments Defendants allege that the stocks were purchased by virtue of a
In this case, it is undisputed that petitioners are members of the resolution of the board of directors of the corporation "when the business
in Pasig City that are engaged in alleged illegal trading of petroleum
board of directors of Omni at the time pertinent. There can be no quibble of the company was going on very well." As to the second cause of action,
products and underfilling of branded LPG cylinders in violation of Batas
that the enumeration of persons who may be held liable for corporate they admit that the dividends were distributed, but alleged that such
Pambansa Blg. (BP) 33, as amended by Presidential Decree No. (PD)
violators of BP 33, as amended, excludes the members of the board of distribution was authorized by the board of directors, "and that the amount
1865.
directors. The board of directors is not directly engaged or charged with the represented by said dividends really constitutes a surplus profit of the
Agents De Jemil and Kawada attested to conducting running of the recurring business affairs of the corporation. Depending on corporation."
surveillance of Omni. They brought eight branded LPG cylinders the powers granted to them by the Articles of Incorporation, the members
of the board generally do not concern themselves with the day-to-day To summarize, the officers purchased their own stocks and
of Shellane, Petron Gasul, Totalgaz, and Superkalan Gaz to Omni for
distributed dividends, in the same meeting, while the corporation has
refilling. The NBI’s test-buy yielded positive results for violations of BP 33, affairs of the corporation, except those corporate officers who are charged
outstanding debts.
Section 2(a) in relation to Secs. 3(c) and 4, and Sec. 2(c) in relation to Sec. with running the business of the corporation and are concomitantly
4. Thus, Agent De Jemil filed an Application for Search Warrant resulting members of the board, like the President. Section 25 of the Corporation ISSUE: Did the directors exercise due diligence in their actions?
in the seizure of several items from Omni’s premises. It was established Code requires the president of a corporation to be also a member of the
that Omni is not an authorized refiller of Shellane, Petron board of directors. HELD: No.
Gasul, Totalgaz and Superkalan Gaz LPG cylinders.
Evidently, petitioner Arnel, as President, who manages the It is, indeed, peculiar that the action of the board in purchasing
Sec. 4 of BP 33, as amended, provides that when the offender business affairs of Omni, can be held liable for probable violations by Omni the stock from the corporation and in declaring the dividends on the stock
is a corporation, partnership, or other juridical person, the president, the of BP 33, as amended. was all done at the same meeting of the board of directors, and it appears
general manager, managing partner, or such other officer charged with the in those minutes that the both Ganzon and Mendaros were formerly
management of the business affairs thereof, or employee responsible for directors and resigned before the board approved the purchase and
the violation shall be criminally liable; in case the offender is an alien, he declared the dividends, and that out of the whole 330 shares purchased,
shall be subject to deportation after serving the sentence. Ganzon, sold 100 and Mendaros 200, or a total of 300 shares out of the
330, which were purchased by the corporation, and for which it paid
Petitioners argue that they cannot be held liable for any P3,300. In other words, that the directors were permitted to resign so that
perceived violations of BP 33, as amended, since they are mere directors they could sell their stock to the corporation. As stated, the authorized
58. C.H. STEINBERG v. GREGORIO VELASCO
of Omni who are not in charge of the management of its business affairs. capital stock was P20,000 divided into 2,000 shares of the par value of P10
G.R. No. L-30460; March 12, 1929
Reasoning that criminal liability is personal, liability attaches to a person each, which only P10,030 was subscribed and paid. Deducting the P3,300
from his personal act or omission but not from the criminal act or paid for the purchase of the stock, there would be left P7,000 of paid up
FACTS: It was alleged that the defendants, Gregorio Velasco, as
stock, from which deduct P3,000 paid in dividends, there would be left
negligence of another. Since Sec. 4 of BP 33, as amended, clearly president, Felix del Castillo, as vice-president, Andres L. Navallo, as
P4,000 only. In this situation and upon this state of facts, it is very apparent
provides and enumerates who are criminally liable, which do not include secretary-treasurer, and Rufino Manuel, as director of Trading Company,
that the directors did not act in good faith or that they were grossly ignorant
members of the board of directors of a corporation, petitioners, as mere at a meeting of the board of directors held on July 24, 1922, approved and
of their duties.
members of the board of directors who are not in charge of Omni’s authorized various unlawful purchases already made of a large portion of
business affairs, maintain that they cannot be held liable for any perceived the capital stock of the company from its various stockholders, thereby
Upon each of those points, the rule is well stated in Ruling Case Law, vol.
violations of BP 33, as amended. diverting its funds to the injury, damage, and in fraud of the creditors of the
7, p. 473, section 454 where it is said:
corporation. It was also alleged that the total amount of the capital stock
Issue: unlawfully purchased was P3,300, and that at the time of such purchase,

48
Derivative Suit
General Duty to Exercise Reasonable Care. — The directors The other eight petitioners are holders of Class B shares. The petitioners
of a corporation are bound to care for its property and manage are part of a group who invested in the purchase of ultrasound equipment, Held: Yes.
its affairs in good faith, and for a violation of these duties resulting the operation of and earnings from which gave rise to the instant
in waste of its assets or injury to the property they are liable to controversy. In the case at bar, the majority of the number of directors, if it is indeed
account the same as other trustees. There can be no doubt thirteen (13), is seven (7), while if it is eleven (11), the majority is six (6).
that if they do acts clearly beyond their power, whereby The laboratory, physical therapy, pulmonary and ultrasound services in During the meetings held by the MCPI Board of Directors i.e. 1) 14 August
loss ensues to the corporation, or dispose of its property or MCP were provided to patients by way of concessions granted to 1998 meeting x x x, twelve (12) directors were present, and of said number,
pay away its money without authority, they will be required independent entities. When the concessions expired, MCPI decided that it seven (7) of them belong to the ultrasound investors x x x, and at which
to make good the loss out of their private estates. This is the would provide on its own the said services, except ultrasound. MCPI’s meeting, the Board decided to make a counter-offer x x x to the ultrasound
rule where the disposition made of money or property of the Board of Directors awarded the operation of the ultrasound unit to a group group and; 2) 05 February 1999 meeting x x x, twelve (12) directors were
corporation is one either not within the lawful power of the of investors (ultrasound investors) composed mostly of Obstetrics- present, and of said number, eight (8) of them belong to the ultrasound
corporation, or, if within the authority of the particular officer or Gynecology (Ob-gyne) doctors. The ultrasound investors held either Class investors x x x, and at which meeting, the Board decided to proceed with
officers. A or Class B shares of MCPI. Among them were nine of the herein the signing of the [MOA] x x x. As can be gleaned from the Minutes of said
petitioners, who were then, likewise, MCPI Board Directors. The group Board meetings, without the presence of the [petitioners]
And section 458 which says: purchased an ultrasound equipment costing P850,000.00 and operated directors/ultrasound investors, there can be no quorum.
the same. Albeit awarded by the Board of Directors, the operation was not
Want of Knowledge, Skill, or Competency. — It has been yet covered by a written contract. “Quorum” is defined as that number of members of a body which, when
said that directors are not liable for losses resulting to the legally assembled in their proper places, will enable the body to transact its
corporation from want of knowledge on their part; or for mistake In the meeting of the MCPI’s Board of Directors, seven (7) of the twelve proper business. “Majority,” when required to constitute a quorum, means
of judgment, provided they were honest, and provided they are (12) Directors present were part of the ultrasound investors. The Board the greater number than half or more than half of any total.
fairly within the scope of the powers and discretion confided to Directors made a counter offer anent the operation of the ultrasound unit.
the managing body. But the acceptance of the office of a Hence, essentially then, the award of the ultrasound operation still bore no
director of a corporation implies a competent knowledge of formal stamp of approval. It is clear that under the “business judgment rule”, the courts are barred
the duties assumed, and directors cannot excuse from intruding into the business judgments of the corporation, when the
imprudence on the ground of their ignorance or A Memorandum of Agreement (MOA) was entered into by and between same are made in good faith. All corporate powers and prerogatives are
inexperience; and if they commit an error of judgment MCPInand the ultrasound investors. Per MOA, the gross income to be vested directly in the BoD.
through mere recklessness or want of ordinary prudence derived from the operation of the ultrasound unit, minus the sonologists’
or skill, they may be held liable for the consequences. Like professional fees, shall be divided between the ultrasound investors and
a mandatory, to whom he has been likened, a director is bound MCPI, in the proportion of 60% and 40%, respectively. Come April 1, 1999, 60. PHILIPPINE STOCK EXCHANGE Inc. vs COURT OF APPEALS
not only to exercise proper care and diligence, but ordinary skill MCPI’s share would be 45%, while the ultrasound investors would receive 281 SCRA 232 [GR No. 125469 October 27, 1997]
and judgment. As he is bound to exercise ordinary skill and 55%. Further, the ownership of the ultrasound machine would eventually
judgment, he cannot set up that he did not possess them. be transferred to MCPI.

Creditors of a corporation have the right to assume that so long as there On October 6, 1999, Flores wrote MCPI’s counsel a letter challenging the FACTS
are outstanding debts and liabilities, the board of directors will not use the Board of Directors’ approval of the MOA for being prejudicial to MCPI’s
assets of the corporation to purchase its own stock, and that it will not interest. Thereafter, on February 7, 2000, Flores manifested to MCPI’s The Puerto Azul Land Inc. (PALI), a domestic real estate corporation, had
declare dividends to stockholders when the corporation is insolvent. Board of Directors and President his view regarding the illegality of the sought to offer its shares to the public in order to raise funds allegedly to
MOA, which, therefore, cannot be validly ratified. develop its properties and pay its loans with several banking institutions. In
January, 1995, PALI was issued a permit to sell its shares to the public by
RTC and CA: MCPI had, in effect, impliedly ratified the MOA by accepting the Securities and Exchange Commission (SEC). To facilitate the trading
59. BALINGHASAY v. CASTILLO or retaining benefits flowing therefrom. Moreover, the elected MCPI’s of its shares among investors, PALI sought to course the trading of its
8 April 2015 Board Directors for the years 1998 to 2000 did not institute legal actions shares through the Philippine Stock Exchange Inc. (PSEi), for which
against the petitioners. MCPI slept on its rights for almost four years, and purpose it filed with the said stock exchange an application to list its shares,
Facts: estoppel had already set in before the derivative suit was filed in 2001. The with supporting documents attached pending the approval of the PALI’s
MCPI, a domestic corporation organized in 1977, operates the Medical RTC likewise stressed that the sharing agreement, per MOA provisions, listing application, a letter was received by PSE from the heirs of Ferdinand
Center Parañaque (MCP). Castillo, Oscar, Flores, Navarro, and Templo was fair, just and reasonable.
Marcos to which the latter claims to be the legal and beneficial owner of
are minority stockholders of MCPI. Each of them holds 25 Class B shares. some of the properties forming part of PALI’s assets. As a result, PSE
On the other hand, nine of the herein petitioners, namely, Balinghasay,
Issue: Was there an error of law in not applying the “business judgment denied PALI’s application which caused the latter to file a complaint before
Bernabe, Alodia, Jimenez, Oblepias, Savet, Villamora, Valdez and
Villareal, are holders of Class A shares and were Board Directors of MCPI. rule”? the SEC. The SEC issued an order to PSE to grant listing application of
49
Derivative Suit
PALI on the ground that PALI have certificate of title over its assets and encouraged and protected and their activities for the promotion of the Pre-Subscription Agreement they entered into, the Ongs and the
properties and that PALI have complied with all the requirements to enlist economic development. Tius agreed to maintain equal shareholdings in FLADC:
with PSE.
a. the Ongs were to subscribe to 1,000,000 million shares at a par
value of P100.00 each while the Tius were to subscribe to an
A corporation is but an association of individuals, allowed to transact under
additional 549,800 shares at P100.00 each in addition to their
ISSUE an assumed corporate name, and with a distinct legal personality. In
already existing subscription of 450,200 shares. (total of ongs
organizing itself as a collective body, it waives no constitutional immunities
Whether or not the denial of PALI’s application is proper and requisites appropriate to such a body as to its corporate and share = 1Million)
management decisions, therefore, the state will generally not interfere with b. Tius were entitled to nominate the Vice-President and the
the same. Questions of policy and management are left to the honest Treasurer plus five directors while the Ongs were entitled to
RULING decision of the officers and directors of a corporation, and the courts are nominate the President, the Secretary and six directors
without authority to substitute their judgements for the judgement of the (including the chairman) to the board of directors of FLADC.
YES. board of directors. The board is the business manager of the corporation Moreover, the Ongs were given the right to manage and operate
and so long as it acts in good faith, its orders are not reviewable by the the mall.
This is in accord with the “Business Judgement Rule” whereby the SEC
courts.
and the courts are barred from intruding into business judgements of
Accordingly, the Ongs paid P100 million in cash for their subscription
corporations, when the same are made in good faith. The same rule to 1,000,000 shares of stock while the Tius committed to contribute to
precludes the reversal of the decision of the PSE, to which PALI had FLADC a four-storey building and two parcels of land respectively valued
previously agreed to comply, the PSE retains the discretion to accept of In matters of application for listing in the market the SEC may exercise such at P20 million (for 200,000 shares), P30 million (for 300,000 shares)
reject applications for listing. Thus, even if an issuer has complied with the power only if the PSE’s judgement is attended by bad faith. and P49.8 million (for 49,800 shares) to cover their additional 549,800
PSE listing rules and requirements, PSE retains the discretion to accept or stock subscription. The Ongs paid in another P70 million to FLADC
reject the issuer’s listing application if the PSE determines that the listing and P20 million to the Tius over and above their P100 million investment,
shall not serve the interests of the investing public. The petitioner was in the right when it refused application of PALI, for a the total sum of which (P190 million) was used to settle the P190 million
mortgage indebtedness of FLADC to PNB.
contrary ruling was not to the best interest of the general public.
The business harmony between the Ongs and the Tius in FLADC,
It is undeniable that the petitioner PSE is not an ordinary corporation, in that however, was shortlived because the Tius rescinded the Pre-Subscription
although it is clothed with the markings of a corporate entity, it functions as Agreement, for the following reasons, that the Ongs:
61. ONG YONG, et al. v. DAVID S. TIU, et al.
the primary channel through which the vessels of capital trade ply. The
G.R. No. 144476. April 8, 2003 (CORONA, J.:)
PSEi’s relevance to the continued operation and filtration of the securities a. Refused to credit to them the FLADC shares covering their real
transaction in the country gives it a distinct color of importance such that DOCTRINE: property contributions.
government intervention in its affairs becomes justified, if not necessarily. “ (C)ontracts intra vires entered into by the board of directors are b. Prevented David S. Tiu and Cely Y. Tiu from assuming the
Indeed, as the only operational stock exchange in the country today, the binding upon the corporation and courts will not interfere unless such positions of and performing their duties as Vice-President and
PSE enjoys monopoly of securities transactions, and as such it yields a contracts are so unconscionable and oppressive as to amount to wanton Treasurer, respectively, and
monopoly of securities transactions, and as such, it yields an immerse destruction to the rights of the minority, as when plaintiffs aver that the c. Refused to give them the office spaces agreed upon.
influence upon the country’s economy. defendants (members of the board), have concluded a transaction among
themselves as will result in serious injury to the plaintiffs stockholders.” Ongs contended that:
a. David S. Tiu and Cely Y. Tiu had in fact assumed the positions
FACTS:
The SEC’s power to look into the subject ruling of the PSE, therefore, may of Vice-President and Treasurer of FLADC but that it was they
In 1994, the construction of the Masagana Citimall in Pasay City
be implied from or be considered as necessary or incidental to the carrying was threatened with stoppage and incompletion when its owner, the First who refused to comply with the corporate duties assigned to
out of the SEC’s express power to insure fair dealing in securities traded Landlink Asia Development Corporation (FLADC), which was owned by them. It was the contention of the Ongs that they wanted the Tius
upon a stock exchange or to ensure the fair administration of such the Tius, encountered dire financial difficulties. It was heavily indebted to to sign the checks of the corporation and undertake their
exchange. It is likewise, observed that the principal function of the SEC is the Philippine National Bank (PNB) for P190 million. management duties but that the Tius shied away from helping
the supervision and control over corporations, partnerships and them manage the corporation.
associations with the end in view that investment in these entities may be To stave off foreclosure of the mortgage on the two lots where the
b. On the issue of office space, the Ongs pointed out that the Tius
mall was being built, the Tius invited the Ongs to invest in FLADC. Under
did in fact already have existing executive offices in the mall since
50
Derivative Suit
they owned it 100% before the Ongs came in. What the Tius capital stock pursuant to Section 38 of the Corporation Code. Section a transaction among themselves as will result in serious injury to the
really wanted were new offices which were anyway 122 of the law provides that (e)xcept by decrease of capital stock, no plaintiffs stockholders.”
subsequently provided to them. corporation shall distribute any of its assets or property except upon lawful
dissolution and after payment of all its debts and liabilities. The Tius claim Reason behind the rule:
c. On the most important issue of their alleged failure to credit the
that their case for rescission, being a petition to decrease capital stock,
Tius with the FLADC shares commensurate to the Tius property does not violate the liquidation procedures under our laws. All that needs Courts and other tribunals are wont to override the business
contributions, the Ongs asserted that, although the Tius to be done, according to them, is for this Court to order (1) FLADC to judgment of the board mainly because, courts are not in the business of
executed a deed of assignment for the 1,902.30 square-meter file with the SEC a petition to issue a certificate of decrease of capital business, and the laissez faire rule or the free enterprise system prevailing
lot in favor of FLADC, they (the Tius) refused to pay P 570,690 stock and (2) the SEC to approve said decrease. in our social and economic set-up dictates that it is better for the State and
for capital gains tax and documentary stamp tax. Without the its organs to leave business to the businessmen; especially so, when
payment thereof, the SEC would not approve the valuation of the ISSUE; courts are ill-equipped to make business decisions. More importantly, the
Can the Tius legally rescind the Pre-Subscription Agreement? social contract in the corporate family to decide the course of the corporate
Tius property contribution (as opposed to cash contribution).
business has been vested in the board and not with courts.
This, in turn, would make it impossible to secure a new Transfer RULING:
Certificate of Title (TCT) over the property in FLADCs name. In It is clear that, when the finances of FLADC improved
any event, it was easy for the Tius to simply pay the said transfer No. the argument of the Tius has no merit. considerably after the equity infusion of the Ongs, the Tius started planning
taxes and, after the new TCT was issued in FLADCs name, they to take over the corporation again and exclude the Ongs from it. It appears
could then be given the corresponding shares of stocks. The Tius case for rescission cannot validly be deemed a petition that the Tius refusal to pay transfer taxes might not have really been at all
d. On the 151 square-meter property, the Tius never executed a to decrease capital stock because such action never complied with the unintentional because, by failing to pay that relatively small amount which
formal requirements for decrease of capital stock under Section 33 of the they could easily afford, the Tius should have expected that they were not
deed of assignment in favor of FLADC. The Tius initially claimed
Corporation Code. No majority vote of the board of directors was ever going to be given the corresponding shares. It was the perfect excuse for
that they could not as yet surrender the TCT because it was still taken. Neither was there any stockholders meeting at which the approval blackballing the Ongs. In other words, the Tius created a problem, then
being reconstituted by the Lichaucos from whom the Tius bought of stockholders owning at least two-thirds of the outstanding capital stock used that same problem as their pretext for showing their partners the
it. The Ongs later on discovered that FLADC had in reality was secured. There was no revised treasurers affidavit and no proof that door. In the process, they stood to be rewarded with a bonanza of
owned the property all along, even before their Pre-Subscription said decrease will not prejudice the creditors rights. On the contrary, all their anywhere between P450 million to P900 million in assets (from an
Agreement was executed in 1994. This meant that the 151 pleadings contained were alleged acts of violations by the Ongs to justify investment of only P45 million which was nearly foreclosed by PNB), to the
an order of rescission. extreme and irreparable damage of the Ongs, FLADC and its creditors.
square-meter property was at that time already the corporate
property of FLADC for which the Tius were not entitled to the Business Judgment rule discussion: After all is said and done, no one can close his eyes to the fact
issuance of new shares of stock. that the Masagana Citimall would not be what it has become today were it
Furthermore, it is an improper judicial intrusion into the not for the timely infusion of P190 million by the Ongs. ithout the Ongs, the
Tius at the Securities and Exchange Commission (SEC), seek internal affairs of the corporation to compel FLADC to file at the SEC Tius would have lost everything they originally invested in said mall.
confirmation of their rescission of the Pre-Subscription Agreement. After a petition for the issuance of a certificate of decrease of stock.
hearing, the SEC, issued a decision confirming the rescission sought by Decreasing a corporations authorized capital stock is an amendment of the
the Tius. SEC en banc confirmed the decision. Articles of Incorporation. It is a decision that only the stockholders and the 62. JAMES IENT and MAHARLIKA SCHULZE v TULLETT PREBON
directors can make, considering that they are the contracting parties (PHILIPPINES), INC.,
The Ongs countered that there was no violation of the Pre- thereto. In this case, the Tius are actually not just asking for a review G.R. Nos. 189158 and 189530 January 11, 2017
Subscription Agreement on the part of the Ongs;that, after more than of the legality and fairness of a corporate decision. They want this
seven years since the mall began its operations, rescission had become Court to make a corporate decision for FLADC. We decline to intervene
not only impractical but would also adversely affect the rights of innocent and order corporate structural changes not voluntarily agreed upon by its
parties; and that it would be highly inequitable and unfair to simply return stockholders and directors. Commercial Law; Corporation Code; Liability on Sections 31 and 34; SC
the P100 million investment of the Ongs and give the remaining assets held that through a thorough scrutinizing of the different provisions in the
now amounting to about P1 billion to the Tius. Truth to tell, a judicial order to decrease capital stock without the Corporation Code including Sections 31 and 34, they only impose civil
assent of FLADCs directors and stockholders is a violation of the business liability aside from Section 74. SC concludes that had it been the intention
The Tius maintain that rescinding the subscription contract is not judgment rule which states that – “(C)ontracts intra vires entered into by the of the drafters of the la to define Sections 31 and 34 as offenses, they could
synonymous to corporate liquidation because all rescission will entail board of directors are binding upon the corporation and courts will not have easily included similar language as that found in Section 74. The
would be the simple restoration of the status quo ante and a return to the interfere unless such contracts are so unconscionable and oppressive as intention can also be gleaned from the floor deliberations of its proponents.
two groups of their cash and property contributions. The Tius claim that to amount to wanton destruction to the rights of the minority, as when
rescission of the agreement will not result in an unauthorized liquidation of plaintiffs aver that the defendants (members of the board), have concluded FACTS:
the corporation because their case is actually a petition to decrease
51
Derivative Suit
Tradition Group, where petitioners herein are employed, and Tullett are criminal sanction. The Court also added that there is no provision in the used by him to acquire the block of shares of SMC stock totaling around
competitors in the inter-dealer broking business. On the Tradition Group's Corporation Code using an emphatic language to compel the SC to 16 million at the time of acquisition (representing 20% of the capital stock
motive of expansion and diversification in Asia, petitioners lent and construe the provision as a penal offense. SC held that through a thorough of SMC)
Schulze were tasked with the establishment Tradition Financial Services scrutinizing of the different provisions in the Corporation Code including
Philippines, Inc. Sections 31 and 34, they only impose civil liability aside from Section 74. Cojuangco purchased a block of 33 million shares of SMC stock (CIFF
SC concludes that had it been the intention of the drafters of the la to define block of shares) through the 14 holding companies owned by the CIIF Oil
However, Tullett, filed a Complaint-Affidavit with the City Prosecution Office Mills.
Sections 31 and 34 as offenses, they could have easily included similar
of Makati City against the officers/employees of the Tradition Group for language as that found in Section 74. The intention can also be gleaned
violation of Sections 31 and 34 of the Corporation Code which made them FACTS:
from the floor deliberations of its proponents. Quite apart that no legislative
criminally liable under Section 144. Impleaded as respondents in the intent to criminalize Sections 31 and 34 was manifested in the deliberations Defendant Eduardo Cojuangco, Jr., (Cojuangco) served as a public officer
Complaint-Affidavit were petitioners lent and Schulze, Jaime Villalon , who on the Corporation Code, it is noteworthy from the same deliberations that during the Marcos administration. During his incumbency, he acquired
was formerly President and Managing Director of Tullett, Mercedes legislators intended to codify the common law concepts of corporate assets, funds, and other property grossly and manifestly disproportionate
Chuidian who was formerly a member of Tullett's Board of Directors. opportunity and fiduciary obligations of corporate officers as found in to his salaries, lawful income and income from legitimately acquired
Villalon and Chuidian were charged with using their former positions in American jurisprudence into said provisions. In common law, the remedies property.
Tullett to sabotage said company by orchestrating the mass resignation of available in the event of a breach of director's fiduciary duties to the
its entire brokering staff in order for them to join Tradition Philippines which corporation are civil remedies. If a director or officer is found to have By taking undue advantage of his association, influence and connection,
was evident on their conduct of several meetings with the employees. breached his duty of loyalty, an injunction may be issued or damages may acting with the Marcoses, and other defendants, they used schemes,
be awarded. A corporate officer guilty of fraud or mismanagement may be including the use of defendant corporations as fronts, to unjustly enrich
According to Tullett, petitioners lent and Schulze have conspired with
held liable for lost profits. A disloyal agent may also suffer forfeiture of his themselves at the expense of Plaintiff and the Filipino people, such as
Villalon and Chuidian in the latter's acts of disloyalty against the company.
compensation. There is nothing in the deliberations to indicate that drafters when he misused coconut levy funds to buy out majority of the outstanding
Petitioners argued that there could be no violation of Sections 31 and 34 of
of the Corporation Code intended to deviate from common law practice shares of stocks (OSS) of San Miguel Corporation (SMC) in order to
the Corporation as these sections refer to corporate acts or corporate
and enforce the fiduciary obligations of directors and corporate officers control the largest agri-business, foods and beverage company in the
opportunity, that Section 144 of the same Code cannot be applied to
through penal sanction aside from civil liability. Philippines, to wit:
Sections 31 and 34 which already contains the penalties or remedies for
their violation; and conspiracy under the Revised Penal Code cannot be a. He entered SMC in 1983; he bought 20 million shares Zobel.
applied to the Sections 31 and 34 of the Corporation Code. The city The shares, worth $49 million, represented 20% of SMC;
prosecutor dismissed the criminal complaint however, on respondent’s 63. REPUBLIC v. COJUANGCO b. Cojuangco acquired the Soriano stocks through secret
appeal to DOJ, the dismissal was reversed finding the arguments of the G.R. Nos. 166859, 169203 & 180702 agreements i.e. other persons or his heir would proxy over the
respondent proper. CA affirmed the decision of the DOJ secretary. April 12, 2011 vote of the shares of Soriano and Cojuangco. This led accounted
for 30% of the OSS of SMC;
ISSUE: The individual defendants were Cojuangco, President Marcos and First c. In exchange for an SMC investment of $45 million in non-voting
Lady Imelda R. Marcos, the ACCRA lawyers, and Ursua. preferred shares in UCPB, Soriano served as the vice-chairman
Whether or not Section 144 of the Corporation Code applies to Sections
31 and 34 of the same code, thus, making it a penal offense so that of the bank of the coconut farmers, UCPB, and in return,
The corporate defendants were Southern Luzon Oil Mills, Cagayan de Oro
conspiracy can be appreciated and the petitioners can be impleaded Cojuangco, for investing funds from the coconut levy, was
Oil Company, Incorporated, Iligan Coconut Industries, Incorporated, San
named vice-chairman of SMC;
Pablo Manufacturing Corporation, Granexport Manufacturing Corporation,
RULING: d. Cojuangco appointed his nominees to the SMC Board, to which
Legaspi Oil Company, Incorporated, (collectively referred to herein as the
CIIF Oil Mills), and their 14 holding companies, namely: Soriano Shares, he appointed key members of the ACCRA Law Firm, instead of
NEGATIVE. The Supreme Court applied rule of lenity as a principle related
Incorporated, Roxas Shares, Incorporated, Arc Investments, Incorporated, coconut farmers whose money really funded the sale;
to liberal interpretation in favor of the accused in criminal cases. The rule
Toda Holdings, Incorporated, ASC Investments, Incorporated, Randy e. All together, Cojuangco purchased 33 million shares of the SMC
applies when the court is faced with two possible interpretations of a penal through the 14 holding companies; the 14 holding companies
Allied Ventures, Incorporated, AP Holdings, Incorporated, San Miguel
statute, one that is prejudicial to the accused and another that is favorable Corporation Officers, Incorporated, Te Deum Resources, Incorporated, were owned by the so-called CIIF Companies;
to him. The rule calls for the adoption of an interpretation which is more Anglo Ventures, Incorporated, Rock Steel Resources, Incorporated, f. Defendant Corporations are but "shell" corporations owned by
lenient to the accused. Valhalla Properties, Incorporated, and First Meridian Development, interlocking shareholders who have admitted that they are just
Incorporated. "nominee stockholders" who do not have any proprietary interest
According to SC, a close reading Section 144 shows that it is not purely a
penal provision because it provides that when the violator is a corporation, over the shares in their names;
The additional corporate defendants (Cojuangco corporations) were g. Cojuangco acquired a total of 16 million shares of SMC from the
an administrative penalty is imposed in form of dissolution, which is not a several corporations alleged to have been under Cojuangco’s control and
Ayala group. The AOI of these corporations under the Ayala,
52
Derivative Suit
showed that 99% of the OSS is owned by Atty. Concepcion of acquire any personal or pecuniary interest in conflict Based on the foregoing, a debtor can appropriate the thing loaned without
ACCRA Law; with their duty as such directors, or trustees shall be any responsibility or duty to his creditor to return the very thing that was
h. The other respondent Corporations are owned by interlocking liable jointly and severally for all damages resulting loaned or to report how the proceeds were used. Nor can he be compelled
shareholders who are likewise lawyers in the ACCRA and had therefrom suffered by the corporation, its stockholders to return the proceeds and fruits of the loan, for there is nothing under our
admitted their status as "nominee stockholders" only; or members and other persons. laws that compel a debtor in a contract of loan to do so. As owner, the
i. These companies were organized by ACCRA for Cojuangco to debtor can dispose of the thing borrowed and his act will not be considered
be able to control more than 60% of SMC shares, they were When a director, trustee or officer attempts to acquire misappropriation of the thing. The only liability on his part is to pay the loan
funded by institutions which depended upon the coconut levy or acquires, in violation of his duty, any interest together with the interest that is either stipulated or provided under existing
such as the UCPB, UNICOM, United Coconut Planters adverse to the corporation in respect of any matter laws.
Assurance Corp. (COCOLIFE), among others. Cojuangco and which has been reposed in him in confidence, as to
his ACCRA lawyers used the funds to borrow money from the which equity imposes a disability upon him to deal in DISSENTING OPINION OF JUSTICE CARPIO-MORALES
UCPB and purchase these holding companies and the SMC his own behalf, he shall be liable as a trustee for the
corporation and must account for the profits which Self-Dealing Director or Officer
stocks. Cojuangco used $150 million from the coconut levy.
j. With this, SMC began to get favors from the Marcos otherwise would have accrued to the corporation”
government, the lowering of the excise taxes (sales and specific In acquiring the loans for himself while he was an officer of UCPB,
The Sandiganbayan could not fairly find that Cojuangco had committed Cojuangco violated his fiduciary obligation under the Corporation Code
taxes) on beer;
breach of any fiduciary duties as an officer and member of the BOD of the and the prohibition on self-dealing under the banking law.
ISSUE:
UCPB.
Whether or not defendant Cojuangco had indeed served in the governing It having been established that Cojuangco engaged in prohibited conflict-
For one, the Amended Complaint contained no clear factual allegation on
bodies of PC, UCPB and/or CIIF Oil Mills at the time the funds used to of-interest transactions by buying the SMC shares using coco levy funds
which to predicate the application of Articles 1455 and 1456 of the Civil
purchase the SMC shares were obtained such that he owed a fiduciary being administered by the UCPB and CIIF Oil Mills for his own benefit, it
Code, and Section 31 of the Corporation Code. Although the trust follows that a constructive trust was formed in favor of the coconut
duty to render an account to these entities as well as to the coconut
relationship supposedly arose from Cojuangco’s being an officer and farmers who should have benefited from such funds.
farmers.
member of the BOD of the UCPB, the link between this alleged fact and
RULING: the borrowings or advances was not established. Nor was there evidence
The Civil Code provides:
on the loans or borrowings, their amounts, the approving authority, etc. As
Cojuangco violated no fiduciary duties. trial court, the Sandiganbayan could not presume his breach of fiduciary
duties without evidence showing so, for fraud or breach of trust is never “Art. 1455. When any trustee, guardian, or other
Republic invoked Article 1455 and 1456 of the Civil Code and Section 31 presumed, but must be alleged and proved. person holding a fiduciary relationship uses trust funds
of the Corporation Code, to wit: for the purchase of property and causes the
The thrust of the Republic that the funds were borrowed or lent might even conveyance to be made to him or to a third person, a
“Article 1455. When any trustee, guardian or other preclude any consequent trust implication. In a contract of loan, one of the trust is established by operation of law in favor of the
person holding a fiduciary relationship uses trust funds parties delivers money or other consumable thing to another on the person to whom the funds belong. (emphasis and
for the purchase of property and causes the underscoring supplied)”
condition that the same amount of the same kind and quality shall be paid.
conveyance to be made to him or to a third person, a Owing to the consumable nature of the thing loaned, the resulting duty of
trust is established by operation of law in favor of the the borrower in a contract of loan is to pay, not to return, to the creditor or A constructive trust is "a right of property, real or personal, held by one party
person to whom the funds belong. lender the very thing loaned. This explains why the ownership of the thing for the benefit of another; that there is a fiduciary relation between a trustee
and a cestui que trust as regards certain property, real, personal, money
loaned is transferred to the debtor upon perfection of the contract.
Article 1456. If property is acquired through mistake or or choses in action." That under Article 1455 there must be a breach of
Ownership of the thing loaned having transferred, the debtor enjoys all the
fraud, the person obtaining it is by force of law, fiduciary relation and profit or gain resulting therefrom in order for a
rights conferred to an owner of property, including the right to use and enjoy constructive trust to be created in favor of that legally entitled to it/
considered a trustee of an implied trust for the benefit
(jus utendi), to consume the thing by its use (jus abutendi), and to dispose
of the person from whom the property comes.”
(jus disponendi), subject to such limitations as may be provided by law.
Fraud in this kind of trust in fact need not even be present. The landmark
“Section 31. Liability of directors, trustees or officers.— Evidently, the resulting relationship between a creditor and debtor in a
case of Severino v. Severino enlightens:
Directors or trustees who willfully and knowingly vote contract of loan cannot be characterized as fiduciary.
for or assent to patently unlawful acts of the
Absent any special facts and circumstances proving a higher degree of “A receiver, trustee, attorney, agent, or any other person
corporation or who are guilty of gross negligence or
responsibility, any dealings between a lender and borrower are not occupying fiduciary relations respecting property or persons, is
bad faith in directing the affairs of the corporation or utterly disabled from acquiring for his own benefit the property
fiduciary in nature.
53
Derivative Suit
committed to his custody for management. This rule is the burden of proving that the transaction was fair and Hence, the assets were transferred to respondent McCullough. However,
entirely independent of the fact whether any fraud has just on the person acquiring the benefit. (emphasis the latter company failed and incurred losses.
intervened. No fraud in fact need be shown, and no excuse will and underscoring supplied)”
be heard from the trustee. It is to avoid the necessity of any such Mead learned of this and so he opposed it by filing a complaint before the
inquiry that the rule takes so general a form. The rule stands on Since Cojuangco was a fiduciary, the burden of evidence on the fairness lower court because the personal properties he contributed were also
the moral obligation to refrain from placing one's self in positions of the questionable transactions was shifted to him. He failed to discharge transferred to respondent McCullough. Primarily, he questioned the power
which ordinarily excite conflicts between self-interest and this burden. of the stockholders to transfer corporate property to one of the members of
integrity. It seeks to remove the temptation that might arise out of the corporation considering that his consent was not obtained to allow such
such a relation to serve one's self-interest at the expense of one's transfer.
integrity and duty to another, by making it impossible to profit by In other words, it was not incumbent upon the Republic to adduce evidence
yielding to temptation. It applies universally to all who come on the particular details of the loans and credit advances for it was ISSUE:
within its principle. (emphasis and underscoring supplied)” Cojuangco’s burden to establish the regularity of these transactions. I am
not "second-guessing," as the majority points out, for I am justified to deem 1. Do majority of the stockholders, who were at the same time
the irregularity or illegality thereof as established after Cojuangco refused majority of the directors of the corporation, have the power under
In the present case, whether Cojuangco committed fraud is no longer to discharge his burden. The intentional concealment of facts as to render
material, what is material and must be established being the existence of the law and its Articles, to sell or transfer to one of its members
secretive the assailed loan transactions entered into by a fiduciary must be,
the fiduciary relation and the use of such position and the attendant abuse the assets of said corporation?
as enunciated by the above-cited rule, taken against Cojuangco, he being
of the confidence reposed in him by virtue of that position which results in the fiduciary. 2. May officer or directors of the corporation purchase the corporate
the constructive trust. property?

Even assuming arguendo that fraud is material, the rule on the burden of RULING:
proof of fraud, as the majority insists, does not apply in the present case.
64. MEAD v. E. C. MCCULLOUGH, et al., and THE PHILIPPINE 1. Yes.
Authorities on evidence cite the existence of a fiduciary relation as an
ENGINEERING AND CONSTRUCTION COMPANY
exception:
G.R. No. 6217 To determine this question, it is necessary to examine, first, the provisions
December 26, 1911 of the Civil Code, and second, those provisions (art. 151 to 174) of the
“However, when a fiduciary relation exists Code of Commerce.
between the parties to a transaction, the burden of FACTS:
proof of its fairness is upon the fiduciary. He must
Articles 1700 to 1708 of the Civil Code deal with the manner of dissolving
show that there was no abuse of confidence, that Petitioner Charles Mead, respondent Edwin McCullough and three others a corporation. There is nothing in these articles which expressly or impliedly
he has acted in good faith, and the act by which organized the corporation called The Philippine Engineering and prohibits the sale of corporate property to one of its members, nor a
he is benefited was the free, voluntary, and Construction Company (PECC). It was engaged in the general dissolution of a corporation in this manner. Neither is there anything in
independent act of the other party, done with full engineering and construction work, primarily in the construction of articles 151 to 174 of the Code of Commerce which prohibits the
knowledge of its purpose and effect. Examples of warehouses and wharf for the US and attempted to rise the sunken dissolution of a corporation by such sale or transfer.
such relationships may be seen in the case of directors Spanish Fleet. The 4 organizers, except petitioner Mead, contributed to the
of a corporation and the corporation, or any other majority of the capital stock of PECC, the remaining shares were offered
relationship of an intimate and fiduciary character. A to the public. Petitioner Mead contributed some personal properties and Article XIII of the corporation's statutes expressly provides that "in all the
fiduciary seeking to profit by a transaction with the one was also assigned as a manager but he resigned when he accepted an meetings of the stockholders, a majority vote of the stockholders present
who confided in him has the burden of showing that engineering job in China. Despite such acceptance, he remained as one shall be necessary to determine any question discussed." The sale or
he communicated to the other not only the fact of his of the five directors of the PECC. transfer to one of its members was a matter which a majority of the
interest in the transaction, but all information he had stockholders could very properly consider. But it is said that if the acts and
which it was important for the other to know in order to At that time, the PECC was already incurring losses. Respondent resolutions of a majority of the stockholders in a corporation are binding in
enable him to judge the value of the property. The McCullough, the president, proposed that he will buy the assets every case upon the minority, the minority would be completely wiped out
formal creation of a fiduciary relationship is not of the corporation, assume all of its obligations and organize another and their rights would be wholly at the mercy of the abuses of the majority.
essential to the application of this rule. The principles association named as Manila Salvage Company. The reason why The resolutions passed within certain limitations by a majority of the
apply to all cases in which confidence is reposed by he organized such association was that PECCs creditors agreed stockholders of a corporation are binding upon the minority.
one party in another, and the trust or confidence is that if McCullough would organize a new association, they would give the
accepted under circumstances which show that it was new association an extension of time to comply with the contract and would Thus, it is well settled, first, that a private corporation, which owes no
founded on intimate, personal, and business relations reconsider the question of forfeiture of the money deposited by PECC with special duty to the public and which has not been given the right of eminent
existing between the parties, which give the one a bank. The three other directors then voted in favor of this proposal. domain, has the absolute right and power as against the whole world
advantage or superiority over the other, and impose except the state, to sell and dispose of all of its property; second, that the
54
Derivative Suit
board of directors, has the power, without reference to the assent or c. Te will pay Prime Php 9.70 per bag of white cement, FOB Under the Corporation Law, all corporate powers shall be exercise by the
authority of the stockholders, when the corporation is in failing Davaor and Cagayan De Oro; and BOD except as otherwise provided by law. However, the BOD may
circumstances or insolvent or when it can no longer continue the business d. Te shall, every time Prime is ready to deliver the goods, open expressly designate specific powers to its President or any of its officers. In
with profit, and when it is regarded as an imperative necessity; third, that a with any bank a letter of credit (LOC) in favor of Prime. the absence of such express delegation, a contract entered into by its
majority of the stockholders or directors, even against the protest of the President, on behalf of the corporation, may still bind the corporation if the
minority, have this power where, from any cause, the business is a failure Right after the execution of the agreement, Te place and ad in a national BOD should ratify the same expressly or impliedly. Furthermore, even in
and the best interest of the corporation and all the stockholders require it. newspaper the fact of being the exclusive dealer of Prime’s goods in the absence of ratification, the President, as a general rule, may bind the
Mindanao area. Thus, he was asked by some of his businessmen friends corporation by a contract in the ordinary course of business, provided that
2. It depends. if they can be his sub-dealer. it is reasonable.

As such, Te entered into agreements with several hardware stores dealing These rules apply where the President or other officer, purportedly acting
While a corporation remains solvent, we can see no reason why a director in buying and selling of cement which would enable him to sell his for the corporation, is dealing with a third person i.e. a person outside the
or officer, by the authority of a majority of the stockholders or board of allocation of 20,000 bags of cement. After execution of such agreement, corporation.
managers, may not deal with the corporation, loan it money or buy property he informed Prime that he is ready to open LOCs pursuant to the
from it, in like manner as a stranger. So long as a purely private corporation dealership agreement they formed. The situation is different where a director or officer is dealing with his own
remains solvent, its directors are agents or trustees for the stockholders. corporation.
They owe no duties or obligations to others. However, Prime decided thru its BOD to impose the following conditions:
Here, Te was not an ordinary stockholder; he was a member of the BOD
But the moment such a corporation becomes insolvent, its directors are a. Delivery shall commence on November 1970; and Auditor of Prime. He is a “self-dealing” director (SDD).
trustees of all the creditors, whether they are members of the corporation b. Only 8,000 bags per month for only a period of 3 months will be
or not, and must manage its property and assets with strict regard to their delivered; A director of a corporation holds a position of trust, he owes a duty of loyalty
interest; and if they are themselves creditors while the insolvent corporation c. The price per bag is priced at Php 13.30; to his corporation. In case of conflict of interest, he cannot sacrifice his duty
is under their management, they will not be permitted to secure to d. The price is subject to readjustment unilaterally on the part of to his own advantage and benefit. This trust relationship is “not a matter of
themselves by purchasing the corporate property or otherwise any Prime; statutory or technical law. It springs from the act that directors have the
personal advantage over the other creditors. Nevertheless, a director or e. The place of delivery shall be Austurias; control and guidance of corporate affairs and property and hence of the
officer may in good faith and for an adequate consideration purchase from f. The LOC may be opened only with the Prudential Bank, Makati property interest of the stockholders.”
a majority of the directors or stockholders the property even of an insolvent branch;
corporation, and a sale thus made to him is valid and binding upon the g. Payment of cement shall be made in advance and it shall be A director’s contract with his own corporation is not in all instances void or
minority. used by Prime as guaranty in the opening of a foreign LOC to voidable. If the contract is fair and reasonable, it may be ratified by the
cover costs and expenses in the procurement of materials in the stockholders. Section 32 of the Corporation Code provides:
manufacture of white cement.
“Sec. 32. Dealings of directors, trustees or officers
Several demands to comply with the dealership agreement were made by with the corporation.- A contract of the corporation
65. PRIME WHITE CEMENT v. IAC and TE Te to Prime. Prime refused which forced Te to cancel his agreement with with one or more of its directors or trustees or officers
G.R. No. L-68555 third parties. More so, Prime, in violation of the exclusive dealership is voidable, at the option of such corporation, unless all
19 March 1993 agreement with Te, entered into an exclusive dealership agreement with a the following conditions are present:
certain Napoleon Co for the marketing of the white cement in Mindanao.
FACTS: 1. That the presence of such director or trustee in
Te filed a suit. the board meeting in which the contract was
In 1969, Prime White Cement Corporation (Prime), thru its President, approved was not necessary to constitute a
Zosimo Falcon (Falcon) and its Chairman, Justo Trazo (Trazo), entered ISSUE: quorum for such meeting;
into a dealership agreement where Alejandro Te (Te) was obligated to act 2. That the vote of such director or trustee was nor
as the executive dealer and/or distributor of Prime of its cement products Is the dealership agreement referred by the President and Chairman of the necessary for the approval of the contract;
in the entire Mindanao area for a term of five (5) years with the following BOD of Prime a valid and enforceable contract? 3. That the contract is fair and reasonable under the
agreements: circumstances; and
RULING: 4. That in case of an officer, the contract has been
a. The delivery shall commence on September 1970; previously authorized by the board of directors.
b. Prime will sell and supply Te with 20,000 bags of white cement No.
per month; Where any of the first two conditions set forth in the
preceding paragraph is absent, in the case of a
55
Derivative Suit
contract with a director or trustee, such contract may (hereafter referred to as SAN JOSE PETROLEUM), a corporation business in the Philippines. Registrant also denied that the offering for sale
be ratified by the vote of the stockholders representing organized and existing in the Republic of Panama. in the Philippines of its shares of capital stock was fraudulent or would work
at least two-thirds (2/3) of the outstanding capital stock or tend to work fraud on the investors. On August 29, 1958, and on
or of at least two-thirds (2/3) of the members in a FACTS: September 9, 1958 the Securities and Exchange Commissioner issued
meeting called for the purpose: Provided, That full the orders object of the present appeal.
disclosure of the adverse interest of the directors or
trustees involved is made at such meeting: Provided, On September 7, 1956, SAN JOSE PETROLEUM filed with the Philippine
Securities and Exchange Commission a sworn registration statement, for ISSUE:
however, That the contract is fair and reasonable
under the circumstances.” the registration and licensing for sale in the Philippines Voting Trust
Certificates representing 2,000,000 shares of its capital stock of a par value Should the registration of the sale be allowed?
Even if the dealership agreement was entered into with a third person of $0.35 a share, at P1.00 per share. It was alleged that the entire proceeds
would be valid, the fact that the other person to the contract was a Director of the sale of said securities will be devoted or used exclusively to finance RULING:
and Auditor of Prime changes the whole situation. the operations of San Jose Oil Company, Inc. (a domestic mining
corporation hereafter to be referred to as SAN JOSE OIL) which has 14
petroleum exploration concessions covering an area of a little less than NO.
First, the contract is not fair nor reasonable. The agreement was contracted
in 1969. Te is a businessman, he must have known that the prices of 1,000,000 hectares. It was the express condition of the sale that every
commodities at that time were not stable and were expected to rise. Prime purchaser of the securities shall not receive a stock certificate, but a The relationship of these corporations involved or affected in this case is
was not even, at that time, starting at the manufacturing of the cement. In registered or bearer-voting-trust certificate from the voting trustees named admitted and established through the papers and documents which are
fact, it shows in Te’s Memorandum, that the price per bag in September therein James L. Buckley and Austin G.E. Taylor, the first residing in parts of the records: SAN JOSE OIL, is a domestic mining corporation,
1970 was Php 14.50, and in 1975, it was already Php 37.50. Despite this, Connecticut, U.S.A., and the second in New York City. While this 90% of the outstanding capital stock of which is owned by respondent SAN
there was no provision for an increase in the price mutually acceptable to application for registration was pending consideration by the Securities and JOSE PETROLEUM, a foreign (Panamanian) corporation, the majority
the parties. Instead, it was pegged at Php 9.70 from 1970 to 1975. Exchange Commission, SAN JOSE PETROLEUM filed an amended interest of which is owned by OIL INVESTMENTS, Inc., another foreign
Moreover, when he entered into sub-dealership agreements, he stipulated Statement on June 20, 1958, for registration of the sale in the Philippines (Panamanian) company. This latter corporation in turn is wholly (100%)
that “the price shall be mutually determined by us in no less than Php 14.00 of its shares of capital stock, which was increased from 2,000,000 to owned by PANTEPEC OIL COMPANY, C.A., and PANCOASTAL
per bag”. 5,000,000, at a reduced offering price of from P1.00 to P0.70 per share. At PETROLEUM COMPANY, C.A., both organized and existing under the
this time the par value of the shares has also been reduced from $.35 to laws of Venezuela. As of September 30, 1956, there were 9,976
Meaning, he protected himself from the fluctuation of prices of the certain $.01 per share.1 stockholders of PANCOASTAL PETROLEUM found in 49 American
commodities. However, why didn’t he protect the corporation in the same states and U.S. territories, holding 3,476,988 shares of stock; whereas, as
manner? As member of the BOD, Te’s bounden duty to act in such manner PALTING: Pedro R. Palting and others, allegedly prospective investors in of November 30, 1956, PANTEPEC OIL COMPANY was said to have
as not to unduly prejudice Prime. the shares of SAN JOSE PETROLEUM, filed with the Securities and 3,077,916 shares held by 12,373 stockholders scattered in 49 American
Exchange Commission an opposition to registration and licensing of the state. In the two lists of stockholders, there is no indication of the citizenship
Hence, he is guilty of disloyalty to his corporation; he was attempting to securities on the ground that the tie-up between the issuer, SAN JOSE of these stockholders,7 or of the total number of authorized stocks of each
enrich himself at the expense of the corporation. Moreover, there is no PETROLEUM, a Panamanian corporation and SAN JOSE OIL, a corporation, for the purpose of determining the corresponding percentage
showing that the “dealership agreement” were ratified by the stockholder, domestic corporation, violates the Constitution of the Philippines, the of these listed stockholders in relation to the respective capital stock of said
or that they were fully aware of its provisions. Corporation Law and the Petroleum Act of 1949. corporation.

The contract was therefore not valid. SAN JOSE P.:the registrant SAN JOSE PETROLEUM claimed that it was PETITIONER: Petitioner, as well as the amicus curiae and the Solicitor
a "business enterprise" enjoying parity rights under the Ordinance General8 contend that the relationship between herein respondent SAN
appended to the Constitution, which parity right, with respect to mineral JOSE PETROLEUM and its subsidiary, SAN JOSE OIL, violates the
resources in the Philippines, may be exercised, pursuant to the Laurel- Petroleum Law of 1949, the Philippine Constitution, and Section 13 of the
66. PATLING v. SAN JOSE PETROLEUM Langley Agreement, only through the medium of a corporation organized Corporation Law, which inhibits a mining corporation from acquiring an
under the laws of the Philippines. Thus, registrant which is allegedly interest in another mining corporation. It is respondent's theory, on the
G.R. No. L-14441 December 17, 1966 qualified to exercise rights under the Parity Amendment, had to do so other hand, that far from violating the Constitution; such relationship
through the medium of a domestic corporation, which is the SAN JOSE between the two corporations is in accordance with the Laurel-Langley
OIL. It refused the contention that the Corporation Law was being Agreement which implemented the Ordinance Appended to the
This is a petition for review of the order of the Securities and Exchange Constitution, and that Section 13 of the Corporation Law is not applicable
violated, by alleging that Section 13 thereof applies only to foreign
Commission denying the opposition to, and instead, granting the because respondent is not licensed to do business, as it is not doing
corporations doing business in the Philippines, and registrant was
registration, and licensing the sale in the Philippines, of 5,000,000 shares business, in the Philippines.
not doing business here. The mere fact that it was a holding company of
of the capital stock of the respondent-appellee San Jose Petroleum, Inc.
SAN JOSE OIL and that registrant undertook the financing of and giving
technical assistance to said corporation did not constitute transaction of
56
Derivative Suit
Respondent SAN JOSE PETROLEUM, whose shares of stock were directors of respondent that "should San Jose Oil Company be granted the These provisions are in direct opposition to our corporation law and
allowed registration for sale in the Philippines, was incorporated under the bulk of the concessions applied for upon reasonable terms, that it would corporate practices in this country. These provisions alone would outlaw
laws of Panama in April, 1956 with an authorized capital stock of have a reasonable value of approximately $10,000,000." 10 Then, of this any corporation locally organized or doing business in this jurisdiction.
$500,000.00, American currency, divided into 50,000,000 shares at par amount, the subscription price of $800,000.00 was deducted and called it Consider the unique and unusual provision that no contract or transaction
value of $0.01 per share. By virtue of a 3-party Agreement of June 14, "difference between the (above) valuation and the subscription price for the between the company and any other association or corporation shall be
1956, respondent was supposed to have received from OIL 8,000,000 shares." Of this $800,000.00 subscription price, they deducted affected except in case of fraud, by the fact that any of the directors or
INVESTMENTS 8,000,000 shares of the capital stock of SAN JOSE OIL the sum of $480,297.97 and the difference was placed as the unpaid officers of the company may be interested in or are directors or officers of
(at par value of $0.01 per share), plus a note for $250,000.00 due in 6 portion of the subscription price. In other words, it was made to appear that such other association or corporation; and that none of such contracts or
months, for which respondent issued in favor of OIL INVESTMENTS they paid in $480,297.97 for the 8,000,000 shares of SAN JOSE OIL. This transactions of this company with any person or persons, firms,
16,000,000 shares of its capital stock, at $0.01 per share or with a value of amount ($480,297.97) was supposedly that $250,000.00 paid by OIL associations or corporations shall be affected by the fact that any director
$160,000.00, plus a note for $230,297.97 maturing in 2 years at 6% per INVESMENTS for 7,500,000 shares of SAN JOSE OIL, embodied in the or officer of this company is a party to or has an interest in such contract or
annum interest,9 and the assumption of payment of the unpaid price of June 14 Agreement, and a sum of $230,297.97 the amount expended or transaction or has any connection with such person or persons, firms
7,500,000 (of the 8,000,000 shares of SAN JOSE OIL). advanced by OIL INVESTMENTS to SAN JOSE OIL. And yet, there is still associations or corporations; and that any and all persons who may
an item among respondent's liabilities, for $230,297.97 appearing as note become directors or officers of this company are hereby relieved of all
payable to Oil Investments, maturing in two (2) years at six percent (6%) responsibility which they would otherwise incur by reason of any contract
per annum. 11 As far as it appears from the records, for the 16,000,000 entered into which this company either for their own benefit, or for the
shares at $0.35 per share issued to OIL INVESTMENTS, respondent SAN benefit of any person, firm, association or corporation in which they may be
On June 27, 1956, the capitalization of SAN JOSE PETROLEUM was JOSE PETROLEUM received from OIL INVESTMENTS only the note for interested.
increased from $500,000.00 to $17,500,000.00 by increasing the par $250,000.00 plus the 8,000,000 shares of SAN JOSE OIL, with par value
of $0.10 per share or a total of $1,050,000.00 — the only assets of the The impact of these provisions upon the traditional judiciary relationship
value of the same 50,000,000 shares, from $0.01 to $0.35. Without any corporation. In other words, respondent actually lost $4,550,000.00, which between the directors and the stockholders of a corporation is too obvious
additional consideration, the 16,000,000 shares of $0.01 previously issued was received by OIL INVESTMENTS. to escape notice by those who are called upon to protect the interest of
to OIL INVESTMENTS with a total value of $160,000.00 were changed investors. The directors and officers of the company can do anything, short
with 16,000,000 shares of the recapitalized stock at $0.35 per share, or Some of the provisions of the Articles of Incorporation of respondent SAN of actual fraud, with the affairs of the corporation even to benefit themselves
valued at $5,600,000.00. And, to make it appear that cash was received JOSE PETROLEUM are noteworthy; viz: directly or other persons or entities in which they are interested, and with
for these re-issued 16,000,000 shares, the board of directors of respondent immunity because of the advance condonation or relief from responsibility
corporation placed a valuation of $5,900,000.00 on the 8,000,000 shares by reason of such acts. This and the other provision which authorizes the
of SAN JOSE OIL (still having par value of $0.10 per share) which were (1) the directors of the Company need not be shareholders;
election of non-stockholders as directors, completely disassociate the
received from OIL INVESTMENTS as part-consideration for the stockholders from the government and management of the business in
16,000,000 shares at $0.01 per share. (2) that in the meetings of the board of directors, any director may be which they have invested.
represented and may vote through a proxy who also need not be a director
In the Balance Sheet of respondent, dated July 12, 1956, from the or stockholder; and
To cap it all on April 17, 1957, admittedly to assure continuity of the
$5,900,000.00, supposedly the value of the 8,000,000 shares of SAN management and stability of SAN JOSE PETROLEUM, OIL
JOSE OIL, the sum of $5,100,000.00 was deducted, corresponding to the (3) that no contract or transaction between the corporation and any other INVESTMENTS, as holder of the only subscribed stock of the former
alleged difference between the "value" of the said shares and the association or partnership will be affected, except in case of fraud, by the corporation and acting "on behalf of all future holders of voting trust
subscription price thereof which is $800,000.00 (at $0.10 per share). From fact that any of the directors or officers of the corporation is interested in, or certificates," entered into a voting trust agreement12 with James L. Buckley
this $800,000.00, the subscription price of the SAN JOSE OIL shares, the is a director or officer of, such other association or partnership, and that no and Austin E. Taylor, whereby said Trustees were given authority to vote
amount of $319,702.03 was deducted, as allegedly unpaid subscription such contract or transaction of the corporation with any other person or the shares represented by the outstanding trust certificates (including those
price, thereby giving a difference of $480,297.97, which was placed as the persons, firm, association or partnership shall be affected by the fact that that may henceforth be issued) in the following manner:
amount allegedly paid in on the subscription price of the 8,000,000 SAN any director or officer of the corporation is a party to or has an interest in,
JOSE OIL shares. Then, by adding thereto the note receivable from OIL such contract or transaction, or has in anyway connected with such other
INVESTMENTS, for $250,000.00 (part-consideration for the 16,000,000 person or persons, firm, association or partnership; and finally, that all and (a) At all elections of directors, the Trustees will designate a suitable proxy
SAN JOSE PETROLEUM shares), and the sum of $6,516.21, as deferred any of the persons who may become director or officer of the corporation or proxies to vote for the election of directors designated by the Trustees in
expenses, SAN JOSE PETROLEUM appeared to have assets in the sum shall be relieved from all responsibility for which they may otherwise be their own discretion, having in mind the best interests of the holders of the
of $736,814.18. liable by reason of any contract entered into with the corporation, whether voting trust certificates, it being understood that any and all of the Trustees
it be for his benefit or for the benefit of any other person, firm, association shall be eligible for election as directors;
These figures are highly questionable. Take the item $5,900,000.00 the or partnership in which he may be interested.
valuation placed on the 8,000,000 shares of SAN JOSE OIL. There (b) On any proposition for removal of a director, the Trustees shall
appears no basis for such valuation other than belief by the board of designate a suitable proxy or proxies to vote for or against such

57
Derivative Suit
proposition as the Trustees in their own discretion may determine, having Maricalum Mining, and the APT as co-defendants warranting the piercing corporations was only necessary to manage and operate the assets
in mind the best interest of the holders of the voting trust certificates; of the veil of corporate fiction since all them are as one and the same entity. acquired in the foreclosure sale lest they deteriorate from non-use and lose
their value.
(c) With respect to all other matters arising at any meeting of stockholders, Issue: Does Remington has a cause of action against DBP or PNB, nor
the Trustees will instruct such proxy or proxies attending such meetings to against their transferees, Nonoc Mining, Island Cement, Maricalum Mining,
vote the shares of stock held by the Trustees in accordance with the written and the APT? 68. JOHN GOKONGWEI, JR. v. SECURITIES AND EXCHANGE
instructions of each holder of voting trust certificates. (Emphasis supplied.) COMMISSION
Ruling: No. G.R. No. L-45911
April 11, 1979
It was also therein provided that the said Agreement shall be binding upon Where the corporations have directors and officers in common,
the parties thereto, their successors, and upon all holders of voting trust there may be circumstances under which their interest as officers in one Facts:
certificates.And these are the voting trust certificates that are offered to company may disqualify them in equity from representing both Petitioner, as stockholder of respondent San Miguel Corporation
investors as authorized by Security and Exchange Commissioner. It can corporations in transactions between the two. Thus, where one corporation (SMC), filed with the SEC a petition for "declaration of nullity of amended
not be doubted that the sale of respondent's securities would, to say was ‗insolvent and indebted to another, it has been held that the directors by-laws, cancellation of certificate of filing of amended by- laws, injunction
the least, work or tend to work fraud to Philippine investors. of the creditor corporation were disqualified, by reason of self-interest, from and damages with prayer for a preliminary injunction" against the majority
acting as directors of the debtor corporation in the authorization of a of the members of the BOD and SMC. Petitioner alleged that the individual
FOR ALL THE FOREGOING CONSIDERATIONS, the motion of mortgage or deed of trust to the former to secure such indebtedness In the respondents amended the by-laws of the corporation, basing their authority
respondent to dismiss this appeal, is denied and the orders of the same manner that when the corporation is insolvent, its directors who are to do so on a resolution of the stockholders in 1961. It was contended that
Securities and Exchange Commissioner, allowing the registration of its creditors cannot secure to themselves any advantage or preference according to section 22 of the Corporation Law and Article VIII of the by-
Respondent's securities and licensing their sale in the Philippines over other creditors. They cannot thus take advantage of their fiduciary laws of the corporation, the power to amend, modify, repeal or adopt new
are hereby set aside. relation and deal directly with themselves, to the injury of others in equal by-laws may be delegated to the BOD only by the affirmative vote of
right. If they do, equity will set aside the transaction at the suit of creditors stockholders representing not less than 2/3 of the subscribed and paid up
67. DBP v. CA of the corporation or their representatives, without reference to the question capital stock of the corporation, which 2/3 should have been computed on
363 S 307 of any actual fraudulent intent on the part of the directors, for the right of the the basis of the capitalization at the time of the amendment. Since the
creditors does not depend upon fraud in fact, but upon the violation of the amendment was based on the 1961 authorization, petitioner contended
Facts: fiduciary relation to the directors. that the Board acted without authority and in usurpation of the power of the
Marinduque Mining obtained from the PNB various loan stockholders. Petitioner alleged that the authority granted in 1961 had
accommodations. To secure the loans, Marinduque Mining mortgaged all In this case, the court did not find any fraud on the part of already been exercised in 1962 and 1963, after which the authority of the
of MarinduqueMinings real properties, located at Surigao del Norte, Marinduque Mining and its transferees to warrant the piercing of the Board ceased to exist. He also averred that the membership of the BOD
Sipalay, Negros Occidental, and at Antipolo, Rizal, including the corporate veil. It bears stressing that PNB and DBP are mandated to had changed since the authority was given in 1961, there being six (6) new
improvements thereon. Also, Marinduque Mining mortgaged all of foreclose on the mortgage when the past due account had incurred directors. It was claimed that prior to the questioned amendment, petitioner
MarinduqueMinings chattels, as well as assets of whatever kind, nature arrearages of more than 20% of the total outstanding obligation. Thus, had all the qualifications to be a director of respondent corporation, being
and description which Marinduque Mining may subsequently acquire. PNB and DBP did not only have a right, but the duty under said law, to a Substantial stockholder thereof; that as a stockholder, petitioner had
foreclose upon the subject properties. The banks had no choice but to obey acquired rights inherent in stock ownership, such as the rights to vote and
For failure of Marinduque Mining to settle its loan obligations, the statutory command. to be voted upon in the election of directors; and that in amending the by-
PNB and DBP instituted extrajudicial foreclosure proceedings over the laws, respondents purposely provided for petitioner's disqualification and
mortgaged properties. In the public auction sale PNB and DBP emerged The Court of Appeals made reference to two principles in deprived him of his vested right. It was alleged that corporations have no
and were declared the highest bidders over the foreclosed properties. PNB corporation law. The first pertains to transactions between corporations inherent power to disqualify a stockholder from being elected as a director
and DBP thereafter assigned and transferred to Nonoc Mining and with interlocking directors resulting in the prejudice to one of the and, therefore, the amended by-laws are null and void.
Industrial Corporation and Maricalum Mining Corp. all their rights, interest corporations. This rule does not apply in this case, however, since the
and participation over the foreclosed properties of MMIC located at Nonoc corporation allegedly prejudiced (Remington) is a third party, not one of the Respondents contended that the power to "amend, modify,
Island, SurigaodelNorte and Sipalay, Negros Occidental respectively. corporations with interlocking directors (Marinduque Mining and DBP). The repeal or adopt new By-laws" delegated to said Board on March 13, 1961
Then they again assigned, transferred and conveyed to the National second principle invoked by respondent court involves directors who are and long prior thereto has never been revoked by San Miguel Corp. and
Government thru the Asset Privatization Trust (APT) all its existing rights creditors which is also inapplicable herein. Here, the creditor of Marinduque that petitioner has not availed of his intra-corporate remedy for the
and interest over the assets of MMIC, earlier assigned to Nonoc Mining, Mining is DBP, not the directors of Marinduque Mining. Neither does the nullification of the amendment, which is to secure its repeal by vote of the
Maricalum Mining and Island Cement Corporation. In the meantime, court discern any bad faith on the part of DBP by its creation of Nonoc stockholders representing a majority of the subscribed capital stock at any
Marinduque Mining purchased from Remignton construction materials but Mining, Maricalum and Island Cement. regular or special meeting, hence the, petition is premature. It also alleged
were unpaid. Remington filed a complaint for a sum of money and that petitioner is estopped from questioning the amendments on the
damages against Marinduque Mining. Remingtons original complaint was As Remington itself concedes, DBP is not authorized by its ground of lack of authority from the Board since he failed to object to other
amended to include PNB and DBP, Nonoc Mining, Island Cement, charter to engage in the mining business. The creation of the three amendments made on the basis of the same 1961 authorization.
58
Derivative Suit
2) Are the amended by-laws of SMC disqualifying a competitor relation is one of trust. This is based upon the principle that where the
Respondents Jose and Andres Soriano, in their answer, alleged from nomination or election to the Board of Directors of SMC director is so employed in the service of a rival company, he cannot serve
that the Universal Robina Corporation (Robina) and Consolidated Foods valid and reasonable exercise of corporate powers? both, but must betray one or the other. Such an amendment advances the
Corporation (CFC), both closed corporations engaged in business benefit of the corporation and is good. Section 21 of the Corporation Law
competitive to that of respondent corporation, where petitioner is the Held: expressly provides that a corporation may make by-laws for the
president and controlling shareholder, purchased 5,000 shares of stock of qualifications of directors. Thus, it has been held that an officer of a
respondent corporation, and thereafter, in behalf of himself, CFC and YES. corporation cannot engage in a business in direct competition with
Robina, conducted malevolent and malicious publicity campaign against that of the corporation where he is a director by utilizing information
SMC to generate support from the stockholders in his effort to secure for It is recognized by authorities that every corporation has the he has received as such officer, under "the established law that a
himself a seat in the Board of Directors of SMC but petitioner was rejected inherent power to adopt by-laws for its internal government, and to regulate director or officer of a corporation may not enter into a competing
by the stockholders because of the basic issue that petitioner was engaged the conduct and prescribe the rights and duties of its members towards enterprise which cripples or injures the business of the corporation
in a competitive business and his securing a seat would have subjected itself and among themselves in reference to the management of its affairs. of which he is an officer or director.
respondent corporation to grave disadvantages. At common law, the rule was "that the power to make and adopt by-laws
was inherent in every corporation as one of its necessary and inseparable The doctrine of "corporate opportunity" is precisely a
In 1976, petitioner filed with the SEC an "Urgent Motion for legal incidents. recognition by the courts that the fiduciary standards could not be upheld
Production and Inspection of Documents", alleging that the Secretary of where the fiduciary was acting for two entities with competing interests.
respondent corporation refused to allow him to inspect its records despite A stockholder has no vested right to be elected as a This doctrine rests fundamentally on the unfairness, in particular
request made by petitioner for production of certain documents director. Any person who buys stock in a corporation does so with the circumstances, of an officer or director taking advantage of an opportunity
enumerated in the request, and that respondent corporation had been knowledge that its affairs are dominated by a majority of the stockholders for his own personal profit when the interest of the corporation justly calls
attempting to suppress information from its stockholders despite a negative and that he impliedly contracts that the will of the majority shall govern in all for protection. It is obviously to prevent the creation of an opportunity for an
reply by the SEC to its query regarding their authority to do so. matters within the limits of the act of incorporation and lawfully enacted by- officer or director of San Miguel Corporation, who is also the officer or
Respondents opposed the urgent motion due to the fact that it has no legal laws and not forbidden by law. To this extent, therefore, the stockholder owner of a competing corporation, from taking advantage of the
basis and that it fails to show good cause and constitutes continued may be considered to have parted with his personal right or privilege to information which he acquires as director to promote his individual or
harassment, and that some of the information sought are not part of the regulate the disposition of his property which he has invested in the capital corporate interests to the prejudice of San Miguel Corporation and its
records of the corporation and, therefore, privileged. stock of the corporation, and surrendered it to the will of the majority of his stockholders, that the questioned amendment of the by-laws was made.
fellow incorporators. It cannot therefore be justly said that the contract, Certainly, where two corporations are competitive in a substantial sense, it
While the petition was yet to be heard, respondent corporation express or implied, between the corporation and the stockholders is would seem improbable, if not impossible, for the director, if he were to
issued a notice of special stockholders' meeting for the purpose of infringed by any act of the former which is authorized by a majority. discharge effectively his duty, to satisfy his loyalty to both corporations and
"ratification and confirmation of the amendment to the By-laws". After Pursuant to section 18 of the Corporation Law, any corporation may place the performance of his corporation duties above his personal
petitioner’s request for summary judgment on the supposed meeting was amend its articles of incorporation by a vote or written assent of the concerns.
opposed, the same filed an Urgent Motion for the Issuance of a TRO stockholders representing at least two-thirds of the subscribed The offer and assurance of petitioner that to avoid any possibility
restraining respondents from holding the special stockholder's meeting as capital stock of the corporation. If the amendment changes, of his taking unfair advantage of his position as director of San Miguel
scheduled which the SEC denied. Subsequently, the meeting proceeded. diminishes or restricts the rights of the existing shareholders then Corporation, he would absent himself from meetings at which confidential
Petitioner then filed with respondent SEC a Manifestation stating that he the dissenting minority has only one right, viz.: to object thereto in matters would be discussed, would not detract from the validity and
intended to run for the position of director of respondent corporation. writing and demand payment for his share. Under section 22 of the reasonableness of the by-laws here involved. Apart from the impractical
Thereafter, respondents filed a Manifestation with the SEC, submitting a same law, the owners of the majority of the subscribed capital stock may results that would ensue from such arrangement, it would be inconsistent
Resolution of the BOD of SMC disqualifying and precluding petitioner from amend or repeal any by-law or adopt new by-laws. It cannot be said, with petitioner's primary motive in running for board membership — which
being a candidate for director unless he could submit evidence that he therefore, that petitioner has a vested right to be elected director, in the face is to protect his investments in San Miguel Corporation. More important,
does not come within the disqualifications specified in the amendment to of the fact that the law at the time such right as stockholder was acquired such a proposed norm of conduct would be against all accepted principles
the by-laws. contained the prescription that the corporate charter and the by-law shall underlying a director's duty of fidelity to the corporation, for the policy of the
be subject to amendment, alteration and modification. law is to encourage and enforce responsible corporate management.
Petitioner alleges that there appears a deliberate and concerted
inability on the part of the SEC to act which amounted to grave abuse of its YES.
discretion when it failed to act with deliberate dispatch on the motions of 69. STRONG v. REPIDE
petitioner. An amendment to the corporation by-law which renders a 213 U.S. 419 May 3, 1909
stockholder ineligible to be director, if he be also director in a
Issues: corporation whose business is in competition with that of the other FACTS:
1) Does the respondent corporation have the power to provide for corporation, has been sustained as valid. As agents entrusted with the
the (additional) qualifications of its directors? management of the corporation for the collective benefit of the
stockholders, directors occupy a fiduciary relation, and in this sense the
59
Derivative Suit
Mrs. Strong was the owner of 800 shares of the capital stock of the Even though a director may not be under the obligation of a fiduciary nature and-file staff. Mapua lost about 95% of her work projects and job
Philippine Sugar Estates Development Company. The defendant was the to disclose to a shareholder his knowledge affecting the value of the responsibilities.9
owner of 30,400 of the 42,030 shares of the company and was one of the shares, that duty may exist in special cases, and did exist upon the facts in
five directors as elected "with exclusive intervention in the management" of this case. If, under all these facts, he purchased the stock from the plaintiff, Mapua consulted these work problems with SPI’s Human Resource
its general business. In 1903 the Philippine government, made an offer of the law would indeed be impotent if the sale could not be set aside or the Director, Lea Villanueva (Villanueva), and asked if she can be transferred
purchase for the total sum of $6,043,219.47 in gold for all the friar lands, defendant cast in damages for his fraud. to another department within SPI. Subsequently, Villanueva informed
though owned by different owners. The offer was rejected by defendant in Mapua that there is an intra-office opening and that she would schedule an
his capacity as majority shareholder, without any consultation with the exploratory interview for her. However, due to postponements not made
other shareholders. by Mapua, the interview did not materialize.
70. SPI TECHNOLOGIES and LEA VILLANUEVA v. VICTORIA
The defendant continued his refusal to accept until the other owners MAPUA On February 28, 2007, Mapua allegedly saw the new table of organization
consented to pay to his company $335,000 of the purchase price for their of the Corporate Development Division which would be renamed as the
land, and until the government consented that a thousand hectares should Marketing Division. The new structure showed that Mapua’s level will be
G.R. No. 191154 April 7, 2014
be excluded from the sale to it of the land of defendant's company. This again downgraded because a new manager will be hired and positioned
being agreed to, the contract for the sale was finally signed by the between her rank and Raina’s.10
defendant as attorney in fact for his company, December 21, 1903. FACTS:

On March 21, 2007, Raina informed Mapua over the phone that her
During the course of said negotiation, on or about October 10, 1903, the Victoria K. Mapua (Mapua) alleged that she was hired in 2003 by SPI
position was considered redundant and that she is terminated from
agent of Strong, sold the 800 shares of stock for $16,000, Mexican Technologies, Inc. (SPI) and was the Corporate Development’s
employment effective immediately. Villanueva notified Mapua that she
currency to defendant. The defendant thus obtained the 800 shares for Research/Business Intelligence Unit Head and Manager of the company.
should cease reporting for work the next day. Her laptop computer and
about one tenth of the amount they became worth by the sale of the lands Subsequently in August 2006, the then Vice President and Corporate
company mobile phone were taken right away and her office phone
between two and three months thereafter. In all the negotiations in regard Development Head, Peter Maquera (Maquera) hired Elizabeth Nolan
ceased to function.11
to the purchase of the stock from Mrs. Strong, not one word of the facts (Nolan) as Mapua’s supervisor.5
affecting the value of this stock was made known to plaintiff's agent by
defendant. The real state of the negotiations with the government was not Mapua was shocked and told Raina and Villanueva that she would sue
Sometime in October 2006, the hard disk on Mapua’s laptop crashed,
mentioned, nor was the fact stated that it rested chiefly with the defendant them.
causing her to lose files and data. Mapua informed Nolan and her
to complete the sale. Moreover, the agent of the plaintiff had no knowledge colleagues that she was working on recovering the lost data and asked for
or suspicion that defendant was the one seeking to purchase the shares. their patience for any possible delay on her part in meeting deadlines.6 Victoria K. Mapua (Mapua) alleged that she was hired in 2003 by SPI
Technologies, Inc. (SPI) and was the Corporate Development’s
Research/Business Intelligence Unit Head and Manager of the company.
On November 13, 2006, Mapua retrieved the lost data with the assistance
RULING: Subsequently in August 2006, the then Vice President and Corporate
of National Bureau of Investigation Anti-Fraud and Computer Crimes
Development Head, Peter Maquera (Maquera) hired Elizabeth Nolan
Division. Yet, Nolan informed Mapua that she was realigning Mapua’s
1. Was it the duty of the defendant, acting in good faith, to disclose to the (Nolan) as Mapua’s supervisor.5
position to become a subordinate of co-manager Sameer Raina (Raina)
agent of the plaintiff the facts bearing upon or which might affect the value due to her missing a work deadline. Nolan also disclosed that Mapua’s
of the stock? colleagues were "demotivated" [sic] because she was "taking things easy Sometime in October 2006, the hard disk on Mapua’s laptop crashed,
while they were working very hard," and that she was "frequently absent, causing her to lose files and data. Mapua informed Nolan and her
No. A director upon whose action the value of the shares depends cannot under timing, and coming in late every time [Maquera] goes on leave or on colleagues that she was working on recovering the lost data and asked for
avail of his knowledge of what his own action will be to acquire shares from vacation."7 their patience for any possible delay on her part in meeting deadlines.6
those whom he intentionally keeps in ignorance of his expected action and
the resulting value of the shares. On November 16, 2006, Mapua obtained a summary of her attendance On November 13, 2006, Mapua retrieved the lost data with the assistance
for the last six months to prove that she did not have frequent absences or of National Bureau of Investigation Anti-Fraud and Computer Crimes
This is a rule of common law, and also of the Spanish law before the under time when Maquera would be on leave or vacation. When shown to Division. Yet, Nolan informed Mapua that she was realigning Mapua’s
adoption of the Philippine Civil Code; and, under ss. 1261-1269 of that Nolan, she was merely told not to give the matter any more importance position to become a subordinate of co-manager Sameer Raina (Raina)
code, a contract obtained under such circumstances can be avoided by and to just move on.8 due to her missing a work deadline. Nolan also disclosed that Mapua’s
the party whose consent would not have been given had he known the colleagues were "demotivated" [sic] because she was "taking things easy
facts within the knowledge of the other party. while they were working very hard," and that she was "frequently absent,
In December 2006, Mapua noticed that her colleagues began to ostracize
under timing, and coming in late every time [Maquera] goes on leave or on
and avoid her. Nolan and Raina started giving out majority of her research
vacation."7
work and other duties under Healthcare and Legal Division to the rank-
60
Derivative Suit
On November 16, 2006, Mapua obtained a summary of her attendance damages to the corporation, its stockholders or other persons; (b) they legitimate issues. This is consistent with the provisions of the Corporation
for the last six months to prove that she did not have frequent absences or consent to the issuance of watered down stocks or when, having Code of the Philippines, which states:
under time when Maquera would be on leave or vacation. When shown to knowledge of such issuance, do not forthwith file with the corporate
Nolan, she was merely told not to give the matter any more importance secretary their written objection; (c) they agree to hold themselves Sec. 31. Liability of directors, trustees or officers. – Directors or
and to just move on.8 personally and solidarily liable with the corporation; or (d) they are made by trustees who wilfully and knowingly vote for or assent to patently unlawful
specific provision of law personally answerable for their corporate action." acts of the corporation or who are guilty of gross negligence or bad faith in
In December 2006, Mapua noticed that her colleagues began to ostracize directing the affairs of the corporation or acquire any personal or pecuniary
and avoid her. Nolan and Raina started giving out majority of her research While the Court finds Mapua’s averments against Villanueva, Nolan, interest in conflict with their duty as such directors or trustees shall be liable
work and other duties under Healthcare and Legal Division to the rank- Maquera and Raina as detailed and exhaustive, the Court takes notice that jointly and severally for all damages resulting therefrom suffered by the
and-file staff. Mapua lost about 95% of her work projects and job these are mostly suppositions on her part. Thus, the Court cannot apply corporation, its stockholders or members and other persons.
responsibilities.9 the above-enumerated exceptions when a corporate officer becomes
personally liable for the obligation of a corporation to this case. Solidary liability will then attach to the directors, officers or
Mapua consulted these work problems with SPI’s Human Resource employees of the corporation in certain circumstances, such as: 1. When
Director, Lea Villanueva (Villanueva), and asked if she can be transferred directors and trustees or, in appropriate cases, the officers of a corporation:
to another department within SPI. Subsequently, Villanueva informed (a) vote for or assent to patently unlawful acts of the corporation; (b) act in
Mapua that there is an intra-office opening and that she would schedule an bad faith or with gross negligence in directing the corporate affairs; and (c)
71. HEIRS OF FE TAN UY v. INTERNATIONAL EXCHANGE BANK
exploratory interview for her. However, due to postponements not made are guilty of conflict of interest to the prejudice of the corporation, its
G.R. No. 166282; February 13, 2013
by Mapua, the interview did not materialize. stockholders or members, and other persons; 2. When a director or officer
has consented to the issuance of watered stocks or who, having
FACTS:
knowledge thereof, did not forthwith file with the corporate secretary his
On February 28, 2007, Mapua allegedly saw the new table of organization International Bank (iBank) granted loans to Hammer Garments
written objection thereto; 3. When a director, trustee or officer has
of the Corporate Development Division which would be renamed as the Corporation (Hammer), covered by promissory notes and deeds of
contractually agreed or stipulated to hold himself personally and solidarily
Marketing Division. The new structure showed that Mapua’s level will be assignment. The loans were secured by a 9-million-peso real estate
liable with the corporation; or 4. When a director, trustee or officer is made,
again downgraded because a new manager will be hired and positioned mortgage by Goldkey Development Corporation (Goldkey) and a 25-
by specific provision of law, personally liable for his corporate action.
between her rank and Raina’s.10 million-peso surety agreement signed by Chua and his wife, Fe Tan Uy
(Uy). Hammer defaulted in the payment of the loans prompting iBank to
foreclose the real estate mortgage leaving an unpaid balance of almost Before a director or officer of a corporation can be held
On March 21, 2007, Raina informed Mapua over the phone that her personally liable for corporate obligations, however, the following requisites
13.5 million pesos. For failure of Hammer to pay the deficiency, iBank filed
position was considered redundant and that she is terminated from must concur: (1) the complainant must allege in the complaint that the
a complaint for sum of money against Hammer, Chua, Uy and Goldkey.
employment effective immediately. Villanueva notified Mapua that she director or officer assented to patently unlawful acts of the corporation, or
Chua and Hammer failed to file their answers and were declared in default.
should cease reporting for work the next day. Her laptop computer and that the officer was guilty of gross negligence or bad faith; and (2) the
Uy claimed that she was not liable to iBank because she never executed
company mobile phone were taken right away and her office phone complainant must clearly and convincingly prove such unlawful acts,
a surety agreement in favor of iBank.
ceased to function.11 negligence or bad faith.
ISSUE:
Mapua was shocked and told Raina and Villanueva that she would sue Is Uy liable for the corporation because she was an officer, In this case, it was not alleged that Uy committed an act as an
them. stockholder thereof? officer of Hammer that would permit the piercing of the corporate veil. A
reading of the complaint reveals that with regard to Uy, iBank did not
RULING: demand that she be held liable for the obligations of Hammer because she
ISSUE:
No. Basic is the rule in corporation law that a corporation is a was a corporate officer who committed bad faith or gross negligence in the
juridical entity which is vested with a legal personality separate and distinct performance of her duties such that the lifting of the corporate mask would
WON the coroporate officers are solidarily liable from those acting for and in its behalf and, in general, from the people be merited. What the complaint simply stated is that she, together with her
comprising it. Following this principle, obligations incurred by the errant husband Chua, acted as surety of Hammer, as evidenced by her
Held: corporation, acting through its directors, officers and employees, are its signature on the Surety Agreement which was later found by the RTC to
sole liabilities. A director, officer or employee of a corporation is generally have been forged.
On the issue of the solidary obligation of the corporate officers impleaded not held personally liable for obligations incurred by the corporation.
vis-à-vis the corporation for Mapua’s illegal dismissal, "[i]t is hornbook Indeed, there is no showing that Uy committed gross negligence.
principle that personal liability of corporate directors, trustees or officers Nevertheless, this legal fiction may be disregarded if it is used as And in the absence of any of the aforementioned requisites for making a
attaches only when: (a) they assent to a patently unlawful act of the a means to perpetrate fraud or an illegal act, or as a vehicle for the evasion corporate officer, director or stockholder personally liable for the obligations
corporation, or when they are guilty of bad faith or gross negligence in of an existing obligation, the circumvention of statutes, or to confuse of a corporation, Uy, as a treasurer and stockholder of Hammer, cannot be
directing its affairs, or when there is a conflict of interest resulting in made to answer for the unpaid debts of the corporation.
61
Derivative Suit
72. EVER ELECTRICAL MANUFACTURING CORP. v SMEE- and, as a result, EEMI’s employees were prevented from entering the had been closed for lack of venue, there would be no one to pay its liability
NAMAWU factory. as the respondents therein claimed that the restaurant was owned by a
G.R. No. 194795 June 13, 2012 different entity, not a party in the case.24 In two subsequent cases, the
Court’s ruling in Restaurante Las Conchas was invoked but the Court
J. MENDOZA refused to consider it reasoning out that it was the exception rather than
the rule. The two cases were Mandaue Dinghow Dimsum House, Co., Inc.
ISSUE/S: and/or Henry Uytengsu v. National Labor Relations Commission25 and
Pantranco Employees Association (PEA-PTGWO) v. National Labor
FACTS:
WHETHER THE CA ERRED IN FINDING VICENTE GO Relations Commission.26 In Mandaue Dinghow Dimsum House, Co., Inc.,
SOLIDARILY LIABLE WITH EEMI the Court declined to apply the ruling in Restaurante Las Conchas because
Petitioner Ever Electrical Manufacturing, Inc. (EEMI) is a there was no evidence that the respondent therein, Henry Uytrengsu,
corporation engaged in the business of manufacturing electrical parts and acted in bad faith or in excess of his authority. It stressed that a corporation
supplies. On the other hand, the respondents are members of Samahang is invested by law with a personality separate and distinct from those of the
Manggagawa ng Ever Electrical/NAMAWU Local 224 (respondents) persons composing it as well as from that of any other legal entity to which
headed by Felimon Panganiban.The controversy started when EEMI RULING: it may be related. For said reason, the doctrine of piercing the veil of
closed its business operations on October 11, 2006 resulting in the corporate fiction must be exercised with caution.27 Citing Malayang
termination of the services of its employees. Aggrieved, respondents filed As a general rule, corporate officers should not be held solidarily Samahan ng mga Manggagawa sa M. Greenfield v. Ramos,28 the Court
a complaint for illegal dismissal with prayer for payment of 13th month pay, liable with the corporation for separation pay for it is settled that a explained that corporate directors and officers are solidarily liable with the
separation pay, damages, and attorney’s fees. Respondents alleged that corporation is invested by law with a personality separate and distinct from corporation for the termination of employees done with malice or bad faith.
the closure was made without any warning, notice or memorandum and in those of the persons composing it as well as from that of any other legal It stressed that bad faith does not connote bad judgment or negligence; it
full disregard of the requirements of the Labor Code.In its defense, EEMI entity to which it may be related. Mere ownership by a single stockholder imports a dishonest purpose or some moral obliquity and conscious doing
explained that it had closed the business due to various factors. In 1995, it or by another corporation of all or nearly all of the capital stock of a of wrong; it means breach of a known duty through some motive or interest
invested in Orient Commercial Banking Corporation (Orient Bank) the sum corporation is not of itself sufficient ground for disregarding the separate or ill will; it partakes of the nature of fraud.
of ₱500,000,000.00 and during the Asian Currency crises, various corporate personality.17 The LA was of the view that Go, as President of
economies in the South East Asian Region were hurt badly. EEMI was one the corporation, actively participated in the management of EEMI’s In Pantranco Employees Association, the Court also rejected the
of those who suffered huge losses. In November 1996, it obtained a loan corporate obligations, and, accordingly, rendered judgment ordering EEMI invocation of Restaurante Las Conchas and refused to pierce the veil of
in the amount of ₱121,400,000.00 from United Coconut Planters Bank and Go "in solidum to pay the complainants"18 their due. He explained that corporate fiction. It explained:
(UCPB). As security for the loan, EEMI’s land and its improvements, "[r]espondent Go’s negligence in not paying the lease rental of the plant in
including the factory, were mortgaged to UCPB. behalf of the lessee EGO Electrical Supply, Inc., where EEMI was
operating and reimburse expenses of UCPB for real estate taxes and the As between PNB and PNEI, petitioners want us to disregard their separate
like, prompted the bank to file an unlawful detainer case against the lessee, personalities, and insist that because the company, PNEI, has already
EEMI’s business suffered further losses due to the continued ceased operations and there is no other way by which the judgment in
entry of cheaper goods from China and other Asian countries. Adding to EGO Electrical Supply Co. This evasion of an existing obligation, made
respondent Go as liable as respondent EEMI, for complainants’ money favor of the employees can be satisfied, corporate officers can be held
EEMI’s financial woes was the closure of Orient Bank where most of its jointly and severally liable with the company. Petitioners rely on the
resources were invested. As a result, EEMI was not able to meet its loan awards."19 Added the LA, "being the President and the one actively
representing respondent EEMI, in major contracts i.e. Real Estate pronouncement of this Court in A.C. Ransom Labor Union-CCLU v. NLRC
obligations with UCPB.In an attempt to save the company, EEMI entered and subsequent cases.
into a dacion en pago arrangement with UCPB which, in effect, transferred Mortgage, loans, dacion en pago, respondent Go has to be liable in the
ownership of the company’s property to UCPB as reflected in TCT No. case."20 As earlier stated, the CA affirmed the LA decision citing the case
429159. Originally, EEMI wanted to lease the premises to continue its of Restaurante Las Conchas v. Llego,21 where it was held that "when the This reliance fails to persuade. We find the aforesaid decisions inapplicable
business operation but under UCPB’s policy, a previous debtor who failed employer corporation is no longer existing and unable to satisfy the to the instant case.
to settle its loan obligation was not eligible to lease its acquired assets. judgment in favor of the employees, the officers should be held liable for
Thus, UCPB agreed to lease it to an affiliate corporation, EGO Electrical acting on behalf of the corporation."22 For one, in the said cases, the persons made liable after the
Supply Co, Inc. (EGO), for and in behalf of EEMI. On February 2, 2002, a company’s cessation of operations were the officers and agents of the
lease agreement was entered into between UCPB and EGO.4 The said A study of Restaurante Las Conchas case, however, bares that corporation. The rationale is that, since the corporation is an artificial
lease came to a halt when UCPB instituted an unlawful detainer suit it was an application of the exception rather than the general rule. As stated person, it must have an officer who can be presumed to be the employer,
against EGO before the Metropolitan Trial Court, Branch 5, Makati City in the said case, "as a rule, the officers and members of a corporation are being the person acting in the interest of the employer. The corporation,
(MeTC) docketed as Civil Case No. 88602. On August 11, 2006, the MeTC not personally liable for acts done in the performance of their duties."23 only in the technical sense, is the employer. In the instant case, what is
ruled in favor of UCPB and ordered EGO to vacate the leased premises The Court therein explained that it applied the exception because of the being made liable is another corporation (PNB) which acquired the debtor
and pay rentals to UCPB in the amount of ₱21,473,843.65.5 On peculiar circumstances of the case. If the rule would be applied, the corporation (PNEI).Moreover, in the recent cases Carag v. National Labor
September 19, 2006, a writ of execution was issued.6 Consequently, on employees would end up in an empty victory because as the restaurant Relations Commission and McLeod v. National Labor Relations
October 11, 2006, the Sheriff implemented the writ by closing the premises Commission, the Court explained the doctrine laid down in AC Ransom
62
Derivative Suit
relative to the personal liability of the officers and agents of the employer has, by law, a personality separate and distinct from that of its owners. As 2. When the director or officer has consented to the
for the debts of the latter. In AC Ransom, the Court imputed liability to the there is no evidence that Go, as EEMI’s President, acted maliciously or in issuance of watered stock or who, having knowledge thereof, did
officers of the corporation on the strength of the definition of an employer bad faith in handling their business affairs and in eventually implementing not forthwith file with the corporate secretary his written objection
in Article 212(c) (now Article 212[e]) of the Labor Code. Under the said the closure of its business, he cannot be held jointly and solidarily liable thereto.
provision, employer includes any person acting in the interest of an with EEMI.
employer, directly or indirectly, but does not include any labor organization 3. When a director, trustee or officer has
or any of its officers or agents except when acting as employer. It was contractually agreed or stipulated to hold himself personally and
clarified in Carag and McLeod that Article 212(e) of the Labor Code, by solidarily liable with the corporation.
itself, does not make a corporate officer personally liable for the debts of
73. HARPOON MARINE SERVICES v. FRANCSICO
the corporation. It added that the governing law on personal liability of 4. When a director, trustee or officer is made, by
G.R. No. 167751; March 2, 2011
directors or officers for debts of the corporation is still Section 31 of the specific provision of law, personally liable for his corporate action.
Corporation Code.More importantly, as aptly observed by this Court in AC
FACTS:
Ransom, it appears that Ransom, foreseeing the possibility or probability
of payment of backwages to its employees, organized Rosario to replace The general rule is grounded on the theory that a corporation has
Harpoon is a company engaged in ship building and repair, with Rosit
Ransom, with the latter to be eventually phased out if the strikers win their a legal personality separate and distinct from the persons comprising it. To
as its president and CEO. Fernan Francisco was hired as yard supervisor
case. The execution could not be implemented against Ransom because warrant the piercing of the veil of corporate fiction, the officers bad faith or
in 1992 but he was unceremoniously dismissed by Rosit in 2001.
of the disposition posthaste of its leviable assets evidently in order to evade wrongdoing must be established clearly and convincingly as bad faith is
Francisco was only offered his separation pay, hence an illegal dismissal
its just and due obligations. Hence, the Court sustained the piercing of the never presumed.
complaint was filed, praying for the payment of his backwages, separation
corporate veil and made the officers of Ransom personally liable for the
pay, unpaid commission, moral and exemplary damages and attorney’s
debts of the latter. In the case at bench, the CA’s basis for petitioner Rosit’s liability
fees.
was that he acted in bad faith when he approached respondent and told
The Labpr Arbiter decided that there was valid dismissal. NLRC
Clearly, what can be inferred from the earlier cases is that the him that the company could no longer afford his salary and that he will be
reversed LA. CA affirmed NLRC. Hence, this petition.
doctrine of piercing the corporate veil applies only in three (3) basic areas, paid instead his separation pay and accrued commissions. This finding,
namely: 1) defeat of public convenience as when the corporate fiction is however, could not substantially justify the holding of any personal liability
ISSUE: Should Rosit be held liable for the dismissal of Fernan Francisco?
used as a vehicle for the evasion of an existing obligation; 2) fraud cases against petitioner Rosit. The records are bereft of any other satisfactory
or when the corporate entity is used to justify a wrong, protect fraud, or evidence that petitioner Rosit acted in bad faith with gross or inexcusable
RULING: No.
defend a crime; or 3) alter ego cases, where a corporation is merely a farce negligence, or that he acted outside the scope of his authority as company
since it is a mere alter ego or business conduit of a person, or where the president. Indeed, petitioner Rosit informed respondent that the company
Rosit should not be held solidarily liable with petitioner Harpoon for the
corporation is so organized and controlled and its affairs are so conducted wishes to terminate his services since it could no longer afford his salary.
payment of respondents backwages and separation pay.
as to make it merely an instrumentality, agency, conduit or adjunct of Moreover, the promise of separation pay, according to petitioners, was out
another corporation. In the absence of malice, bad faith, or a specific of goodwill and magnanimity. At the most, petitioner Rosits actuations only
As held in the case of MAM Realty Development Corporation v.
provision of law making a corporate officer liable, such corporate officer show the illegality of the manner of effecting respondents termination from
National Labor Relations Commission, obligations incurred by [corporate
cannot be made personally liable for corporate liabilities.29 [Emphasis service due to absence of just or valid cause and non-observance of
officers], acting as such corporate agents, are not theirs but the direct
supplied]Similarly, in the case at bench, the records do not warrant an procedural due process but do not point to any malice or bad faith on his
accountabilities of the corporation they represent. As such, they should not
application of the exception.1âwphi1 The rule, which requires the presence part. Besides, good faith is still presumed. In addition, liability only attaches
be generally held jointly and solidarily liable with the corporation. The Court,
of malice or bad faith, must still prevail. In the recent case of Wensha Spa if the officer has assented to patently unlawful acts of the corporation.
however, cited circumstances when solidary liabilities may be imposed, as
Center and/or Xu Zhi Jie v. Yung,30 the Court absolved the corporation’s Thus, it was error for the CA to hold petitioner Rosit solidarily
exceptions:
president from liability in the absence of bad faith or malice. In the said liable with petitioner Harpoon for illegally dismissing respondent.
case, the Court stated:In labor cases, corporate directors and officers may 1. When directors and trustees or, in appropriate
be held solidarily liable with the corporation for the termination of cases, the officers of a corporation
employment only if done with malice or in bad faith.31 Bad faith does not
connote bad judgment or negligence; it imports a dishonest purpose or (a) vote for or assent to [patently] unlawful acts of the 74. QUEENSLAND-TOKYO COMMODITIES v. GEORGE
some moral obliquity and conscious doing of wrong; it means breach of a corporation; GR 172727, 08 September 2010
known duty through some motive or interest or ill will; it partakes of the (b) act in bad faith or with gross negligence in directing
nature of fraud.32 In the present case, Go may have acted in behalf of the corporate affairs;
EEMI but the company’s failure to operate cannot be equated to bad faith. (c) are guilty of conflict of interest to the prejudice of the FACTS
Cessation of business operation is brought about by various causes like corporation, its stockholders or members, and other persons.
mismanagement, lack of demand, negligence, or lack of business QTCI is a duly licensed broker engaged in the trading of commodity
foresight. Unless it can be shown that the closure was deliberate, malicious
futures. In 1995, Guillermo Mendoza, Jr. (Mendoza) and Oniler Lontoc
and in bad faith, the Court must apply the general rule that a corporation
63
Derivative Suit
(Lontoc) of QTCI met with respondent Thomas George (respondent), as a rule, only when - (1) he assents to a patently unlawful act of the work. Thus, the burden was on him to substantiate his claims for illegal
encouraging the latter to invest with QTCI. corporation, or when he is guilty of bad faith or gross negligence in directing dismissal.
its affairs, or when there is a conflict of interest resulting in damages to the
On July 7, 1995, upon Mendoza's prodding, respondent finally invested corporation, its stockholders, or other persons; (2) he consents to the
with QTCI. On the same day, Collado, in behalf of QTCI, and respondent issuance of watered down stocks or who, having knowledge thereof, does
signed the Customer's Agreement. Forming part of the agreement was the Issue:
not forthwith file with the corporate secretary his written objection thereto;
Special Power of Attorney executed by respondent, appointing Mendoza (3) he agrees to hold himself personally and solidarily liable with the Whether or not respondent Cando the President of Bill Sender
as his attorney-in-fact with full authority to trade and manage his account. corporation; or (4) he is made by a specific provision of law personally Corporation could be made personally and solidarily liable with the
answerable for his corporate action. corporation for the monetary claims of Geraldo.
On June 20, 1996, the Securities and Exchange Commission (SEC)
issued a Cease-and-Desist Order against QTCI. Alarmed by the issuance Romeo Lau, as president of [petitioner] QTCI, cannot feign innocence on
of the CDO, respondent demanded from QTCI the return of his investment, the existence of these unlawful activities within the company, especially so
but it was not heeded. QTCI claimed that they were not aware of, nor were that Collado, himself a ranking officer of QTCI, is involved in the unlawful Held:
they privy to, any arrangement which resulted in the account of respondent execution of customers orders. Lau, being the chief operating officer,
being handled by unlicensed brokers. They pointed out that respondent It must be noted, however, that respondent Cando cannot be
cannot escape the fact that had he exercised a modicum of care and
transacted business with QTCI for almost a year, without questioning the held personally and solidarity liable with the company for the monetary
discretion in supervising the operations of QTCI, he could have detected
license or the authority of the traders handling his account, rendering him claims of Geraldo. As a general rule, a corporate officer cannot be held
and prevented the unlawful acts of Collado and Mendoza.
estopped. It was only after it became apparent that QTCI liable for acts done in his official capacity because a corporation, by legal
fiction, has a personality separate and distinct from its officers,
could no longer resume its business transactions by reason of the CDO stockholders, and members. To pierce this fictional veil, it must be shown
that respondent raised the alleged lack of authority of the brokers or traders 75. GERALDO vs. BILL SENDER that the corporate personality was used to perpetuate fraud or an illegal
handling his account. GR No. 222219 October 03, 2018 act, or to evade an existing obligation, or to confuse a legitimate issue. In
illegal dismissal cases, corporate officers may be held solidarily liable with
the corporation if the termination was done with malice or bad faith. To
ISSUE hold a director or officer personally liable for corporate obligations, two
Facts:
requisites must concur, to wit: (1) the complaint must allege that the
Whether or not QTCI should be held liable for the loss incurred by George On June 20, 1997, respondent The Bill Sender Corporation, director or officer assented to the patently unlawful acts of the corporation,
in the investment he made with the corporation engaged in the business of delivering bills and other mail matters for and or that the director or officer was guilty of gross negligence or bad faith;
in behalf of their customers, employed petitioner Reynaldo S. Geraldo as and (2) there must be proof that the director or officer acted in bad faith. In
a delivery/messenger man to deliver the bills of its client, the Philippine the instant case, however, there is no showing that Cando, as President
Long Distance Telephone Company (PLDT). He was paid on a "per- of the company, was guilty of malice or bad faith in terminating the
RULING
piece basis," the amount of his salary depending on the number of bills employment of Geraldo. Thus, she should not be held personally liable
YES. he delivered. for his monetary claims.

It recognized Mendoza and Collado as its brokers. Petitioners did not On February 6, 2012, Geraldo filed a complaint for illegal
object to, and in fact recognized, Mendoza's appointment as respondent's dismissal alleging that on August 7, 2011, the company's operations
attorney-in-fact. manager, Mr. Nicolas Constantino, suddenly informed him that his
employment was being terminated because he failed to deliver certain
Collado, in behalf of QTCI, concluded the Customer's Agreement despite bills. He explained that he was not the messenger assigned to deliver the
the fact that the appointed attorney-in-fact was not a licensed dealer. said bills but the manager refused to reconsider and proceeded with his
Worse, petitioners permitted Mendoza to handle respondent's account. termination. Thus, he claims that his dismissal was illegal for being done
Doctrine dictates that a corporation is invested by law with a personality without the required due process under the law and that the company
separate and distinct from those of the persons composing it, such that, and its president, respondent Lourdes Ner Cando, be held liable for his
save for certain exceptions, corporate officers who entered into contracts monetary claims.
in behalf of the corporation cannot be held personally liable for the liabilities The company asserts that he was not illegally dismissed for he
of the latter. Personal liability of a corporate director, trustee, or officer, was the one who abandoned his job when he no longer reported for
along (although not necessarily) with the corporation, may validly attach,
64
Derivative Suit
76. WENSHA SPA CENTER and/or XU ZHI JIE v. LORETA YUNG Is Xu, as the president of Wensha, solidarily liable with the company? Upon maturity, the seven checks were presented for payment by
G.R. No. 185122 Tsukahara, but the same were dishonored by PNB, the drawee bank. After
August 16, 2010 RULING: several failed attempts to collect the loan amount totaling P16,500,000,
Tsukahara filed the instant case for collection of sum of money against
FACTS: No. CMMCI and Sugimoto.

Xu Zhi Jie (Xu) is the president of Wensha Spa Center, Inc. (Wensha) and Elementary is the rule that a corporation is invested by law with a Tsukahara alleged that the amount of P16,500,000 was used by CMMCI
the sole owner of the company. Respondent Loreta Yung (Loreta) was its personality separate and distinct from those of the persons composing it for the improvement of its beach resort, which included the construction of
administrative manager at the time of her termination. and from that of any other legal entity to which it may be related. "Mere a wave fence, the purchase of airconditioners and curtains, and the
ownership by a single stockholder or by another corporation of all or nearly provision of salaries of resort employees. He also asserted that Sugimoto,
Loreta used to be employed in Manmen Services Co., Ltd. (Manmen) all of the capital stock of a corporation is not of itself sufficient ground for as the President of CMMCI, "has the power to borrow money for said
where Xu was a client. Xu was impressed by Loreta, hence, he convinced disregarding the separate corporate personality." corporation by any legal means whatsoever and to sign, endorse and
her to work for him. Loreta was initially reluctant for her work at Manmen deliver all checks and promissory notes on behalf of the corporation."
was stable and she had been there for seven (7) years. However, she In labor cases, corporate directors and officers may be held solidarily liable
accepted the work from Xu when she was offered a higher salary. She with the corporation for the termination of employment only if done with
started as a personal assistant of Xu until she was promoted as the malice or in bad faith. Bad faith does not connote bad judgment or CMMCI, on the other hand, denied borrowing the amount from Tsukahara,
administrative manager of Wensha when her performance indicated a negligence; it imports a dishonest purpose or some moral obliquity and and claimed that both loans were personal loans of Sugimoto. The
positive changes to the company. conscious doing of wrong; it means breach of a known duty through some company also contended that if the loans were those of CMMCI, the same
motive or interest or ill will; it partakes of the nature of fraud. should have been supported by resolutions issued by CMMCI's Board of
In August 2004, she was asked to leave her office because Xu and a Feng Directors.
Shui master was exploring the premises. Thereafter, Xu asked Loreta to In the subject decision, the CA concluded that petitioner Xu and Wensha
go on leave with pay for one (1) month. Upon her return in September are jointly and severally liable to Loreta. The Court have read the decision Issue:
2004, Xu and his wife asked her to resign because according to the Feng in its entirety but simply failed to come across any finding of bad faith or Whether the Court of Appeals erred in holding that CMMCI is liable for the
Shui Master, her aura did not match that of Xu (lol). Loreta refused but to malice on the part of Xu. There is, therefore, no justification for such a loan contracted by its President without a resolution issued by the CMMCI
no avail. ruling. To sustain such a finding, there should be an evidence on record Board of Directors.
that an officer or director acted maliciously or in bad faith in terminating the Ruling:
Hence, she filed a case for illegal dismissal against Wensha and Xu before services of an employee. Moreover, the finding or indication that the No.
the NLRC. She alleged that she has done several improvements in dismissal was effected with malice or bad faith should be stated in the
Wensha such as uplifting the morale and efficiency of its employees and decision itself. In this case, there was none. A corporation, being a juridical entity, may act through its board of directors,
increasing respondents’ clientele, and that she was offered twice a which exercises almost all corporate powers, lays down all corporate
promotion but she nevertheless declined. It would be against human Thus, Xu cannot be held solidarily liable with its corporation, Wensha. business policies and is responsible for the efficiency of management. The
experience and contrary to business acumen to let go of someone, who general rule is that, in the absence of authority from the board of directors,
was an asset and has done so much for the company merely on the no person, not even its officers, can validly bind a corporation. Section 23
ground that she is a "mismatch" to the business 77. CEBU MACTAN v. MASAHIRO TSUKAHARA of the Corporation Code of the Philippines provides:
17 July 2009
Wensha and Xu denied illegally terminating Loreta. They claimed that they
received various complaints against her from employees, hence, they Facts: SEC. 23. The Board of Directors or Trustees. - Unless otherwise provided
advised her to take a leave to conduct an investigation. Thereafter, they In February 1994, petitioner Cebu Mactan Members Center, Inc. (CMMCI), in this Code, the corporate powers of all corporations formed under this
terminated her due to loss of trust and confidence. through Mitsumasa Sugimoto (Sugimoto), the President and Chairman of Code shall be exercised, all business conducted and all property of such
the Board of Directors of CMMCI, obtained a loan amounting corporations controlled and held by the board of directors or trustees x x x.
The Labor Arbiter dismissed the complaint of Loreta on the sole ground to P6,500,000 from respondent Masahiro Tsukahara. As payment for the
that the ground of “mismatch” aura is hard to believe and was dubious. The loan, CMMCI issued seven postdated checks of CMMCI payable to However, just as a natural person may authorize another to do certain acts
ruling was also affirmed by NLRC. Tsukahara. for and on his behalf, the board of directors may validly delegate some of
CMMCI, through Sugimoto, obtained another loan amounting its functions and powers to officers, committees or agents. The authority
However, CA noticed some irregularities and inconsistencies with to P10,000,000 from Tsukahara. Sugimoto executed and signed a of such individuals to bind the corporation is generally derived from
Wensha’s position. Thus finds the decision in favor of Loreta. It ruled that promissory note in his capacity as CMMCI President and Chairman, as law, corporate by-laws or authorization from the board, either expressly
Xu is solidarily liable with Wensha. well as in his personal capacity. or impliedly by habit, custom or acquiescence in the general course of
business. This Court has held, thus:
ISSUE:

65
Derivative Suit
A corporate officer or agent may represent and bind the corporation in judgment award in its favor. heard and to present his evidence.
transactions with third persons to the extent that [the] authority to do so has 6. MACs counsel argued that Carag and David should not be held liable Hence, the Labor Arbiter and the NLRC did not have jurisdiction
been conferred upon him, and this includes powers which have been because MAC is owned by a consortium of banks. Carags and over David and making its decision void.
intentionally conferred, and also such powers as, in the usual course of the
Davids ownership of MAC shares only served to qualify them as
particular business, are incidental to, or may be implied from, the powers
intentionally conferred, powers added by custom and usage, as usually officers in MAC. 79. HILARIO P. SORIANO and ROSALINDA ILAGAN vs. PEOPLE,
pertaining to the particular officer or agent, and such apparent powers as 7. The LA and NLRC ruled in favor of NAFLU and MACLU. It held that BSP and PDIC
the corporation has caused persons dealing with the officer or agent to “Normally, officers acting for and in behalf of a corporation are not held G.R. No. 159517-18
believe that it has conferred. personally liable for the obligation of the corporation. However, where June 30, 2009
corporate officers dismissed employees in bad faith or wantonly
To insist that a board resolution is still required in order to bind the violate labor standard laws or when the company had already ceased FACTS:
corporation with respect to the obligations contracted by its president is to operations and there is no way by which a judgment in favor of
defeat the purpose of the by-laws. By-laws of a corporation should be employees could be satisfied, corporate officers can be held jointly Hilario P. Soriano (Soriano) and Rosalinda Ilagan (Ilagan) were the
construed and given effect according to the general rules governing the
and severally liable with the company.” President and General Manager, respectively, of the Rural Bank of San
construction of contracts. They, as the self-imposed private laws of a Miguel (Bulacan), Inc. (RBSM).
corporation, have, when valid, substantially the same force and effect as
laws of the corporation, as have the provisions of its charter insofar as the ISSUE:
corporation and the persons within it are concerned. They are in effect 1Is David liable for the obligations of MAC to its employees? June 27, 1997 and August 21, 1997, during their incumbency as president
written into the charter and in this sense, they become part of the 2Was there a violation of the right to due process? and manager of the bank, petitioners indirectly obtained loans from RBSM.
fundamental law of the corporation. And the corporation and its directors They falsified the loan applications and other bank records, and made it
(or trustees) and officers are bound by and must comply with them. RULING: appear that Virgilio J. Malang and Rogelio Mañaol obtained loans of
1. NO. ₱15,000,000.00 each, when in fact they did not.
Article 212(e) of the Labor Code does not make a corporate
The corporation is now estopped from denying the authority of its president officer personally liable for the debts of the corporation because Section 31
to bind the former into contractual relations. May 4, 2000, State Prosecutor Josefino A. Subia charged Soriano with
of the Corporation Code is the governing law on personal liability of officers violation of the DOSRI rules. An information for estafa thru falsification of
for the debts of the corporation. commercial document was also filed against Soriano and Ilagan.
Section 31 of the Corporation Code provides that: “Directors or
trustees who willfully and knowingly vote for or assent to patently unlawful Another information for violation of Section 83 of R.A. No. 337, as
78. ARMANDO DAVID v. NAFLU acts of the corporation or who are guilty of gross negligence or bad faith in amended, was filed against Soriano, this time, covering the
April 21, 2009 G.R. Nos. 148263 and 148271-72 directing the affairs of the corporation or acquire any personal interest in ₱15,000,000.00 loan obtained in the name of Rogelio Mañaol.
conflict with their duty shall be liable jointly and severally for all damages
FACTS: suffered by the corporation, its stockholders or members and other Soriano and Ilagan were also indicted for estafa thru falsification of
1. September 16 1988 Mariveles Apparel Corporation (MAC) hired persons.” commercial document for obtaining said loan.
Armando David (David) as IMPEX and Treasury Manager In the case, there was no showing of David willingly and
2. May 1990 David began serving as MACs President in the nature of a knowingly voting for or assenting to patently unlawful acts of the Petitioners moved to quash the informations arguing that the prosecutor
corporation, or that David was guilty of gross negligence or bad faith. charged more than one offense for a single act. Soriano was charged with
nominee as he did not own any of MACs shares. The LA based its ruling under Article 212(e) of the Labor Code violation of DOSRI rules and estafa thru falsification of commercial
3. September 30 1993 David tendered his irrevocable resignation from which provides that an “Employer includes any person acting in the interest document for allegedly securing fictitious loans. They further argued that
MAC and took effect on October 15 1993. of an employer, directly or indirectly. The term shall not include any labor the facts as alleged in the information do not constitute an offense.
4. In a complaint for illegal dismissal National Federation of Labor organization or any of its officers or agents except when acting as
Unions (NAFLU) and MACLU alleged that MAC ceased operations employer.” Petitioners also contend that Soriano should be charged with one offense
on July 8 1993 without prior notice to its employees. MAC allegedly ---------------------------------------------------------------------------------- only, because all the charges filed against him proceed from and are based
2. YES. The Court based its ruling pursuant to the Sections 2 on a single act of obtaining fictitious loans. Thus, Soriano argues that he
gave notice of its closure on the same day that it ceased operations.
3, 4, 5(b), and 11(c) of Rule V of the New Rules of Procedure of the cannot be charged with estafa thru falsification of commercial document,
It further alleged that, at the time of MACs closure, employees who NLRC. considering that he is already being prosecuted for obtaining a DOSRI
had rendered one to two weeks work were not paid their The records of the present case fail to show any order from LA loan.
corresponding salaries. summoning David to attend the preliminary conference. Despite this lack
5. January 3, 1994, MACLU and NAFLU filed their position paper and of summons, LA not only granted MACLU and NAFLUs motion to implead November 15, 2000, RTC Branch 77 denied the motion to quash stating :
moved to implead Carag and David to guarantee satisfaction of any but also held them solidarily liable with MAC. Moreover, David was not assuming that the two (2) cases arose from the same facts, if they violate
ordered to submit a position paper, was not given the opportunity to be
66
Derivative Suit
two (2) or more provisions of the law, a prosecution under one will not bar The petition for review is denied. Is the Country Club and its Directors liable to pay damages to Elizagaque
a prosecution under another. in disapproving his application for proprietary membership?

All other motions to quash filed by petitioners for the other information was Ruling:
also dismissed by the court. 80. CEBU COUNTRY CLUB v. ELIZAGAQUE
Cebu Country Club, Inc., Sabino R. Dapat, Ruben D. Almendras, The Country Club and its Directors are liable to pay damages to
Petitioners appealed the case to the CA via Certiorari but it was also Julius Z. Neri, Douglas L. Luym, Cesar T. Libi, Ramontito* E. Garcia
dismissed. Elizagaque in the disapproval of his application because it was done in
And Jose B. Sala v. Ricardo F. Elizagaque fraud and evident bad faith.
Thus the petition before the Supreme Court.
Facts: According to the Articles of Incorporation of the Country Club, it provides
ISSUE: that:
Cebu Country Club, Inc.(Country club) is a non-stock, non-profit domestic
Whether or not the dismissal of the motion to quash was proper. corporation and a private membership club and its co-petitioners are its Section 3, Article 1 of CCCI’s Amended By-Laws provides:
board of directors while Ricardo Elizagaque (Elizagaque) is San Miguel
HELD: Corporation’s Senior Vice President and Operations Manager for the SECTION 3. HOW MEMBERS ARE ELECTED –
Visayas and Mindanao. The procedure for the admission of new members of the Club
Yes.
shall be as follows:
Soriano was faced not with one information charging more than one In 1987, Elizagaque was designated as a special non-proprietary member
offense, but with more than one information, each charging a different of the Country Club and his designation was approved by the Country (a) Any proprietary member, seconded by another
offense - violation of DOSRI rules in one, and estafa thru falsification of club’s Board of directors. In 1996, Elizagaque himself filed with CCCI an voting proprietary member, shall submit to the Secretary a
commercial documents in the others. Ilagan, on the other hand, was application for proprietary membership and such application was indorsed written proposal for the admission of a candidate to the "Eligible-
charged with estafa thru falsification of commercial documents in separate by Edmunto Misa (Edmundo). Benito Unchuan, then president of CCCI, for-Membership List";
informations. Thus, petitioners erroneously invoke duplicity of charges as offered to sell Elizagaque a share for only P3.5 million. However,
a ground to quash the Informations. (b) Such proposal shall be posted by the Secretary for
Elizagaque purchased the share of a certain Dr. Butalid for only P3 million.
There are differences between the two (2) offenses. A DOSRI violation Subsequently, a certificate of Proprietary Ownership was issued by CCCI a period of thirty (30) days on the Club bulletin board during
consists in the failure to observe and comply with procedural, reportorial or in favor of Elizagaque. which time any member may interpose objections to the
ceiling requirements prescribed by law in the grant of a loan to a director, admission of the applicant by communicating the same to the
officer, stockholder and other related interests in the bank, i.e. lack of During the meeting of the board of directors of the Country Club, Board of Directors;
written approval of the majority of the directors of the bank and failure to Elizagaque’s application for membership was voted upon. Subsequently,
enter such approval into corporate records and to transmit a copy thereof Elizacague received a letter from Country Club’s corporate secretary Julius (c) After the expiration of the aforesaid thirty (30) days,
to the BSP supervising department. The elements of abuse of confidence, if no objections have been filed or if there are, the Board
Neri that his application for proprietary membership was disapproved.
deceit, fraud or false pretenses, and damage, which are essential to the
Edmundo, in behalf of Elizagaque, wrote a letter for reconsideration to the considers the objections unmeritorious, the candidate shall be
prosecution for estafa, are not elements of a DOSRI violation. The filing of
several charges against Soriano was, therefore, proper. Country Club, and the latter did not answer. Elizagaque wrote another qualified for inclusion in the "Eligible-for-Membership List";
letter to the Country Club, but the latter kept its silence. Elizagaque again
In the criminal cases filed against petitioner, the violation of the DOSRI wrote another letter, but this time inquiring whether any member of the In March 1, 1978, Section 3(c) was amended as follows:
rules were evident. Soriano and Ilagan were officers of RBSM that during Board objected to his application. Again, the Country Club did not reply.
their tenure both were able to indirectly obtain loans without complying with (c) After the expiration of the aforesaid thirty (30) days,
the requisite board approval, reportorial and ceiling requirements. This prompted Elizacaque to file for an action for damages against the the Board may, by unanimous vote of all directors present at
Country Club and its directors with the Regional Trial Court (RTC). The a regular or special meeting, approve the inclusion of the
The crime of estafa filed against them were based on separate provisions candidate in the "Eligible-for-Membership List".
of the Revised penal code namely Article 315 (1)(b) and Article 315 (2) (a) RTC rendered its decision ordering the Country Club and its directors to
. both stating different elements. pay Elizagaque damages. On appeal by the Country Club and its directors,
In the above amended by-laws, the Board adopted a secret balloting
the Court of Appeals affirmed the RTC’s decision but modified the award
Verily, there is no justification for the quashal of the Information filed against known as the "black ball system" of voting wherein each member will drop
of damages.
petitioners. The RTC committed no grave abuse of discretion in denying a ball in the ballot box. A white ball represents conformity to the admission
the motions. Issue: of an applicant, while a black ball means disapproval. When Elizagaque’s

67
Derivative Suit
application for proprietary membership was voted upon during the Board the period starting October 1996 to December 1998. The letter alleged that there must be factual basis; that the records did not disclose any evidence
meeting on the ballot box contained one (1) black ball. Thus, for lack of the redundancy program is a response to the market situation which to show basis for respondent’s termination; that neither did petitioner send
unanimity, his application was disapproved. In accordance with its Articles constrained petitioner to rationalize and simplify its business processes; notice to DOLE one month prior to respondent’s dismissal.
that petitioner undertook a review, restructuring and streamlining of its
of Incorporation, the board of directors of the Country Club has the right to
organization which resulted in consolidation, abolition and outsourcing of The CA ruled that there was no reason to deviate from the findings of the
approve or disapprove an application for proprietary membership. certain functions and in the identification of certain redundant positions. The NLRC since the pieces of evidence presented by Caltex are not only
letter also states that petitioner will provide the DOLE a list of affected insufficient but also baseless and self-serving and Caltex failed to send
However, in rejecting Elizagaque’s application, the directors violated Article employees as it implements each phase of the redundancy program. DOLE a written notice of its implementation of the redundancy program
19 and 21 of the Civil Code, since they committed fraud and evident bad one month prior to the intended date thereof since petitioner had admitted
faith, which is contrary to morals, good customs or public policy. The Caltex, through a letter dated June 30, 1997, notified private respondent of such failure in its Answer to respondent’s appeal to the NLRC.
amendment to Section 3(c) of CCCI’s Amended By-Laws requiring the his termination effective July 31, 1997 due to the redundancy of his position
unanimous vote of the directors present at a special or regular meeting was and awarded him a separation package in the amount of ₱559,458.90. ISSUE:
not printed on the application form that Elizagaque filled up and submitted
On June 8, 1998, Sto Tomas filed with the Labor Arbiter a complaint for Was private respondent illegally dismissed?
to the Country Club. What was printed thereon was the original provision illegal dismissal against petitioner and its President and Chief Executive
of Section 3(c) which was silent on the required number of votes needed Officer, Mr. Clifton Hon, alleging that: being Caltex’s regular employee, he RULING:
for admission of an applicant as a proprietary member. The directors is entitled to security of tenure; he did not commit any serious misconduct,
reasoned why it was not printed because of economic reasons. But the willful disobedience, gross and habitual neglect of duty or fraud and willful Private respondent was dismissed by Caltex on the ground of redundancy,
Court did not believe such reason and said that how can a prestigious breach of trust to warrant the penalty of dismissal from employment; there one of the authorized causes for dismissal under Article 283 of the Labor
country club did not have enough money to cause the printing of an was no independent proof or evidence presented by Caltex to substantiate Code.
its claim of redundancy nor was he afforded due process as he was not
updated application form.
given any opportunity to present his side; he was dismissed due to his Redundancy in an employer’s personnel force necessarily or even
active participation in union activities; petitioner opened positions for hiring ordinarily refers to duplication of work. That no other person was holding
It is thus clear that Elizagaque was left groping in the dark wondering why some of which offered jobs that are the same as what private respondent the same position that private respondent held prior to the termination of
his application was disapproved. He was not even informed that a was performing; Caltex failed to give written notice to him and DOLE at his services, does not show that his position had not become redundant.
unanimous vote of the Board members was required. When he sent a least one month before the intended date of termination as required by the Indeed, in any well organized business enterprise, it would be surprising to
letter for reconsideration and an inquiry whether there was an objection to Labor Code. find duplication of work and two (2) or more people doing the work of one
his application, petitioners apparently ignored him. Certainly, Elizagaque person. We believe that redundancy, for purposes of the Labor Code,
did not deserve this kind of treatment. Having been designated by San Caltex and Mr. Hon averred that private respondent’s dismissal from the exists where the services of an employee are in excess of what is
service was due to redundancy of his position which was determined after reasonably demanded by the actual requirements of the enterprise.
Miguel Corporation as a special non-proprietary member of the Country
petitioner’s business process re-engineering study and organization Succinctly put, a position is redundant where it is superfluous, and
Club, he should have been treated by the Country Club and its directors review, conducted with private respondent’s knowledge; that redundancy superfluity of a position or positions may be the outcome of a number of
with courtesy and civility. At the very least, they should have informed him is an authorized cause to terminate an employee which is a management factors, such as overhiring of workers, decrease in volume of business, or
why his application was disapproved. prerogative and cannot be interfered with absent any abuse of discretion; dropping of a particular product line or service activity previously
and that there is nothing in the law that requires petitioner to conduct manufactured or undertaken by the enterprise.
impartial investigation or hearing to terminate an employee due to
redundancy. We are mindful of the rule that the characterization of an employee’s
81. CALTEX (PHILS.), INC. (now CHEVRON PHILIPPINES, INC.) vs. services as no longer necessary or sustainable, and therefore, properly
NLRC AND ROMEO T. STO. TOMAS The Labor Arbiter rendered a decision dismissing the complaint on the terminable, is an exercise of business judgment on the part of the
G.R. No. 159641 ground that the redundancy was done in good faith and a valid exercise of employer, and that the wisdom or soundness of such characterization or
October 15, 2007 management prerogative; and that redundancy did not deter the employer decision is not subject to discretionary review. However, such
to hire additional workers when it is deemed best for proper management. characterization may be rejected if the same is found to be in violation of
FACTS: While the LA found that Caltex failed to give notice to DOLE one month law or is arbitrary or malicious.
Romeo T. Sto Tomas (private respondent) was a regular employee of before the intended date of private respondent’s termination, the LA ruled
petitioner since February 2, 1984. He was a Senior Accounting Analyst that non-compliance with the procedural requirement will not per se make We have held that the employer must comply with the following requisites
receiving a monthly salary of ₱29,860.00 at the time of his termination on the termination illegal and held that requirement of procedural process was to ensure the validity of the implementation of a redundancy program: 1) a
July 31, 1997. not totally disregarded. written notice served on both the employees and the Department of Labor
and Employment (DOLE) at least one month prior to the intended date of
Caltex informed DOLE of its plan to implement a redundancy program in The NLRC reversed the decision of the LA, expounding that although retrenchment; 2) payment of separation pay equivalent to at least one
its Marketing Division and some departments in its Batangas Refinery for Article 283 of the Labor Code authorizes termination due to redundancy, month pay or at least one month pay for every year of service, whichever
68
Derivative Suit
is higher; 3) good faith in abolishing the redundant positions; and 4) fair and that petitioner failed to show proof of fair and reasonable criteria for the In a letter dated 23 February 1994 ARBC informed TBSS of its
reasonable criteria in ascertaining what positions are to be declared implementation of a valid redundancy program. Thus, whether it is desire to terminate the Service Contracts effective thirty (30) days after
redundant and accordingly abolished. retrenchment or redundancy, or any of the other authorized causes, no receipt of the letter. ARBC through its Vice President for Operations, Mark
employee may be dismissed without observance of the fundamentals of Molina, informed TBSS that it was replacing its security guards with those
In the instant case, we find no reversible error committed by the CA in fair play. of Global Security Investigation Agency (GSIA).
upholding the findings of the NLRC that there was no substantial evidence
presented by Caltex to justify private respondent's dismissal due to Caltex committed a fatal error when it failed to give a written notice to DOLE TBSS informed ARBC that the latter could not preterminate the
redundancy. As correctly found by the CA, petitioner’s evidence to show as required under Article 283 of the Labor Code. All three, the LA, NLRC Service Contracts nor could it post security guards from GSIA as it would
redundancy merely consisted of a copy of petitioner’s letter to the DOLE and the CA, found the absence of notice sent by petitioner to DOLE one run counter to the provisions of their Service Contracts. Nevertheless,
informing the latter of its intention to implement a redundancy program and month before the intended date of private respondent’s termination. While Molina decreased the security guards to only one. ARBC claimed that it
nothing more. The letter which merely stated that petitioner undertook a petitioner claims that it sent a notice to the DOLE through a letter dated
decreased the number of security guards being posted at its
review, restructuring and streamlining of its organization which resulted in June 30, 1997, Caltex failed to show that the same was actually received
consolidation, abolition and outsourcing of certain functions; and which by DOLE. The purpose of the written notice to the DOLE is to give it the establishments to only one (1) as the security guards assigned by TSBB
resulted in identified and redundant positions instead of simplifying its opportunity to ascertain the verity of the alleged authorized cause of were found to be grossly negligent and inefficient because a Mitsubishi
business process restructuring, does not satisfy the requirement of termination. roadgrader of herein defendant was stripped of parts amounting to
substantial evidence, that is, the amount of evidence which a reasonable P58,642.00 and a concrete vibrator and mercury light assembly were
mind might accept as adequate to justify a conclusion. Caltex’s claim that private respondent consented to his termination by stolen from the construction site of the Multipurpose Hall which is worth
accepting his separation pay deserves scant consideration. Private P2800.
Caltex failed to demonstrate the superfluity of private respondent’s position respondent had no other recourse but to accept his separation pay since
as there was nothing in the records that would establish any concrete and petitioner’s letter made it clear that his position had been determined to be Molina allegedly applied P171,853.80 payable to private
real factors recognized by law and relevant jurisprudence, such as redundant and his services shall be terminated effective July 31, 1997. As respondent to the losses suffered by petitioner ARB (ARBC) due to the
overhiring of workers, decreased volume of business, or dropping of a private respondent was dismissed allegedly due to redundancy, he is negligence and indifference of the private respondent's security guards.
particular product line or service activity previously manufactured or entitled to separation pay under Article 283 of the Labor Code. And since
undertaken by the enterprise, which were adopted by petitioner in there was no extra consideration for the private respondent to give up his TBSS thereafter filed a case for breach of contract against ARBC
implementing the redundancy program. employment, such undertaking cannot be allowed to bar the action for and Mark Molina.
illegal dismissal.
Caltex also failed to show any fair and reasonable criteria in ascertaining Issue:
what positions are redundant and how the selection of employees to be
dismissed was made. Is the complaint sufficient to hold Petitioner Molina liable to
82. ATRIUM MANAGEMENT v. CA
Private Respondent in his personal capacity?
Moreover, Caltex failed to refute private respondent’s assertion that it Case is the same as case #87 Ruling:
opened positions of accountants for hiring to which he could have qualified
rather than be dismissed. In petitioner’s Memorandum dated May 28, 1997 Click/Tap here No. A corporation is invested by law with a personality separate
and July 4, 1997, it declared vacant the positions of Terminal Accountant
and distinct from those of the persons composing it as well as from that of
and Internal Auditor, respectively, the minimum requirements of which are
any other legal entity to which it may be related. However, the veil of
being accountants and having 4-5 years experience in handling
accounting and supervisory functions, among others. There is no showing corporate fiction may be pierced when it is used as a shield to further an
83. ARB CONSTRUCTION CO. INC. and MARK MOLINA v. CA end subversive of justice; to defeat public convenience, justify wrong,
that private respondent could not perform the functions demanded of the G.R. No. 126554 May 31, 2000
vacant positions considering his experience as petitioner’s Senior protect fraud, or defend crime.
Accounting Analyst for 13 years and to which he could be transferred Facts:
instead of being dismissed. We find such hiring of accountants inconsistent Petitioner Molina could not be held jointly and severally liable for
with respondent’s termination due to redundancy. TBS Security and Investigation Agency (TBSS) entered into two any obligation which petitioner ARBC may be held accountable for, absent
(2) Service Contracts with ARBC wherein TBSS agreed to provide and any proof of bad faith or malice on his part. Corollarily, it is also incorrect on
There is merit in Caltex’s claim that the CA’s finding "that it (petitioner) failed post security guards in the five (5) establishments being maintained by the part of the Court of Appeals to conclude that there was a sufficient
to provide proof that it truly had an extensive reengineering study on ARBC. The contract shall be effective for a period of one (1) year cause of action against Molina as to make him personally liable for his
account of business losses arising out of massive oil deregulation" is commencing from 15th August 1993 and shall be considered actuations as Vice President for Operations of ARBC. A cursory reading of
misplaced considering that Article 283 of the Labor Code does not require automatically renewed for the same period unless otherwise a written the records of the instant case would reveal that Molina did not summarily
that the employer should be suffering financial losses before he can withhold certain amounts from the payroll of TBSS.
notice of termination shall have been given by one party to the other party
terminate the services of the employee on the ground of redundancy.
Nevertheless, the CA finding on this matter does not detract from the fact thirty (30) days in advance.

69
Derivative Suit
84. RUFINA LUY LIM v. CA of Deeds of Caloocan City issued new transfer certificates of title in the
G.R. No. 124715; January 24, 2000 Moreover, to disregard the separate juridical personality of a corporation, name of Merryland which did not bear a memorandum of the mortgage
the wrong-doing must be clearly and convincingly established. It cannot be liens in favor of Gutierrez.
FACTS: Pastor Y. Lim died intestate. Rufina Luy Lim is the surviving presumed.
spouse. Apparently, Merryland is a corporation in which Francisco was the
85. FRANCISCO v. MEJIA President and majority stockholder. Mejia then sought to nullify the
Auto Truk Corp., Alliance Marketing Corp., Speed Distributing, 14 August 2001 auction sale on the ground that Francisco used the two corporations
Inc., Active Distributing, Inc., and Action Company are corporations under as dummies to defraud the estate of Gutierrez especially so that
Philippine Laws and owned real properties covered under the Torrens Facts: these circumstances are present: 1. Francisco did not inform the lower
system. Their properties were included in the inventory of Pastor’s estate. Andrea Cordova Vda. de Gutierrez (Gutierrez) was the registered owner court that the properties were delinquent in taxes; 2. That there was
They filed a motion to have their properties excluded from the estate. of a parcel of land. Gutierrez and Cardale Financing and Realty notice for an auction sale and Francisco did not inform the Gutierrez
Corporation (Cardale) executed a Deed of Sale with Mortgage for the estate and as such, the estate was not able to perform appropriate
Rufina filed a petition which alleged that Pastor owned the consideration of P800,000.00. Upon the execution of the deed, Cardale acts to remedy the same; 3. That without knowledge of the auction,
subject properties, that even though the respondent corporations present paid Gutierrez P171,000.00. It was agreed that the balance of the Gutierrez estate cannot exercise their right of redemption; 4. That
themselves as corporations, all their capital, assets and equity were P629,000.00 would be paid in several installments within five years from Francisco failed to inform the court that the highest bidder in the
personally owned by the Pastor Lim, and that they were listed therein only the date of the deed, at an interest of nine percent per annum “based on auction sale was Merryland, her other company; 5. That thereafter,
for purposes of registration with the SEC. the successive unpaid principal balances.” Thereafter, the titles of Cardale was dissolved and the subject properties were divided and
Gutierrez were cancelled and in lieu thereof TCTs were issued in favor of sold to other people.
The probate court denied the motion for exclusion citing the case Cardale. To secure payment of the balance of the purchase price, Cardale
of Cease v. Court of Appeals which ruled that the corporation is a mere constituted a mortgage on three of the four parcels of land. The Trial court rendered a decision in favor of the defendants, dismissing the
extension of the decedent in that case and that therefore the assets of the encumbrance was annotated upon the certificates of title and the owner’s complaint for damages filed by Mejia.
corporation are also assets of the estate. duplicate certificates. The owner’s duplicates were retained by Gutierrez.
The Court of Appeals held that the corporate veil of Cardale and Merryland
ISSUE: Can the properties of the respondent corporations be included in Owing to Cardale’s failure to settle its mortgage obligation, Gutierrez filed must be pierced in order to hold Francisco and Merryland solidarily liable
the estate of Pastor Y. Lim? a complaint for rescission of the contract and during the pendency of the since these two corporations were used as dummies by Francisco, who
rescission case, Gutierrez died and was substituted by her executrix, Rita employed fraud in allowing Cardale to default on the realty taxes for the
HELD: No. C. Mejia (Mejia). In 1971, plaintiff’s presentation of evidence was properties mortgaged to Gutierrez so that Merryland could acquire the
terminated. However, Cardale, which was represented by petitioner Adalia same free from all liens and encumbrances in the tax delinquency sale
It is settled that a corporation is clothed with personality separate B. Francisco (Francisco) in her capacity as Vice-President and Treasurer and, as a consequence thereof, frustrating Gutierrez’s rights as a
and distinct from that of the persons composing it. It may not generally be of Cardale, lost interest in proceeding with the presentation of its evidence mortgagee over the subject properties.
held liable for that of the persons composing it. It may not be held liable for and the case lapsed into inactive status for a period of about fourteen
the personal indebtedness of its stockholders or those of the entities years. Issue: Did Francisco acted in bad faith in her dealings?
connected with it.
In the meantime, the mortgaged parcels of land became delinquent in the Held: Yes. The Court, after an assiduous study of this case, is convinced
Piercing the veil of corporate entity requires the court to see payment of real estate taxes which culminated in their levy and auction sale that the totality of the circumstances appertaining conduce to the inevitable
through the protective shroud which exempts its stockholders from in satisfaction of the tax arrears. The highest bidder for the three parcels of conclusion that Francisco acted in bad faith.
liabilities that ordinarily, they could be subject to, or distinguishes one land was Merryland Development Corporation (Merryland), whose
corporation from a seemingly separate one, were it not for the existing President and majority stockholder is Francisco. A memorandum based To reiterate, notwithstanding receipt of the aforesaid notices, appellee
corporate fiction. upon the certificate of sale was then made upon the original copies of the Francisco did not inform the Estate of Gutierrez or her executrix about the
TCTs. Before the expiration of the one year redemption period, Mejia filed tax delinquencies and of the impending auction sale of the said properties.
The corporate mask may be lifted and the corporate veil may be a Motion for Decision with the trial court. The hearing of said motion was Even a modicum of good faith and fair play should have encouraged
pierced when a corporation is just but the alter ego of a person or of another deferred due to a Motion for Postponement filed by Cardale through Francisco to at least advise Gutierrez’s Estate through her executrix and
corporation. Where badges of fraud exist, where public convenience is Francisco, who signed the motion in her capacity as “officer-in-charge,” the trial court which was hearing the complaint for rescission and recovery
defeated; where a wrong is sought to be justified thereby, the corporate claiming that Cardale needed time to hire new counsel. However, of said properties of such fact, so that the Estate of Gutierrez, which had a
fiction or the notion of legal entity should come to naught. Francisco did not mention the tax delinquencies and sale in favor of real interest on the properties and could at least take steps to forestall the
Merryland. Subsequently, the redemption period expired and Merryland, auction sale and thereby preserve the properties and protect its interests
Mere ownership by a single stockholder or by another acting through Francisco, filed petitions for consolidation of title, which thereon. And not only did Francisco allow the auction sale to take place,
corporation of all or nearly all of the capital stock of a corporation is not of culminated in the issuance of certain orders and the issuance of new but she used her other corporation (Merryland) in participating in the
itself a sufficient reason for disregarding the fiction of separate corporate transfer certificates of title “free from any encumbrance or third-party claim auction sale and in acquiring the very properties which her first corporation
personalities. whatsoever” in favor of Merryland. Pursuant to such orders, the Register (Cardale) had mortgaged to Gutierrez. Again, Francisco did not thereafter
70
Derivative Suit
inform the Estate of Gutierrez or its executrix about the auction sale, thus construction materials and other merchandise purchased by Marinduque
precluding the Estate from exercising its right of redemption. Francisco’s The only act imputable to Merryland in relation to the mortgaged properties Mining, as well as interest, attorneys fees and the costs of suit.
deception is further shown by her concealment of the tax delinquency sale is that it purchased the same and this by itself is not a fraudulent or wrongful
of the properties from the estate or its executrix, thus preventing the latter act. No evidence has been adduced to establish that Merryland was a Remington filed an amended complaint including the Maricalum Mining
from availing of the right of redemption of said properties. mere alter ego or business conduit of Francisco. Time and again it has Corporation (Maricalum Mining) and Island Cement Corporation (Island
been reiterated that mere ownership by a single stockholder or by another Cement) as co-defendants. Remington asserted that Marinduque Mining,
It is dicta in corporation law that a corporation is a juridical person with a corporation of all or nearly all of the capital stock of a corporation is not of PNB, DBP, Nonoc Mining, Maricalum Mining and Island Cement must be
separate and distinct personality from that of the stockholders or members itself sufficient ground for disregarding the separate corporate personality. treated in law as one and the same entity by disregarding the veil of
who compose it. However, when the legal fiction of the separate corporate Neither has it been alleged or proven that Merryland is so organized and corporate fiction since:
personality is abused, such as controlled and its affairs are so conducted as to make it merely an
when the same is used for fraudulent or wrongful ends, the courts have not instrumentality, agency, conduit or adjunct of Cardale. Even assuming that 1. Co-defendants NMIC, Maricalum and Island Cement which are newly
hesitated to pierce the corporate veil. If any general rule can be laid down, the businesses of Cardale and Merryland are interrelated, this alone is not created entities are practically owned wholly by defendants PNB and DBP,
in the present state of authority, it is that a corporation will be looked upon justification for disregarding their separate personalities, absent any and managed by their officers, aside from the fact that the aforesaid co-
as a legal entity as a general rule, and until sufficient reason to the contrary showing that Merryland was purposely used as a shield to defraud defendants NMIC, Maricalum and Island Cement were organized in such
appears; but, when the notion of legal entity is used to defeat public creditors and third persons of their rights. Thus, Merryland’s separate a hurry and in such suspicious circumstances by co-defendants PNB and
convenience, justify wrong, protect fraud, or defend crime, the law will juridical personality must be upheld. DBP after the supposed extra-judicial foreclosure of MMICs assets as to
regard the corporation as an association of persons. make their supposed projects assets, machineries and equipment which
were originally owned by co-defendant MMIC beyond the reach of
With specific regard to corporate officers, the general rule is that the officer 86. DBP v. CA creditors of the latter.
cannot be held personally liable with the corporation, whether civilly or 16 August 2001
otherwise, for the consequences of his acts, if he acted for and in behalf of 2. The personnel, key officers and rank-and-file workers and employees of
the corporation, within the scope of his authority and in good faith. In such Facts: co-defendants NMIC, Maricalum and Island Cement creations of co-
cases, the officer’s acts are properly attributed to the corporation. However, Marinduque Mining Industrial Corporation (Marinduque Mining), a defendants PNB and DBP were the personnel of co-defendant MMIC such
if it is proven that the officer has used the corporate fiction to defraud a third corporation engaged in the manufacture of pure and refined nickel, nickel that x x x practically there has only been a change of name for all legal
party, or that he has acted negligently, maliciously or in bad faith, then the and cobalt in mixed sulfides, copper ore/concentrates, cement and pyrite purpose and intents.
corporate veil shall be lifted and he shall be held personally liable for the conc., obtained from the Philippine National Bank (PNB) various loan
particular corporate obligation involved. accommodations. To secure the loans, Marinduque Mining executed on
October 9, 1978 a Deed of Real Estate Mortgage and Chattel Mortgage in 3. The places of business not to mention the mining claims and project
The totality of Franciso’s actions clearly betray an intention to conceal the favor of PNB. premises of co-defendants NMIC, Maricalum and Island Cement likewise
tax delinquencies, levy and public auction of the subject properties from the On July 13, 1981, Marinduque Mining executed in favor of PNB and the used to be the places of business, mining claims and project premises of
estate of Gutierrez and the trial court until after the expiration of the Development Bank of the Philippines (DBP) a second Mortgage Trust co-defendant MMIC as to make the aforesaid co-defendants NMIC,
redemption period when the remotest possibility for the recovery of the Agreement. In said agreement, Marinduque Mining mortgaged to PNB Maricalum and Island Cement mere adjuncts and subsidiaries of co-
properties would be extinguished. Consequently, Francisco had effectively and DBP. defendants PNB and DBP, and subject to their control and management.
deprived the estate of Gutierrez of its rights as mortgagee over the three The mortgage also covered all of Marinduque Minings chattels, as well as
parcels of land which were sold to Cardale. If Francisco was acting in good assets of whatever kind, nature and description which Marinduque Mining On top of everything, co-defendants PNB, DBP NMIC, Maricalum and
faith, then she should have disclosed the status of the mortgaged may subsequently acquire in substitution or replenishment or in addition to Island Cement being all corporations created by the government in the
properties to the trial court - especially after Mejia had filed a Motion for the properties covered by the previous Deed of Real and Chattel pursuit of business ventures should not be allowed to ignore, x x x or
Decision, in response to which she filed a motion for postponement Mortgage. obliterate with impunity nay illegally, the financial obligations of x x x MMIC
wherein she could easily have mentioned the tax sale - since this action For failure of Marinduque Mining to settle its loan obligations, PNB and whose operations co-defendants PNB and DBP had highly financed
directly affected such properties which were the subject of both the sale DBP instituted sometime on July and August 1984 extrajudicial foreclosure before the alleged extrajudicial foreclosure of defendant MMICs assets,
and mortgage. proceedings over the mortgaged properties.The public auction sale machineries and equipment to the extent that major policies of co-
conducted on September 18, 1984 on the foreclosed personal properties defendant MMIC were being decided upon by co-defendants PNB and
That Merryland acquired the property at the public auction only serves to of MMIC, the same were sold to PNB and DBP as the highest bidder in the DBP as major financiers who were represented in its board of directors
shed more light upon Francisco’s fraudulent purposes. Based on the sum of P678,772,000.00 forming part of the majority thereof which through the alleged extrajudicial
findings of the Court of Appeals, Francisco is the controlling stockholder In the meantime, between July 16, 1982 to October 4, 1983, Marinduque foreclosure culminated in a complete take-over by co-defendants PNB and
and President of Merryland. Thus, aside from the instrumental role she Mining purchased and caused to be delivered construction materials and DBP bringing about the organization of their co-defendants NMIC,
played as an officer of Cardale, in evading that corporation’s legitimate other merchandise from Remington Industrial Sales Corporation Maricalum and Island Cement to which were transferred all the assets,
obligations to Gutierrez, it appears that Francisco’s actions were also (Remington) worth P921,755.95. The purchases remained unpaid as of machineries and pieces of equipment of co-defendant MMIC used in its
oriented towards securing advantages for another corporation in which she August 1, 1984 when Remington filed a complaint for a sum of money and nickel mining project in Surigao del Norte, copper mining operation in
had a substantial interest. damages against Marinduque Mining for the value of the unpaid Sipalay, Negros Occidental and cement factory in Antipolo, Rizal to the
71
Derivative Suit
prejudice of creditors of co-defendant MMIC such as plaintiff Remington than 20% of the total outstanding obligation. Section 1 of Presidential indorsers, was questioned by other creditors of the corporation, they
Industrial Sales Corporation whose stockholders, officers and rank-and-file Decree No. 385 (The Law on Mandatory Foreclosure) provides: should have been classed as instruments rendered void by the legal
workers in the legitimate pursuit of its business activities, invested principle which prevents directors of an insolvent corporation from giving
considerable time, sweat and private money to supply, among others, co- It shall be mandatory for government financial institutions, after the lapse themselves a preference over outside creditors. x x x (page 106-107 of the
defendant MMIC with some of its vital needs for its operation, which co- of sixty (60) days from the issuance of this decree, to foreclose the collateral Appellees Brief.)
defendant MMIC during the time of the transactions material to this case and/or securities for any loan, credit accommodation, and/or guarantees
became x x x co-defendants PNB and DBPs instrumentality, business granted by them whenever the arrearages on such account, including The second principle invoked by respondent court involves directors
conduit, alter ego, agency (sic), subsidiary or auxiliary corporation, by virtue accrued interest and other charges, amount to at least twenty percent who are creditors which is also inapplicable herein. Here, the creditor of
of which it becomes doubly necessary to disregard the corporation fiction (20%) of the total outstanding obligations, including interest and other Marinduque Mining is DBP, not the directors of Marinduque Mining.
that co-defendants PNB, DBP, MMIC, NMIC, Maricalum and Island charges, as appearing in the books of account and/or related records of
Cement, six (6) distinct and separate entities, when in fact and in law, they the financial institution concerned. This shall be without prejudice to the Neither do we discern any bad faith on the part of DBP by its creation
should be treated as one and the same at least as far as plaintiffs exercise by the government financial institution of such rights and/or of Nonoc Mining, Maricalum and Island Cement. As Remington itself
transactions with co-defendant MMIC are concerned, so as not to defeat remedies available to them under their respective contracts with their concedes, DBP is not authorized by its charter to engage in the mining
public convenience, justify wrong, subvert justice, protect fraud or confuse debtors, including the right to foreclose on loans, credits, accomodations business. The creation of the three corporations was necessary to manage
legitimate issues involving creditors such as plaintiff, a fact which all and/or guarantees on which the arrearages are less than twenty (20%) and operate the assets acquired in the foreclosure sale lest they deteriorate
defendants were as (sic) still are aware of during all the time material to the percent. from non-use and lose their value. In the absence of any entity willing to
transactions subject of this case. purchase these assets from the bank, what else would it do with these
properties in the meantime? Sound business practice required that they be
Thus, PNB and DBP did not only have a right, but the duty under said law, utilized for the purposes for which they were intended.
The Regional Trial Court (RTC) rendered a decision in favor of Remington. to foreclose upon the subject properties. The banks had no choice but to
Upon appeal by PNB, DBP, Nonoc Mining, Maricalum Mining, Island obey the statutory command.
Cement and APT, the Court of Appeals affirmed the decision of the
RTC. Petitioner filed a Motion for Reconsideration, which was denied in the
As aptly stated by the appellee in its brief, x x x where the corporations 87. ATRIUM MANAGEMENT CORPORATION v. COURT OF
Resolution.
have directors and officers in common, there may be circumstances under APPEALS
Hence, this petition, DBP maintaining that Remington has no cause which their interest as officers in one company may disqualify them in G.R. No. 109491. February 28, 2001 (PARDO, J.:)
of action against it or PNB, nor against their transferees, Nonoc Mining, equity from representing both corporations in transactions between the
Island Cement, Maricalum Mining, and the APT. two. Thus, where one corporation was insolvent and indebted to another, DOCTRINE:
it has been held that the directors of the creditor corporation were
On the other hand, private respondent Remington submits that the “Personal liability of a corporate director, trustee or officer along
disqualified, by reason of self-interest, from acting as directors of the debtor
transfer of the properties was made in fraud of creditors. The presence of (although not necessarily) with the corporation may so validly attach, as a
corporation in the authorization of a mortgage or deed of trust to the former
fraud, according to Remington, warrants the piercing of the corporate veil rule, only when: 1. He assents (a) to a patently unlawful act of the
to secure such indebtedness x x x (page 105 of the Appellees Brief). In the
such that Marinduque Mining and its transferees could be considered as corporation, or (b) for bad faith or gross negligence in directing its affairs, or
same manner that x x x when the corporation is insolvent, its directors who
one and the same corporation. The transferees, therefore, are also liable (c) for conflict of interest, resulting in damages to the corporation, its
are its creditors cannot secure to themselves any advantage or preference
for the value of Marinduque Minings purchases. stockholders or other persons; 2. He consents to the issuance of watered
over other creditors. They cannot thus take advantage of their fiduciary
down stocks or who, having knowledge thereof, does not forthwith file with
relation and deal directly with themselves, to the injury of others in equal
the corporate secretary his written objection thereto; 3. He agrees to hold
Issue: right. If they do, equity will set aside the transaction at the suit of creditors
himself personally and solidarily liable with the corporation; or 4. He is
Whether or not there is fraud on the part of Marinduque Mining to warrant of the corporation or their representatives, without reference to the question
made, by a specific provision of law, to personally answer for his corporate
the piercing of the corporate veil. of any actual fraudulent intent on the part of the directors, for the right of the
action.”
creditors does not depend upon fraud in fact, but upon the violation of the
Ruling: fiduciary relation to the directors. xxx. (page 106 of the Appellees Brief.)
No. FACTS:
We also concede that x x x directors of insolvent corporation, who are The case arose when Hi-Cement Corporation through its
This Court has disregarded the separate personality of the corporation creditors of the company, can not secure to themselves any preference or corporate signatories, petitioner Lourdes M. de Leon, treasurer, and the
where the corporate entity was used to escape liability to third parties. In advantage over other creditors in the payment of their claims. It is not good late Antonio de las Alas, Chairman, issued checks in favor of E.T. Henry
this case, however, we do not find any fraud on the part of Marinduque morals or good law. The governing body of officers thereof are charged and Co. Inc., as payee. E.T. Henry and Co., Inc., in turn, endorsed four
Mining and its transferees to warrant the piercing of the corporate veil. with the duty of conducting its affairs strictly in the interest of its existing checks amounting to 2 million pesos to petitioner Atrium Management
creditors, and it would be a breach of such trust for them to undertake to Corporation for valuable consideration. Upon presentment for payment,
It bears stressing that PNB and DBP are mandated to foreclose on the
give any one of its members any advantage over any other creditors in the drawee bank dishonored all four checks for the common reason
mortgage when the past due account had incurred arrearages of more
securing the payment of his debts in preference to all others. When validity payment stopped. Atrium then filed an action for collection of the proceeds
of these mortgages, to secure debts upon which the directors were of four postdated checks in the total amount of P2 million.
72
Derivative Suit
as treasurer and Chairman of Hi-Cement were authorized to issue the and personal hand in giving "bribe money" he was dismissed. He then
According to Atrium, through his witness Carlos C. Syquia that checks. However, Ms. de Leon was negligent when she signed the implicated AHS
Enrique Tan of E.T. Henry approached Atrium for financial assistance, confirmation letter requested by Mr. Yap of Atrium and Mr. Henry of E.T. President Gervacio Amistoso and Vice President Constancio Halili as
offering to discount four RCBC checks in the total amount of P2 million, Henry for the rediscounting of the crossed checks issued in favor of E.T. responsible for
issued by Hi-Cement in favor of E.T. Henry. Atrium agreed to discount the Henry. She was aware that the checks were strictly endorsed for deposit his illegal dismissal.
checks, provided it be allowed to confirm with Hi-Cement the fact that the only to the payees account and not to be further negotiated. What is more,
checks represented payment for petroleum products which E.T. Henry the confirmation letter contained a clause that was not true, that is, that the
delivered to Hi-Cement. Respondent Hi-Cement presented as witness Ms. checks issued to E.T. Henry were in payment of Hydro oil bought by Hi- ISSUE:
Erlinda Yap, She testified that E.T. Henry offered to give Hi-Cement a loan Cement from E.T. Henry. Her negligence resulted in damage to the Whether or not Amistoso and Halili be held solidarily liable with the
which the subject checks would secure as collateral. corporation. Hence, Ms. de Leon may be held personally liable therefor. corporation
the trial court rendered a decision ordering Lourdes M. de Leon, However, the issuance of checks is not a ultra vires act.
her husband Rafael de Leon, E.T. Henry and Co., Inc. and Hi-Cement There is basis to rule that the act of issuing the checks was well within the RULING:
Corporation to pay petitioner Atrium, jointly and severally, the amount of P2 ambit of a valid corporate act, for it was for securing a loan to finance the NO.
million corresponding to the value of the four checks. The CA absolved Hi- activities of the corporation, hence, not an ultra vires act. An ultra vires act
Corporate officers are not personally liable for money claims of discharged
Cement Corporation from liability and dismissing the complaint as against is one committed outside the object for which a corporation is created as
it. The appellate court ruled that: (1) Lourdes M. de Leon was not defined by the law of its organization and therefore beyond the power corporate employees unless they acted with evident malice and bad faith
authorized to issue the subject checks in favor of E.T. Henry, Inc.; (2) The conferred upon it by law. The term ultra vires is distinguished from an illegal in
issuance of the subject checks by Lourdes M. de Leon and the late Antonio act for the former is merely voidable which may be enforced by terminating their employment. In the case at bar, while petitioners Amistoso
de las Alas constituted ultra vires acts; and (3) The subject checks were performance, ratification, or estoppel, while the latter is void and cannot be and
not issued for valuable consideration. validated. Halili may have had a hand in the relief of respondent. Bayani, there are no
indications of malice and bad faith on their part. We take exception to the
ISSUE: conclusion
88. AMERICAN HOSPITAL SUPPLIES/PHILIPPINES et al. vs. of respondent Court of Appeals that "the manner by which Halili and
Is Lourdes as a signatory of the checks is personally liable for the COURT OF APPEALS, ALFONSO BAYANI Amistoso acted is
value of the checks, which were declared to be issued without GR 111807 14 June 1996 characterized by bad faith and malice, thus binding them personally liable
consideration? FACTS: to plaintiff appellee,'' On the contrary it is apparent that the relief order was
American Hospital Supplies was engaged in the sale and manufacture of a business judgment
Is the issuance of the questioned checks an ultra vires act? medicines and pharmaceuticals in the country and did substantial business on the part of the officers, with the best interest of the corporation in mind,
with based on
RULING: government hospitals. On 1 June 1970 it hired Alfonso Bayani as an Area their opinion that respondent Bayani had failed to perform the duties
Manager for expected of him.
Yes. Lourdes is personally liable.
Visayas and Mindanao, and later appointed him Manager of its Cebu Hence both the trial court and respondent Court of Appeals committed a
“Personal liability of a corporate director, trustee or officer along branch. On 30 reversible
(although not necessarily) with the corporation may so validly attach, as a January 1978 private respondent was dismissed from the service. At that error in holding petitioners Amistoso and Halili jointly and solidarily liable
rule, only when: 1. He assents (a) to a patently unlawful act of the time he was with
corporation, or (b) for bad faith or gross negligence in directing its affairs, or receiving a monthly compensation of P3,180.00. Petitioner Corporation.
(c) for conflict of interest, resulting in damages to the corporation, its On 5 May 1978 private respondent filed a complaint for damages before
stockholders or other persons; 2. He consents to the issuance of watered the 89. COMPLEX ELECTRONICS v. NLRC
down stocks or who, having knowledge thereof, does not forthwith file with trial court alleging that in the course of their business petitioners were COMPLEX ELECTRONICS EMPLOYEES ASSOCIATION (CEEA)
the corporate secretary his written objection thereto; 3. He agrees to hold directly represented by its union president CECILIA TALAVERA,
himself personally and solidarily liable with the corporation; or 4. He is encouraging, abetting and promoting bribery in the guise of "commissions," GEORGE ARSOLA, MARIO DIAGO AND SOCORRO
made, by a specific provision of law, to personally answer for his corporate "entertainment expenses" and "representation expenses" which were BONCAYAO, petitioners, vs. THE NATIONAL LABOR
action.” given to various RELATIONS COMMISSION, COMPLEX ELECTRONICS
government hospital officials in exchange for favorable recommendations, CORPORATION, IONICS CIRCUIT, INC., LAWRENCE QUA,
Lourdes M. de Leon is the treasurer of the corporation and is REMEDIOS DE JESUS, MANUEL GONZAGA, ROMY DELA
approvals
authorized to sign checks for the corporation. At the time of the issuance of ROSA, TERESITA ANDINO, ARMAN
the checks, there were sufficient funds in the bank to cover payment of the and actual purchases of medicines and pharmaceuticals. For his refusal to
take direct CABACUNGAN,GERRY GABANA, EUSEBIA MARANAN
amount of P2 million pesos. Lourdes M. de Leon and Antonio de las Alas and BERNADETH GACAD, respondents.July 19, 1999]
73
Derivative Suit
COMPLEX ELECTRONICS CORPORATION, petitioner, On November 10, 1997, the Union presented additional
vs. NATIONAL LABOR RELATIONS COMMISSION, Ionics contended that it was an entity separate and distinct from documentary evidence which consisted of a newspaper clipping in the
COMPLEX ELECTRONICS EMPLOYEES ASSOCIATION Complex and had been in existence since July 5, 1984 or eight (8) years Manila Bulletin, dated August 18, 1997 bearing the picture of Lawrence
(CEEA), represented by Union President, CECILIA before the labor dispute arose at Complex. Like Complex, it was also Qua with the following inscription:
TALAVERA, respondents. engaged in the semi-conductor business where the machinery, equipment
[G.R. No. 121315. July 19, 1999] and materials were consigned to them by their customers. While admitting RECERTIFICATION. The Cabuyao (Laguna) operation of Ionic Circuits,
that Lawrence Qua, the President of Complex was also the President of Inc. consisting of plants 2, 3, 4 and 5 was recertified to ISO 9002 as
FACTS: Ionics, the latter denied having Qua as their owner since he had no electronics contract manufacturer by the TUV, a rating firm with
Complex Electronics Corporation (Complex) was engaged in recorded subscription of P1,200,000.00 in Ionics as claimed by the Union. headquarters in Munich, Germany. Lawrence Qua, Ionics president and
the manufacture of electronic products. It was actually a subcontractor of chief executive officer, holds the plaque of recertification presented by
electronic products where its customers gave their job orders, sent their On April 30, 1993, the Labor Arbiter rendered a decision the Gunther Theisz (3rd from left), regional manager of TUV Products Services
own materials and consigned their equipment to it. The customers were dispositive portion of which reads: Asia during ceremonies held at Sta. Elena Golf Club. This is the first of its
foreign-based companies with different product lines and specifications kind in the country that four plants were certified at the same time.[12]
requiring the employment of workers with specific skills for each product WHEREFORE, all the foregoing premises being considered,
line. Thus, there was the AMS Line for the Adaptive Micro System, Inc., judgment is hereby rendered ordering the respondent Complex The Union claimed that the said clipping showed that both
the Heril Line for Heril Co., Ltd., the Lite-On Line for the Lite-On Philippines Electronics Corporation and/or Ionics Circuit Incorporated and/or corporations, Ionics and Complex are one and the same.
Electronics Co., etc Lawrence Qua, to reinstate the 531 above-listed employees to their former
position with all the rights, privileges and benefits appertaining thereto, and In answer to this allegation, Ionics explained that the photo which
The rank and file workers of Complex were organized into a to pay said complainants-employees the aggregate backwages appeared at the Manila Bulletin issue of August 18, 1997 pertained only to
union known as the Complex Electronics Employees Association, herein amounting P26,949,891.80 as of April 6, 1993 and to such further respondent Ionics recertification of ISO 9002. There was no mention about
referred to as the Union. backwages until their actual reinstatement. In the event reinstatement is no Complex Electronics Corporation. Ionics claimed that a mere photo is
longer feasible for reasons not attributable to the complainants, said insufficient to conclude that Ionics and Complex are one and the same.
On March 4, 1992, Complex received a facsimile message from respondents are also liable to pay complainants-employees their
Lite-On Philippines Electronics Co., requiring it to lower its price by 10%. separation pay to be computed at the rate of one (1) month pay for every ISSUE:
year of service, a fraction of at least six (6) months to be considered as one 1. Whether or not Ionics was merely a runaway shop.
Consequently, on March 9, 1992, a meeting was held between whole year.
Complex and the personnel of the Lite-On Production Line. Complex 2. Whether or not Lawrence Qua is personally Liable to the union.
informed its Lite-On personnel that such request of lowering their selling Separate appeals were filed by Complex, Ionics and Lawrence Qua
price by 10% was not feasible as they were already incurring losses at the before the respondent NLRC which rendered the questioned decision on
present prices of their products. Under such circumstances, Complex March 10, 1995, the decretal portion of which states: RULING:
regretfully informed the employees that it was left with no alternative but to 1. The Union's contentions are untenable.
close down the operations of the Lite-On Line. WHEREFORE, premises considered, the assailed decision is hereby
ordered vacated and set aside, and a new one entered ordering A runaway shop is defined as an industrial plant moved by its owners
In the evening of April 6, 1992, the machinery, equipment and respondent Complex Electronics Corporation to pay 531 complainants from one location to another to escape union labor regulations or state
materials being used for production at Complex were pulled-out from the equivalent to one month pay in lieu of notice and separation pay equivalent laws, but the term is also used to describe a plant removed to a new
company premises and transferred to the premises of Ionics Circuit, Inc. to one month pay for every year of service and a fraction of six months location in order to discriminate against employees at the old plant because
(Ionics) at Cabuyao, Laguna. The following day, a total closure of company considered as one whole year. of their union activities.[14] It is one wherein the employer moves its
operation was effected at Complex. business to another location or it temporarily closes its business for anti-
Respondents Ionics Circuit Incorporated and Lawrence Qua are hereby union purposes.[15] A runaway shop in this sense, is a relocation motivated
A complaint was, thereafter, filed with the Labor Arbitration ordered excluded as parties solidarily liable with Complex Electronics by anti-union animus rather than for business reasons. In this case,
Branch of the NLRC for unfair labor practice, illegal closure/illegal lockout, Corporation however, Ionics was not set up merely for the purpose of transferring the
money claims for vacation leave, sick leave, unpaid wages, 13th month business of Complex. At the time the labor dispute arose at Complex,
pay, damages and attorney's fees. The Union alleged that the pull-out of Complex, Ionics and the Union filed their motions for reconsideration of the Ionics was already existing as an independent company. As earlier
the machinery, equipment and materials from the company premises, above decision which were denied by the respondent NLRC. mentioned, it has been in existence since July 5, 1984. It cannot, therefore,
which resulted to the sudden closure of the company was in violation of be said that the temporary closure in Complex and its subsequent transfer
Section 3 and 8, Rule XIII, Book V of the Labor Code of the On December 23, 1996, the Union filed a motion for consolidation of of business to Ionics was for anti-union purposes. The Union failed to show
Philippines[4] and the existing CBA. Ionics was impleaded as a party G.R. No. 122136 with G.R. No. 121315.[10] The motion was granted by this that the primary reason for the closure of the establishment was due to the
defendant because the officers and management personnel of Complex Court in a Resolution dated June 23, 1997.[11] union activities of the employees.
were also holding office at Ionics with Lawrence Qua as the President of
both companies.
74
Derivative Suit
Mere ownership by a single stockholder or by another corporation of all or
nearly all of the capital stock of a corporation is not of itself sufficient ground Petitioner Jose averred that it is not Santos who is the accommodation
for disregarding the separate corporate personality. party to the instrument but the corporation itself. But assuming arguendo
90. ERNESTINA CRISOLOGO-JOSE v. CA that the corporation is the accommodation party, it cannot be held liable to
Ionics may be engaged in the same business as that of Complex, G.R. No. 80599 the check issued in favor of petitioner. The provision of the Negotiable
but this fact alone is not enough reason to pierce the veil of corporate fiction September 15, 1989 Instruments Law which holds an accommodation party liable on the
of the corporation. Well-settled is the rule that a corporation has a instrument to a holder for value, although such holder at the time of taking
personality separate and distinct from that of its officers and FACTS: the instrument knew him to be only an accommodation party, does not
stockholders. This fiction of corporate entity can only be disregarded in include nor apply to corporations which are accommodation parties. This
certain cases such as when it is used to defeat public convenience, justify is because the issue or indorsement of another is ultra vires. Hence, one
Respondent Ricardo S. Santos, Jr. was the vice-president of Mover
wrong, protect fraud, or defend crime.[19] To disregard said separate who has taken the instrument with knowledge of the accommodation
Enterprises, Inc. in-charge of marketing and sales; and the president of the
juridical personality of a corporation, the wrongdoing must be clearly and nature thereof cannot recover against a corporation where it is only an
said corporation was Atty. Oscar Z. Benares. Atty. Benares, in
convincingly established. accommodation party. If the form of the instrument, or the nature of the
accommodation of his clients, the spouses Jaime and Clarita Ong, issued
transaction, is such as to charge the indorsee with the knowledge that the
Check drawn against Traders Royal Bank, in the amount of P45,000.00
we agree with respondent Ionics that the photo/newspaper clipping issue or indorsement of the instrument by the corporation is for the
payable to defendant Ernestina Crisologo-Jose. Since at that time, the
itself does not prove that Ionics and Complex are one and the same accommodation of another, he cannot recover against the corporation
treasurer of Mover Enterprises was not available, Atty. Benares prevailed
entity. The photo/newspaper clipping merely showed that some plants of thereon.
upon Ricardo S. Santos, Jr., to sign the aforesaid check.
Ionics were recertified to ISO 9002 and does not show that there is a
relation between Complex and Ionics except for the fact that Lawrence By way of exception, an officer or agent of a corporation shall have the
Qua was also the president of Ionics. The check was issued to petitioner Ernestina Crisologo-Jose in power to execute or indorse a negotiable paper in the name of the
consideration of the waiver or quitclaim by said petitioner over a certain corporation for the accommodation of a third party only is specifically
2. Going now to the issue of personal liability of Lawrence Qua, it is property which the Government Service Insurance System (GSIS) agreed authorized to do so. Corollarily, corporate officers have no power to
settled that in the absence of malice or bad faith, a stockholder or an officer to sell to the clients of Atty. Oscar Benares, the spouses Jaime and Clarita execute for mere accommodation a negotiable instrument of the
of a corporation cannot be made personally liable for corporate Ong, with the understanding that upon approval by the GSIS of the corporation for their individual debts and transactions arising from or in
liabilities.[25] In the present case, while it may be true that the equipment, compromise agreement with the spouses Ong, the check will be encashed relation to matters in which the corporation has no legitimate concern.
materials and machinery were pulled-out of Complex and transferred to accordingly. However, since the compromise agreement was not Since such accommodation paper cannot be enforced against the
Ionics during the night, their action was sufficiently explained by Lawrence approved within the expected period of time, the aforesaid check for corporation, the signatories thereof shall be personally liable therefore, as
Qua in his Comment to the petition filed by the Union. We quote: P45,000.00 was replaced by Atty. Benares with another Traders Royal well as the consequences arising from their acts in connection therewith.
Bank check in the same amount of P45,000.00 also payable to Jose. This
The fact that the pull-out of the machinery, equipment and materials was replacement check was also signed by Atty. Oscar Z. Benares and by the
effected during nighttime is not per se an indicia of bad faith on the part of Ricardo S. Santos, Jr. When Jose deposited this replacement, it was
respondent Qua since he had no other recourse, and the same was dishonored for insufficiency of funds. Hence, petitioner Jose filed a criminal 91. FCY CONSTRUCTION and FRANCIS YU v. CA and LEY
dictated by the prevailing mood of unrest as the laborers were already complaint for violation of Batas Pambansa Blg. 22 against Atty. Oscar Z. CONSTRUCTION
vandalizing the equipment, bent on picketing the company premises and Benares and Ricardo S. Santos, Jr. G.R. No. 123358
threats to lock out the company officers were being made. Such acts of February 1, 2000
respondent Qua were, in fact, made pursuant to the demands of Meanwhile, during the preliminary investigation, Santos tried to tender a
Complex's customers who were already alarmed by the pending labor cashier’s check for the value of the dishonored check but petitioner refused FACTS:
dispute and imminent strike to be stage by the laborers, to have their to accept such. This was consigned by Santos with the clerk of court and
equipment, machinery and materials pull out of Complex. As such, these he instituted charges against petitioner. The trial court held that In June 1993, Ley Construction and Development Construction (Ley) filed
acts were merely done pursuant to his official functions and were not, in consignation wasn't applicable to the case at bar but was reversed by the a complaint for collection of sum of money against FCY Construction
any way, made with evident bad faith. CA. Group, Inc. (FCY) and Francis Yu (Yu), the president of FCY. Ley alleged
that it had a joint venture agreement with FCY over the Tandang Sora
We perceive no intention on the part of Lawrence Qua and the other Commonwealth Flyover government project (Project), for which Ley had
ISSUE:
officers of Complex to defraud the employees and the Union. They were provided funds and construction materials.
compelled to act upon the instructions of their customers who were the real
owners of the equipment, materials and machinery. The prevailing labor Is Santos personally liable?
The complaint was filed in order to compel FCY to pay its share (half) in
unrest permeating within the premises of Complex left the officers with no the collections it received in the project. They also alleged that petitioners
RULING:
other choice but to pull them out of Complex at night to prevent their were guilty of fraud in incurring the obligation.
destruction. Thus, we see no reason to declare Lawrence Qua personally
Yes.
liable to the Union.
75
Derivative Suit
FCY and Yu claimed that there was no fraud. Moreover, they claimed that Petitioner Ricardo A. Llamado was Treasurer of Pan Asia Finance Probation may be granted whether the sentence imposes a term of
it was the DPWH that induced Ley to deliver materials and cash for the Corporation. Together with Jacinto N. Pascual, Sr., President of the same imprisonment or a fine with subsidiary imprisonment in case of insolvency.
Project. They also claim that Yu should be dropped as a party-defendant corporation, petitioner Llamado was prosecuted for violation of Batas An application for probation shall be filed with the trial court, with notice to
considering the hornbook law that corporate personality is a shield against Pambansa Blg. 22. The two (2) had co-signed a postdated check payable the appellate court if an appeal has been taken from the sentence of
personal liability of its officers. to private respondent Leon Gaw in the amount of P186,500.00, which conviction. The filing of the application shall be deemed a waiver of the right
check was dishonored for lack of sufficient funds. to appeal, or the automatic withdrawal of a pending appeal. In the latter
ISSUES: case, however, if the application is filed on or after the date of the judgment
The trial court convicted the petitioner alone, since jurisdiction over the of the appellate court, said application shall be acted upon by the trial court
Can Yu be made liable in his individual capacity if he indeed entered into person of Pascual, who had thoughtfully fled the country, had not been on the basis of the judgment of the appellate court.
and signed the contract in his official capacity as President of FCY? obtained. Petitioner was sentenced to imprisonment and to pay a fine.
In its present form, Section 4 establishes a much narrower period during
RULING: which an application for probation may be filed with the trial court: "after [the
Petitioner then filed with the Court of Appeals Manifestation and Petition for
Probation". Petitioner asked the Court of Appeals to grant his Petition for trial court] shall have convicted and sentenced a defendant and — within
No. The Court held that Francis Yu cannot be made liable in his individual the period for perfecting an appeal — ." As if to provide emphasis, a new
capacity, in the absence of stipulation to that effect, due to the personality Probation or, in the alternative, to remand the Petition back to the trial court.
In a "Manifestation and Motion" and filed with the Court of Appeals, proviso was appended to the first paragraph of Section 4 that
of the corporation being separate and distinct from the persons composing expressly prohibits the grant of an application for probation "if the
it. petitioner formally withdrew his appeal conditioned, however, on the
approval of his Petition for Probation. 2 defendant has perfected an appeal from the judgment of conviction." It is
worthy of note too that Section 4 in its present form has dropped the phrase
However, while the Court agrees that Yu cannot be held solidarily liable which said that the filing of an application for probation means "the
with FCY merely because he is the President thereof and was involved in The Office of the Solicitor General filed a Comment stating that it had no automatic withdrawal of a pending appeal". The deletion is quite logical
the transactions with Ley, the Court also notes that there exists instances objection to petitioner Llamado's application for probation. The Court of since an application for probation can no longer be filed once an appeal is
when corporate officers may be held personally liable for corporate acts. Appeals, through Mr. Justice Magsino, denied the Petition for Probation. perfected; there can, therefore, be no pending appeal that would have to
Such exceptions were outlined in Tramat Mercantile, Inc. vs. Court of be withdrawn.
Appeals, as follows: ISSUE:
1. He assents (a) to a patently unlawful act of the corporation, or (b) In applying Section 4 in the form it exists today (and at the time petitioner
for bad faith or gross negligence in directing its affairs, or (c) for Should the motion for probation be granted? Llamado was convicted by the trial court), to the instant case, we must then
conflict of interest, resulting in damages to the corporation, its inquire whether petitioner Llamado had submitted his application for
stockholders or other persons; RULING: probation "within the period for perfecting an appeal." Put a little differently,
2. He consents to the issuance of watered down stocks or who, the question is whether by the time petitioner Llamado's application was
having knowledge thereof, does not forthwith file with the filed, he had already "perfected an appeal" from the judgment of conviction
NO, it should be denied. of the Regional Trial Court of Manila.
corporate secretary his written objection thereto;
3. He agrees to hold himself personally and solidarily liable with the
corporation; or Probation may be granted whether the sentence imposes a term of The period for perfecting an appeal from a judgment rendered by the
4. He is made, by a specific provision of law, to personally answer imprisonment or a fine only. An application for probation shall be filed with Regional Trial Court, under Section 39 of Batas Pambansa Blg. 129,
for his corporate action. the trial court, with notice to the appellate court if an appeal has been taken Section 19 of the Interim Rules and Guidelines for the Implementation of
from the sentence of conviction. The filing of the application shall be B.P. Blg. 129 and under the 1985 Rules on Criminal Procedure, as
The attendance of these circumstances, however, cannot be determined deemed a waiver of the right to appeal, or the automatic withdrawal of a amended, or more specifically Section 5 of Rule 122 of the Revised Rules
at this stage and should properly be threshed out during the trial on the pending appeal. An order granting or denying probation shall not be of Court, is fifteen (15) days from the promulgation or notice of the judgment
merits. Hence, whether Yu should be held personally and solidarily liable appealable. It will be noted that under Section 4 of P.D. No. 968, the trial appealed from. It is also clear from Section 3 (a) of Rule 122 that such
with FCY is a matter that should be left to the trial court's discretion, court could grant an application for probation "at any time" "after it shall appeal is taken or perfected by simply filing a notice of appeal with the
dependent as it is on evidence during trial. have convicted and sentenced a defendant" and certainly after "an appeal Regional Trial Court which rendered the judgment appealed from and by
has been taken from the sentence of conviction." Thus, the filing of the serving a copy thereof upon the People of the Philippines. As noted earlier,
application for probation was "deemed [to constitute] automatic withdrawal petitioner Llamado had manifested orally and in open court his intention to
92. LLAMADO v. CA and GAW of a pending appeal." appeal at the time of promulgation of the judgment of conviction, a
G.R. No. 84850 June 29, 1989 manifestation at least equivalent to a written notice of appeal and treated
as such by the Regional Trial Court.
FACTS:
PETITIONER’S DEFENSES: Petitioner urges, however, that the phrase
"period for perfecting an appeal" and the clause "if the defendant has
76
Derivative Suit
perfected an appeal from the judgment of conviction" found in Section 4 in The trial court lost jurisdiction over the case when petitioner perfected his incurred but only when exceptional circumstances warrant such as,
its current form, should not be interpreted to refer to Rule 122 of the appeal. The Court of Appeals was not, therefore, in a position to remand generally, in the following cases:
Revised Rules of Court; and that the "whereas" or preambulatory clauses the case except for execution of judgment. Moreover, having invoked the
of P.D. No. 1990 did not specify a period of fifteen (15) days for perfecting jurisdiction of the Court of Appeals, petitioner is not at liberty casually to 1. When directors and trustees or, in appropriate cases, the officers of a
an appeal. 3 It is also urged that "the true legislative intent of the attack that jurisdiction when exercised adversely to him. In any case, the corporation
amendment (P.D. No. 1990) should not apply to petitioner who filed his argument is mooted by the conclusion that we have reached, that is, that (a) vote for or assent to patently unlawful acts of the corporation;
Petition for probation at the earliest opportunity then prevailing and petitioner's right to apply for probation was lost when he perfected his (b) act in bad faith or with gross negligence in directing the corporate
withdrew his appeal." 4 appeal from the judgment of conviction. affairs;
(c) are guilty of conflict of interest to the prejudice of the corporation, its
Petitioner invokes the dissenting opinion rendered by Mr. Justice Bellosillo 93. MAM REALTY DEVELOPMENT CORPORATION and MANUEL stockholders or members, and other persons.
in the Court of Appeals. Petitioner then asks us to have recourse to "the CENTENO, v. NLRC and CELSO B. BALBASTRO 2. When a director or officer has consented to the issuance of watered
cardinal rule in statutory construction" that "penal laws [should] G.R. No. 114787, June 2, 1995 stocks or who, having knowledge thereof, did not forthwith file with the
be liberally construed in favor of the accused," and to avoid "a too literal corporate secretary his written objection thereto.
and strict application of the proviso in P.D. No. 1990" which would "defeat Facts: 3. When a director, trustee or officer has contractually agreed or stipulated
the manifest purpose or policy for which the [probation law] was enacted-." Celso B. Balbastro filed a case against MAM Realty to hold himself personally and solidarily liable with the Corporation.
Development Corporation (MAM) and its Vice President Manuel P. 4 When a director, trustee or officer is made, by specific provision of law,
Centeno, for unfair labor practice in violation of the Labor Code. Balbastro personally liable for his corporate action.1
Section 4 as it now stands, in authorizing the trial court to grant probation
"upon application by [the] defendant within the period for perfecting an alleged that he was employed by MAM as a pump operator in 1982 and
had since performed such work at its Rancho Estate, Marikina, Metro In labor cases, for instance, the Court has held corporate
appeal" and in reiterating in the proviso that no application for directors and officers solidarily liable with the corporation for the termination
probation shall be entertained or granted if the defendant has perfected an manila.
of employment of employees done with malice or in bad faith.
appeal from the judgment of conviction. Daid not really mean to refer to the
fifteen-day period established, as indicated above, by B.P. Blg. 129, the MAM countered that Balbastro had previously been employed
by Francisco Cacho and Co., Inc., the developer of Rancho Estates. In the case at Bench, there is nothing substantial on record that
Interim Rules and Guidelines Implementing B.P. Blg. 129 and the 1985 can justify, prescinding from the foregoing, petitioner Centeno's solidary
Rules on Criminal Procedure, but rather to some vague and undefined Sometime in May 1982, his services were contracted by MAM for the
operation of the Rancho Estates' water pump. He was engaged, however, liability with the corporation.
time, i.e., "the earliest opportunity" to withdraw the defendant's appeal.
The whereas clauses invoked by petitioner did not, of course, refer to the not as an employee, but as a service contractor, at an agreed fee of
fifteen-day period. There was absolutely no reason why they should have P1,590.00 a month. Under the agreement, Balbastro was merely made to
so referred to that period for the operative words of Section 4 already do open and close on a daily basis the water supply system of the different
phases of the subdivision in accordance with its water rationing scheme. 94. SERGIO F. NAGUIAT, doing business under the name and style
refer, in our view, to such fifteen-day period. Whereas clauses do not form
He worked for only a maximum period of three hours a day, and he made SERGIO F. NAGUIAT ENT., INC., & CLARK FIELD TAXI, INC. v.
part of a statute, strictly speaking; they are not part of the operative
use of his free time by offering plumbing services to the residents of the NLRC
language of the statute. 5 Nonetheless, whereas clauses may be helpful to
subdivision. He was not at all subject to the control or supervision of MAM G.R. No. 116123
the extent they articulate the general purpose or reason underlying a new
for. Prior to the filing of the complaint, MAM executed a Deed of Transfer, March 13, 1997
enactment, in the present case, an enactment which drastically but clearly
changed the substantive content of Section 4 existing before the in favor of the Rancho Estates Phase III Homeowners Association, Inc.,
conveying to the latter all its rights and interests over the water system in Facts:
promulgation of P.D. No. 1990. Whereas clauses, however, cannot control
the subdivision. Petitioner Clark Field Taxi, Inc. (CFTI) held a concessionaire's
the specific terms of the statute; in the instant case, the whereas clauses
contract with the Army Air Force Exchange Services ("AAFES") for the
of P.D. No. 1990 do not purport to control or modify the terms of Section 4
NLRC found the corporation guilty as charged, and likewise held operation of taxi services within Clark Air Base. Sergio F. Naguiat was
as amended. Upon the other hand, the term "period for perfecting an
Centeno liable together with said corporation. CFTI's president, while Antolin T. Naguiat was its vice-president. Like
appeal" used in Section 4 may be seen to furnish specification for the loose
Sergio F. Naguiat Enterprises, Incorporated (Naguiat Enterprises), a
language "first opportunity" employed in the fourth whereas clause.
Issue: Should Centeno be held liable together with MAM Realty? trading firm, it was a family-owned corporation. Respondents were
"Perfection of an appeal" is, of course, a term of art but it is a term of art
previously employed by CFTI as taxi drivers. They were required to pay a
widely understood by lawyers and judges and Section 4 of the Probation
Ruling: daily "boundary fee" and all incidental expenses for the maintenance of the
Law addresses itself essentially to judges and lawyers. "Perfecting an
No. vehicles they were driving were accounted against them, including
appeal" has no sensible meaning apart from the meaning given to those
gasoline expenses. Subsequently, AAFES was dissolved as a result of the
words in our procedural law and so the law-making agency could only have
The NLRC erred in holding Centeno jointly and severally liable US military bases phase-out in the Philippines and the services of
intended to refer to the meaning of those words in the context of procedural
with MAM. A corporation, being a juridical entity, may act only through its individual respondents were officially terminated in 1991.
law.
directors, officers and employees. Obligations incurred by them, acting as
such corporate agents, are not theirs but the direct accountabilities of the The AAFES Taxi Drivers Association (driver’s union)held
corporation they represent. True, solidary liabilities may at times be negotiations as regards separation benefits that should be awarded in
77
Derivative Suit
favor of the drivers. They arrived at an agreement that the separated retrenchment to prevent losses and in cases of closures or cessation of of the business was proffered. In this light, he cannot be held solidarily liable
drivers will be given P500.00 for every year of service as severance pay. operations of establishment or undertaking not due to serious business for the obligations of CFTI and Sergio Naguiat to the private respondents.
Most of the drivers accepted said amount, however, individual respondents losses or financial reverses, the separation pay shall be equivalent to one
herein refused to accept theirs. Instead, after disaffiliating themselves from (1) month pay or at least one-half (1/2) month pay for every year of service,
the drivers' union, individual respondents, through the National whichever is higher. A fraction of at least six (6) months shall be considered A corporation, being a juridical entity, may act only through its
Organization of Workingmen (NOWM), a labor organization which they one whole year. The Court found that NLRC did not commit grave abuse directors, officers and employees. Obligations incurred by them, acting as
subsequently joined, filed a complaint against Sergio F. Naguiat (Naguiat of discretion in ruling that individual respondents were entitled to separation such corporate agents, are not theirs but the direct accountabilities of the
Enterprises), AAFES and the driver’s union for payment of separation pay pay in the amount $120.00 (one-half of $240.00 monthly pay) or its peso corporation they represent. True, solidary liabilities may at times be
due to termination/phase-out. Private respondents alleged that they were equivalent for every year of service. incurred but only when exceptional circumstances warrant such as,
regular employees of Naguiat Enterprises, although their individual generally, in the following cases:
applications for employment were approved by CFTI. They claimed to
have been assigned to Naguiat Enterprises after having been hired by Naguiat Enterprises, not liable 1. When directors and trustees or, in appropriate cases, the officers
CFTI, and that the former thence managed, controlled and supervised their There is no substantial basis to hold that Naguiat Enterprises is of a corporation —
employment. They averred further that they were entitled to separation pay an indirect employer of individual respondents much less a labor only
based on their latest daily earnings of US$15.00 for working sixteen (16) contractor. Private respondents failed to substantiate their claim that (a) vote for or assent to patently unlawful acts of the corporation;
days a month. Petitioners claimed that the cessation of business of CFTI Naguiat Enterprises managed, supervised and controlled their
was due to "great financial losses and lost business opportunity" resulting (b) act in bad faith or with gross negligence in directing the corporate
employment. It appears that they were confused on the personalities of
from the phase-out of Clark Air Base brought about by the Mt. Pinatubo Sergio F. Naguiat as an individual who was the president of CFTI, and affairs;
eruption and the expiration of the RP-US military bases agreement. They Sergio F. Naguiat Enterprises, Inc., as a separate corporate entity with a
admitted that CFTI had agreed with the drivers' union to grant its taxi driver- (c) are guilty of conflict of interest to the prejudice of the corporation,
separate business. They presumed that Sergio F. Naguiat, who was at the
employees separation pay equivalent to P500.00 for every year of service. its stockholders or members, and other persons.
same time a stockholder and director of Sergio F. Naguiat Enterprises, Inc.,
was managing and controlling the taxi business on behalf of the latter. A
Labor Arbiter ruled in favor of the respondents, ordering CFTI to closer scrutiny and analysis of the records, however, evince the truth of the
pay respondents P1,200/year of service for humanitarian consideration. 2. When a director or officer has consented to the issuance of
matter: that Sergio F. Naguiat, in supervising the-taxi drivers and
NLRC affirmed LA's decision with modification by granting separation pay watered stocks or who, having knowledge thereof, did not
determining their employment terms, was rather carrying out his
$120/year of service, and held that Naguiat Enterprises, S. Naguiat, and A. forthwith file with the corporate secretary his written objection
responsibilities as president of CFTI. Hence, Naguiat Enterprises as a
Naguiat are jointly and severally liable with CFTI. NLRC issued a second separate corporation does not appear to be involved at all in the taxi thereto.
resolution denying the MR of the petitioners. business.
3. When a director, trustee or officer has contractually agreed or
CFTI president (Sergio Naguiat), solidarily liable
Sergio F. Naguiat, admittedly, was the president of CFTI who stipulated to hold himself personally and solidarily liable with the
Issue: actively managed the business. Thus, he falls within the meaning of an Corporation.
Are private respondent-employees of petitioner Clark Field Taxi, Inc., "employer" as contemplated by the Labor Code, who may be held jointly
who were separated from service due to the closure of Clark Air Base, and severally liable for the obligations of the corporation to its dismissed
entitled to separation pay and, if so, in what amount? 4. When a director, trustee or officer is made, by specific provision
employees. Moreover, petitioners also conceded that both CFTI and
Naguiat Enterprises were "close family corporations" owned by the of law, personally liable for his corporate action.
Held: Naguiat family. Section 100, paragraph 5 of the Corporation Code which
specifically imposes personal liability for corporate torts upon the
YES. In labor cases, for instance, the Court has held that corporate directors and
stockholder actively managing or operating the business and affairs of the
officers are solidarily liable with the corporation for the termination of
close corporation unless the corporation has obtained reasonably
Well-settled is the rule that business losses or financial reverses, employment of employees done with malice or in bad faith.
adequate liability insurance. CFTI failed to comply with this law-imposed
in order to sustain retrenchment of personnel or closure of business and duty or obligation. Consequently, its stockholder who was actively engaged
warrant exemption from payment of separation pay, must be proved with in the management or operation of the business should be held personally
clear and satisfactory evidence. The records, however, are devoid of such liable.
evidence. The Labor Arbiter; as affirmed by NLRC, correctly found that 95. PROGRESS HOMES AND ERMELO ALMEDA v. NLRC
petitioners stopped their taxi business within Clark Air Base because of the CFTI Vice-President (Antolin Naguiat), not personally liable 269 SCRA 274
phase-out of U.S. military presence thereat. It was not due to any great Although he carried the title of "general manager" as well, it had
financial loss because petitioners' taxi business was earning profitably at not been shown that he had acted in such capacity. Furthermore, no FACTS:
the time of its closure. With respect to the amount of separation pay that evidence on the extent of his participation in the management or operation
should be granted, Article 283 of the Labor Code provides that in case of
78
Derivative Suit
Petitioner is a non-stock organization duly registered with the 96. REAHS CORP, SEVERO CASTULO, et al. v. NLRC, BONIFACIO bar, the thrust of petitioners' arguments was aimed at confining liability
SEC. The other petitioner, Ermelo Almeda, is the President and General RED, et al solely to the corporation, as if the entity were an automaton designed to
Manager of Progress Homes and the owner of the land where the GR 117473, 15 April 1997 perform functions at the push of a button. The issue, however, is not limited
Progress Homes Subdivision is located. to payment of separation pay under Article 283 but also payment of labor
FACTS:
standard benefits such as underpayment of wages, holiday pay and 13th
Private respondents allegedly were among the workers employed
by petitioners in their construction and development of the subdivision from Private respondents sued Reahs Corp. for unfair labor practice month pay to two of the private respondents. While there is no sufficient
1986 to 1988. Forty of these workers, including private respondents, filed and illegal dismissal. They claim that they were unlawfully dismissed and evidence to conclude that petitioners have indiscriminately stopped the
before the NLRC Arbitration Branch a petition for reinstatement, salary were not awarded nor given any separation pay. On the other hand, entity's business, at the same time, petitioners have opted to abstain from
adjustment, ECOLA, overtime pay and 13th month pay. Petitioners respondents allege that sometime in 1986, a certain Ms Soledad Domingo, presenting sufficient evidence to establish the serious and adverse
amicably settled the case with thirty-three of the laborers, leaving private the sole proprietress and operator of Rainbow Sauna located at 316 financial condition of the company.
respondents as the only claimants. Private respondents alleged that they Araneta Avenue, Quezon City, offered to sell her business to respondent
worked as laborers and carpenters for 8.5 hours a day at a salary below Reah's Corporation After the sale, all the assets of Ms Domingo were 97. BENJAMIN A. SANTOS vs. NLRC
the minimum wage and that when they demanded payment of the benefits G.R. No. 101699; March 13, 1996
turned over to respondent Reah's, which put a sing-along coffee shop and
due them, they were summarily dismissed and barred from entering the
massage clinic; that complainant Red started his employment on the first
workplace. FACTS:
week of December 1988 as a room boy at P50.00/day and was given living
Melvin Millena was hired to be the project accountant for
Petitioners seek to set aside the decision of the NLRC which affirmed quarters inside the premises as he requested; that sometime in March
MMDCs mining operations in Gatbo, Bacon, Sorsogon. On 12 August
the decision of the Labor Arbiter declaring them (Progress Homes and 1989, complainant Red asked permission to go to Bicol for a period of ten 1986, he sent to Mr. Gil Abao, the MMDC corporate treasurer, a
Ermelo Almeda) jointly and severally liable to private respondents for their (10) days, which was granted, and was given an advance money of memorandum calling the latters attention to the failure of the company to
backwages and separation pay. P1,200.00 to bring some girls from the province to work as attendants at comply with the withholding tax requirements of, and to make the
the respondent's massage clinic, that it was only on January 1, 1990 that corresponding monthly remittances to the Bureau of Internal Revenue
complainant Red returned and was re-hired under the same terms and (BIR) on account of delayed payments of accrued salaries to the
conditions of his previous employment with the understanding that he will company’s laborers and employees.
RULING: have to refund the P1,200.00 cash advance given to him; that due to poor
business, increase in the rental cost and the failure of Meralco to reconnect In a letter, dated 08 September 1986, Abano advised Millena to
1. Did the NLRC properly held Ermelo Almeda to be solidarily liable with the electrical services in the establishment, it suffered losses leading to its resign from the corporation. Millena expressed shock over the termination
Progress Homes? closure. The NLRC ruled in favor of respondents. Together with the of his employment. He complained that he would not have resigned from
corporation, the NLRC also held Castulo, Romeo Pascua, and Daniel the Sycip, Gorres & Velayo accounting firm, where he was already a senior
No. The Court has held that corporate directors and officers are solidarily Valenzuela solidarily liable due to their capacity as Chairman, Board staff auditor, had it not been for the assurance of a continuous job by
liable with the corporation for the termination of employment of employees Member and Accountant, and Acting Manager, respectively. MMDCs Engr. Rodillano E. Velasquez. He requested that he be
only if the termination is done with malice or in bad faith. The Labor Arbiter's reimbursed the advances he had made for the company and be paid his
decision failed to disclose why Almeda was made personally liable. There ISSUE: accrued salaries/claims.
appears no evidence on record that he acted maliciously or in bad faith in
terminating the services of private respondents. Petitioner Almeda, Whether or not Pascua, Castulo, and Valenzuela, may be held The claim was not heeded, hence, Millena filed with the NLRC
therefore, should not have been made personally answerable for the liable. Regional Arbitration a complaint for illegal dismissal, unpaid salaries, 13th
payment of private respondents' salaries. month pay, overtime pay, separation pay, and incentive leave pay against
RULING: MMDC and its two top officials Benjamin A. Santos ,the President, and
Rodillano A. Velasquez ,the executive vice-president,.
2. Was there illegal dismissal? YES. They acted in bad faith in dismissing the respondents. As
a general rule established by legal fiction, the corporation has a personality A copy of the notice and summons was served to MMDC,
No. The labor arbiter relied solely on the bare allegations of the parties in separate and distinct from its officers, stockholders and members. Hence, Santos and Velasquez. At the initial hearing before the Labor Arbiter, only
their position papers. There is nothing in the labor arbiter's decision to show officers of a corporation are not personally liable for their official acts unless the complainant, Millena, appeared; however, Atty. Romeo Perez, in
how he arrived at the conclusion that there is employer-employee it is shown that they have exceeded their authority. This fictional veil, representation of the respondents, requested by telegram that the hearing
relationship. The employment contract presented by petitioners, while however, can be pierced by the very same law which created it when "the be reset. Although the request was granted by the Labor Arbiter, Millena
admittedly defective, did not refer to any of the private respondents. No notion of the legal entity is used as a means to perpetrate fraud, an illegal was allowed, nevertheless, to present his evidence ex-parte at that initial
evidence was presented to show that petitioners engaged the services of act, as a vehicle for the evasion of an existing obligation, and to confuse hearing.
private respondents. legitimate issues". Under the Labor Code, for instance, when a corporation
violates a provision declared to be penal in nature, the penalty shall be Labor Arbiter Fructouso T. Aurellano, finding no valid cause for
terminating complainants employment, ruled, citing this Courts
imposed upon the guilty officer or officers of the corporation. In the case at
79
Derivative Suit
pronouncement in Construction & Development Corporation of the situation might arise when a corporation is used to evade a just and due amount of $18,300, Exhibit D; and the goods arrived sometime in July,
Philippines vs. Leogardo, Jr. that a partial closure of an establishment due obligation or to justify a wrong, to shield or perpetrate fraud, to carry out 1963 according to accused himself, tsn. II:7; now from here on there is
to losses was a retrenchment measure that rendered the employer liable similar other unjustifiable aims or intentions, or as a subterfuge to commit some debate on the evidence; according to Complainant Bank, there was
for unpaid salaries and other monetary claims. injustice and so circumvent the law. In Tramat Mercantile, Inc., vs. Court of permitted delivery of the steel sheets only upon execution of a trust receipt,
Appeals, the Court has collated the settled instances when, without Exhibit A; while according to the accused, the goods were delivered to him
Santos reiterates that he should not have been adjudged necessarily piercing the veil of corporate fiction, personal civil liability can sometime before he executed that trust receipt in fact they had already
personally liable by public respondents, the latter not having validly also be said to lawfully attach to a corporate director, trustee or officer; to
been converted into steel office equipment by the time he signed said trust
acquired jurisdiction over his person whether by personal service of wit: When
receipt, tsn. II:8; but there is no question - and this is not debated - that the
summons or by substituted service under Rule 19 of the Rules of Court. (1) He assents (a) to a patently unlawful act of the corporation,
His contention is unacceptable. The fact that Atty. Romeo B. Perez has or (b) for bad faith or gross negligence in directing its affairs, or (c) for conflict bill of exchange issued for the purpose of collecting the unpaid account
been able to timely ask for a deferment of the initial hearing coupled with of interest, resulting in damages to the corporation, its stockholders or other thereon having fallen due (see Exh. B) neither accused nor his company
his subsequent active participation in the proceedings, should disprove the persons; having made payment thereon notwithstanding demands, Exh. C and C-
supposed want of service of legal process. Although as a rule, modes of (2) He consents to the issuance of watered stocks or who, 1, dated 17 and 27 December, 1963, and the accounts having reached the
service of summons are strictly followed in order that the court may acquire having knowledge thereof, does not forthwith file with the corporate sum in pesos of P46,818.68 after deducting his deposit valued at
jurisdiction over the person of a defendant, such procedural modes, secretary his written objection thereto; P28,736.47; that was the reason why upon complaint by Continental Bank,
however, are liberally construed in quasi-judicial proceedings, substantial (3) He agrees to hold himself personally and solidarily liable with the Fiscal filed the information after preliminary investigation as has been
compliance with the same being considered adequate. Moreover, the corporation; or said on 22 October, 1964.
jurisdiction over the person of the defendant in civil cases is acquired not (4) He is made, by a specific provision of law, to personally
only by service of summons but also by voluntary appearance in court and answer for his corporate action.
submission to its authority. Appearance by a legal advocate is such
voluntary submission to a courts jurisdiction. It may be made not only by The case of petitioner is way off these exceptional instances. It ISSUE/S:
actual physical appearance but likewise by the submission of pleadings in is not even shown that Santos has had a direct hand in the dismissal of
compliance with the order of the court or tribunal. To say that petitioner did private respondent enough to attribute to Millena a patently unlawful act WHETHER PETITIONER JOSE O. SIA, HAVING ONLY
not authorize Atty. Perez to represent him in the case is to unduly tax while acting for the corporation. Neither can Article 289 of the Labor Code ACTED FOR AND IN BEHALF OF THE METAL MANUFACTURING
credulity. Like the Solicitor General, the Court likewise considers it unlikely be applied since this law specifically refers only to the imposition of COMPANY OF THE PHILIPPINES (METAL COMPANY, FOR SHORT)
that Atty. Perez would have been so irresponsible as to represent petitioner penalties under the Code. It is undisputed that the termination of petitioner’s AS PRESIDENT THEREOF IN DEALING WITH THE COMPLAINANT,
if he were not, in fact, authorized. Atty. Perez is an officer of the court, and employment has, instead, been due, collectively, to the need for a further THE CONTINENTAL BANK, (BANK FOR SHORT) HE MAY BE
he must be presumed to have acted with due propriety. The employment mitigation of losses, the onset of the rainy season, the insurgency problem LIABLE FOR THE CRIME CHARGED.
of a counsel or the authority to employ an attorney, it might be pointed out, in Sorsogon and the lack of funds to further support the mining operation
need not be proved in writing; such fact could inferred from circumstantial in Gatbo.
evidence. Santos was not just an ordinary official of the MMDC; he was the
President of the company. RULING:
98. SIA v PEOPLE, G.R. No. L-30896
Santos, in any event, argues that public respondents have April 28, 1983 It is worthy of note that the civil liability imposed by the trust
gravely abused their discretion in finding petitioner solidarily liable with receipt is exclusively on the Metal Company. Speaking of such liability
MMDC even in the absence of bad faith and malice on his part. There is J. DE CASTRO alone, as one arising from the contract, as distinguished from the civil
merit in this plea. liability arising out of a crime, the petitioner was never intended to be
equally liable as the corporation. Without being made so liable personally
Issue: as the corporation is, there would then be no basis for holding him
Is Santos solidarily liable with MMDC even in the absence of bad FACTS:
criminally liable, for any violation of the trust receipt. This is made clearly so
faith and malice on his part? upon consideration of the fact that in the violation of the trust agreement
There is no debate on certain antecedents: Accused Jose 0. Sia
sometime prior to 24 May, 1963, was General Manager of the Metal and in the absence of positive evidence to the contrary, only the
Held:
Manufacturing Company of the Philippines, Inc. engaged in the corporation benefited, not the petitioner personally, yet, the allegation of the
No. A corporation is a juridical entity with legal personality
manufacture of steel office equipment; on 31 May, 1963, because his information is to effect that the misappropriation or conversion was for the
separate and distinct from those acting for and in its behalf and, in general,
from the people comprising it. The rule is that obligations incurred by the company was in need of raw materials to be imported from abroad, he personal use and benefit of the petitioner, with respect to which there is
corporation, acting through its directors, officers and employees, are its applied for a letter of credit to import steel sheets from Mitsui Bussan variance between the allegation and the evidence.
sole liabilities. Nevertheless, being a mere fiction of law, peculiar situations Kaisha, Ltd. of Tokyo, Japan, the application being directed to the
or valid grounds can exist to warrant, albeit done sparingly, the disregard Continental Bank, herein complainant, Exhibit B and his application having 99. TRAMAT MERCANTILE v. COURT OF APPEALS
of its independent being and the lifting of the corporate veil. As a rule, this been approved, the letter of credit was opened on 5 June, 1963 in the G.R. No. 111008; November 7, 1994
80
Derivative Suit
100. LIRAG TEXTILE MILLS and BASILIO LIRAG vs. SSS, HON. preferred shares at the specified dates constitutes a debt which is defined
FACTS: PACIFICO DE CASTRO "as an obligation to pay money at some fixed future time, or at a time which
Melchor de la Cuesta, doing business under the name and style of GR L-33205, 31 August 1987 becomes definite and fixed by acts of either party and which they expressly
"Farmers Machineries," sold to Tramat, 1 unit HINOMOTO TRACTOR. or impliedly, agree to perform in the contract.
David Ong, Tramat's president and manager, issued check for P33,500
payment (apparently replacing an earlier postdated check for A stockholder sinks or swims with the corporation and there is no obligation
P33,080.00). Tramat, in turn, sold the tractor, together with an FACTS
to return the value of his shares by means of repurchase if the corporation
attached lawn mower fabricated by it, to the Metropolitan Waterworks and incurs losses and financial reverses, much less guarantee such
That on September 4, 1961, the SSS and Lirag Textile Mills, Inc. and
Sewerage System ("NAWASA") for P67,000.4. David Ong caused a "stop repurchase through a surety.
payment" of the check when NAWASA refused to pay the tractor and lawn Basilio Lirag entered into a Purchase Agreement under which the plaintiff
mower after discovering that, aside from some stated defects of the agreed to purchase from the said defendant preferred shares of
P1,000,000.00 subject to the conditions set forth in such agreement. 101. NAVA vs. PEERS MARKETING
attached lawn mower, the engine was a reconditioned unit. De la Cuesta
Pursuant to the Purchase Agreement of September 4, 1961, SSS, on GR No. L-28120 November 25, 1976
filed an action for the recovery of P33,500.
January 31, 1962, paid Lirag Textile Mills, Inc. the sum of P500,000.00 for Facts:
ONG averred that de la Cuesta had no cause of action; that the questioned which the said defendant issued to plaintiff 5,000 preferred shares with a
transaction was between plaintiff and Tramat Mercantile, Inc., and not with par value of P100.00 per share. This is a mandamus case, Teofilo Po as an incorporator
Ong in his personal capacity; and that the payment of the check was subscribed to eighty shares of Peers Marketing Corporation at one
stopped because the subject tractor had been priced as a brand new, not To guarantee the redemption of the stocks purchased by the plaintiff, the
hundred pesos a share or a total par value of eight thousand pesos. Po
as a reconditioned unit. The trial court ruled in favor of payment of dividends, as well as the other obligations of the Lirag Textile
paid two thousand pesos or twenty-five percent of the amount of his
petitioner. CA affirmed. MR was denied. Mills, Basilio signed the Purchase Agreement of September 4, 1961 not
subscription. No certificate of stock was issued to him or, for that matter, to
only as president of the defendant corporation, but also as surety so that
ISSUE: Should Ong be held jointly and severally liable with Tramat? any incorporator, subscriber or stockholder.
should the Lirag Textile Mills, Inc. fail to perform any of its obligations in the
said Purchase Agreement, the surety shall immediately pay to the vendee On April 2, 1966 Po sold to Ricardo A. Nava for two thousand
HELD:
the amounts then outstanding. pesos twenty of his eighty shares. In the deed of sale Po represented that
No.
he was "the absolute and registered owner of twenty shares" of Peers
Notwithstanding such letters of demand to the defendant Basilio L. Lirag,
Ong acted, not in his personal capacity, but as an officer of a Marketing Corporation.
Stock Certificates Nos. 128 and 139 issued to plaintiff are still unredeemed
corporation, TRAMAT, with a distinct and separate personality. As such, it and no dividends have been paid on said stock certificates.
should only be the corporation, not the person acting for and on its behalf, Nava requested the officers of the corporation to register the sale
that properly could be made liable thereon. Personal liability of a corporate in the books of the corporation. The request was denied because Po has
director, trustee or officer along (although not necessarily) with the not paid fully the amount of his subscription. Nava was informed that Po
corporation may so validly attach, as a rule, only when: ISSUE was delinquent in the payment of the balance due on his subscription and
1. He assents (a) to a patently unlawful act of the corporation, or that the corporation had a claim on his entire subscription of eighty shares
(b) for bad faith, or gross negligence in directing its affairs, or (c) for conflict Whether or not Lirag Textile is liable to SSS which included the twenty shares that had been sold to Nava.
of interest, resulting in damages to the corporation, its stockholders or other
persons; On December 21, 1966 Nava filed this mandamus action in the
2. He consents to the issuance of watered stocks or who, having Court of First Instance of Negros Occidental, Bacolod City Branch to
knowledge thereof, does not forthwith file with the corporate secretary his RULING compel the corporation and Renato R. Cusi and Amparo Cusi, its
written objection thereto; executive vice-president and secretary, respectively, to register the said
YES.
3. He agrees to hold himself personally and solidarily liable with twenty shares in Nava's name in the corporation's transfer book.
the corporation; or It failed to comply with its contractual stipulations. The Purchase
4. He is made, by a specific provision of law, to personally The respondents in their answer pleaded the defense that no
Agreement is, indeed, a debt instrument. Its terms and conditions
answer for his corporate action. shares of stock against which the corporation holds an unpaid claim are
In the case at bench, there is no indication that petitioner David unmistakably show that the parties intended the repurchase of the
transferable in the books of the corporation.
Ong could be held personally accountable under any of the preferred shares on the respective scheduled dates to be an absolute
abovementioned cases. obligation which does not depend upon the financial ability of petitioner
corporation. This absolute obligation on the part of petitioner corporation is
made manifest by the fact that a surety was required to see to it that the Issue:
obligation is fulfilled in the event of the principal debtor's inability to do so.
The unconditional undertaking of petitioner corporation to redeem the
81
Derivative Suit
W/N officers of Peers Marketing Corporation can be compelled by Roberto Ang 8,750 President Rachel replied that the Hi-Yield Case is different because the complaining
mandamus to enter in its stock and transfer book the sale made. Rachel Ang 3,750 Secretary stockholder was a minority stockholder. However, in the case at bar,
Total 25,000 Juanito Ang is one of the biggest stockholders of SMBI. He is a member of
BOD and is even the VP thereof.
Juanito and Roberto are siblings. Anecita is Juanito’s wife and Jeannevie
Held: is their daughter. Rachel is Juanito’s wife. ISSUE:
There‘s no certificate of stock issued in favor of Po. Shares of stock may Nancy Ang (Nancy), the sister of Juanito and Roberto, extended a loan Is the nature of the case a derivative suit?
be transferred by delivery to the transferee of the certificate properly (the contract was not written in view of their close relationship) to settle the
indorsed. "Title may be vested in the transferee by delivery of the certificate obligations of SMBI and other corporations owned by the Ang family, in the RULING:
with a written assignment or indorsement thereof" There should be amount of $1,000,000 payable to Sps. Roberto and Rachel Ang (Sps.
compliance with the mode of transfer prescribed by law. The usual practice Roberto) and Sps. Juanito and Anecita Ang (Sps. Juanito). Part of the loan The complaint is not a derivative suit.
is for the stockholder to sign the form on the back of the stock certificate. was also used to purchase real properties for SMBI, for Juanito and for
The certificate may thereafter be transferred from one person to another. If Roberto. Nancy was a former stockholder of SMBI, but she no longer A derivative suit is an action brought by a stockholder on behalf of the
the holder of the certificate desires to assume the legal rights of a appears in the General Information Sheet (GIS) of SMBI since 1996. corporation to enforce corporate rights against the corporation’s directors,
shareholder to enable him to vote at corporate elections and to receive officers or other insiders. Under Sections 23 and 36 of the Corporation
dividends, he fills up the blanks in the form by inserting his own name as Juanito claimed that payments to Nancy ceased after 2006. Thus, in 2008, Code, the directors or officers, as provided under the by-laws, have the
transferee. Then he delivers the certificate to the secretary of the Nancy demanded to Sps. Juanito and Sps. Roberto for the payment of the right to decide whether or not a corporation should sue. Since these
loan. However, Sps. Roberto sent a letter to Nancy saying that they are not directors or officers will never be willing to sue themselves, or impugn their
corporation so that the transfer may be entered in the corporation's books.
complying to anything because they have not personally contracted a loan wrongful or fraudulent decisions, stockholders are permitted by law to bring
The certificate is then surrendered and a new one issued to the transferee. from Nancy. an action in the name of the corporation to hold these directors and officers
That procedure cannot be followed in the instant case because, as already accountable. In derivative suits, the real party in interest is the corporation,
noted, the twenty shares in question are not covered by any certificate of On the other hand, Sps. Juanito executed a Deed of Acknowledgment and while the stockholder is a mere nominal party.
stock in Po's name. Moreover, the corporation has a claim on the said Settlement Agreement (Settlement) and an Extra Judicial REM. They also
shares for the unpaid balance of Po's subscription. A stock subscription is admitted that they contracted a loan together with Sps. Roberto and such Section 1, Rule 8 of the Interim Rules imposes the following requirements
a subsisting liability from the time the subscription is made. The subscriber loan shall be secured by the the real properties of Sps. Juanito and Sps. for derivative suits:
is as much bound to pay his subscription as he would be to pay any other Roberto. (1) The person filing the suit must be a stockholder or member at
debt. The right of the corporation to demand payment is no less the time the acts or transactions subject of the action occurred
incontestable. In this case no stock certificate was issued to Po. Without Thereafter, Juanito filed a “Stockholder Derivate Suit” against Sps. and the time the action was filed;
the stock certificate, which is the evidence of ownership of corporate stock, Roberto. He alleged that “the intentional and malicious refusal of defendant (2) He must have exerted all reasonable efforts, and alleges the
the assignment of corporate shares is effective only between the parties to Sps. Roberto to settle their 50% share x x x of the total obligation x x x will same with particularity in the complaint, to exhaust all remedies
definitely affect the financial viability of plaintiff SMBI." available under the articles of incorporation, by-laws, laws or
the transaction.
rules governing the corporation or partnership to obtain the relief
In their answer, Rachel claimed that the suit was not a bona fide derivative he desires;
suit as defined under the Interim Rules of Procedure for Intra-Corporate (3) No appraisal rights are available for the act or acts complained
Controversies (Interim Rules); that the Complaint, although labelled as a of; and
102. JUANITO ANG (in behalf of SUNRISE) v. SPS. ROBERTO and
derivative suit, is actually a collection suit since the real party in interest is (4) The suit is not a nuisance or harassment suit.
RACHEL ANG
not SMBI, but Nancy and her husband.
G.R. No. 201675
First, the Complaint failed to show how the acts of Sps. Roberto resulted in
June 19, 2013
She also argued that the Complaint failed to allege that Juanito "exerted all any detriment to SMBI. The loan was not a corporate obligation, but a
reasonable efforts to exhaust all intra-corporate remedies available under personal debt of the Ang brothers and their spouses. The check was
FACTS:
the articles of incorporation, by-laws, laws or rules governing the issued to Sps. Juanito and Sps. Roberto, and not SMBI. The proceeds
corporation to obtain the relief he desires," as required by the Interim Rules. were used for payment of the obligations of the other corporations owned
Sunrise Marketing (Bacolod), Inc. (SMBI) is owned by the Ang family. Its
by the Angs as well as the purchase of real properties for the Ang brothers.
current stockholders are the following:
Juanito replied that, the requirement for exhaustion of intra-corporate SMBI was never a party to the Settlement Agreement or the Mortgage. It
remedies is no longer needed when the corporation itself is "under the was never named as a co-debtor or guarantor of the loan. Both instruments
Stockholder Number of Share Position
complete control of the persons against whom the suit is filed." citing Hi- were executed by Juanito and Anecita in their personal capacity, and not
Juanito Ang 8,750 Vice President
Yield Realty, Inc. v. CA. He also claimed that Rachel’s Motion is disallowed in their capacity as directors or officers of SMBI. Thus, SMBI is under no
Anecita Ang 1,250 Treasurer
under the Interim Rules. legal obligation to satisfy the obligation.
Jeannevie Ang 2,500

82
Derivative Suit
Moreover, the fact that Juanito and Anecita attempted to constitute a there appears to be no damage to SMBI if the loan extended by Nancy actions, PPC's assets were ". . . not only in imminent danger, but have
mortgage over "their" share in a corporate asset cannot affect SMBI. and Theodore remains unpaid. actually been dissipated, lost, wasted and destroyed."
Juanito and Anecita, as stockholders of SMBI, are not co-owners of SMBI
assets. They do not own pro-indiviso shares, and therefore, cannot Thus, a plain reading of the allegations in the Complaint would show that Respondent Balmores prayed that a receiver be appointed from his list of
mortgage the same except in their capacity as directors or officers of SMBI. the case was mainly filed to collect a debt allegedly extended by the Nancy nominees. He also prayed for petitioners' prohibition from "selling,
to SMBI. Thus, the aggrieved party is not SMBI but Nancy, who are not encumbering, transferring or disposing in any manner any of [PPC's]
Since damage to the corporation was not sufficiently proven by Juanito, even stockholders. properties, including the MC Home [Depot] checks and/or their
the Complaint cannot be considered a bona fide derivative suit. A proceeds." He prayed for the accounting and remittance to PPC of the MC
derivative suit is one that seeks redress for injury to the corporation, and Home Depot checks or their proceeds and for the annulment of the board's
not the stockholder. No such injury was proven in this case. 103. VILLAMOR v. UMALE resolution "waiving PPC's rights in favor of Villamor's law firm.
24 September 2014 According to the trial court, PPC's entitlement to the checks was doubtful.
Second, the Complaint also failed to allege that all available corporate
The resolution issued by PPC's board of directors; waiving its rights to the
remedies under the articles of incorporation, by-laws, laws or rules Facts:
option to lease contract in favor of Villamor's law firm, must be accorded
governing the corporation were exhausted, as required under the Interim
MC Home Depot occupied a prime property (Rockland area) in Pasig. The prima facie validity.
Rules.
property was part of the area owned by Mid-Pasig Development
The trial court also noted that there was a pending case filed by one
No written demand was ever made for the BOD to address Juanito’s Corporation (Mid-Pasig). PPC obtained an option to lease portions of Mid-
Pasig's property, including the Rockland area. Leonardo Umale against Villamor, involving the same checks. Umale was
concerns. The fact that SMBI is a family corporation does not exempt
also claiming ownership of the checks. This, according to the trial court,
Juanito from complying with the Interim Rules. In the Yu case, the Court
PPC's board of directors issued a resolution waiving all its rights, interests, weakened respondent Balmores' claim that the checks were properties of
held that a family corporation is not exempt from complying with the clear
and participation in the option to lease contract in favor of the law firm of PPC.
requirements and formalities of the rules for filing a derivative suit. There is
nothing in the pertinent laws or rules which state that there is a distinction Atty. Alfredo Villamor, Jr. (Villamor), petitioner in G.R. No. 172843. PPC
between family corporations and other types of corporations in the received no consideration for this waiver in favor of Villamor's law firm.
The trial court also found that there was "no clear and positive showing of
institution by a stockholder of a derivative suit. dissipation, loss, wastage, or destruction of [PPC's] assets . . . [that was]
On November 22, 2004, PPC, represented by Villamor, entered into a prejudicial to the interest of the minority stockholders, parties-litigants or the
Lastly, the complaint is considered as a nuisance or harassment suit under memorandum of agreement (MOA) with MC Home Depot. Under the general public."26 The board's failure to recover the disputed amounts was
Section 1(b) of the Interim Rules. MOA, MC Home Depot would continue to occupy the area as PPC's sub-- not an indication of mismanagement resulting in the dissipation of assets.
lessee for four (4) years, renewable for another four (4) years, at a monthly
“Prohibition against nuisance and harassment suits. - rental of P4,500,000.00 plus goodwill of P18,000,000.00. The trial court noted that PPC was earni The Court of Appeals ruled that
Nuisance and harassment suits are prohibited. In the case filed by respondent Balmores with the trial court "[was] a derivative
determining whether a suit is a nuisance or In compliance with the terms of the MOA, MC Home Depot issued 20 post- suit because there were allegations of fraud or ultra vires acts ... by [PPC's
harassment suit, the court shall consider, among dated checks representing rental payments for one year and the goodwill directors]."36cralawlawlibrary
others, the following: money. The checks were given to Villamor who did not turn these or the
(1) The extent of the shareholding or interest of the equivalent amount over to PPC, upon encashment. According to the Court of Appeals, the trial court abandoned its duty to the
initiating stockholder or member; stockholders in a derivative suit when it refused to appoint a receiver or
(2) Subject matter of the suit; Hernando Balmores, respondent in G.R. No. 172843 and G.R. No. create a management committee, all during the pendency of the
(3) Legal and factual basis of the complaint; 172881 and a stockholder and director of PPC, wrote a letter addressed to proceedings. The assailed order of the trial court removed from the
(4) Availability of appraisal rights for the act or acts PPJC's directors, petitioners in G.R. No. 172881, on April 4, 2005. He stockholders their right, in an intra-corporate controversy, to be allowed the
complained of; and informed them that Villamor should be made to deliver to PPC and account remedy of appointment of a receiver during the pendency of a derivative
(5) Prejudice or damage to the corporation, for MC Home Depot's checks or their equivalent value. suit, leaving the corporation under the control of an outsider and its assets
partnership, or association in relation to the relief prone to dissipation.ng substantial rental income from its other sub-
sought. Due to the alleged inaction of the directors, respondent Balmores filed with lessees.
the Regional Trial Court an intra-corporate controversy complaint under
In case of nuisance or harassment suits, the court Rule 1, Section 1(a)(1) of the Interim Rules for Intra-Corporate
may, motu proprio or upon motion, forthwith dismiss Controversies (Interim Rules) against petitioners for their alleged devices
the case.” or schemes amounting to fraud or misrepresentation "detrimental to the
Issue:
interest of the Corporation and its stockholders." Whether the Court of Appeals correctly characterized respondent
Records show that Juanito, apart from being VP, owns the highest number
of shares, equal to those owned by Roberto. Also, as explained earlier, Balmores' action as a derivative suit.
Respondent Balmores alleged in his complaint that because of petitioners'
83
Derivative Suit
Ruling: (4) The suit is not a nuisance or harassment suit. However, an amendment to the AI was approved by the SEC,
which provides “No profit shall inure to the exclusive benefit of any of its
Respondent Balmores' action in the trial court failed to satisfy all the The fifth requisite for filing derivative suits, while not included in the shareholders, hence, no dividends shall be declared in their favor. In
requisites of a derivative suit. enumeration, is implied in the first paragraph of Rule 8, Section 1 of the accordance with the Lease and Development Agreement by and between
Interim Rules: The action brought by the stockholder or member must be Subic Bay Metropolitan Authority and The Universal International Group of
Respondent Balmores failed to exhaust all available remedies to obtain the "in the name of [the] corporation or association. ..." This requirement has Taiwan, where the golf course and clubhouse component thereof was
reliefs he prayed for. Though he tried to communicate with PPC's directors already been settled in jurisprudence. assigned to the Club, the shareholders shall not have proprietary rights or
about the checks in Villamor's possession before he filed an action with the interests over the properties of the Club.”
trial court, respondent Balmores was not able to show that this comprised Neither did respondent Balmores implead PPC as party in the case nor did Petitioners claimed in the Complaint that defendant corporation
-all the remedies available under the articles of incorporation, bylaws, laws, he allege that he was filing on behalf of the corporation. In this case, did not disclose to them the above amendment which allegedly makes the
or rules governing PPC. respondent Balmores filed an individual suit. shares non-proprietary, as it takes away the right of the shareholders to
participate in the pro-rata distribution of the assets of the corporation after
An allegation that appraisal rights were not available for the acts Respondent Balmores did not bring the action for the benefit of the
its dissolution. According to petitioners, this is in fraud of the stockholders
complained of is another requisite for filing derivative suits under Rule 8, corporation. Instead, he was alleging that the acts of PPC's directors,
who only discovered the amendment when they filed a case for injunction
Section 1(3) of the Interim Rules. specifically the waiver of rights in favor of Villamor's law firm and their failure
to restrain the corporation from suspending their rights to use all the
to take back the MC Home Depot checks from Villamor, were detrimental
facilities of the club. Furthermore, petitioners alleged that the Board of
A derivative suit is an action filed by stockholders to enforce a corporate to his individual interest as a stockholder. In filing an action,
Directors and officers of the corporation did not call any stockholders’
action. It is an exception to the general rule that the corporation's power to therefore, his intention was to vindicate his individual interest and not
meeting from the time of the incorporation, in violation of Section 50 of the
sue is exercised only by the board of directors or trustees. PPC's or a group of stockholders'.
Corporation Code and the By-Laws of the corporation. Neither did the
defendant directors and officers furnish the stockholders with the financial
Individual stockholders may be allowed to sue on behalf of the corporation
The essence of a derivative suit is that it must be filed on behalf of the statements of the corporation nor the financial report of the operation of the
whenever the directors or officers of the corporation refuse to sue to
corporation. This is because the cause of action belongs, primarily, to the corporation in violation of Section 75 of the Corporation Code. Petitioners
vindicate the rights of the corporation or are the ones to be sued and are in
corporation. The stockholder who sues on behalf of a corporation is merely also claim that on August 15, 1997, SBGCCI presented to the SEC an
control of the corporation. It is allowed when the "directors [or officers] are
a nominal party. amendment to the By-Laws of the corporation suspending the voting rights
guilty of breach of . . . trust, [and] not of mere error of judgment." In
of the shareholders except for the five founders’ shares. Said amendment
derivative suits, the real party in interest is the corporation, and the suing
was allegedly passed without any stockholders’ meeting or notices to the
stockholder is a mere nominal party.
stockholders in violation of Section 48 of the Corporation Code.
104. CHING v. SUBIC Alleging that the stockholders suffered damages as a result of
The Court has recognized that a stockholder's right to institute a derivative
NESTOR CHING ANDREW WELLINGTON AND THE SUBIC BAY the fraudulent mismanagement of the corporation, petitioners prayed in
suit is not based on any express provision of the Corporation Code, or even
GOLFERS AND SHAREHOLDERS, INC. v. SUBIC BAY GOLF AND their Complaint for the following: TRO against the defendants from acting
the Securities Regulation Code, but is impliedly recognized when the said
COUNTRY CLUB, INC. as Officers and Board of Directors of the Corporation. After hearing, a writ
laws make corporate directors or officers liable for damages suffered by
G.R. No. 174353 September 10, 2014 of preliminary injunction be issued enjoining defendants to act as Board of
the corporation and its stockholders for violation of their fiduciary duties. In
Directors and Officers of the Corporation. In the meantime a Receiver be
effect, the suit is an action for specific performance of an obligation, owed
FACTS: appointed by the Court to act as such until a duly constituted Board of
by the corporation to the stockholders, to assist its rights of action when the
Nestor Ching and Andrew Wellington filed a Complaint with the Directors and Officers of the Corporation be elected and qualified plus
corporation has been put in default by the wrongful refusal of the directors
RTC on behalf of the members of Subic Bay Golf and Country Club, Inc. damages.
or management to adopt suitable measures for its protection.
(SBGCCI) against the said country club and its Board of Directors and Respondents specifically denied the allegations of the Complaint
SECTION 1. Derivative action. - A stockholder or member may officers under the provisions of PD No. 902-A in relation to Section 5.2 of claiming that petitioners failed (a) to show that it was authorized by SBGSI
bring an action in the name of a corporation or association, as the SRC. The Subic Bay Golfers and Shareholders Incorporated (SBGSI, to file the Complaint on the said corporation’s behalf; (b) to comply with the
the case may be, provided, that: plaintiff), a corporation composed of shareholders of the defendant requisites for filing a derivative suit and an action for receivership; and (c)
corporation, was also named as plaintiff. The officers impleaded as to justify their prayer for injunctive relief since the Complaint may be
(1) He was a stockholder or member at the time the acts or defendants were the following: (1) President, Susan Hu; (2) its treasurer, considered a nuisance or harassment suit under Section 1(b), Rule 1 of
transactions subject of the action occurred and at the time the action Jack Hu; (3) corporate secretary Reynald Suarez; and (4) directors Hu the Interim Rules of Procedure for Intra-Corporate Controversies.16 Thus,
was filed; Tsung Hui and Hu Tsung Tzu. prayed for the dismissal of the Complaint.
(2) He exerted all reasonable efforts, and alleges the same with The complaint alleged that the defendant corporation sold The RTC issued an Order dismissing the Complaint. The RTC
particularity in the complaint, to exhaust all remedies available shares to SBGSI at US$22,000.00 per share, presenting to them the AI held that the action is a derivative suit. The CA affirmed the decision of the
under the articles of incorporation, by-laws, laws or rules governing which contained the following provision: “No profit shall inure to the RTC.
the corporation or partnership to obtain the relief he desires; exclusive benefit of any of its shareholders, hence, no dividends shall be
(3) No appraisal rights are available for the act or acts complained of; declared in their favor. Shareholders shall be entitled only to a pro-rata ISSUE:
and share of the assets of the Club at the time of its dissolution or liquidation.” 1. Is the case filed a deivative suit?
84
Derivative Suit
2. Should the case at hand be dismissed? In the case, the RTC dismissed the Complaint for failure to Out of a total number of 5,723 members who were entitled to vote, 1,358
comply with the second and fourth requisites above. The Court finds that were supposed to vote through their respective proxies and their votes
RULING: the same should not have been dismissed on the ground that it is a were critical in determining the existence of a quorum, which was at least
1. YES. In Nelson v. Anderson(1999), the **289 minority nuisance or harassment suit. Although the shareholdings of petitioners are 2,863 (50% plus 1).
shareholder alleged that the other shareholder of the corporation indeed only two out of the 409 alleged outstanding shares or 0.24%, the
negligently managed the business, resulting in its total failure. The Court has held that it is enough that a member or a minority of stockholders However the Committee on Elections of the corporation found that the
appellate court concluded that the plaintiff could not maintain the suit as a file a derivative suit for and in behalf of a corporation. proxy votes were irregular and questionable. Since there is lack of time to
direct action: "Because the gravamen of the complaint is injury to the whole With regard, however, to the second requisite, we find that authenticate such, the BOD adjourned the meeting for lack of quorum.
body of its stockholders, it was for the corporation to institute and maintain petitioners failed to state with particularity in the Complaint that they had
a remedial action. A derivative action would have been appropriate if its exerted all reasonable efforts to exhaust all remedies available under the However, respondents challenged the adjournment of the meeting.
responsible officials had refused or failed to act." The court wenton to note articles of incorporation, by-laws, and laws or rules governing the Despite no quorum was obtained, the respondents pushed through with
that the damages shown at trial were the loss of corporate profits. Since corporation to obtain the relief they desire. The Complaint contained no the scheduled election and were elected as the new Board of Directors and
"shareholders own neither the property nor the earnings of the allegation whatsoever of any effort to avail of intra-corporate remedies. officers of Legaspi Towers 300, Inc.
corporation," any damages that the plaintiff alleged that resulted from such Indeed, even if petitioners thought it was futile to exhaust intra-corporate
loss of corporate profits "were incidental to the injury to the corporation." remedies, they should have stated the same in the Complaint and A general information sheet was submitted to SEC with the list of the newly
In the case, the reliefs sought in the Complaint, namely that of enjoining specified the reasons for such opinion. Failure to do so allows the RTC to elected officers.
defendants from acting as officers and Board of Directors of the dismiss the Complaint, even motu proprio, in accordance with the Interim
corporation, the appointment of a receiver, and the prayer for damages in Rules. April 13, 2004, petitioners filed a Complaint for the Declaration of Nullity of
the amount of the decrease in the value of the sharesof stock, clearly show The wordings of Section 1, Rule8 of the Interim Rules of Elections with Prayers for the lssuance of Temporary Restraining Orders
that the Complaint was filed to curb the alleged mismanagement of Procedure Governing Intra-Corporate Controversies are simple and do not and Writ of Preliminary Injunction and Damages against respondents with
SBGCCI. The causes of action pleaded by petitioners do not accrue to a leave room for statutory construction. The second paragraph thereof the RTC of Manila.
single shareholder or a class of shareholders but to the corporation itself. requires that the stockholder filing a derivative suit should have exerted all
However, as minority stockholders, petitioners do not have any reasonable efforts to exhaust all remedies available under the articles of April 14, 2004 before an answer could be filed, an amended complaint was
statutory right to override the business judgments of SBGCCI’s officers and incorporation, by-laws, laws or rules governing the corporation or submitted.
Board of Directors on the ground of the latter’s alleged lackof qualification partnership to obtain the relief he desires; and to allege such fact with
to manage a golf course. Contraryto the arguments of petitioners, particularity in the complaint. The obvious intent behind the rule is to make April 20, 2004, before respondents could submit an Answer to the
Presidential Decree No. 902-A, which is entitled REORGANIZATION OF the derivative suit the final recourse of the stockholder, after all other Amended Complaint, petitioners again filed an Urgent Ex-Parte Motion to
THE SECURITIES AND EXCHANGE COMMISSION WITH remedies to obtain the relief sought had failed. Admit Second Amended Complaint and for the lssuance of Ex-Parte
ADDITIONAL POWERS AND PLACING THE SAID AGENCY UNDER Temporary Restraining Order Effective only for Seventy-Two (72) Hours.
THE ADMINISTRATIVE SUPERVISION OF THE OFFICE OF THE
PRESIDENT, does not grant minority stockholders a cause of action Both judges of branch 24 and 46 ( Presiding Judge Antonio Eugenio, Jr.
against waste and diversion by the Board of Directors, but merely identifies 105. LEGASPI TOWERS 300 v. MUER, et. al. and Presiding Judge Artemio S. Tipon respectively) inhibited from hearing
the jurisdiction of the SEC over actionsalready authorized by law or LEGASPI TOWERS 300, INC., LILIA MARQUINEZ PALANCA, the said case.
jurisprudence. It is settled that a stockholder’s right to institute a derivative ROSANNA D. IMAI, GLORIA DOMINGO and RAY
suit is not based on any express provisionof the Corporation Code, or even VINCENT, Petitioners, April 21, 2004, Executive Judge Enrico A. Lanzanas of the RTC of Manila
the Securities Regulation Code, but is impliedly recognized when the said vs. acted on the Motion for the Issuance of an Ex Parte Temporary
laws make corporate directors or officers liable for damages suffered by AMELIA P. MUER, SAMUEL M. TANCHOCO, ROMEO TANKIANG, Restraining Order enjoining respondents from taking over management for
the corporation and its stockholders for violation of their fiduciary duties. RUDEL PANGANIBAN, DOLORES AGBAYANI, ARLENEDAL A. 72 hours in order to prevent further irreparable damages and prejudice to
----------------------------------------------------------------------------------------------------- YASUMA, GODOFREDO M. CAGUIOA and EDGARDO M. the corporation, as day-to-day activities will be disrupted and will be
------------------------------------------------------------- SALANDANAN,Respondents. paralyzed due to the legal controversy.
2. Yes. However, derivative suit cannot prosper without first G.R. No. 170783
complying with the legal requisites for its institution (1) He was a April 21, 2004, respondents filed their Answer alleging that the election
June 18, 2012
stockholder or member at the time the acts or transactions subject of the was lawfully conducted citing the report of SEC counsel Nicanor Patricio
action occurred and at the time the action was filed; (2) He exerted all who attended the election. Reports would state that since there is no time
Facts: to authenticate the proxy votes, the BOD adjourned the meeting despite
reasonable efforts, and alleges the same with particularity in the complaint,
to exhaust all remedies available under the articles of incorporation, by- query from the attendees. Proxies were counted and recorded, and there
laws, laws or rules governing the corporation or partnership to obtain the April 2, 2004, pursuant to by-laws of Legaspi Towers 300, Inc., petitioners was a declaration of a quorum – out of a total of 5,721 votes, 2,938 were
relief he desires; (3) No appraisal rights are available for the act or acts as Board of Directors set the annual meeting of the members of the present either in person or proxy. Ballots were prepared and votes cast.
complained of; and (4) The suit is not a nuisance or harassment suit. corporation and the election of the new Board of Directors for Year 2004-
2005 at the lobby of Legaspi Towers 300 Inc.
85
Derivative Suit
April 22, 2004, the case was assigned to Judge Antonio I. De Castro of as well as a violation of the priority rights of creditors. Furthermore, there is in behalf of the condominium corporation in the Second Amended
the RTC of Manila, Branch 3 (trial court). the difficulty of determining the amount of damages that should be paid to Complaint is improper.
each individual stockholder.
July 21, 2004, the pre trial was initiated and that petitioners motion to The stockholder’s right to file a derivative suit is not based on any express
inmplead the corporation. However it was denied for being improper. However, in cases of mismanagement where the wrongful acts are provision of The Corporation Code, but is impliedly recognized when the
committed by the directors or trustees themselves, a stockholder or law makes corporate directors or officers liable for damages suffered by
A second order was also issued, reiterating that to implead the corporation member may find that he has no redress because the former are vested the corporation and its stockholders for violation of their fiduciary duties,
is improper. Petitioners filed a motion for reconsideration but was denied. by law with the right to decide whether or not the corporation should sue, which is not the issue in this case.
and they will never be willing to sue themselves. The corporation would
Petition for certiorari before the CA was filed and denied becase the CA thus be helpless to seek remedy. Because of the frequent occurrence of Petition is denied.
stated that the right to vote is to be exercised by the stockholders not the such a situation, the common law gradually recognized the right of a
corporation. Hence impleading the corporation is improper. stockholder to sue on behalf of a corporation in what eventually became 106. LISAM ENTERPRISES v. BDO
known as a "derivative suit." It has been proven to be an effective remedy LISAM ENTERPRISES INC., REPRESENTED BY LOLITA SORIANO
Thus the petition before the SC. of the minority against the abuses of management. Thus, an individual & LOLITA SORIANO vs. BDO (FORMERLY PHILIPPINE
stockholder is permitted to institute a derivative suit on behalf of the COMMERCIAL INTERNATIONAL BANK) & SPS. SORIANO
corporation wherein he holds stock in order to protect or vindicate GR No. 143264
corporate rights, whenever officials of the corporation refuse to sue or are April 23, 2012
ISSUE: the ones to be sued or hold the control of the corporation. In such actions,
the suing stockholder is regarded as the nominal party, with the corporation PERALTA, J.
1. Whether or not there is grave abuse on the act of the court not as the party-in- interest.
impleading the corporation as party-plaintiff to the case. Facts:
2. Is a derivative suit proper in this case? Since it is the corporation that is the real party-in-interest in a derivative suit, This case stemmed from an action for annulment of mortgage
then the reliefs prayed for must be for the benefit or interest of the executed by the defendant spouses Sorianos, subjecting a property
corporation. When the reliefs prayed for do not pertain to the corporation, owned by Lisam Enterprises Inc. (LEI) as security for their loan from BDO.
HELD: then it is an improper derivative suit
Lolita Soriano, representing LEI in her capacity as Corporate
1. No. The requisites for a derivative suit are as follows: Secretary, alleged that the spouses, in their personal capacity loaned from
Why should Legaspi Towers 300, Inc. x x x be included as party-plaintiff BDO the amount of 20 Million pesos; that the spouses in their capacity as
when defendants are members thereof too like plaintiffs. Both parties are President and Treasurer, falsified a resolution and the secretary’s
a) the party bringing suit should be a shareholder as of the time
deemed to be acting in their personal capacities as they both claim to be certificate allowing the LEI property to be mortgage by the spouses. Lolita
of the act or transaction complained of, the number of his shares
the lawful board of directors. also imputes BDO with gross negligence for not inquiring into the status of
not being material;
the LEI property as corporate property; it should have been acting in
2. No. circumspect in allowing corporate property as a security for a personal
Suits by stockholders or members of a corporation based on wrongful or b) he has tried to exhaust intra-corporate remedies, i.e., has loan.
fraudulent acts of directors or other persons may be classified into made a demand on the board of directors for the appropriate
individual suits, class suits, and derivative suits. Where a stockholder or relief but the latter has failed or refused to heed his plea; and Furthermore, Lolita alleges that in a subsequent arrangement,
member is denied the right of inspection, his suit would the Spouses Soriano and BDO colluded to transfer the liability for the loan
be individual because the wrong is done to him personally and not to the c) the cause of action actually devolves on the corporation, the to the corporation. These facts prompted Lolita to initiate derivative suits for
other stockholders or the corporation. Where the wrong is done to a group wrongdoing or harm having been, or being caused to the Fraudulent Scheme & Unlawful Machinations with the SEC, and the
of stockholders, as where preferred stockholders' rights are violated, corporation and not to the particular stockholder bringing the suit. annulment of mortgage suit in the RTC.
a class or representative suit will be proper for the protection of all
stockholders belonging to the same group. But where the acts complained The RTC ruled in favor of the Spouses and BDO. Lolita moved
The party-in-interest are the petitioners as stockholders, who wield such for its reconsideration. During the pendency of the MR, Lolita moved for
of constitute a wrong to the corporation itself, the cause of action belongs right to vote. The cause of action devolves on petitioners, not the
to the corporation and not to the individual stockholder or member. the admission of an amendment of the complaint adding the following
condominium corporation, which did not have the right to vote. Hence, the portions:
Although in most every case of wrong to the corporation, each stockholder complaint for nullification of the election is a direct action by petitioners, who
is necessarily affected because the value of his interest therein would be were the members of the Board of Directors of the corporation before the
impaired, this fact of itself is not sufficient to give him an individual cause of “xxx; that plaintiff Lolita A. Soriano likewise made
election, against respondents, who are the newly-elected Board of demands upon the Board of Directors of Lisam
action since the corporation is a person distinct and separate from him, and Directors. Under the circumstances, the derivative suit filed by petitioners
can and should itself sue the wrongdoer. Otherwise, not only would the Enterprises, Inc., to make legal steps to protect the
theory of separate entity be violated, but there would be multiplicity of suits interest of the corporation from said fraudulent
86
Derivative Suit
transaction, but unfortunately, until now, no such legal Prologue: This case is an anatomy of a ₱6.185 billion1 pillage of the public
step was ever taken by the Board, hence this action a) The party bringing the suit should be a shareholder as of coffers that ranks among one of the most brazen and hideous in the history
for the benefit and in behalf of the corporation; xxx” the time of the act or transaction complained of, the number of this country. This case answers the questions why our Government
of shares not being material perennially runs out of funds to provide basic services to our people, why
The RTC denied the motion to admit amended complaint as it b) The party has tried to exhaust intra-corporate remedies, i.e. the great masses of the Filipino people wallow in poverty, and why a very
will change the cause of action Lolita, and denied the MR for lack of merit. he made a demand on the board of directors for the select few amass unimaginable wealth at the expense of the Filipino
This decision was appealed to the SC on pure questions of law. appropriate relief but the latter has failed or refused to heed people. On 1 May 2007, the 30-year old franchise of Philippine National
his plea; and Construction Corporation (PNCC) under Presidential Decree No. 1113
Issues: c) The cause of action actually devolves on the corporation, (PD 1113), as amended by Presidential Decree No. 1894 (PD 1894),
1. Is the denial of the motion to admit amended complaint the wrongdoing or harm having been, or being caused to expired. During the 13th Congress, PNCC sought to extend its franchise.
proper? the corporation and not to the particular stockholder PNCC won approval from the House of Representatives, which passed
2. Is the derivative suit properly initiated? bringing the suit. House Bill No. 57492 renewing PNCC’s franchise for another 25 years.
3. Should the case be dismissed for forum-shopping. However, PNCC failed to secure approval from the Senate, dooming the
extension of PNCC’s franchise.
Ruling: A reading of the amended complaint will reveal that the foregoing
1. The denial of the motion to admit amended complaint is not requisites have been properly alleged by Lolita. Her being a corporate FACTS:
proper. secretary signifies her status as shareholder; she has exhausted her
remedies as alleged in the amended complaint; and that the transaction PNCC was incorporated in 1966 for a term of fifty years under the
There is no ground to deny the motion. The RTC’s rationale for the was alleged to be at prejudice of LEI. Corporation Code with the name Construction Development Corporation
denial is banking on the fact that it changes the cause of action of Lolita. of the Philippines (CDCP). PD 1113, issued on 31 March 1977, granted
This was, however, the old rule before the 1997 Rules of Civil Procedure. 3. No, the case should not be dismissed for forum-shopping CDCP a 30-year franchise to construct, operate and maintain toll facilities
Prior to the 1997 Rules, a motion to amend the complaint is only disallowed in the North and South Luzon Tollways. PD 1894, issued on 22 December
if the amendment will “change the cause of action” as expressly provided; While it is true that a case has been filed each with the SEC and RTC, 1983, amended PD 1113 to include in CDCP’s franchise the Metro Manila
however, in the 1997 Rules, that part of the provision is already removed, jurisprudence guided that Court that the causes of action on both suits differ Expressway, which would "serve as an additional artery in the
thereby prompting the interpretation of such rules to shift towards liberality in nature. transportation of trade and commerce in the Metro Manila area."
in the interest of higher justice.
What is filed before the SEC is questioning the due execution, Sometime between 1978 and 1981, Basay Mining Corporation, an
Since the amendment was done during the MR, it is now an authenticity or validity of the board resolutions that facilitated the mortgage affiliate of CDCP, obtained loans from Marubeni Corporation of
amendment that requires leave of court, but the court must have a and assumption of liability by the LEI. This case, initially filed with the RTC, Japan amounting to 5,460,000,000 yen and US$5 million. A CDCP
reasonable ground to deny such amendment, and it must rule on such is for the annulment of mortgage, questioning the validity of said mortgage, official issued letters of guarantee for the loans, committing CDCP to pay
motion with liberality to promote higher justice. In this case, there is no which is within the jurisdiction of the regular courts. In sum, there is no solidarily for the full amount of the 5,460,000,000 yen loan and to the extent
reasonable ground for the RTC to deny the motion for amendments since identity of causes of action. of ₱20 million for the US$5 million loan. However, there was no CDCP
there is no inexcusable delay or there are surreptitious attempts to surprise Board Resolution authorizing the issuance of the letters of guarantee.
the spouses or BDO with allegation they cannot refute. The Court therefore Later, Basay Mining changed its name to CDCP Mining Corporation
allows the amendment (CDCP Mining). CDCP Mining secured the Marubeni loans when CDCP
107. STRADEC v. RADSTOCK & PNCC and CDCP Mining were still privately owned and managed.
2. Yes, the derivative suit was properly initiated.
STRATEGIC ALLIANCE DEVELOPMENT CORPORATION vs. Subsequently in 1983, CDCP changed its corporate name to PNCC to
Since the amendment is hereby allowed, the complaint now reads RADSTOCK SECURITIES LIMITED and PHILIPPINE NATIONAL reflect the extent of the Government's equity investment in the company,
with the following allegations: CONSTRUCTION CORPORATION, ASIAVEST MERCHANT which arose when government financial institutions converted their loans
BANKERS BERHAD, to PNCC into equity following PNCC’s inability to pay the loans. In fine, the
“xxx; that plaintiff Lolita A. Soriano likewise made x - - - - - - - - - - - - - - - - - - - - - - -x Government owns 90.3% of the equity of PNCC and only 9.70% of
demands upon the Board of Directors of Lisam Enterprises, Inc., G.R. No. 180428 PNCC’s voting equity is under private ownership.
to make legal steps to protect the interest of the corporation from LUIS SISON vs. PHILIPPINE NATIONAL CONSTRUCTION
said fraudulent transaction, but unfortunately, until now, no such CORPORATION and RADSTOCK SECURITIES LIMITED Meanwhile, the Marubeni loans to CDCP Mining remained unpaid. On 20
legal step was ever taken by the Board, hence this action for the October 2000, during the short-lived Estrada Administration, the PNCC
benefit and in behalf of the corporation; xxx” G.R. No. 178158 Board of Directors (PNCC Board) passed a Board Resolution
December 4, 2009 admitting PNCC’s liability to Marubeni for ₱10,743,103,388 as of 30
Jurisprudence with establish the requisites for a derivative suit, September 1999. This was the first PNCC Board Resolution admitting
viz: PNCC’s liability for the Marubeni loans. Previously, for two decades the
87
Derivative Suit
PNCC Board consistently refused to admit any liability for the Marubeni annul a final and executory judgment also rendered by the Court of affected by a distribution or other disposition of property in the
loans. Appeals. custody of the court or of an officer thereof may, with leave of court,
be allowed to intervene in the action. The Court shall consider
Less than two months later, the PNCC Board passed another Board Asiavest, a judgment creditor of PNCC, filed an Urgent Motion for Leave whether or not the intervention will unduly delay or prejudice the
Resolution amending the first Board which still admitted its obligations for to Intervene and to File the Attached Opposition and Motion-in-Intervention adjudication of the rights of the original parties, and whether or
the Marubeni loans but now included a clause which states “…subject to before the Court of Appeals, which was denied. not the intervenor’s rights may be fully protected in a separate
the final determination by the Commission on Audit (COA) of the amount proceeding.
of obligation involved, and subject further to the declaration of the legality ISSUE:
of said obligations by the Office of the Government Corporate Counsel STRADEC’s interest is dependent on the outcome of Civil Case No. 05-
(OGCC)…” Do STRADEC, Asiavest, and Sison have legal standing to file suit? 882. Unless STRADEC can show that RTC Branch 146 had already
In January 2001, barely three months after the PNCC Board first admitted decided in its favor, its legal interest is simply contingent and expectant.
liability for the Marubeni loans, Marubeni assigned its entire credit to RULING:
Radstock for US$2 million or less than ₱100 million. In short, Radstock However, Asiavest has a direct and material interest in the approval or
paid Marubeni less than 10% of the ₱10.743 billion admitted amount. The Court of Appeals denied STRADEC’s motion for intervention on the disapproval of the Compromise Agreement. Asiavest is a judgment
Radstock immediately sent a notice and demand letter to PNCC. ground that the motion was filed only after the Court of Appeals and the creditor of PNCC in G.R. No. 110263 and a court has already issued a writ
trial court had promulgated their respective decisions. of execution in its favor. Asiavest’s interest is actual and material, direct and
On 15 January 2001, Radstock filed an action for collection and damages Section 2, Rule 19 of the 1997 Rules of Civil Procedure provides: immediate characterized by either gain or loss from the judgment that this
against PNCC before the Regional Trial Court. The trial court issued a writ SECTION 2. Time to intervene.– The motion to intervene may Court may render. Considering that the Compromise Agreement involves
of preliminary attachment against PNCC, and ordered PNCC’s bank be filed at any time before rendition of judgment by the trial court. A copy the disposition of all or substantially all of the assets of PNCC, Asiavest, as
accounts garnished and several of its real properties attached. of the pleading-in-intervention shall be attached to the motion PNCC’s judgment creditor, will be greatly prejudiced if the Compromise
and served on the original parties. Agreement is eventually upheld.
Meanwhile, on 19 June 2001, at the start of the Arroyo Administration, the
PNCC Board, under a new President and Chairman, revoked the The rule is not absolute. The rule on intervention, like all other rules of Sison has legal standing to challenge the Compromise Agreement.
Board Resolution admitting liability to the Marubeni loans. procedure, is intended to make the powers of the Court completely Although there was no allegation that Sison filed the case as a
available for justice. It is aimed to facilitate a comprehensive adjudication derivative suit in the name of PNCC, it could be fairly deduced that
The trial court continued to hear the main case. On 10 December 2002, of rival claims, overriding technicalities on the timeliness of the filing of the Sison was assailing the Compromise Agreement as a stockholder of
the trial court ruled in favor of Radstock. claims. This Court has ruled: Allowance or disallowance of a motion for PNCC. In such a situation, a stockholder of PNCC can sue on behalf
intervention rests on the sound discretion of the court after consideration of of PNCC to annul the Compromise Agreement.
On 17 August 2006, PNCC and Radstock entered into the the appropriate circumstances. Rule 19 of the Rules of Court is a rule of
Compromise Agreement where they agreed to reduce PNCC’s procedure whose object is to make the powers of the court fully and A derivative action is a suit by a stockholder to enforce a corporate
liability to Radstock, supposedly from ₱17,040,843,968, to completely available for justice. Its purpose is not to hinder or delay but to cause of action.25 Under the Corporation Code, where a corporation
₱6,185,000,000, which was later approved by the court. facilitate and promote the administration of justice. Thus, interventions is an injured party, its power to sue is lodged with its board of
have been allowed even beyond the prescribed period in the Rule in the directors or trustees. However, an individual stockholder may file a
STRADEC moved for reconsideration alleging that it has a claim against higher interest of justice. Interventions have been granted to afford derivative suit on behalf of the corporation to protect or vindicate corporate
PNCC as a bidder of the National Government’s shares, receivables, indispensable parties, who have not been impleaded, the right to be heard rights whenever the officials of the corporation refuse to sue, or are the
securities and interests in PNCC. The matter is subject of a complaint filed even after a decision has been rendered by the trial court, when the petition ones to be sued, or hold control of the corporation. In such actions, the
by STRADEC against PNCC and the Privatization and Management for review of the judgment was already submitted for decision before the corporation is the real party-in-interest while the suing stockholder, on
Office (PMO) for the issuance of a Notice of Award of Sale to Dong-A Supreme Court, and even where the assailed order has already become behalf of the corporation, is only a nominal party.
Consortium of which STRADEC is a partner. The Court of Appeals treated final and executory. In Lim v. Pacquing (310 Phil. 722 (1995)], the motion
STRADEC’s motion for reconsideration as a motion for intervention and for intervention filed by the Republic of the Philippines was allowed by this In this case, the PNCC Board cannot conceivably be expected to attack
denied it. Court to avoid grave injustice and injury and to settle once and for all the the validity of the Compromise Agreement since the PNCC Board itself
substantive issues raised by the parties. approved the Compromise Agreement. In fact, the PNCC Board
Rodolfo Cuenca (Cuenca), a stockholder and former PNCC President and steadfastly defends the Compromise Agreement for allegedly being
Board Chairman, filed an intervention alleging that PNCC had no obligation Concededly, STRADEC has no legal interest in the subject matter of the advantageous to PNCC.
to pay Radstock, which was denied by the court. Compromise Agreement. Section 1, Rule 19 of the 1997 Rules of Civil
Procedure states: Besides, the circumstances in this case are peculiar. Sison, as former
Meanwhile, on 20 February 2007, Sison, also a stockholder and former PNCC President and Chairman of the PNCC Board, was responsible for
PNCC President and Board Chairman, filed a Petition for Annulment of SECTION 1. Who may intervene. - A person who has a legal the approval of the Board Resolution issued on 19 June 2001 revoking the
Judgment Approving Compromise Agreement before the Court of interest in the matter in litigation, or in the success of either of the previous Board Resolution admitting PNCC’s liability for the Marubeni
Appeals, which was dismissed on the ground that it had no jurisdiction to parties, or an interest against both, or is so situated as to be adversely loans. Such revocation, however, came after Radstock had filed an action
88
Derivative Suit
for collection and damages against PNCC on 15 January 2001. Then, There is no dispute that the Marubeni loans, once  In approving PNCC Board Resolution Nos. BD-092-2000 and
when the trial court rendered its decision on 10 December 2002 in favor of recognized, would wipe out the assets of PNCC, "virtually BD-099-2000, the PNCC Board caused undue injury to the
Radstock, Sison was no longer the PNCC President and Chairman, emptying the coffers of the PNCC." Government and gave unwarranted benefits to Radstock,
although he remains a stockholder of PNCC. (c) In a debilitating self-inflicted injury, the PNCC Board revived through manifest partiality, evident bad faith or gross inexcusable
what appeared to have been a dead claim by abandoning negligence of the PNCC Board. Such acts are declared under
When the case was on appeal before the Court of Appeals, there was no one of PNCC’s strong defenses, which is the prescription of Section 3(e) of RA 3019 or the Anti-Graft and Corrupt Practices
need for Sison to avail of any remedy, until PNCC and Radstock entered the action to collect the Marubeni loans. Thus, more than Act, as "corrupt practices xxx and xxx unlawful." Being unlawful
into the Compromise Agreement, which disposed of all or substantially all ten years would have already lapsed between Marubeni’s and criminal acts, these PNCC Board Resolutions are void ab
of PNCC’s assets. Sison came to know of the Compromise Agreement extrajudicial demands in 1984 and 1986 and the initio and cannot be implemented or in any way given effect by
only in December 2006. PNCC and Radstock submitted the Compromise acknowledgment by the PNCC Board of the Marubeni the Executive or Judicial branch of the Government.
Agreement to the Court of Appeals for approval on 10 January 2007. The loans in 2000. However, the PNCC Board suddenly passed  Not content with forcing PNCC to commit corporate suicide with
Court of Appeals approved the Compromise Agreement on 25 January Board Resolution No. BD-092-2000 expressly admitting the admission of liability for the Marubeni loans under Board
2007. To require Sison at this stage to exhaust all the remedies within the liability for the Marubeni loans. In short, the PNCC Board Resolution Nos. BD-092-2000 and BD-099-2000, the PNCC
corporation will render such remedies useless as the Compromise admitted liability for the Marubeni loans despite the fact that Board drove the last nail on PNCC’s coffin when the PNCC
Agreement had already been approved by the Court of Appeals. PNCC’s the same might no longer be judicially collectible. Although Board entered into the manifestly and grossly disadvantageous
assets are in danger of being dissipated in favor of a private foreign the legal advantage was obviously on its side, the PNCC Compromise Agreement with Radstock. This time, the OGCC,
corporation. Thus, Sison had no recourse but to avail of an extraordinary Board threw in the towel even before the fight could begin. headed by Agnes DST Devanadera, reversed itself and
remedy to protect PNCC’s assets. (d) The basis for the admission of liability for the Marubeni recommended approval of the Compromise Agreement to the
loans, which was an opinion of the Feria Law Office, was PNCC Board.
Besides, in the interest of substantial justice and for compelling reasons, not even shown to the PNCC Board.
such as the nature and importance of the issues raised in this case, this  Atty. Francisco’s act of recommending to the 2) The Compromise Agreement is Void for Being Contrary to the
Court must take cognizance of Sison’s action. This Court should exercise PNCC Board the acknowledgment of the Constitution, Existing Laws, and Public Policy
its prerogative to set aside technicalities in the Rules, because after all, the Marubeni loans based only on an opinion of a
power of this Court to suspend its own rules whenever the interest of justice private law firm, without consulting the OGCC  Under the Compromise Agreement, PNCC shall pay Radstock
requires is well recognized. and without showing this opinion to the members the reduced amount of ₱6,185,000,000.00 in full settlement of
of the PNCC Board except to Atty. Valdecantos, PNCC’s guarantee of CDCP Mining’s debt allegedly totaling
NOTES: reflects how shockingly little his concern was for ₱17,040,843,968.00 as of 31 July 2006. To satisfy its reduced
PNCC, contrary to his claim that "he only had the obligation, PNCC undertakes to (1) "assign to a third party
1) The PNCC Board Acted in Bad Faith and with Gross Negligence interest of PNCC at heart." In fact, if what was assignee to be designated by Radstock all its rights and
in Directing the Affairs of PNCC involved was his own money, Atty. Francisco interests" to the listed real properties therein; (2) issue to
would have preferred not just two, but at least Radstock or its assignee common shares of the capital stock of
 In this jurisdiction, the members of the board of directors have a three different opinions on how to deal with the PNCC issued at par value which shall comprise 20% of the
three-fold duty: duty of obedience, duty of diligence, and duty of matter, and he would have maintained his non- outstanding capital stock of PNCC; and (3) assign to Radstock
loyalty. Accordingly, the members of the board of directors (1) liability. or its assignee 50% of PNCC’s 6% share, for the next 27 years
shall direct the affairs of the corporation only in accordance with  Section 1 of Memorandum Circular No. 9 dated (2008-2035), in the gross toll revenues of the Manila North
the purposes for which it was organized; (2) shall not willfully and 27 August 1998 issued by the President states: Tollways Corporation.
knowingly vote for or assent to patently unlawful acts of the All legal matters pertaining to government-owned  The PNCC Board has no power to compromise the ₱6.185
corporation or act in bad faith or with gross negligence in or controlled corporations, their subsidiaries, billion amount. Under Section 20(1), Chapter IV, Subtitle B,
directing the affairs of the corporation; and (3) shall not acquire other corporate off-springs and government Title I, Book V of Executive Order No. 292 or the
any personal or pecuniary interest in conflict with their duty as acquired asset corporations (GOCCs) shall be Administrative Code of 1987, the authority to compromise a
such directors or trustees exclusively referred to and handled by the Office settled claim or liability exceeding ₱100,000.00 involving a
 In the present case, the PNCC Board blatantly violated its duty of the Government Corporate Counsel (OGCC). government agency, as in this case where the liability
of diligence as it miserably failed to act in good faith in handling  The PNCC Board acted in bad faith in relying on amounts to ₱6.185 billion, is vested not in COA but
the affairs of PNCC. the opinion of a private lawyer knowing that exclusively in Congress. Congress alone has the power to
(a) For almost two decades, the PNCC Board had consistently PNCC is required to rely "exclusively" on the compromise the ₱6.185 billion purported liability of PNCC.
refused to admit liability for the Marubeni loans because of OGCC’s opinion. Worse, the PNCC Board, in Without congressional approval, the Compromise
the absence of a PNCC Board resolution authorizing the admitting liability for ₱10.743 billion, relied on the Agreement between PNCC and Radstock involving ₱6.185
issuance of the letters of guarantee. recommendation of a private lawyer whose billion is void for being contrary to Section 20(1), Chapter IV,
(b) The PNCC Board admitted liability for the Marubeni loans opinion the PNCC Board members have not Subtitle B, Title I, Book V of the Administrative Code of 1987.
despite PNCC’s total liabilities far exceeding its assets. even seen.
89
Derivative Suit
 PNCC’s toll fees are public funds. PD 1113 granted PNCC land itself. Otherwise, there will be a blatant circumvention because PNCC merely holds such toll fees in trust for the
a 30-year franchise to construct, operate and maintain toll of the Constitution, which prohibits a foreign private National Government. PNCC does not own the toll fees,
facilities in the North and South Luzon Expressways. With corporation from owning land in the Philippines. In and such toll fees do not form part of PNCC’s assets.
the expiration of PNCC’s franchise, the assets and facilities addition, Radstock cannot transfer the rights to ownership  The ₱36 billion debt to the National Government was
of PNCC were automatically turned over, by operation of of land in the Philippines if it cannot own the land itself. It is acknowledged by the PNCC Board in the same board
law, to the government at no cost. basic that an assignor or seller cannot assign or sell resolution that recognized the Marubeni loans. Since
 Forming part of the General Fund, the toll fees can only be something he does not own at the time the ownership, or PNCC is clearly insolvent with a huge negative net worth,
disposed of in accordance with the fundamental principles the rights to the ownership, are to be transferred to the the government enjoys preference over Radstock in the
governing financial transactions and operations of any assignee or buyer. satisfaction of PNCC’s liability arising from taxes and
government agency, to wit: (1) no money shall be paid out  PNCC must follow rules on preference of credit. duties, pursuant to the provisions of the Civil Code on
of the Treasury except in pursuance of an appropriation  In giving priority and preference to Radstock, the concurrence and preference of credits. Articles 2241,
made by law, as expressly mandated by Section 29(1), Compromise Agreement is certainly in fraud of PNCC’s 2242 and 2243 of the Civil Code expressly mandate that
Article VI of the Constitution; and (2) government funds or other creditors, including the National Government, and taxes and fees due the National Government "shall be
property shall be spent or used solely for public purposes, violates the provisions of the Civil Code on concurrence preferred" and "shall first be satisfied" over claims like
as expressly mandated by Section 4(2) of PD 1445 or the and preference of credits. those arising from the Marubeni loans which "shall enjoy
Government Auditing Code.  This Court has held that while the Corporation Code no preference" under Article 2244.
 Applying Section 29(1), Article VI of the Constitution, as allows the transfer of all or substantially all of the assets of
implanted in Sections 84 and 85 of the Government a corporation, the transfer should not prejudice the
Auditing Code, a law must first be enacted by Congress creditors of the assignor corporation. Assuming that
appropriating ₱6.185 billion as compromise money before PNCC may transfer all or substantially all its assets, to 108. YU v. YUKAYGUAN
payment to Radstock can be made. Otherwise, such allow PNCC to do so without the consent of its creditors or
payment violates a prohibitory law and thus void under without requiring Radstock to assume PNCC’s debts will Anthony S. Yu, Rosita G. Yu And Jason G. Yu (Petitioners) v.
Article 5 of the Civil Code which states that "[a]cts executed defraud the other PNCC creditors since the assignment Joseph S. Yukayguan, Nancy L. Yukayguan, Jerald Nerwin L.
against the provisions of mandatory or prohibitory laws will place PNCC’s assets beyond the reach of its other Yukayguan, And Jill Neslie L. Yukayguan, [On Their Own Behalf
shall be void, except when the law itself authorizes their creditors.
validity." Indisputably, funds held in trust by PNCC for the  Asiavest is a judgment creditor of PNCC in G.R. No. And On Behalf Of] Winchester Industrial Supply, Inc.,
National Government cannot be used by PNCC to pay a 110263 and a court has already issued a writ of execution (Respondents)
private debt of CDCP Mining to Radstock, otherwise the in its favor. Thus, when PNCC entered into the
PNCC Board will be liable for malversation of public funds. Compromise Agreement conveying several prime lots in G.R. No. 177549 June 18, 2009
 Funds held in trust by PNCC for the National Government favor of Radstock, by way of dacion en pago, there is a
cannot be used by PNCC to pay a private debt of CDCP legal presumption that such conveyance is fraudulent Facts:
Mining to Radstock, otherwise the PNCC Board will be under Article 1387 of the Civil Code.
liable for malversation of public funds. In addition, to pay  In this case, PNCC’s huge negative net worth - at least ₱6 Petitioners and Respondents herein are all stockholders of Winchester
Radstock ₱6.185 billion violates the fundamental public billion as expressly admitted by PNCC’s counsel during Industrial Supply, Inc.,(Winchester) a domestic corporation engaged in the
policy, expressly articulated in Section 4(2) of the the oral arguments, or ₱14 billion based on the 2006 COA operation of a general hardware and industrial supply and equipment
Government Auditing Code, that government funds or Audit Report - necessarily translates to an extremely
business. Anthony Yu is the older half brother of Joseph Yukayguan, who
property shall be spent or used solely for public purposes. embarrassing financial situation. With its huge negative
PNCC cannot use public funds, like toll fees that net worth arising from unpaid billions of pesos in debt, is the corporate secretary and treasurer of Winchester.
indisputably form part of the General Fund, to pay a private PNCC cannot claim that it is financially stable. As a
debt of CDCP Mining to Radstock. Such payment cannot consequence, the Compromise Agreement stipulating a Respondents filed a derivative suit in behalf of Winchester with a verified
qualify as expenditure for a public purpose. The toll fees transfer in favor of Radstock of substantially all of PNCC’s Complaint for Accounting, Inspection of Corporate Books and Damages
are merely held in trust by PNCC for the National assets constitutes fraud. To legitimize the Compromise through Embezzlement and Falsification of Corporate Records and
Government, which is the owner of the toll fees. Agreement just because there is still no judicial declaration Accounts on the ground that the petitioners are the ones who actually
 Radstock is not qualified to own land in the Philippines. of PNCC’s insolvency will work fraud on PNCC’s other controlled Winchester and were misappropriating the funds and properties
 There is no dispute that Radstock is disqualified to own creditors, the biggest creditor of which is the National
lands in the Philippines. Consequently, Radstock is also Government. To insist that PNCC is very much liquid, of Winchester, Inc. by understating the sales, charging their personal and
disqualified to own the rights to ownership of lands in the given its admitted huge negative net worth, is nothing but family expenses to the said corporation, and withdrawing stocks for their
Philippines. Contrary to the OGCC’s claim, Radstock denial of the truth. The toll fees that PNCC collects belong personal use without paying for the same. They prayed for the trial curt to
cannot own the rights to ownership of any land in the to the National Government. Obviously, PNCC cannot render an accounting of all the funds of Winchester, Inc. which petitioners
Philippines because Radstock cannot lawfully own the claim it is liquid based on its collection of such toll fees, misappropriated; (3) reimburse the personal and family expenses which
90
Derivative Suit
petitioners charged to Winchester, Inc., as well as the properties of the Ruling:
corporation which petitioners withheld without payment.
No. The Respondents failed to do so. Facts:
Petitioner contended their Answer an argued that since respondents’
Felix Gochan and Sons Realty Corporation (Gochan Realty)
Complaint purportedly constituted a derivative suit, it noticeably failed to Rule 8 of the Interim Rules of Procedure Governing Intra-Corporate
was registered with the SEC with Felix Gochan, Sr., Maria Pan Nuy Go
allege that respondents exerted effort to exhaust all available remedies in Controversies provides:
Tiong, Pedro Gochan, Tomasa Gochan, Esteban Gochan and Crispo
the Articles of Incorporation and By-Laws of Winchester, Inc., as well as in Gochan as its incorporators.
the Corporation Code. And third, given that respondents’ Complaint was Section 1. Derivative action. — A stockholder or member may bring an
also for inspection of corporate books, it lacked the allegation that action in the name of a corporation or association, as the case may be, Felix Gochan Sr.'s daughter, Alice, mother of the respondents,
respondents made a previous demand upon petitioners to inspect the provided, that: inherited 50 shares of stock in Gochan Realty from the former. Alice died
corporate books but petitioners refused. (1) He was a stockholder or member at the time the acts or transactions in 1955, leaving the 50 shares to her husband, John Young, Sr.
subject of the action occurred and the time the action was filed; The Regional Trial Court of Cebu adjudicated 6/14 of these
The parties went to a mediation and arrived at an amicable settlement. (2) He exerted all reasonable efforts, and alleges the same with particularity shares to her children, herein respondents Richard Young, David Young,
They agreed to a division of the stocks in trade, the real properties, and the in the complaint, to exhaust all remedies available under the articles of Jane Young Llaban, John Young Jr., Mary Young Hsu and Alexander
other assets of Winchester. However, Respondents repudiated the incorporation, by-laws, laws or rules governing the corporation or Thomas Young. Having earned dividends, these stocks numbered 179.
settlement for failure to divide the remaining assets of Winchester. Hence,
partnership to obtain the relief he desires;
the parties submitted their respective memoranda and pleadings
(3) No appraisal rights are available for the acts or acts complained of; and Their father John Sr., requested Gochan Realty to partition the
reiterating their allegations to each other. (4) The suits is not a nuisance or harassment suit. shares of his late wife by cancelling the stock certificates in his name and
issuing in lieu thereof, new stock certificates in the names of the
The Regional Trial Court (RTC) rendered a decision dismissing the In this case, the allegation of respondent Joseph in his Affidavit of his respondents. However, Gochan Realty refused citing as reason, the right
complaint filed by the respondents on the ground that they failed to comply repeated attempts to talk to petitioner Anthony regarding their dispute does of first refusal granted to the remaining stockholders by the Articles of
with the essential requisites for filing a derivative suit and noted that Incorporation.
not constitute "all reasonable efforts to exhaust all remedies
respondent Joseph was the corporate secretary of Winchester, Inc. and, available. Respondents did not mention at all any other remedy under the
as such, he was supposed to be the custodian of the corporate books and Cecilia Gochan Uy and Miguel Uy filed a complaint with the SEC
articles of incorporation or by-laws of Winchester, Inc., available for dispute
records; therefore, a court order for respondents’ inspection of the same for issuance of shares of stock to the rightful owners, nullification of shares
resolution among stockholders, which respondents unsuccessfully availed of stock, reconveyance of property impressed with trust, accounting,
was no longer necessary. themselves of and the Court does not believe that the articles of removal of officers and directors and damages against respondents.
incorporation and by-laws of Winchester, Inc. absolutely failed to provide
The respondents filed a petition with the Court of Appeals (CA), and the
for such remedies. Petitioners argue that Spouses Cecilia and Miguel Uy had no
CA initially dismissing the petition. However, by reconsideration, The CA
capacity or legal standing to bring the suit before the SEC on February 8,
remanded the case back to the RTC to take the necessary proceedings The fact that Winchester, Inc. is a family corporation should not in any way 1994, because the latter were no longer stockholders at the time. Allegedly,
on the ground that since the dispute involves the settlement and the closing exempt respondents from complying with the clear requirements and the stocks had already been purchased by the corporation.
of Wichester’s affairs, disposition and distribution of its remaining assets, formalities of the rules for filing a derivative suit. There is nothing in the
and there is no board of directors for purposes of dissolution and pertinent laws or rules supporting the distinction between, and the Issue:
liquidation, it is the RTC who have jurisdiction to settle the remaining assets difference in the requirements for, family corporations vis-à-vis other types
of the corporation as conferred by A.M. No. 00-8-10-SC. Hence, this of corporations, in the institution by a stockholder of a derivative suit Whether or not respondents have the legal personality to file a
petition for Review on Certiorari filed by the Petitioners with the Supreme derivative suit on behalf of the corporation.
Court. With respect to the third and fourth requirements of Section 1, Rule 8, the
respondents’ Complaint failed to allege, explicitly or otherwise, the fact that Ruling:
Issue: there were no appraisal rights available for the acts of petitioners
complained of, as well as a categorical statement that the suit was not a Yes. The respondents have personality to file a derivative suit.
Did Respondents exerted all reasonable efforts to exhaust all remedies
nuisance or a harassment suit.
available under the articles of incorporation, by-laws, laws or rules
governing Winchester, to obtain the relief they desire before filing a Where corporate directors have committed a breach of trust
109. GOCHAN et. al. v. YOUNG et. al either by their frauds, ultra vires acts, or negligence, and the corporation is
derivative suit in behalf of the corporation? G.R. No. 131889 March 12, 2001 unable or unwilling to institute suit to remedy the wrong, a single
91
Derivative Suit
stockholder may institute that suit, suing on behalf of himself and other On March 1991, Villasis and other co-petitioners filed two to entertain the complaint. The ease should have been filed with the SEC
stockholders and for the benefit of the corporation, to bring about a redress separate criminal informations, one for falsification of a public document which exercises original and exclusive jurisdiction over derivative suits,
of the wrong done directly to the corporation and indirectly to the anchored on the private respondents' submission of WIT's income they being intra-corporate disputes.
stockholders. statement for the fiscal year 1985-1986 with the SEC reflecting therein the
disbursement of corporate funds for the compensation of corporate
In the present case, the Complaint alleges all the components of officers, making it appear that the same was passed by the board on March
a derivative suit. The allegations of injury to the Spouses Uy can coexist 30, 1986, when in truth, the same was actually passed on June 1, 1986, a 111. FIRST INTERNATIONAL BANK v. CA
with those pertaining to the corporation. The personal injury suffered by the date not covered by the corporation's fiscal year 1985-1986. 252 S 259
spouses cannot disqualify them from filing a derivative suit on behalf of the G.R. No. 115849, January 24, 1996
corporation. It merely gives rise to an additional cause of action for On the other hand, an information for estafa was also filed for the
damages against the erring directors. This cause of action is also included private respondents’ alleged fraud committed against WIT and its Facts:
in the Complaint filed before the SEC. stockholders for disbursing corporate funds knowing fully well that they In the course of its banking operations, Producer Bank of the Philippines
have no sufficient, lawful authority to disburse them for the subsequent acquired 6 parcels of land at Laguna. The property used to be owned by
collective salaries of the corporate officers. BYME Corporation which was mortgaged as collateral for a loan.
Cecilia Uy's averment in the Complaint that the purchase of her
Demetrio Demetria and Jose O. Janolo wanted to purchase the property
stocks by the corporation was null and void ab initio is deemed admitted. It The trial court acquitted the accused corporate officers on both and thus initiated negotiations for that purpose. Upon the suggestion of
is elementary that a void contract produces no effect either against or in charges without imposing any civil liability against them. WIT filed an MR BYME Investment’s legal counsel, Jose Fajardo, met with Mercurio Rivera,
favor of anyone. It cannot create, modify or extinguish the juridical relation on the civil aspect which was denied, hence, this petition. Manager of the Property Management Department of the bank. The
to which it refers. Thus, Cecilia remains a stockholder of the corporation in
meeting was held in pursuant to plaintiffs’ plan to buy the property. After
view of the nullity of the Contract of Sale. ISSUE: Is the action a derivative suit? the meeting, plaintiff Janolo, following the advice of defendant Rivera,
made a formal purchase offer to the bank. Negotiations took place and an
The Spouses Uy have the capacity to file a derivative suit in HELD: No. offer price was fixed at P5.5million. During the course of the negotiations,
behalf of and for the benefit of the corporation. The reason is that the the bank was placed under conservatorship and a new conservator (under
allegations of the Complaint make them out as stockholders at the time the A derivative suit is an action brought by minority shareholders in conservatorship by the Central Bank since 1984) was appointed to which
questioned transaction occurred, as well as at the time the action was filed the name of the corporation to redress wrongs committed against it, for the name has been refused to recognize. It was replaced by Leonida
and during the pendency of the action. which the directors refuse to sue. It is a remedy designed by equity and Encarnacionas wherein she sought the repudiation of the agreement as
has been the principal defense of the minority shareholders against abuses it alleged that Rivera was not authorized to enter into such an agreement,
by the majority. Here, however, the case is not a derivative suit but is hence there was no valid contract of sale. Demetria and Janolo sued
merely an appeal on the civil aspect of the criminal cases. Producers Bank, its Manager Rivera and Acting Conservator
110. WESTERN INSTITUTE OF TECHNOLOGY v. RICARDO T. Encarnacion. A derivative suit has been filed against Rivera for the
Among the basic requirements for a derivative suit to prosper is damages suffered from the alleged perfect contract of sale involving the 6
SALAS
that the minority shareholder who is suing for and on behalf of the parcels of land.
G.R. No. 113032; August 21, 1997
corporation must allege in his complaint before the proper forum that he is
suing on a derivative cause of action on behalf of the corporation and all The regional trial court ruled in favor of Demetria et al.
FACTS: Ricardo, Salvador, Soledad, Antonio, and Richard S. Salas,
other shareholders similarly situated who wish to join. This is necessary to
belonging to the same family, are the majority and controlling members of vest jurisdiction upon the tribunal in line with the rule that it is the allegations Issue: May derivative suit lie involving the bank and its stockholders?
the Board of Trustees of Western Institute of Technology, a stock in the complaint that vests jurisdiction upon the court or quasi-judicial body
corporation engaged in the operation of an educational institution. concerned over the subject matter and nature of the action. This was not
According to petitioners, the minority stockholders of WIT, sometime on Held: No. An individual stockholder is permitted to institute a derivative suit
complied with by the WIT either in their complaint before the court a quo on behalf of the corporation wherein he hold stock in order to protect or
June 1, 1986 in the principal office of WIT at La Paz, Iloilo City, a Special nor in the instant petition which, in part, merely states that "this is a petition
Board Meeting was held. In attendance were other members of the Board vindicate corporate rights, whenever the officials of the corporation refuse
for review on certiorari on pure questions of law to set aside a portion of the to sue, or are the ones, to be sued or hold the control of the corporation. In
including one of the petitioners Reginald Villasis. Prior to aforesaid Special RTC decision in Criminal Cases Nos. 37097 and 37098" since the trial
Board Meeting, copies of notice thereof, dated May 24, 1986, were such actions, the suing stockholder is regarded as a nominal party with the
court's judgment of acquittal failed to impose any civil liability against the corporation as the real party in interest. First Philippine International Bank
distributed to all Board Members. The notice allegedly indicated that the private respondents. By no amount of equity considerations, if at all
meeting to be held on June 1, 1986. tried to seek refuge in the corporate fiction that the personality of the
deserved, can a mere appeal on the civil aspect of a criminal case be Bank is separate and distinct from its shareholders. But the rulings of
treated as a derivative suit. this Court are consistent: When the fiction is urged as a means of
In said meeting, the Board of Trustees passed a resolution
granting monthly compensation to the private respondents as corporate perpetrating a fraud or an illegal act or as a vehicle for the evasion
Granting, for purposes of discussion, that this is a derivative suit of an existing obligation, the circumvention of statutes, the
officers retroactive June 1, 1985. as insisted by petitioners, which it is not, the same is outrightly dismissible achievement or perfection of a monopoly or generally the perpetration
for having been wrongfully filed in the regular court devoid of any jurisdiction of knavery or crime, the veil with which the law covers and isolates
92
Derivative Suit
the corporation from the members or stockholders who compose it Mariano allegedly discovered that for several years, Jesus and his wife
will be lifted to allow for its consideration merely as an aggregation of Section 28-A of BP 68 merely gives the conservator power to revoke Corazon (who was herself a director) had been siphoning and diverting to
individuals. contracts that are, under existing law, deemed not to be effective – i.e void, their private bank accounts in the United States and in Hongkong
voidable, unenforceable or rescissible. Hence, the conservator merely gargantuan amounts sliced off from commissions due Commart from
In the face of the damaging admissions taken from the complaint in the takes the place of a bank’s board of directors. What the said board cannot some foreign suppliers.
second case, First Philippine International Bank, quite strangely, sought to do – such as repudiating a contract validly entered into under the doctrine
deny that the second case was a derivative suit, reasoning that it was of implied authority – the conservator cannot do either. August 22, 1989, spouses Mariano and Alice Maglutac filed a complaint
brought not by the minority shareholders, but by Henry Co. etal. who not (SEC Case No. 2673) with the SEC against Jesus T. Maglutac, Victor
only hold or control over 80% of the outstanding capital stock, but also Cipriano, Clemente Ramos, Carolina de los Reyes, Corazon Maglutac,
constitute the majority in the board of directors of petitioners bank. That Alberto Maglutac and Bernardo Maglutac (Jesus as Chairman) and the
being so, then they really represent the bank, so whether they sued 112. COMMART PHILS v. SEC rest as members of the Board of Directors of Commart.
derivatively or directly, there is undeniably an identity of interest/entity
represented. It is alleged that Jesus, by means of secret arrangements with foreign
COMMART (PHILS.) INC., JESUS, CORAZON, ALBERTO, AND
suppliers, has been diverting into his private bank accounts and converting
BERNARD all surnamed MAGLUTAC,petitioners,
In addition to the many cases, where the corporate fiction has been to his own personal benefit and advantage substantial portions of the
vs. commission income of the corporation, to the prejudice of the corporation,
regarded, we now add the instant case, and declare herewith that the
SECURITIES & EXCHANGE COMMISSION and ALICE
corporate veil cannot be used to shield an otherwise blatant violation of the its stockholders and its creditors.
MAGLUTAC, respondents.
prohibition against forum shopping. Shareholders, whether suing as the
majority in direct actions or as the minority in a derivative suit, cannot be Two motions to dismiss were filed by petitioner stating that Mariano had no
allowed to trifle with court processes particularly where, as in this case, the G.R. No. 85318 capacity to file and that SEC had no jurisdiction. Both were opposed by the
corporation itself has not been remiss in vigorously prosecuting or complainant spouses.
defending corporate causes and in using and applying remedies available June 3, 1991
to it. To rule otherwise would be to encourage corporate litigants to use May 10, 1985 Commart filed a Manifestation/Notice of Dismissal,
their shareholders as fronts to circumvent the stringent rules against forum manifesting that "it withdraws and dismisses the action taken in its behalf
FACTS: by complainants Mariano T. Maglutac and Alice M. Maglutac against all
shopping.
respondents.
From the facts, the official bank price, at any rte, the bank placed its official, Commart (Phils.), Inc., (Command for short) is a corporation organized by
Rivera is a position of authority to accept offers to buy and negotiate the two brothers, Jesus and Mariano Maglutac, to engage in the brokerage This was opposed by complainants on the ground, among other doctrines,
sale by having the offer officially acted upon by the bank. The bank cannot business for the importation of fertilizers and other products/commodities. that in a derivative suit the corporation is not allowed to be an active
turn around and say, as it now does, that what Rivera states as the bank’s participant and has no control over the suit against the real defendants; that
action on the matter is not in fact so. It is a familiar doctrine, the doctrine of the suing shareholder has the right of control.
Commart's principal income came from commissions paid to it in U.S.
ostensible authority, that if a corporation on knowingly permits one of its dollars by foreign suppliers of fertilizers and other commodities imported by
officers, or any other agent, to do acts within the scope of apparent May 27, 1985, the Hearing Panel issued an Order denying all the motions
Planters Products, Inc. and other local importers.
authority, and thus holds him out to the public as possessing power to do to dismiss as well as the so called manifestation/notice of dismissal.
those acts, the corporation will, as against any one who has in good faith
dealt with the corporation through such agent, he estopped from denying June 1984, the two brothers agreed to go their separate ways, with A motion for reconsideration was filed by Commart and Petitioners.
his authority. Mariano being persuaded to sell to Jesus his shareholdings in Commart
amounting to 25% of the outstanding capital stock. As part of the deal, a November 12, 1985, the Hearing Panel issued an Order modifying its
A bank is liable for wrongful acts of its officers done in the interest of the "Cooperative Agreement" was signed, between Commart (represented by previous order "by dismissing this case insofar as Mariano T. Maglutac is
bank or in he course of dealings of the officers in their representative Jesus) and Mariano, in which, among others, Commart ceded to Mariano concerned" but affirming the said order "in all other respects.
capacity but not for acts outside the scope of their authority. A bank holding or to an "acceptable entity" he may create, a portion of its business, with a
out its officers and agents as worthy of confidence will not be permitted to pledge of mutual cooperation for a certain period so as to enable Mariano Petitioners filed a petition for Certiorari, prohibition, Mandamus. The
profit by the frauds they my thus be enabled to perpetrate in the apparent to get his own corporation off the ground. commission en banc issued an order denying the aforesaid petition and
scope of their employment; nor will it be permitted to shrink its responsibility remanding the case to the Securities Investigation and Clearing
for such fraud even through no benefit may accrue to the bank therefrom. Mariano's wife, Alice M. Maglutac (private respondent herein) who has Department for further proceedings.
Accordingly, a banking corporation is liable to innocent third persons where been for years a stockholder and director of Commart, did not dispose of
the representation is made in the course of its business by an agent acting her shareholdings, and thus continued as such even after the sale of Thus the petition.
within the general scope of its authority even though, in the particular case, Mariano's equity.
the agent is secretly abusing his authority and attempting to perpetrate ISSUE:
fraud upon his principal or some other person, for his own ultimate benefit.
93
Derivative Suit
Whether alice have the legal standing to file the derivative suit. 113. CHASE v. BUENCAMINO These negotiations culminated in a final agreement to the effect
ELTON W. CHASE, as minority Stockholder and on behalf of other that-Elton Chase was to be paid One Hundred Thousand Dollars
Stockholders similarly situated and for the benefit of AMERICAN ($100,000.00) and he would also be given a one-third interest in Amparts,
MACHINERY AND PARTS MANUFACTURING, INC., v. DR. VICTOR with the other two, Dr. Buencamino and Cranker, as the owner of the other
HELD:
BUENCAMINO, SR., two-thirds (2/3) interest, 1/3 interest each; and that in exchange for said
Yes. A derivative suit has been the principal defense of the minority G.R. No. L-20395 May 13, 1985 (CUEVAS, J.:) $100,000.00 and the 1/3 interest, Chase was to transfer to Amparts his
shareholder against abuses by the majority.1âwphi1 It is a remedy tractor plant, ship his machineries to Manila, assuming all costs of
designed by equity for those situations where the management, through DOCTRINE: dismantling, preserving and crating for shipment to Manila, install said
fraud, neglect of duty, or other cause, declines to take the proper and “An action brought by minority shareholders in the name of the machineries at Amparts plant with the aid of five technicians and finally, he
necessary steps to assert the corporation's rights. Indeed, to grant to corporation to redress wrongs committed against the corporation, for which has to be the production manager of Amparts. But since five were
Commart the right of withdrawing or dismissing the suit, at the instance of the directors refuse to sue. It is a remedy designed by equity and has been
majority stockholders and directors who themselves are the persons necessary to organize a corporation, Buencamino and Cranker took in
the principal defense of the minority shareholders against abuses by the
alleged to have committed breaches of trust against the interest of the majority. Those brought by one or more stockholders/members in the their respective wives.
corporation, would be to emasculate the right of minority stockholders to name and on behalf of the corporation to redress wrongs committed
seek redress for the corporation. To consider the Notice of Dismissal filed against it, or protect/vindicate corporate rights whenever the officials of the Meanwhile, Chase had already shipped his machineries and
by Commart as quashing the complaint filed by Alice Maglutac in favor of corporation refuse to sue, or the ones to be sued has control of the had them installed in the Amparts plant in Pasig, Rizal. Amparts then
the corporation would be to defeat the very nature and function of a corporation. ” began operation with Dr. Buencamino as President, William Cranker as
derivative suit and render the right to institute the action illusory.
Manager and Elton Chase as Production Manager. For sometime the
FACTS: three maintained harmonious relations but later on distrust came in until
In any case, the suit is for the benefit of Commart itself, for a judgment in
The evidence on record discloses that defendant Dr. finally Chase tendered his letter of resignation as Production Manager.
favor of the complainants will necessarily mean recovery by the corporation
of the US$2.5 million alleged to have been diverted from its coffers to the Buencamino, Sr., a Filipino and William Cranker, an American, even prior
private bank accounts of its top managers and directors. Thus, the prayer to the year 1954 were already business associates. They owned two firms Chase filed an action against Cranker with the Superior Court of
in the Amended Complaint is for judgment ordering respondents Jesus namely, the Philippine American Machinery and Equipment Corporation Los Angeles seeking to recover the sum of $ 150,000.00 as alleged
and Corazon Maglutac, as well as Victor Cipriano, "to account for and to (PAMEC) which was organized in 1947 and the BUCRA which means balance of the purchase price of his plant. This case however died a natural
turn over or deliver to the Corporation" the aforesaid sum, with legal Buencamino and Cranker. While, Plaintiff Elton Chase was the owner of
interest, and "ordering all the respondent, as members of the Board of death because Cranker left and was never reached by process from the
Production Manufacturing Company, of Portland, Oregon, USA, a California Court. Then, sometime in August 1958, Cranker sold out all his
Directors to take such remedial steps as would protect the corporation from
corporation primarily dedicated to the operation of a machine shop and interest in Amparts to Dr. Buencamino.
further depredation of the funds and property."
heat-treating plant for the production of tractor parts.
CONFLICT OF INTEREST RULE ISSUE: Finally, Chase filed this case before the Court of First Instance of
Sometime in 1954, Chase was notified by the Highway Manila, alleging various acts of fraud which he claimed had been
On the "conflict of interest" issue, petitioners allege that private respondent Commission of the State of Oregon that his factory was going to be in the committed by both Dr. Buencamino and Cranker. He sought for the
Alice Maglutac "is a majority stockholder of M.M. International Sales, a path of a proposed highway. He was then advised to sell or face dissolution of the corporation.
business rival/competitor of Commart and holds only less than one percent
expropriation and warned to remove his plant within a year. His distributor
(1%) of the entire shareholdings of Commart." According to petitioners, this
being the case it is easier to believe that this so called derivative suit was Craig Carrol told him of a Dr. Buencamino of Manila who he said was
interested in establishing a manufacturing plant in the Philippines. Craig ISSUE:
filed because it is to the best interest of the company where she has a
bigger and substantial interest, which in this case is M.M. International Carrol contacted Buencamino who told him to contact his associate Is the dissolution of the corporation proper?
Sales, Inc. William Cranker in the United States.
RULING:
No real prejudice has been inflicted upon petitioners' right to be heard on
Thus, a series of negotiations took place both here in Manila, and
this matter raised by them, since the same can still be looked into during No. The Court cannot grant dissolution because the action is a
the hearing of a derivative suit on the merits. There was, therefore, neither in the United States, between Chase on the one hand, and Cranker and
Buencamino, on the other, for the purchase of Chase's factory (Production derivative one for the benefit of Amparts and not for the personal benefit of
error nor grave abuse of discretion in the decision of the Securities &
Exchange Commission not to dismiss the case but to remand it instead to Manufacturing Company) and the establishment of a new factory in Manila Chase, and Amparts cannot be benefited by its extinction. Also, as to the
the Hearing Panel for further proceedings. which was to be called the American Machinery Engineering Parts, Inc. ouster of Dr. Buencamino from management, it should not be forgotten
(Amparts for short). that Dr. Buencamino is not only a manager, but is in fact 2/3 owner of
Amparts and to oust him from management would amount to his
94
Derivative Suit
disenfranchisement as owner of the majority of the enterprise apart from o That the teal and company (The Company) was a domestic
the fact that it is also established in the proofs that Amparts is already Whether or not de los Angeles can file a derivative suit in behalf of the corporation duly incorporated under the laws of the Philippine and
picking up and has been a going concern after Cranker left unto him the corporation. principal office and place of business at manila. The plaintiffs Everett,
direction of its affairs. Clifford, teal and robinson were the principal stockholders in the
company that the defendant Barclay was the only other stockholder,
RULING: owning one share.
The Court therefore having in mind all these finds that the
solution most equitable and just would be to limit its decision to imposing a o In 1921, the bank persuaded the company and the said H.W
YES. Peabody and Co. and smith, Kirkpatrick and Co. to enter into a so
monetary judgment upon the guilty parties for the benefit of Amparts.
The Court ruled that since de los Angeles’ 20 shares represent only called “creditors agreement” with itself, wherein it mutually agreed that
.00001644% of the total number of outstanding shares (1 21,645,860), he neither of the parties should take action to collect its debt from the
cannot be deemed to fairly and adequately represent the interests of the Company for the term of two years after the date thereof. The plaintiffs
minority stockholders. The implicit argument — that a stockholder, to be have no copy of said agreement but beg leave to refer to the original
114. SMC v. KAHN
considered as qualified to bring a derivative suit, must hold a substantial or of same, in possession of the bank, for greater certainty.
SAN MIGUEL CORPORATION, represented by EDUARDO DE LOS
significant block of stock — finds no support whatever in the law. The o In December 1922, said company was solvent and in the enjoyment
ANGELES requisites for a derivative suit are as follows: (a) the party bringing suit of a large, growing, and lucrative business and in the possession of a
vs. should be a shareholder as of the time of the act or transaction complained valuable reputation and good-will and had done its banking business
ERNEST KAHN et al., of, the number of his shares not being material; (b) he has tried to exhaust and financing almost exclusively thru and with the bank and by reason
intra-corporate remedies, i.e., has made a demand on the board of of the company had acquired trust and confidence in the integrity and
G.R. No. 85339. August 11, 1989 directors for the appropriate relief but the latter has failed or refused to heed
good intentions of the said bank and its officers and the other
his plea; and (c) the cause of action actually devolves on the corporation,
defendants.
the wrongdoing
FACTS:
or harm having been, or being caused to the corporation and not to the o Towards the end of the year 1922, the bank, through its defendant
particular stockholder bringing the suit. The bona fide ownership by a mullen represented to the Company and its managers that for the
stockholder of stock in his own right suffices to invest him with standing to protection both of the Bank and the Company it was advisable for
33,133,266 shares of the outstanding capital stock of SMC were acquired them both that the Bank should temporarily obtain control of the
bring a derivative action for the benefit of the corporation. The number of
14 his shares is immaterial since he is not suing in his own behalf, or for the management and affairs of the Company in order that the affairs of
other corporations, and were placed under a Voting Trust Agreement in protection or vindication of his own particular right, or the redress of a wrong the Company could be conducted by the Bank without interference or
favor of the late Andres Soriano, Jr. However, 33,133,266 SMC shares committed against him, individually, but in behalf and for the benefit of the hindrance from outside, and to this end that it would be necessary for
were sequestered by the PCGG, on the ground that the stock belonged to corporation. the stockholders in the Company to place their shares therein in a
Eduardo Cojuangco, Jr., allegedly a close associate and dummy of former Voting Trust to be held by the Bank would then finance the Company
President Marcos. SMC promptly suspended payment of the other under its own supervision and that if and when the same were
installments of the price to the 14 seller corporations. On December, 1986, 115. HARRIE S. EVERETT, CRAL G. CLIFFORD, ELLIS H. TEAL and
successful and in position to resume independent operation the said
the SMC Board, by Resolution No. 86-122, "decided to assume the loans GEORGE W. ROBINSON v. THE ASIA BANKING CORPORATION,
trust would be terminated and the stock returned to its true owners,
incurred by Neptunia for the down payment ((P500M)) on the 33,133,266 NICHOLAS E. MULLEN, ERIC BARCLAY, ALFRED F. KELLY, JOHN
W. MEARS and CHARLES D. MACINTOSH and in case the Bank decided to discontinue operation under the said
shares." The Board opined that there was "nothing illegal in this trust then the stock also would be so returned.
assumption (of liability for the loans)," since Neptunia was "an indirectly G.R. No. L-25241 November 3, 1926
Facts: o It was also stated by mullen that in order to protect the mutual interests
wholly owned subsidiary of SMC," there "was no additional expense or of the Bank and the Company, it was necessary to carry into effect
exposure for the SMC Group, The plaintiff, by Thomas Cary Welch, complaint of the the proposed voting trust without knowledge of the creditors place the
and there were tax and other benefits which would redound to the SMC defendants and for cause of action against them allege: Bank in advantageous position with regard to them. The plaintiffs
group of companies. However, at the meeting of the SMC Board, Eduardo were induced to sign and did sign and deliver to the Bank
de los Angeles, one of o That the defendant the Asia Banking Corporation (The Bank), was a simultaneously a so-called “Voting Trust Agreement” executed by the
the PCGG representatives in the SMC board, impugned said Resolution foreign banking corporation duly licensed to transact banking plaintiff stockholders and a memorandum of agreement executed by
No. 86-122. business in the Philippines and its principal office and place of the Company.
business at manila but never has been empowered by law or licensed o Subsequently to the execution and delivery of the voting trust and
to do any business other than commercial banking in the Philippine MOA, the defendant mullen caused procured by virtue of the powers
ISSUE: islands. That the defendants Mullen, Alfred f. Kelly, mears and delegated in the said voting trust, the displacement and removal from
macintosh were officers, agents and employees of the said bank. the Board of Directors of the Company of each and every person who

95
Derivative Suit
was at the time of the execution of the said voting trust a stockholder the same effectively would been useless, and the law does not require
in the Company and the substitution in their places as such directors, litigants to perform useless acts. An individual stockholder is permitted to institute a derivative suit on behalf
the defendants, or employees of the Bank and no subsequent time of the corporation wherein he holds stock in order to protect or vindicate
did the trustee allow to act as a director of the Company any person corporate rights, whenever the officials of the corporation refuse to sue, or
who was in fact a stockholder in the Company, and the latter has been are the ones to be sued or hold the control of the corporation. In such
exclusively controlled and managed by the said defendants. 116. RICARDO L. GAMBOA, et al. v. Hon. OSCAR R. VICTORIANO, actions, the suing stockholder is regarded as a nominal party, with the
et al. corporation as the real party in interest.
o To defraud these plaintiffs, the new so-called directors proceeded to
G.R. No. L-40620
remove from office the Secretary of the Company, and to discharge
May 5, 1979 In the case at bar, however, the plaintiffs are alleging and vindicating their
from employment all of the old managers and the stockholders own individual interests or prejudice, and not that of the corporation. At any
(plaintiffs, who were also the real owners.) FACTS: rate, it is yet too early in the proceedings since the issues have not been
o The said defendants gave pledges and mortgages from the joined. Besides, misjoinder of parties is not a ground to dismiss an action.
Company to the Bank and entered contracts foreclose and to sell the The herein petitioners were sued by herein defendants to nullify the
property of the Company without knowing the interests of the issuance of 823 shares of stock of the Inocentes de la Rama, Inc. in favor
Company in which the Company was not represented by anyone. of the petitioners.
Defendants tricked and deluded the courts into giving judgments in 117. REYES v. TAN
which the rights of the real parties were concealed and unknown to The respondents, are the owners of 1,328 shares of stock of the Inocentes GR No.L-16982 September 30, 1961
the courts. de la Rama, Inc., a domestic corporation, with an authorized capital stock
o In august 1923, said defendants, filed in the Bureau of Commerce of 3,000 shares, with a par value of P100.00 per share, 2,177 of which FACTS:
and Industry of the Philippine Islands, articles of incorporation of a were subscribed and issued, thus leaving 823 shares unissued. Then
President and Vice-President of the corporation, respectively, the The corporation, Roxas-Kalaw Textile Mills, Inc., was organized
corporation called the “Philippine Motors Corporation” and the
defendants Mercedes R. Borromeo, Honorio de la Rama, and Ricardo on June 5, 1954 by defendants Cesar K. Roxas, Adelia K. Roxas,
defendants were officers or employee of the Bank. Such incorporation Benjamin M. Roxas, Jose Ma. Barcelona and Morris Wilson, for and on
Gamboa, remaining members of the board of directors of the corporation,
was a fraud upon these plaintiffs for the reason that it was intended in order to forestall the takeover by the plaintiffs of the afore-named behalf of the following primary principals with the following shareholdings:
for the sole purpose of taking over the assets of the Company and corporation, surreptitiously met and elected Ricardo L. Gamboa and Adelia K. Roxas, 1200 Class A shares; I. Sherman, 900 Class A shares;
said defendants were enabled to effectuate such intent by reason of Honorio de la Rama as president and vice-president of the corporation, Robert W. Born, 450 Class A shares and Morris Wilson, 450 Class A
their positions as officers and employees of the Bank and because respectively, and passed a resolution authorizing the sale of the 823 shares; that the respondent holds both Class A and Class B shares and
each of them were de facto directors of the Company, by reason of unissued shares of the corporation to the defendants, at par value, after number and value thereof are is follows: Class A — 50 shares, Class B —
their appointments by defendant Mullen, Through the Voting Trustee. which the petitioners were elected to the board of directors of the 1,250 shares. On May 8, 1957, the Board of Directors approved a
o Thereafter, said Bank turned over to the Philippine Motors corporation.
resolution designating one Dayaram as co-manager and Morris Wilson
Corporation all of the business and assets of the company of every
was likewise designated as co-manager with responsibilities for the
name, nature and description. Since then, The PMC has continued to The respondents claimed that the sale of the unissued 823 shares of stock
of the corporation was in violation of the plaintiffs' and pre-emptive rights management of the factory only‘. An office in New York was opened for the
conduct and advantage itself of the business of the Company.
and made without the approval of the board of directors representing 2/3 purpose of supervising purchases, which purchases must have the
of the outstanding capital stock, and is in disregard of the strictest relation unanimous agreement of Cesar K. Roxas, New York resident member of
ISSUE: Whether or not Plaintiffs have the capacity to sue.
of trust existing between the defendants, as stockholders. The the board of directors, Robert Born and Wadhumal Dalamal or their
RULING: respondents prayed that a writ of preliminary injunction be issued respective representatives. Several purchases aggregating $289,678.86
restraining the defendants from committing, or continuing the performance were made in New York for raw materials and shipped to the Philippines,
Invoking the well-known rule that shareholders cannot ordinarily of an act tending to prejudice, diminish or otherwise injure the plaintiffs' which shipment were found out to consist not of raw materials but already
sue in equity to redress wrongs done to the corporation, but that the action rights in the corporate properties and funds of the corporation, and from finished products, for which reasons the Central Bank of the Philippines
must be brought by the Board of Directors, the appelles argue- and the disposing, transferring, selling, or otherwise impairing the value of the 823 stopped all dollar allocations for raw materials for the corporation which
court below held that the corporation Teal and Company is a necessary shares of stock illegally issued. The respondent court granted the prayer. necessarily led to the paralyzation of the operation of the textile mill and its
party plaintiff and that the plaintiff stockholders, not having made any business.
ISSUE:
demand on the Board to bring the action, are not the proper parties plaintiff.
But, like most rule, the rule in question has its exceptions. It is alleged in the ISSUE:
Is derivative the proper action in this case?
complaint and, consequently, admitted through the demurrer that the Whether or not a derivative suit will prosper.
corporation teal and company is under the complete control of the principal RULING:
defendants in the case, and, in these circumstances, it is obvious that a HELD:
demand upon the Board of Directors to institute an action and prosecute Yes.
96
Derivative Suit
No. The claim that respondent Justiniani did not take steps to The court below sustained the demurrer as to the first and second causes of this character upon a cause of action pertaining to the corporation when
remedy the illegal importation for a period of two years is without merit. of action on the ground that in actions of this character the plaintiff must it appears that he was not a stockholder at the time of the occurrence of
During that period of time respondent had the right to assume and expect aver in his complaint that he was the owner of stock in the corporation at the acts complained of and upon which the action is based, the authorities
that the directors would remedy the anomalous situation of the corporation the time of the occurrences complained of, or else that the stock has since do not agree.
brought about by their own wrong doing. Only after such period of time had devolved upon him by operation of law.
elapsed could respondent conclude that the directors were remiss in their The decision in the Hawes case is that among other necessary averments,
This action was brought by the plaintiff Pascual, in his own right as a the bill should contain "an allegation that the plaintiff was a shareholder at
duty to protect the corporation property and business. The fraud consisted
stockholder of the bank, for the benefit of the bank, and all the other the time of the transaction of which he complains ... and that the suit is not
in importing finished textile instead of raw cotton for the textile mill; the fraud,
stockholders thereof. The plaintiff sues on behalf of the corporation, which, a collusive one to confer jurisdiction on a court of the United States in a
therefore, was committed by the manager of the business and was even though nominally a defendant, is to all intents and purposes the real case in which it would otherwise have no cognizance ... ." The language of
consented to by the directors, evidently beyond reach of respondent as plaintiff in this case. the 94th Equity Rule is practically identical with this. It provides, in terms
treasurer for that period. The directors permitted the fraudulent transaction that a stockholder's bill in cases of this character "must contain an
to go unpunished and nothing appears to have been done to remove the It is alleged in the amended complaint that the only compensation allegation that the plaintiff was a stockholder at the time of the transaction
erring purchasing managers. In a way the appointment of a receiver may contemplated or provided for the managing officers of the bank was a of which he complains ... and that the suit is not a collusive one to confer
have been thought of by the court below so that the dollar allocation for raw certain per cent of the net profits resulting from the bank's operations. jurisdiction . . . ."
material may be revived and the textile mill placed on an operating basis.
ISSUE: This is, obviously, a mere rule of pleading — it requires averments of facts
118. PASCUAL v. OROZCO Does Plaintiff Pascual have the cause of action of file the Derivative Suit? upon which the plaintiff's cause of action and the jurisdiction of the court
G.R. No. L-5174 March 17, 1911 rest. It assumes, as the court had already decided, that the ownership of
RULING: the stock at the time of the transaction is a fact essential to the maintenance
FACTS: NO. of the suit in any event. Unless that fact exists no cause of action exists,
That during the years 1903, 1904, 1905, and 1907 the defendants and whether the suit is collusive or not. Even if the stock was owned prior to the
appellees, without the knowledge, consent, or acquiescence of the In suits of this character, the corporation itself and not the plaintiff transaction complained of, if the suit is collusive — as it would be, for
stockholders, deducted their respective compensation from the gross stockholder is the real party in interest. The rights of the individual instance if one of the defendants had acquired a merely colorable domicile
income instead of from the net profits of the bank, thereby defrauding the stockholder are merged into that of the corporation. It is a universally in another State to support the allegation of diversity of citizenship — the
bank and its stockholders of approximately P20,000 per annum; that recognized doctrine that a stockholder in a corporation has no title legal or plaintiff has no right to maintain the action in a Federal court. Consequently,
though due demands has been made upon them therefor, defendants equitable to the corporate property; that both of these are in the corporation the rule requires that these two facts be distinctly averred. The requirement
refuse to refund to the bank the sums so misappropriated, or any part itself for the benefit of all the stockholders. Text writers illustrate this rule by that they be pleaded is procedural. The necessity of the existence of the
thereof; that defendants constitute a majority of the present board of the familiar example of one person or entity owning all the stock and still facts in order to give rise to the right of action is substantive.
directors of the bank, who alone can authorize an action against them in having no greater or essentially different title than if he owned but one
the name of the corporation, and that prior to the filing of the present suit single share. Since, therefore, the stockholder has no title, it is evident that As a general proposition, the purchaser of stock in a corporation is not
plaintiff exhausted every remedy in the premises within this banking what he does have, with respect to the corporation and his fellow allowed to attack the acts and management of the company prior to the
corporation. stockholder, are certain rights sui generis. acquisition of his stock; otherwise we might have a case where stock duly
represented in a corporation consented to and participated in bad
The second cause of action sets forth that defendants' and appellees' So it is clear that the plaintiff, by reason of the fact that he is a stockholder management and waste, and after reaping the benefits from such
immediate predecessors in office in this bank during the years 1899, 1900, in the bank (corporation) has a right to maintain a suit for and on behalf of transaction, could be easily passed into the hands of a subsequent
1901, and 1902, committed the same illegality as to their compensation as the bank, but the extent of such a right must depend upon when, how, and purchaser, who could make his harvest by appearing and contesting the
is charged against the defendants themselves; that in the four years for what purpose he acquired the shares which he now owns. very acts and conducts which his vendor had consented to.
immediately following the year 1902, the defendants and appellees were
the only officials or representatives of the bank who could and should It is alleged that the plaintiff became a stockholder on the 13th of Where stock is required for the purpose of bringing suit it has been held
investigate and take action in regard to the sums of money thus November, 1903; that the defendants, as members of the board of that the complainant is a mere interloper and entitled to no consideration.
fraudulently appropriated by their predecessors; that they were the only directors and board of government, respectively, during each and all the And stockholder suits not brought in good faith in the interest of the
persons interested in the bank who knew of the fraudulent appropriation by years 1903, 1904, 1905, 1906, and 1907, did fraudulently, and to the great corporation have been dismissed on the ground. (Home Fire Ins. Co. vs.
their predecessors; that they wholly neglected to take any action in the prejudice of the bank and its stockholders, appropriate to their own use Baker, supra, and cases cited therein.) Some of the State courts hold that
premises or inform the stockholders thereof; that due demand has been from the profits of the bank sums of money amounting approximately to a purchaser of shares in a corporation acquires all the rights of the vendor.
made upon defendants to reimburse the bank for this loss; that the bank P20,000 per annum.
itself can not bring an action in its own name against the defendants and If the transferee purchased the shares in good faith, and without notice of
appellees, for the reason already stated, and that there remains no remedy It affirmatively appears from the complaint that the plaintiff was not a the fact that the prior holder had precluded himself from suing, he would
within the corporation itself. stockholder during any of the time in question in this second cause of have as just a title to relief as if he had purchased from a shareholder who
action. Upon the question whether or not a stockholder can maintain a suit was under no disability; but if the purchaser was aware that the prior holder
97
Derivative Suit
had barred his right to relief, neither justice nor public policy would require whether any case, going as far as this, can be found. No such case has
that the transferee, under these circumstances, should be accorded any been cited in the argument. Dicta of judges to that effect may undoubtedly
greater rights than his transferrer. be produced, but they are not supported by the facts of the cases under
consideration.
If a stockholder participates in a wrongful or fraudulent contract, or silently
acquiesces until the contract becomes executed, he can not then come It seems clear that subsequent creditors have no better right than
into a court of equity to cancel the contract, and more especially if the subsequent purchasers, to question a previous transaction in which the
company, or himself, as a stockholder, has reaped a benefit from the debtor's property was obtained from him by fraud, which he has
contract; and this rule holds good, although the consideration of the acquiesced in, and which he has manifested no desire to disturb. Yet, in
contract may be one expressly prohibited by statute. The same disability such a case, subsequent purchasers have no such right.
would attach to the transferee of his stock who bought with notice.
So it seems to be settled by the Supreme Court of the United States, as a
This rule, in the main, is correctly stated, but we think that the latter part of matter of substantive law, that a stockholder in a corporation who
the same should be modified so as to read: "The same disability would was not such at the time of the transactions complained of, or whose
attach to the transferee of his stock who bought with or without notice." We shares had not devolved upon him since by operation of law, can not
base our modification of this rule upon the ground that a transferee could maintain suits of this character, unless such transactions continue
not sue as being a bona fide purchaser in ignorance of the disability and are injurious to the stockholder, or affect him especially and
attaching to his vendor, because shares of stock, strictly speaking, are not specifically in some other way.
negotiable, and the sale can not pass greater rights than those possessed
by the vendor. (Clark vs. American Coal Co., 86 Iowa, 436; 4 Thomp.
Corp., 3410.)

It is self-evident that the plaintiff in the case at bar was not, before he
acquired in September, 1903, the shares which he now owns, injured or
affected in any manner by the transactions set forth in the second cause of
action. His vendor could have complained of these transactions, but he did
not choose to do so. The discretion whether to sue to set them aside, or to
acquiesce in and agree to them, is, in our opinion, incapable of transfer. If
the plaintiff himself had been injured by the acts of defendants'
predecessors that is another matter. He ought to take things as he found
them when he voluntarily acquired his ten shares. If he was defrauded in
the purchase of these shares he should sue his vendor.

If the party himself, who is the victim of fraud or usury, chooses to waive
his remedy and release the party, it does not belong to a subsequent
purchaser under him to recall and assume the remedy for him. (Quoted
with approval in the case of the Graham vs. La Crosse and Milwaukee
R.R. Co., 102., U.S., 148.)

But it is contended that this is a case in which the debtor corporation was
defrauded of its property, and that as the company had a right of
proceeding for its recovery, any of its judgment and execution creditors
have an equal right; that it is a property right, and one that inures to the
benefit of creditors.

Conceding that creditors who were such when the fraudulent procurement
of the debtor's property occurred — and cases to that effect have been
cited — the question still remains, whether, the debtor being unwilling to
disturb the transaction, subsequent creditors have such an interest that
they can reach the property for the satisfaction of their debts. We doubt
98

You might also like